Super Set 1

Réussis tes devoirs et examens dès maintenant avec Quizwiz!

Team Stage--Forming

Team Stage--Forming 1) Member are Inexperienced Anxious Proud 2) "Honeymoon"----- Exploration of boundaries of acceptable group behavior. 3) Each team member looks to team leader (or Facilitator) for guidance on his role or responsiblities.

Twenty ports in a housing may each have dimensional errors (defects). A control chart is being set up to monitor the number of unsatisfactory ports in each housing, where each housing is a subgroup. Which control chart is most appropriate to use? np p u c

The np chart is used when subgroup sizes are constant and you are measuring defectives. The p chart is used when subgroup sizes vary and you are measuring defectives. The u chart is used when subgroup sizes vary and you are measuring defects. In this case, subgroup sizes are constant. The c chart is used when subgroup sizes are constant and you are measuring defects.

In order to improve its services, a passport issuing authority started accepting applications for passports renewals trough local post offices in remote areas. The post office clerks accept renewal applications, collect supporting documents and fees, and forward completed applications to the passport authority for processing. Which of the following describes the passport authority's relationship with the post offices? A) The post offices are internal customers for the passport authority. B) The passport authority is a supplier to the post offices. C) The post offices are external customers for the passport authority. D) The passport authority is an external customer for the post offices.

The passport authority is using the post offices services, and hence it is their external customer. The post offices are suppliers to the passport authority and not customers.

Tools for managing the project and helping to define problem & current state

Tools for managing the project and helping to define problem & current state a. Affinity Diagram b. Interrelationship diagrams c. Tree Diagrams d. Prioritization Matrices e. Matrix Diagrams f. Process Decision program Chart (PDPC)

The results of a gage study calculate the Total Gage R&R study variation (6*standard deviation) as 0.36. If the tolerance is +/- 0.25 then what is the percent of tolerance? 72% 144% 12% 24%

% of Tolerance is the Total Gage R&R study variation / tolerance width. The tolerance width is 0.50 (0.25 on either side). 0.36/0.50 = 0.72 x 100% = 72%.

2 Books by Deming

2 Books by Deming: 1. Quality, Productivity, and Competitive Position 2. Out of Crisis

5 Things that Project Statement must define

5 Things that Project Statement must define: 1) Objectives 2) Scope 3) Measures 4) Milestones 5) Resources

Types of Variation in Measurements: Bias

5) Bias- (difference b/w absolute value and real value) with respect to a standard master at various measurement points of the measuring range. 6) Wide-spread understanding of measurement system analysis was accepted after the introduction of QS-9000

The least squares regression equation relating appraisal cost growth to year of production is: Appraisal Cost Growth Rate = 5.67 - (0.75) year. The model was constructed with 5 years of data. Predict the appraisal cost growth rate in year 3. 1.92 4.92 3.42 4.17

5.67 - 0.75(3) = 3.42

Attribute Data/ Discrete

Attribute Data 1) Data values can only be integers--eg. No. of defects, absent people, kind of perfomance.

Descriptive Statistics

Descriptive Statistics Includes: 1) Central Tendency 2) Measures of Dispersion 3) Probability Density function 4) Frequency Distribution 5) Cumulative Distribution

Attribute data are also referred to as: Representative Continuous Discrete Variable

Discrete is also referred to as attribute data, which have only a few values. Continuous is variable type data or measured data. Attribute data, however, is when the characteristics being studied have only a few values

Causes of "Within Appraiser" inconsistencies due to human error

"Within Appraiser" inconsistencies due to human error : 1. Misunderstanding of the criteria 2. Human mood swings 3. Ergonomics of the inspection area 4. Fatigue. iv. Types of Attribute study involving ranking of

When interpreting the results and preparing the report from a design of experiment, which of the following is important when presenting the findings? Present all available sources of information. Limit conclusions to object summary of evidence. Outline possible alternative outcomes. State possible interrelationships of the factors.

Correct! The conclusions must be backed by the data from the experiment performed. So limit conclusion to object summary of evidence In the design of the experimental program, stating possible interrelationships of factors and alternative outcomes is performed DURING the DOE and not at the reporting stage.

Deming and True Metrics

Deming and True Metrics: Deming stated" The problem with obtaining true value of measurements is the presence of variation in all meausrement and one must be septical of how data is collected. Must answer the when, where, and how the product will impact accuracy.

First -Order

First -Order 1) Refers to the power to which a factor appears in a model 2) First-order models cannot account from curvature or interaction. 3) If "X1" represent a factor and "B" is a factor effect then the model: Y= B0 + B1X1 + B2X2 + E

Measurement System Considerations

Measurement System Considerations 1. Select less than 20 metrics 2. Metrics should focus on past, present, and future 3. Metrics should be linked to the needs of shareholders on the business level , customers on operational , & employees on the process level. 4.

Design of Experimentation Randomization

Randomization a. Organizes the experiment to have treatment combinations done in a chance manner, improving statistical validity.

Taguch---Statement of Quality

Taguchi---Statement of Quality Quality is related to the financial loss to society caused by a product during its life cyvle

List the 9 types of Waste in Lean System

Types of Wastes 1) Overproduction 2) Excess Motion 3) Waiting 4) Inventory 5) Excess Movement of Materials 6) Defect Correction 7) Excess Processing 8) Lost Creativity

X-bar and R-Chart

X-bar and R-Chart

Armand Feigenbaum 6 books

Armand Feigenbaum---6 books

Best use of Attribute chart

Best use of Attribute chart 1) Flow trends and cycles 2) Evaluate any change in the process

New Product Term---Cost Reductions

New Product Term---Cost Reductions New Products designed to replace existing products but a lower price

Null Hypothesis

Null Hypothesis-→ believed to be true unless evidence contrary.

6 Tools for Problem-Solving

6 Tools for Problem-Solving: 1) PCDA/PDSA 2) 8D eight disciplines 3) FMEA/ FMECA 4) QFD 5) DOE 6)Hypothesis Testing *** Not competing techniques but are complementary to each other. **Standardized problem-solving methods result in uniform reporting of results.

Motion/ Muda

Motion/ Muda

Duties of Champion

Duties of Champion i. Resource allocation ii. Determine project selection criteria iii. Approval of completed projects iv. Removal of project barriers

Inner Array

Inner Array In Taguchi style factional factorial experiment, these are the factors that can be controlled in a process.

Tools for Business performance measures: Other measures of performance of business:

Other measures of performance of business: i. Defects per single line of code for software projects ii. Customer complaints iii. Inquiry response time iv. Defect containment

Design of Experimentation

1. Factor a. Variable controlled by the experimenter.

Project Elements

Project Elements

What are Black Belts?

"Black Belts"---key people trained in "statistical thinking" at all levels and have training in advanced statistics and project management.

Treatment

Treatment In an experiment, the various factor levels that describe how an experiment is to be carried out.

DMADV Process Define Stage

DMADV Process a. Define i. Evaluation and prioritization of primary design objectives to reach Six Sigma targets

Project Team

Project Team member a. Selected by process owner and trained in 6 Sigma methodologies, quality, basic statistical tools, and process improvement techniques.

Armand Feigenbaum Key contribution

Armand Feigenbaum Key contribution 1. Concept of "Total Quality Control" in 1940 at GE. 2. His TQC statement was first published in 1961 but the concept was so new that no one listened

Matrix Diagram

Matrix Diagram 1) Allows illustration and discovery of relationships between two groups. Each square in matrix is examined for relationship between items in the row or column.

42. Which of the following distributions would be appropriate for discrete data? a. Exponential b. Poisson c. Normal d. Johnson"

"42. B: Poisson. A Poisson distribution would be most appropriate for discrete data. Binomial distributions may also be used for discrete data. Continuous data, on the other hand, should be handled with a normal, exponential, Johnson, or Pearson distribution. Continuous data is obtained from measurement, while discrete data is based on observation. A discrete data set, for instance, would only indicate the number of times an event occurred, but "but would not give any indication of the size or intensity of the event.

"Complete factorial designs

"Complete factorial designs 1) Formula for No. of Runs needed= (number of levels for each factor) ^ power equal to the number of factors. --->( eg. Each factor has 3 levels & 4 factors so 3 ^ 4th power = 81( 81 required runs) 2) Factorial design = standard table, with each row representing a different configuration of the factors. 3) Exponential relationship between number of factors & Number of the levels for each factor,---> small increases in (# of factors) ~ GIANT increase in (# of required runs) . a) For this reason, complete factorial designs quickly can become unwieldy and inefficient--> Rarely used in business. b) Better to start with a fractional factorial design to identify multi-factor interactions that require more study.

Creation of P-chart

"Creation of P-chart: 1) Gather samples from the same point in the process every time. 2) Key Difference= Np chart =samples require same number of units,---- P chart = samples with varying numbers of units. 3) P chart= the plotted statistic is % of items in each sample that have the target characteristic. 4) Formula for plotted statistic p(j) = (count) j / (nj) where nj is the number of units taken from group j. 5) The centerline is calculated with (lookup the formula) . a) Again, nj = sample size of group j, and m is the number of groups included in analysis. 6) Look up the formula for The upper control limit is calculated with , and the lower control limit with . In both of these equations, p-bar is the average percent."

Eliminating quotas and ineffective management styles:

"Eliminating quotas and ineffective management styles: Deming's eleventh point for management is split into two parts. a) Declaration that production quotas should be eliminated. ----> For Deming, this sort of leadership is ineffective because continuous improvement always will affect the appropriateness of targets. + better to encourage employee pride and promote quality is than mere focus on quotas. b) Second part= Deming believed establishing a positive long-term plan and adhering to this plan made more sense than pursuing immediate gratification in the form of quotas or sales targets. By emphasizing continuous quality improvement, managers create the conditions for success. "

"Implementation of X-bar

"Implementation of X-bar charts 1) Used when subgroups of more than two observations can be measured. a) time = x-axis ---> X-chart= chronological model of the process, moving from left to right. b) To detect special cause variation, must assign a time to all data c) If subgroup size > 10, ---> Use sigma chart (avoid using range chart to monitor process variation= can't estimate process sigma) d) . If subgroup sizes = 1, an exponentially weighted moving average or individual-X/moving-range chart may be used. "

Process when the historical standard deviation is unknown

"Process when the historical standard deviation is unknown a) If the historical standard deviation is unknown, must start by calculating the average and the sample standard deviation s for n sample units. b) The confidence interval is calculated with the formula ( see formula) . c) Similar to when historical standard deviation known, it is assumed that the samples are taken from a population with a normal distribution. d) Again, the confidence interval will be two-sided, so confidence level should be divided by two.

Purpose of Hypothesis testing

1. Purpose of Hypothesis testing--→ statistical conclusion about accepting or not accepting statements.

5 Things that a facilitator must avoid doing

5 Things that a facilitator must avoid doing 1) Being judgmental of team/ ideas/ comments 2) Taking sides 3) Dominating group discussions 4) Giving the answer or solving problem for th eteam 5) Making suggestions on the task instead of on the process

5 Advantages of Project Management:

6 Advantages of Project Management 1) Easy to use and low cost 2) Best for monitoring schedules and event timings 3) Hands-on feel of the project status 4) Customizable 5) Little training needed

Hardness of a rubber product doesn't follow the Normal distribution, but the mean and standard deviation of sample of 36 are 70 and 6 Shore A (the hardness scale). What is the 95% confidence interval for population mean? (Z0.025=1.96, Z0.05=1.645) A) 68.355 < µ < 71.345 B) The population standard deviation is not known so we cannot calculate the confidence interval. C) 68.04 < µ < 71.96 D) The distribution is not Normal so we cannot calculate the confidence interval.

68.04 < µ < 71.96 Mean is between +1.96 ( 6/ square root 36) = (1.96) (6/6) = 70 + 1.96= 71.96 and - 1.96 ( 6/ square root 36) = (-1.96) (6/6) =[ 70 - 1.96]= 68.04

7. Which is typically the first category to be identified in SIPOC analysis? a. Suppliers b. Inputs c. Outputs d. Processes

7. C: Outputs. In SIPOC analysis, the first category to be identified is outputs. SIPOC (suppliers, inputs, processes, outputs, and customers) analysis is typically performed during the define stage of DMAIC. Its intention is to identify the most important processes and the relevant stakeholders. At the beginning of SIPOC analysis, it is typical to create a process map or flowchart. Outputs are the first category to be identified, because the identification of outputs facilitates the identification of suppliers, inputs, and customers.

9 Common Causes of Conflict

9 Common Causes of Conflict 1. Organizational Structure 2. Value differences 3. Difference in ideals or perceptions 4. Divergent goals 5. Differences in priorities 6. Changes in procedures 7. Role pressures 8. Status threats 9. Personality Clashes

What is the estimated percent defective for the upper specification area of a normal distribution that has two-sided limits and a Cp=1 and Cpk=1? 135ppm 35ppm 2650ppm 1350ppm

99.73 percent of the area is under the normal distribution which has two sided limits with a Cp=1 and Cpk=1. Thus, .27% is the percent defective for the entire curve, thus for the upper tail it would be .135% or 1350 ppm.

Six Sigma Manager is scheduling and tracking the progress of critical project tasks within a project. Which of the following is most likely to be the best tool he can use? Critical Path Method Project charter Gantt chart PERT chart

A Gantt chart is a bar chart that shows the tasks of a project - when each must take place and how long each will take. As the project progresses, bars are shaded to show which tasks have been completed? It is useful to indicate the critical path on the chart with bold or colored outlines of the bars for the steps on the critical path. CPM is not used for ongoing progress tracking. CPM is used to identify which activities drive end-to-end project completion time and which do not (are or are not on the critical path for the project). project charter has a timeline, but is too high-level to track specific progress of deliverables.

Which of the following elements should be included in a control plan? Plan for the control phase Project Charter Tests for significance Identified sample sizes

A control plan is used for inspection activates, not the project's control phase. The project charter is a component of the Define Phase and not typically included in the control plan. Tests for significance are usually used in the Analyze Phase and not an element of the control plan.

Adaptive Approaches to Poke/ Yoke

Adaptive Approaches to Poke/ Yoke: 1) Gadgets/ Device can stop machines from working if a part or operation sequence has been missed by operator. 2) Specialized tray or dish used prior to assembly to ensure all parts are present (eg. surgical tray) 3) Service-oriented check-lists that can be used to assist an attendant is case of interruption.

Which of the following statements best describes how Six Sigma and lean tools are applied? A) Six Sigma defines the project and quality levels and lean tools identify non-value added waste. B) Lean tools are only applicable in manufacturing or transactional situations, whereas Six Sigma tools can be used throughout. C) It is best to apply Six Sigma and lean separately with experts in each discipline working together on projects. D) Lean tools define the project and quality levels with Six Sigma identifying non-value added waste

Although lean tools can be used to define projects, they typically are more effectively used to measure process performance. High-level Six Sigma metrics define projects that then can use lean tools to measure process performance. Close

The theory of constraints method starts with which of the following steps? A)Start by optimizing all processes in the process chain. B) Start by identifying the weakest process in the process chain. C) Start by adjusting the processes to match the weakest process. D) Start by improving all the slow processes.

B= CORRECT!! ----The first step in theory of constraints is to identify the constraining (weakest) process. Theory of constraints recognizes that optimizing the pieces of a system sub-optimizes the whole system. Theory of constraints can be repeated to improve the next weak process after the weakest process is improved. Theory of constraints recognizes that optimizing the pieces of a system sub-optimizes the whole system.

Benefits of Initial Project

Benefits of Initial Project 1) Team members learn about team work 2) Organization achieves its objectives 3) Individuals gain experience in project selection and completion 4) Project that is selected may solve a problem ---> eg. cost reduction, reduced cycle time, or efficient use of resources. 5) Scope of project can vary and can even be extended to many years.

"9. How many runs would be required in a complete factorial design if there are four levels and three factors? a. 7 "b. 12 c. 64 d. 81

C "9. C: 64. If there are four levels and three factors in a complete factorial design, 64 runs would be required. The number of required runs is calculated by raising the number of levels to a power equal to the number of factors. In this case, then, the calculation is performed 43 = 64. "If the complete factorial design had five levels and three factors, the number of runs would be calculated 53 = 125."

In an experimental design, there are 8 machines that are run by 4 operators where the switching mechanism with 2 positions will be observed. Identify the maximum number of factors in this experiment. 3 4 8 2

Correct! There are three factors: machine, operator, and switching mechanism.

What is a single level assigned to a single factor during an experimental run? Treatment Block Version Unit

Correct! A treatment is a single level assigned to a single factor during an experimental run of a designed experiment. A treatment is applied to a unit. A block is a factor that has influence as a source of variability.

Describe Critical-to-Cost metrics

Describe Critical-to-Cost metrics 1) identifies areas of the process that raise the expense 2) Should not include only the typical cost of task, but also the increased cost of errors in the performance of this task. 3) Should include the probability of the error. 4) If lag time is present---> Include the cost of keeping product on hand

How are FMEAs Used?

How are FMEAs Used? 1. New designs and Technologies 2. Modification of existing design. process 3. New environments 4. New application of existing product ***Customer should be the end-user, other design teams, or internal operation. ***Pareto Analysis can be used to rank the potential Failure modes.

Jumps in Process Level

Jumps in Process Level A. X-Bar Chart Causes 1) Changes in proportions of materials coming different sources 2) New operator or machine 3) Modification of production method or process 4) Change in inspection device or method B R- Chart Causes 1) Change in material, method, operator, or inspection C. Corrective Action 1) Keep material supply consistent 2) Investigate source of material 3) Check out machine capability 4) Examine operator methods and instruction 5) Check calbiration of measurement device

Objectives of Statistical Process Control

Objectives of Statistical Process Control 1) Determine process capability 2) Monitor process 3) identify whether is process is operating as expected or whether the process has changed and is need of corrective action.

Parallel Experiments

Parallel Experiments 1) Experiment done at the same time, not one after the another. 2) Opposite of Sequential experimentation.

Precision

Precision Same as Repeatability ---can repeat the same number by the same operator every time. ***To improve accuracy and precision, must have define test method and must be statistically stable.

Process Capability Indices

Process Capability Indices Discuss the various process capability indices that are commonly used as baseline measurements in the MEASURE phase and in the CONTROL phase. The concept of process capability pertains only to processes that are in statistical control. There are two types of data being analyzed: CONTINUOUS DATA: PPM DPMO Z-short term and Z-long term scores VARIABLE DATA: Cp Cpk Pp Ppk Cpm Capability estimates are strongest (but not always required) when: 1) Process is in stable and in statistical control - use SPC charts to verify 2) Data is normally distributed (however data does not have to be normally distributed to use control charts) If not normal, the data is transformable Satisfies the Central Limit Theorem and normality assumptions apply The capability indices of Ppk and Cpk use the mean and standard deviation to estimate probability. A target value from historical performance or the customer can be used to estimate the Cpm. Cp and Pp are measurements that do not account for the mean being centered around the tolerance midpoint. The higher these values means the narrower the spread (more precise) of the process. That spread being centered around the midpoint is part of the Cpk and Ppk calculations. The midpoint = (USL-LSL) / 2 The addition of "k" quantifies the amount of which a distribution is centered. A perfectly centered process where the mean is the same as the midpoint will have a "k" value of 0. The minimum value of "k" is 0 and the maximum is 1.0. An estimate for Cpk = Cp(1-k). and since the maximum value for k is 1.0, then the value for Cpk is always equal to or less than Cp. Cp and Cpk are coined as "within subgroup", "short term", or "potential capability" measurements of process capability because they use a smoothing estimate for sigma. These indices should measure only inherent variation, that is common cause variation within the subgroup. Plotting subgroup to subgroup data as individuals (I-MR) will likely show an out of control chart and process that is likely in control just showing lot-lot variation, shift-shift variation or day to day variation which is expected. Therefore the SEQUENCE of gathering and measuring is mandatory to have a correct calculation of Cp and Cpk. The subgroups should be the same size and an the highest value is most likely obtained for Cp when the samples are collected with one operator on one shift on one machine with one set of tools, etc. Most common estimate for Cp and Cpk uses an average of the subgroup ranges, R-bar, in a process with only inherent variation (no special causes) formula that lowers the width of the data distribution (sigma) from the X-bar & R chart. This optimization of sigma reduces its spread and value further increasing the value Cp and Cpk over Pp and Ppk. Pp and Ppk use an estimate for sigma that takes into account all or total process variation including special causes (should they exist) and this estimate of sigma is the sample standard deviation, s, applies to most all situations. This estimation accounts for "within subgroup" and "between subgroup" variation. Cp, since it is a short term index and not dependent on centering and uses an optimal smoothed and reduced estimate for sigma, represents the process entitlement. Process entitlement the best a process can be expected to perform in terms of minimal variation under existing conditions. Cpk value can never exceed Cp. A perfectly centered distribution on the midpoint will have a Cpk = Cp. Any movement either way from the midpoint will have a "k" value of <1.0 and Cpk < Cp. In Cp and Pp, consider the numerator (USL-LSL) as a constant. As the estimate for standard deviation (sigma) of a distribution reduces and approaches zero the value of Cp and Pp will increase towards infinity. Cp and Pp are meaningless if only unilateral tolerances are provided, in other words if only the USL or LSL are provided. Both tolerances (bilateral) must be provided to calculate a meaningful Cp and Pp. A boundary can be used (such as 0 lower limit) but the meaning of Cp to Cpk will differ from the meaning using bilateral tolerances. The overall process performance indices, Pp and Ppk, most often uses the sample standard deviation, s, formula as an estimate for sigma. There are other methods available for estimating the overall (total) process sigma. The Cpk and Ppk will require two calculations, selecting the minimum is the value use as baseline and to compare to customer acceptability level. These can be calculated using unilateral or bilateral tolerances. Shown in the table below is the formula for bilateral tolerances where a LSL and USL are provided. If only one specification is provided (unilateral) then the value used for Cpk and Ppk is provided by the calculation that involves the specification limit provided. Pp and Ppk are rarely used compared to Cp and Cpk. They should only be used as relative comparisons to their counterparts. Capability indices, Cp and Cpk, should be compared to one another to assess the differences over a period of time. The goal is to have a high Cp, and get the process centered so the Cpk increases and approaches Cp. The same applies for Pp and Ppk. Cpk and Ppk account for centering of the process among the midpoint of the specifications. However, this performance index may not be optimal if the customer wants another point as the target other than the midpoint. The calculation of Cpm accounts for the addition of a target value.

Purpose of 6 sigma academy?

Purpose of 6 sigma academy? To accelerate the efforts of corporations to achieve world class standards.

How would you best ensure that everyone in your nation-wide company receives the same information about the change in health care benefits? Give all managers a summary of the changes and ask them to communicate this information to their staff asq/learn_check.gif Send a written memo to all employees, followed by Q&A sessions at each location Show how the changes would support the company's mission, vision and values This is a task for the health insurance company

Sending a written memo to all employees, followed by Q&A sessions at each location is an important matter and should include the opportunity for a two-way dialogue. Giving all managers a summary of the changes and asking them to communicate this information to their staff assumes that the managers will communicate the news consistently and that they will schedule meetings with their staff.

Six Sigma and other team roles & responsibilities. Executive Sponsor

Six Sigma and other team roles & responsibilities. 1) Executive Sponsor a. Business unit leader responsible for profit and loss. b. Duties: i. Set the direction/vision/ objectives of the Six sigma project or organization ii. Initiate incentive programs and reward successful projects. iii. Resource allocation iv. Monitoring of the program's project. 5) Black Belt a. Skills i. 6 Sigma methods & statistics ii. Basic financial tools iii. Project management iv. Risk assessment

Steps for Process Capability Studies Measurement System Verification

Steps for Process Capability Studies 1) Measurement System Verification a. Measurement system analysis (1st step in process capability study)----Variations in the measurement system can mislead the process capability study & process stability monitoring b. Remove sources of variation in the measurement system-→ reduces the variations due to measurements ( reduces measurement variation) present in the overall process variation and process tolerance.

Symptoms of Waning management Support

Symptoms of Waning management Support 1) Lip service without action by mid-management 2) Delays in projects getting implemented 3) Supporting Champion leaves 4) Increasing need for top management stimuli 5) Passive, rather than active, management support.

System FMECA

System FMECA 1) Uses only 2 probability components---D= Design Weakness & P= probability that failure mode will occur

T-distribution

T-distribution 1) The t distribution allows us to conduct statistical analyses on certain data sets that are not appropriate for analysis, using the normal distribution. 2) But sample sizes are sometimes small, and often we do not know the standard deviation of the population. When either of these problems occur, statisticians rely on the distribution of the t statistic (also known as the t score),

Team Empowerment

Team Empowerment 1. Team charter clarify what team members are empowered to do by management 2. Control, Information, and Access to Resources are 3 sources of power for team member given to them by management a) Team Control over performance & behavior b) Information ---> financial conditions, market conditions, organizational changes, ect. c) Freedom to use organizational resources influences team success.

Team Problems & Solution---Wanderlust (tangents & digressions)

Team Problems & Solution---Wanderlust (tangents & digressions) 1) Digressions---> Follow written agenda 2) Sensitive issues avoided----> Enforce team operating guidelines 3) Group digresses from topic---> redirect the topic

Team Stage---Norming

Team Stage---Norming 1. Members are cooperative, talk things out, focused objectives so fewer conflicts 2. Group cohesion develops 3. Use to data analysis, hypothesis testing, and root cause analysis.

Team Stages

Team Stages 1. Forming 2. Storming 3. Norming 4, Performing

Test Coverage

Test Coverage Percentage of all possible combinations of input factors in an experimental test

There are many sources of variation in the process. However, these sources can be grouped into three categories:

There are many sources of variation in the process. However, these sources can be grouped into three categories: a. variation due to the process itself b. variation due to sampling c. variation due to the measurement system

Two calculation used to identify how capable a process is

Two calculation used to identify how capable a process is: i. Capability index (Cp) ii. Process performance (Cpk)

Use of Analytical statistics to estimation of process baseline

Use of Analytical statistics to estimation of process baseline 1) Analytical statics ---used to distinguish special and common cause variation in dynamic, moving processes. 2) Statistical process control---operational defintion of special cause variation---notes the location and level of variation.

Term: Tool that describes all actions (both value-added & NVA) required to take a product from raw materials and deliver to the customer

Value Stream Map Tool that describes all actions (both value-added & NVA) required to take a product from raw materials and deliver to the customer

15. In what order are the process steps presented in a process decision program chart? a. Right to left b. Left to right c. Top to bottom d. Bottom to top

a 15. A: Right to left. In a process decision program chart, the process steps are presented from right to left. A process decision program chart is used to isolate possible problems with a particular process or strategy. These charts are typically used during the Analyze and Improve stages of DMAIC. At the top of the chart, the process is named. The steps in the process are then presented from right to left, with any necessary substeps mentioned underneath. Then the potential problems in each step are listed, along with some brainstormed solutions.

"19. Which of the following characteristics of a team most often results in groupthink? a. Frequent communication b. Lack of accountability c. Lack of subject expertise d. Undefined roles

c 9. C: Lack of subject expertise. When a team has a lack of subject expertise, it is more likely to suffer from groupthink. Groupthink[...]" "Groupthink is a phenomenon in which team members agree too readily, without adequately challenging each other's ideas. When groupthink occurs, a team will often select the first proposed solution to a problem, even if it has serious weaknesses. Groupthink is more likely to be a problem when the team members do not have enough experience or expertise in the subject area to come up with alternatives to a recommendation. Also, if the team members do not feel empowered to offer their views, they may be more likely to engage in groupthink.

If the characteristic or process variation is centered between its specification limits

c. If the characteristic or process variation is centered between its specification limits, the calculated value for CPK is equal to the calculated value for CP. But as soon as the process variation moves off the specification center, it's penalized in proportion to how far it's offset. CPK is very useful and very widely used. i. That's because it compares the width of the specification with the width of the process while also accounting for any error in the location of the central tendency. This approach is much more realistic than the one the CP method offers. ii. Generally, a CPK greater than 1.33 indicates that a process or characteristic is capable in the short term. Values less than 1.33 tell you that the variation is either too wide compared to the specification or that the location of the variation is offset from the center of the specification. It may be a combination of both width and location.

Sometimes quality improvements cause reductions in workforce. Select the most appropriate way for this situation to be communicated to employees: A) Bilateral communication B) Bottom Up communication C) Top Down communication D) Horizontal communication

op Down communication is the correct answer since communication of this type needs to come from the top of the company. Horizontal communication is used in flat organizations to communicate across the organization.

Value Stream Mapping

Value Stream Mapping 1) Identifies all of the activities involved in product manufacturing from start to finish. 2) Includes suppliers, production operations, and the end customer 3) For product development, value stream mapping includes the design flow from product concept to launch

A Green Belt wishes to sample castings due to some parts being out of tolerance. Which of the following multi-vari sampling techniques would be used for batch to batch sampling? Positional DOE Cyclical Temporal

A cyclical study is to assess variance from part to part. A positional study is to assess variance within a single part. A temporal study is to assess variance over time. A Design of Experiment (DOE) is used for designing and conducting experiments to determine independent and response variables.

Select the best example of vertical communication: A) Quality improvement team reporting on a cycle time reduction to the production team B) Field sales person notifying a production scheduler of pending large order C) Manager communicating the result of performance review to his direct report D) Cross-functional design team soliciting input from customers on a new product

A manager communicating the result of a performance review to his direct report is an example of vertical communication. A cross functional team soliciting input from customers on a new product AND THE REMAINING CHOICES = example of horizontal communication.

AIAG Reference: Process Control Situations

AIAG Reference: Process Control Situations i. Measurement result & decision criteria determine the "process stability, direction, and compliance with the natural process variation" ( that is , SPC, process monitoring, capability, and process improvement), the availability of samples over the entire operating range becomes very important. ii. Process Capability Study = independent estimate of process variation used to assess the adequacy of the measurement system to process control; tool used to calculate percent GR&R to process variation.

The ANOVA method for analyzing a gage study is preferred over the Average and Range method because it: makes calculations easier to perform by hand is more likely to identify inconsistent measurements within the study includes interactions between operators and items being measured uses constant values from lookup tables to estimate variation

ANOVA method can calculate interactions between operators and samples, and the Average and Range method cannot. Average and Range method uses a Range chart to identify inconsistent measurements in the study, whereas the ANOVA method does not generate a Range chart by default. Average and Range method is easier to calculate by hand. The ANOVA method is typically done with computer software.

SIPOC-- Advantage

Advantage of using SIPOC 1) Display of cross-functional activities in single, simple diagram 2) "Big Picture" perspective to which additional detail can be added. 3) Framework applicable to either large organizations or smaller processes.

Advantages and Disadvantages of Latin Square Designs

Advantages and Disadvantages of Latin Square Designs The advantages of Latin square designs are: They handle the case when we have several nuisance factors and we either cannot combine them into a single factor or we wish to keep them separate. They allow experiments with a relatively small number of runs. The disadvantages are: The number of levels of each blocking variable must equal the number of levels of the treatment factor. The Latin square model assumes that there are no interactions between the blocking variables or between the treatment variable and the blocking variable.

Types of Variation in Measurements: Reproducibility-

3) Reproducibility---("between appraiser" variation) -----Variation in measurement when measured by two or more appraisers multiple times. Measurements are taken from the same equipment by more than one appraiser over a short period of time. It is the variation in the average of the measurements made by the different appraisers when measuring the same characteristic on the same part. This is also called the "between system" variation. Appraiser variability is an estimate of standard deviation due to reproducibility.

5 Categories of Measurement Error

5 Categories of Measurement Error 1) Bias 2) Repeatability 3) Reproducibility 4) Stability 5) Linearity

5 ways that business customer can be identified

5 ways that business customer can be identified: 1) Very small number of business constomers 2) Large dollar amounts for most transactions 3) Specialized personnel handle the purchases 4) Customer may know more about the requirements than the producer 5) Supplier may allow the customer access to all sorts of information.

5S--Sort

5S--Sort 1) Schedule to target each area 2) Red tag everything and document things thrown out. 3) Keep repaired items and list the items that need repairs 4) Inspect for problems, breakages, rust, scratches, and grime

6 ways that management can support teams

6 ways that management can support teams 1) Ensuring constancy of purpose 2) Sharing business results 3) Reinforcing positive results 4) Giving people as sense of mission 5) Providing direction and support 6) Developing an integrated plan

Control Charts for Variables: X-bar & R-charts

Control Charts for Variables: a. X-bar & R-charts-→ data is readily available b. In statistical quality control, the X-bar and R chart is a type of control chart used to monitor variables data when samples are collected at regular intervals from a business or industrial process. c. The chart is advantageous in the following situations:[2] i. The sample size is relatively small (say, n ≤ 10—X bar and s charts are typically used for larger sample sizes) ii. The sample size is constant iii. Humans must perform the calculations for the chart

Benefits of Value Stream Mapping

Benefits of Value Stream Mapping: 1) Seeing the complete process 2) Identifying sources and location of waste 3) Providing common terminology for process discussions 4) Helping to make decision about flow 5) Tying multiple lean concepts and techniques together 6) Showing linkage between information and material flows 7) Describing how process can change 8) Determining the effects of various metrics 9) Provides blueprint for lean ideas

10 Survey Pitfalls

10 Survey Pitfalls 1) Poor survey design 2) Survey issues not well-defined 3) Sampling errors due to sampling techniques 4) Incorrect analysis methods 5) Treating survey as event, not a process 6) Using nonspecific questions or asking the wrong questions 7) Incorrect usage or ignoring results 8) Failing to provide feedback when necessary 9)Using too many questions 10_ Using temporary employee to conduct interviews

4 LEAN Principles that allow the firm to move toward perfection

4 LEAN Principles that allow the firm to move toward perfection: 1. Solving Customer Value Problems 2. Working the Value Stream 3. Converting To Flow 4. Making Pull Occur ***All of the above eliminate Muda

5S--Straighten

5S--Straighten 1) Analyze existing conditions for tooling, equipment, and inventory/ supplies 2)

A Six Sigma Manager assesses and evaluates the outcomes of the project, such as cycle time reduction. Which of the following methodologies should the manager select? Post-project audit Customer audit Post-project outcome assessment Budget review

A post-project outcome assessment is used at some predetermined point to assess and evaluate the quantitative and qualitative outcomes of the project. Post-project audit is a project cash/capital budgeting process.

Which type of team member behavior is considered beneficial to team success? Challenging Reluctant Overbearing Dominating

A team in which team members are challenging each other can be considered a healthy team - not a problem behavior.

Activity network diagrams

A. Activity network diagrams 1) Graphically shows interdependencies between tasks and highlights key tasks, task timeline, flow paths, ect. 2) Similar to PERT & Critical path analysis, it provides detailed data and top-level overview.

Statistical Process Control

A. Statistical Process Control 1. Control Process a. Feedback loop which measures the actual performances, compares with a standard, and acts on the difference

According to Juran, the devleopment of any measurement system should take onto account the following 5 factors

According to Juran, the development of any measurement system should take onto account the following 5 factors: 1. There should be standardized meaning of measurement 2. Data should help the decision-making process 3. It should provide worthwhile information 4. It should be easy to install 5. It can be benchmarked or used elsewhere. ***Any mechanical and electical instruments for data collection must undergo caliberation procedures. *** In many application, the appropriate metrics are qualitatiove based on customer, supplier, appraisal feedback forms

Attributes of sample data: Center of Sample

Attributes of Sample Data------Center of sample i. Center is quantified in 3 ways: Mean, Mode, Median

Design FMEA Vs. Process FMEA:

Brainstorm potential failure modes a. Design FMEA----->Potential failure mode= any manner in which product component could fail to perform its intended function. b. Process FMEA------>Potential failure mode= any process stem that could fail to perform its intended function

CPM Cost & Time Trade off

CPM Cost & Time Trade off 1) Crashing activities beyond a certain activity (eg activity I) may increase cost without further reduction in time----> leading to useless waste in resources. 2) Assumption made in crashing an activity is that it is independent of other activities

Central Limit Theorem and Sampling Distribution of the Mean

Central Limit Theorem and Sampling Distribution of the Mean 1) Central limit theorem is an important principle used in statistical process control. 2) Foundation of statistical procedures. 3) The distribution of averages tends to be normal even though the distribution from which the average data are computed is from non-normal distribution.

Champion

Champion a. Upper-level managers with training in 6 Sigma concepts/ tools/ strategies/operations methods (emphasis on management tools) that function as the liaison to senior management b. Vital in the implementation of quality management methodologies. Champions also serve as mentors to Black Belt certified professionals. c

A basic quality control tool that is used to collect data on the frequency or patterns of events, problems, or defects from observation is called a: Checklist Histogram Cause and Effect diagrams Check sheet

Check sheet----Check sheets are a tool to tally data from observations. While checklists are considered a type of check sheet. Checklists are not uses to collect frequency data, but instead are used to help ensure steps are completed in a process.

Conducting Customer Surveys

Conducting Customer Surveys: 1) Multiple measurements aid in validating the initial result 2) Customer survey sample sizes and frequency should be chosen to balance business resources and the need to monitor changes in business enviroment. 3) Adequate samples= 30 questions 4) L-type matrix survey= numerical scale quantifies results. 5) To get long-term precision and short-term sensitivity to changes in customer attitudes, divide the customer survey sample into 12 monthly subsamples and conduct them on rotating sample 6) Likert Scale can be used to evaluate changes in customer attitudes and determine shifts in business attitudes.

Control Charts for Attribute Data

Control Charts for Attribute Data P-charts Np charts C-charts U charts Short run varieties of above 4 ***Generally more costly since separate variable must have data gathered and analyzed and requires larger data

What can workers do to meet time requirements in a continuous flow environment? Work faster. Work in only one position the entire time. Communicate with each other. Fix problems as fast as possible and restart the machine.

Correct! Workers should communicate with each other to maintain the required speed.

Design for Maintainability—

Design for Maintainability— consider aspects of routine maintenance especially if long time for diagnosis and repair is needed. Maintainability includes modularity, decoupling, and component standardization.

End Users

End Users 1) Those who purchase product/service for personal use 2) Examples: a. Potential test takers b. People interested in self-development c. People who need to know about quality d. Course/University instructors

Flowchart Structure:

Flowchart Structure: a. Process boundaries= process inputs/output or suppliers/customers of the process b. Process sequence showing both alternative steps in process and parallel processes.

Future State Map

Future State Map 1) Use of creativity and teamwork to identify creative solution 2) Topics covered in this map: a) Required Takt time b) Continuous flow processing applicable c) Movement of manufactured items to shipping d) Leveling of the process e) Pacemaker process to control the tempo of value stream f) Customer pull

Guidance in Project selection & Implementation

Guidance in Project selection & Implementation for the team can come from------> Team leader, green belt, black belt, or facilitator

Guideline for Collecting Data

Guidelines---Methods of Collecting Data 1) Formulate a clear problem statement 2) Define what is measured 3) List all the important measurements to be measured. 4) Select the right measurement technique 5) Construct an uncomplicated data form 6) Decide who will collect the data 7) Arrange for appropriate sampling method 8( Decise who will analyze and report the results *** Data needs an operational definition to have meaning and the ability to control quality ** Manual Data Collection is laborious and prone to error so inferior to automatic data collection. *****Large amounts of data are difficult to analyze unless organized is digestable format---graphs, charts, histograms, and Pareto Diagrams

Impact Severity vs. Risk Probability

Impact severity is determined by analyzing "like" risks from other projects, historical data, and brainstorming Risk probability is determined by based on the likelihood that the risk will occur during execution of the project.

If events A and B are mutually exclusive, then how does this impact the occurrence of A and B? At least one of them will occur. P(A - B) = P(B) P(A ∩ B) = P(A)P(B) A and B cannot occur simultaneously.

In mutually exclusive relationships, A and B cannot occur at the same time. It means: P(A ∩ B) = 0

Input Factor

Input Factor An independent variable which may affect (dependent) response variable and is included at different levels in the experiment.

Inputs & Outputs of a Process

Inputs are categorized as 5 M's (man, material, methods/machines, mother nature, management, money, & measurement system. Output = products (hardware, software , systems, ect) & services

Interpreting Kano Model

Interpreting Kano Model According to the Kano Model (developed by Dr Noriaki Kano in the 1980s), a product or service can have three types of attribute (or property): Threshold Attributes: Which customers expect to be present in a product. Performance Attributes: Which are not absolutely necessary, but which are known about and increase the customer's enjoyment of the product. Excitement Attributes: Which customers don't even know they want, but are delighted when they find them. Threshold Attributes affect customers' satisfaction with the product or service by their absence: If they're not present, customers are dissatisfied. And even if they're present, if no other attributes are present, customers aren't particularly happy (you can see this as the bottom curve on the graph below).

Juran's contribution to Japan's quality efforts

Juran's contribution to Japan's quality efforts 1. Japan adopted Juran's application of Quality control to management , rather than a specialist technique.

Lean Concepts & Tools--

Lean Concepts & Tools--

Liker & LEAN Thinking

Liker & LEAN Thinking: 1. Liker points out that if a company is not implementing LEAN to gain benefits, they should ask if their competitors are benefiting from LEAN

MIL-STD-1629A

MIL-STD-1629A (now obsolete)= formal military document used for FMEAs by aerospace and defense application (earliest FMEA document)

Main Effect

Main Effect An estimate of the effect of a factor independent of any other factors.

Multiple Linear Regression

Multiple Linear Regression a. Explains the higher portion of y b. Requires finding out the closeness of the calculated b-values to the actual coifficient for the population.

Operating Characteristic Curve

Operating Characteristic Curve a. Plot of the true value of a process parameter against the probability that a single sample will fall within the control limits b. Show the ability of the chart to detect process changes.

The reliability of each independent component during a certain time is .99. What is the reliability of the following system? 0.99[1 - 01.02] 0.99[2(0.99) - 0.992] 0.99[2(0.99)] 0.99[0.992]

P(A1).P(Parallel) ; P(Parallel)=P(A2)+P(A3)-P(A2 & A3)= 2P(A)-P(A)^2

Percent Agreement

Percent Agreement 1) Percent agreement between measurement system and either reference value or variable being measured can be estimated using correlation coiefficient "r" 2) r= 0 then 0% agreement ; r= +/- 1, then 100% agreement

Term: Complete elimination of muda so that all activities along the value stream create value

Perfection

First VP of quality in the country

Philip Crosby

Probability Density Function

Probability Density Function (PDF) 1) Describes the behavior of a random variable ---- For continuous variables, the pdf is the probability that a variate assumes the value x, expressed in terms of an integral between two points. 2) "Shape" of distribution----grouped frequency distribution. 3) Sampling errors or lack of randomness can create data points that above the "Smooth curve" outline of the bell-shaped curve. 4) Since probability density function represents the area under the curve, the area under the probability density function must equal one.

Process Performance Indices

Process Performance Indices a. Provides picture of current process operations b. Used to compare & prioritize improvement efforts c.

Define Project Benchmarking

Project Benchmarking 1) Easier than many business processes, because of the opportunities for selection outside the groupof direct competitors. ----eg. new product introduction, construction, or new services. 2) Useful in selecting new techniques for planning, scheduling and controlling the project.

Quality Teams

Quality Teams Structure= 8-10 members from single department ---> may evolve into self-directed teams a) Can work on quality topics or overall department performance.

Randomized Block Plans

Randomized Block Plans *** Key objective ---------->If it is desired to neutralize the effect of single factor, assembly line on the response, as it is acting as a nuisance factor 1) Comparing a number of factor treatments requires keeping all other conditions constant. 2) If the required number of tests is too large to carry out----> consider dividing the experiment into blocks or planned homogenouous groups 3) Randomized block plans = each group in the experiment has exactly one measurement of every treatment

Range Method

Range Method 1) Reproducibility is the variability introduced into the measurement system by bias difference of different operators 2) Rang method ---simple way to quantify the combined repeatability and reproducibility of a system (can't do it separately---need Average and range method to separate it)

Reproducibility: Appraiser Variation (AV)

Reproducibility: Appraiser Variation (AV) This is the "between appraisers" variation. It is the variation in the average of the measurements made by the different appraisers when measuring the same characteristic on the same part.

Residual Error

Residual Error The difference between the observed and predicted value for that result, based on an empirically determined model. It can be variation in outcomes of irtually identical test conditions.

Resolution IV

Resolution IV Fractional factorial design in which the main effects and two factor interaction effects are not confounded but the two factor effects may be confouned with each other.

Results of Lean Thinking

Results of Lean Thinking 1. 100% Greater Labor productivity 2. 90% reduction in inventory & Throughput times 3. 50% reduction in: a) Customer errors b) Safety Injuries c) Product Development time d) In-house Scrap

Design FMEA Vs. Process FMEA: Review the design/ process

Review the design/ process a. Design FMEA---------Schematic diagram and functional block diagram→ id main design componetns, determine function(s) of components, and interface b/w components. b. Process FMEA-------------Flowcharts used to find the scope and assists team members in understanding the process steps

Process Performance Measures: Rolled throughput yield (RYT)

Rolled throughput yield (RYT)= yield from a series of processes i. Individual process yields are multiplied.

Self-directed Teams

Self-directed Teams Structure= 6-15 members. May need staff support 1) Minimal direction from management---> maximum latitude in meeting deadlines. 2) After team understands charters and the normal process, it then works on finding solutions. 3) Team selects its own leader who coordinates interfacing with other teams 4) Team leader has an equal position as other team members ---> option to even rotate the responsibility of being leader.

Six Sigma Structure options---Coach

Six Sigma Structure options ---Coach Master Black Belt or Black Belt

Six Sigma Structure options--Sponsor

Six Sigma Structure options --Sponsor Process Owner or Champion

Six Sigma Structure options---Executive Direction

Six Sigma Structure options---Executive Direction 1. Six Sigma steering committee 2. Quality Council 3. Executive Steering Council

Sources of Measurement Variation:

Sources of Measurement Variation: a. Part-to Part Variability b. Measurement system variability c. Variation due to gage (repeatability) d. Variation due to operations (Reproducibility)

Statistical tests

Statistical tests 1) Uses confidence level to display shifts in customer preferences. 2) Most normal statistical tests may be used on numerical surveys, like Likert surveys

Taguchi Awards

Taguchi Awards 1. Deming Price 2. Rockwell Award 3. MITI Purple Ribbon Award 4. Indigo Award 5 ASME award

Taguchi's influence on American Quality Efforts

Taguchi's influence on American Quality Efforts 1. First to develop quality engineering techniques which allowed products and processes to be made quicker 2. Appealing to USA because it was a complete system that started with the product concept and continued to product design, and into the manufacturing operations. It is a system to optimize the design or product and processes in cost-effective manner.

Test Statistic-

Test Statistic-→calculated value made from sample information used to test the null hypothesis

General method for calculating process cycle efficiency

The general method for calculating process cycle efficiency is to divide value-added time by process lead time.

Lean Processes & Transparency

The lean philosophy emphasizes three areas: transparency, velocity, and value. Transparency is easy access to useful information. A business has achieved transparency when processes can be observed and measured. Errors should be noticed immediately and rectified.

Tools used in Project Risk Analysis:

Tools used in Project Risk Analysis: a. SWOT analysis b. Risk priority number of risk priority matrix c. FMEA d. Formula for expected profit.

Types I Error

Types I Error 1) Alpha error---> when a true hypothesis is rejected. 2) Also known as "Producer's Risk"

3 Ways that Stakeholders Impact the health of the Business

Ways that Stakeholders Impact the health of the Business: 1) Stakeholder choose to invest based on expected returns in the near term ( dividends & increased stock price) or longer term (growth) 2) When customer purchase good/service that satisfy their needs, they contribute to the process by positive word-of-mouth, new product ideas, referrals, and additional orders. Negative feedback can harm the company. 3) Supplier/ Customer sets the product/service's value by defining the final specifications----> percieved value is the function of Quality, cost, features, and Availability.

Weighted Pareto Analysis

Weighted Pareto Analysis 1) Used to help improvement items with selection of serious problems 2) Assumes that there will be segreation of a significant few from the trivial many 3) Based on number of even occurences and organized by criticality factors

e. Two primary focuses for FMEA

e. Two primary focuses for FMEA= Design & Process.

Design for Cost

i. Design for Cost—Search for alternative processes, materials, and methods via use of cost accounting & purchasing skills.

7 categories of MUDA

7 categories of MUDA 1) Overproduction 2) Inventory 3) Motion 4) Repairs/ rejects 5) Transport 6) Waiting 7) Processing

Which distribution listed below is formed by the distribution of sample variances: Exponential F Student t Chi-Square

Chi-Square is formed by the distribution of sample variances. Student t is used for hypothesis testing of means for small data sets. F distributions are the distribution created by the ratio of sampling variances from a normal distribution Exponential distributions are highly skewed right used for reliability.

What is the biggest benefit of competitive benchmarking? A) It is a quick, one-time improvement activity. B) It focuses on the weakest link of a process C) It monitors large amounts of information for problems and trends. D) It provides opportunities for improvement breakthroughs

Competitive benchmarking helps establish ambitious goals that are realistic (after all, someone else is already achieving them) and gives you are starting point for developing and implementing changes rather than starting from scratch. Monitoring large amounts of information for problems and trends is a performance index

Control mechanism & Poke/ Yoke

Control mechanism & Poke/ Yoke: 1) Designing a par so it cannot be used by mistake 2) Using tools and fixtures that will not load a mis-positioned part 3) Having a work procedure controlled by electric relay

A kanban is which of the following? Used for determining effecting combinations of operations Method for mistake proofing Signal for a particular action Used for low-cost gradual improvements

Correct!---Kanban is a signal for a particular action such as "move material" or "produce a part." Standard work is used to determine the most efficient combinations of operations. Poka-yoke is a method for mistake proofing Kaizen is used for low-cost gradual improvements.

Which of the following financial metrics provides the best measure of the long-term success of an organization's Six Sigma program? asq/learn_check.gif Cost of Poor Quality Cost Benefit Analysis Net Present Value of Project Outcomes Market Share

Cost reductions related to reductions in internal failures, external failures, appraisal cost, prevention cost and the cost of non-value added activities are a direct, and the best measure of improved performance related to Six Sigma projects. Close

Cumulative Distribution Function

Cumulative Distribution Function

DFSS Roadmaps

DFSS Roadmaps 1. Method for bringing order to product design 2. 70-80% of quality problems are due to design 3. Design (front end) is cheaper to fix than manufacturing processes (back end) Example---NIST auto supply chain lost money due to poorly designed product's lack of interoperability.

Kaizen & " Pull"

In this Kaizen system, pull means that each activity in a process should receive only the necessary materials and resources when subsequent activities are demanding the process be completed. In other words, instead of pushing resources through the process chain, it is more effective to have them be drawn along by the vacancies created by completed projects down the line. Finally, in kaizen the term perfection is defined as the ultimate goal of continuous improvement. By eliminating waste and streamlining processes, an organization can approach perfection."

Juran & Customer Needs

Juran & Customer Needs: 1) Stated Needs---What the customer says they want (eg car) 2) Real needs-- What customer really wants (eg. transpotation ) 3) Percieved Needs--customer desires (new car) 4) Cultural needs---status of product (BMW) 5) Unintended needs---customer uses the product in an unintended manner (BMW used to haul concrete blocks)

Juran---Books

Juran---Books 1. Juran on Planning for Quality 2. Juarn on Leadership for Quality 3. Juran on Quality by Design 4. Juran's Quality Control Handbook

PROCESS FMEA

PROCESS FMEA 1) Focuses on reducing failure modes by manufacturing processes or assembly operations 2) Accounts for all manufacturing process 3) Initiated before OR after feasibility stage, prior to tooling for production 4) Requires input from the flow chart of the process and the DFMEA document

Pattern of Residuals

Pattern of Residuals: 1) Should be similar to bell-shaped patterns 2) Departures from assumptions usually mean that the residuals contain structure that is not accounted for in the model. 3) Any graph suitable for displaying the distribution of data set is suitable for judging the normality of the distribution of a group of residuals. 4) three most common patterns: a) Histograms b) Normal Probability Plots c) Dot Plots

Population"= collection of data to be considered

Population"= collection of data to be considered a. Sample= subset of populations b. Samples randomly selected so that they represent the population from which they are drawn. c. Population parameters= population mean and standard deviation. d. Sample Statistic= statistics and quantitative research methodology, a data sample is a set of data collected and/or selected from a statistical population by a defined procedure.

Problems due to not Coding:

Problems due to not Coding: a) Inspectors trying to squeeze too many digits into small blocks on a check sheet form b) Reduced throughput and increased errors by clerks at keyboards reading and entering large sequences of digits for a single observation. c) Insensitivity of analytic results due to rounding of large sequence of digits.

Process mapping

Process mapping= a. 1st step in improving a process b. 1st step in risk analysis tools like FMEA c. Helps to visualize redundancy in the process, non-value added steps, and unnecessary complexities

Term: Systems of cascading production and delivery instructions from downstream to upstream activities in which nothing is produced by the upstream supplier until the downstream customer signals a need

Pull System Systems of cascading production and delivery instructions from downstream to upstream activities in which nothing is produced by the upstream supplier until the downstream customer signals a need

Quality Function Deployment Definition

Quality Function Deployment (aka House of Quality) 1) Organizes customer requirements & desires and allows them to be traced to specifications. a. Captures customer requirements in the earliest states and allow the captured information to cascade with subsequent stages to ensure no gaps in customer satisfaction. 2) Assurance that linked specifications will result in customer satisfaction is ensures by linking the voice of customer 3) Process for planning new or re-designed products/services, re-designing the preference of the VOC 4) Team must discover the needs & desires of customer

SPC Theory

SPC Theory 1. X-bar & R charts are most common type of control chart, and are calculated for each subgroup and plotted order of production on separate charts.

Simple Linear Correlation

Simple Linear Correlation a. Relationship between 2 or more data sets b. Measures the strength and direction of the relationship between variables

A study compared a ranking of the tax system friendliness of all 50 states to the number of businesses per million population per state. A regression analysis showed the correlation coefficient was 0.108. What conclusion can be made from the analysis? Tax system friendliness is negatively correlated to the number of businesses There is little if any association between tax system friendliness and the number of businesses Tax system friendliness predicts more privately held businesses A strong association between tax system friendliness and the number of businesses

There is a low r indicating a low association between variables.

Ultimate Goal of SIPOC

Ultimate Goal of SIPOC 1) Ultimate goal is to identify essential work flows and sources of variation in the work over time 2) Can be adapted to a number of essential support processes. 3) Process mapping, flow charting, and affinity diagrams can be used to identify the major blacoks or steps in a process of system.

Lean Concepts & Tools-- Value chain diagram

Value chain diagram a. Boxes labeled with each step in the process. Each process box has information about timing and inventory 6) Eliminate unnecessary steps (non-value added activities) in the value stream. 7) Move towards value streams in which production decisions are based on the pull of customer demand. 8) Perfect pull-based flow-→ customer order will trigger production of a replacement item in finished goods inventory before it is needed by the customer and be delivered in timely manner.

value stream analysis

Value stream 1) Sries of activities that create value in a product or service from the perspective of the customer rather than from the perspective of employees or management within the company. 2) During process mapping, the Six Sigma team will divide the activities into three categories: value-added activities, business-value-added activities, and non-value-added activities.

Which method of analyzing detailed flow chart is suitable for identifying physical inefficiencies? Value stream map SIPOC Deployment chart Work -flow diagram

We can identify physical inefficiencies by converting a flow chart to a work-flow diagram Deployment flow chart show who performs which step. SIPOC focuses on input, process and output. Value stream mapping is used for analyzing waste and value-added processes.

"D" phase of IDOV

Design Phase i. Concept design ii. Id risk via use of FMEA iii. Design parameters iv. Procurement of raw materials & integration

X-R chart

X-R chart

A Green Belt is creating a regression model for estimation. He thinks that one has to determine the closeness of the calculated z-scores to the actual coefficient values for the population. The error in his thinking is that: It is easy to calculate coefficient values The coefficient values should be compared against the population The calculated values should be closely aligned when compared to the population He should calculate z-scores for the coefficient values

z-scores are not used in regression models.The calculated values should be closely aligned when compared to the population The coefficient values should be compared against the population

Juan

11) Joseph Juran: a. Juran Triology= Managerial processes for quality management= quality planning, quality control, & quality improvement. b. Regular staff progress reviews + Make quality improvement part of job description + recognition of winning teams

5S organization method

5S organization method (sort, set-in-order, shine, standardize, sustain): a. Improves efficiency and management of operations. b. Environment impacts processes and the ability of staff to respond to process change. c. Process control data are filtered thru the measurement system so must maintain the a clutter free space. d. Untidy workspace leads to poor quality and waste

6 issues to be considered in the Kaizen strategy

6 issues to be considered in the Kaizen strategy 1) Management maintains and improves operating standards 2) Process improvement is the key to success 3) PCDA improvement cycles are used 4) Quality is highest priority 5) Problems are solved with hard data 6) Next process is provided with good parts/information

Auditing Method:

Auditing Method: a. Define the auditing criteria i. Process Audit-→ need standard operating procedure and other process sheets ii. Safety Audit-→ need written safety rules iii. Other types of Audits= project closures, product quality, system knowledge, emergency procedures, & cleanliness.

What is the Sigma level of average American companies?

Average American company operates at 4 sigmas which is equivalent to 0.6% defective or 6,210 defects per million.

The design failure mode and effects analysis should be completed before which of the following? A) Severity changes B) Performance testing C) Production drawings are released for tooling D) Prototype testing

C= CORRECT ! Production drawings are released for tooling after the design failure mode and effects analysis is complete. Prototype testing will validate FMEA and may identify additional failure modes. Severity changes can be affected only through design changes. The product of severity, occurrence, and detection is the risk priority number. Failure modes should be identified before performance is tested for the customer.

Which the following best represents the benefits of capturing the lessons learned before project closure? Quantify project savings Celebrate the project closure party Review the team's process and identify what worked best. Provide input for performance appraisal

C= Providing the team with the opportunity to review and document best practices is a benefit of a lessons learned review.

Co-variates

Co-variates 1) Things which change during an experiment which had not been planned to change, such as temperature or humidity. 2) Randomize the test order to alleviate this problem. 3) Record the value of the c0variate for possible use in regression analysis.

Confidence Interval for Mean

Confidence Interval for Mean ***Confidence intervals for the mean are symmetrical about the average. 1) Continuous Data/ Large Samples----> use Normal Distribution 2) Continuous Data/ Small Samples ( less than 30) -------> use t- test

Confidence Interval for Proportion

Confidence Interval for Proportion For large sample sizes, with n(p) and n(1-p) greater than or equal to 4 or 5, the normal distribution can be used to claculate a confience intercal for proportion.

Constructing a Cause-and-Effect Matrix

Constructing a Cause-and-Effect Matrix 1) List out the Key output variables (KPOV) for the process and then assign numbers to note the order of priority 2) List the KPIV that may cause variability or non-conformance in the process. 3) Assign---BY TEAM CONSENSUS---numbers to organize the effect that each KPIV has on each KPOV. 4) For Each KPIV Column------> Multiply a) (KPOV PROCESS Priority Number/ Top Row) (KPIV effect value in each cell) b) Sum across the row to compute TOTAL for the KPIV and reach a consensus on which KPIV should get a priority attention.

Which of the following financial metrics provides the best measure of the long-term success of an organization's Six Sigma program? Cost of Poor Quality Market Share Cost Benefit Analysis Net Present Value of Project Outcomes

Cost reductions related to reductions in internal failures, external failures, appraisal cost, prevention cost and the cost of non-value added activities are a direct, and the best measure of improved performance related to Six Sigma projects.

Creating a Pareto chart

Creating a Pareto chart 1st step --Identifying the correct ( non-overlapping and fundable.) categories. a) Data should be collected over the same interval for each category. c) Left Axis = The count/cost metric = identities of the categories will be along the bottom axis and categories placed in descending order from left to right. d) Right vertical axis = An indication of the percent demarcations, or the percent of total cost/count represented by each variable, *****Line ascending from left to right= The cumulative percentages from each bar ( cumulative aspect of the Pareto chart makes percentages and rates unacceptable data.) *** In order for the chart to work, the data must be additive"

Design and Analysis of One-factor Experiments

Design and Analysis of One-factor Experiments a. Completely randomized when no tests are omitted b. ANOVA analysis of the results c. Significant values exceed the F-statistic derived from samples i. F-test analysis is the basis for model evaluation of both single factor and multi-factor experiments.

2 Disadvantage of PERT

Disadvantage of PERT 1) Complexity of PERT increases implementation problems 2) More data is required as network inputs

Examples of processes suitable for monitoring with a c-chart include:

Examples of processes suitable for monitoring with a c-chart include: Monitoring the number of voids per inspection unit in injection molding or casting processes Monitoring the number of discrete components that must be re-soldered per printed circuit board Monitoring the number of product returns per day

Examples of standards that go beyond procedures and work instructions include

Examples of standards that go beyond procedures and work instructions include: 1) Yellow Lines on Floor 2) Color Coding 3) Production Control Boards 4) Inventory Indicators 5) Cross-training matrices 6) Trouble lights

Experimental Assumptions

Experimental Assumptions 1) Are the measurements systems capable for all responses? 2) Is the process stable? 3) Are the residuals ( the difference between the model predictions and the actual observations well behaved?)

Experimental Error

Experimental Error Variation in response or outcome of virtually identical test conditions ---aka Residual Error

Gant Charts (Bar Charts) --4 Advantages

Gant Charts (Bar Charts) ---4 Advantages: 1) Charts are easy to understand and change 2) Each bar represents a single activity ( each activity shown as a horizontal bar with ends positioned at the start and end dates for that activity) 3) Only minimal data needed to construct 4) Program task progress verses date is shown

Hyper Graec0- Latin Designs

Hyper Graec0- Latin Designs 1) Hyper-Graeco-Latin squares, as described earlier, are efficient designs to study the effect of one treatment factor in the presence of 4 nuisance factors. They are restricted, however, to the case in which all the factors have the same number of levels. 2) Seldom used because experimental units cannot be balanced into appropriate number of groupings.

Interpreting a hypothesis test

Interpreting a hypothesis test a) If the calculated statistic > critical value of the test statistic , then reject Ho----> (experiment weakly supports conclusion because the means may /may not be equal. ) i) If the p value is small, then the chances of obtaining results similar to those gathered during the experiment are so small that the null hypothesis should be rejected. b) If the calculated statistic < the critical value of the test statistic, then accept the HO:----> means are not equal and experiment provides strong support for the conclusion.

Leader Role

Leader Role 1. Ensures implementation of the team's mission & charter 2. Teams can have both leaders & facilitators in manufacturing industry 3) Facilitation & leadership diminish as the team develops capability

Linking Projects to Organizational Goals

Linking Projects to Organizational Goals a. Projection selection Team (set the criteria for project selection and team assignments): 1. Master Black Belts 2. Black Belts 3. Champions 4. Executive supporters b. Project selection criteria= key element to furthering organizational goals.

Lost Creativity (source of Waste)

Lost Creativity a. Most unfortunate waste b. Lean supports the involvement of employees in teams to reward their input, supports mistakes as learning experiences, and empower employees to make changes. c. Nontangible benefits of Lean thinking= reduced staff turnover and improved employee morale. 9) Perfection a. Continuous improvement to reach perfection.

Main Objective of the Measure State

Main Objective of the Measure State: 1) Data gathering to complete project. 2) Team defines each relevant process in great detail. 3) Group of metrics for the process is developed 4) Measurement analysis is conducted to identify and quantify any common errors in the metric that is selected. 5) Estimation of process baselines to determine a reasonable starting point.

Ordinal Data Scale

Ordinal Data Scale i. Interval between adjacent scale values are indeterminate (eg. Categorization of defect by criticality, functional failures, performance degradation, cosmetic defects) ii. Application: a) "Greater that" & "Lesser than " operations, b) Median, c) Interquartile range, d) Sign test

Pareto Diagram

Pareto Diagram 1) Creator = Vilfredo Pareto (Parisian economist who wrote about unequal distribution of wealth and contibuted to Fascism in Italy)----> Pareto only applied his principles to unequal income distribution 2) Juran applied the Pareto princeple as "universal"

5 States of Team Development --Performing

Performing i. Team is effective, skills complement, synergy is created. ii. Team interdependence iii. Team work to solve problems and lots of work accomplished.

Process Inputs, Outputs, & Feedback:

Process Inputs, Outputs, & Feedback: a. Feedback loops aid in process control b. Flowcharts emphasize the inputs & outputs for each process. c. Value lies in the quality of the inputs/outputs and the efficiency of their management d. Function of process management = collection/ analysis of data about inputs & outputs, using information as feedback to the process for adjustment & improvements

Project Charter Role in Development of the Improvement Team

Project Charter in Development of the Improvement Team : 1) Acceptance & Development of project charter -----> critical element to forming an improvement team b/c it defines the following: a) Mission b) Scope of Operation c) Objectives d) Time Frame e) Consequences **** Purpose Statement of Charter explains why the team is being formed and identifies the objectives for Team

Project Closure

Project Closure 1) Final step based on proving that project goals and objectives were met , ensuring required documentation is completed and stored, and conducting a closure meeting with project sponsors to ensure agreement that the project is completed.

Quality Function Deployment or QFD :

Quality Function Deployment or QFD : Definition of QFD Matrix: Method of using market segmentation to improve on design quality. An iterative process for continually refining customer requirement to ever-increasing levels of detail & specificity Key Takeaway: Companies can use QFD to transform information about their customers such as segmentation into product characteristics. For example, soap producers can use information about the age of their intended clients to incorporate into popular scents.

Types of Variation in Measurements: Repeatability

Repeatability: 1. Variation in measurement that occurs when the SAME measuring system (appraiser, equipment, materials) are used and is reflected in R values 2. Repeatability is referred to as equipment variation and R averages indicated difference in appraisers. 3. If two R averages are compared, the smaller R value means that there were less errors and the appraiser is more accurate (inverse relationship between R-value and accuracy)

Risk Management Phases & Definitions ---Risk Planning

Risk Management Phases & Definitions ---Risk Definition-----> Development and Documentation of a systematic methodology for identifying, analyzing, and tracking risk issues. Includes development of contigency plans, assessments, and resource allocation plan. Inputs: " FT RRRN"----> Focust Points, Techniques, Responsibilities, Resources, Requirements, Needs

Six Sigma

Six Sigma 1) Program for improving business processes such that six standard deviations will fit between the mean and both the upper and lower acceptable limits. 2) Standard deviation suggests the amount of variation between the members of the data set or described in sigma units 3) Imagine a process that can be represented graphically as a bell curve. The centerline of the process is a time of 25 minutes, meaning that this is the average time required to complete the process. If the standard deviation is 30 seconds, then six standard deviations, or 6σ, would be either 22 minutes or 28 minutes.

Term: Work cell visual control depicting all work activities. Used to cross-train team members

Skills Matrix: Work cell visual control depicting all work activities. Used to cross-train team members

Term: Effectively reducing lot size until the optimum of one piece flow is realized.

Small Lot Principles Effectively reducing lot size until the optimum of one piece flow is realized.

The different measures for dispersion include which of the following? Mode, Range, Standard deviation. Range. Mean, Standard deviation. Range, Variance, Standard deviation. Variance, Mode, Standard deviation.

The range is the maximum to minimum measure of dispersion. Variance is the square of standard deviation and a measure of dispersion. Standard deviation is the most important measure for dispersion. Mean is a measure of central tendency and not a measure of dispersion.

In a process failure mode and effects analysis, when is the resulting RPN calculated? A) After corrective actions have been identified and completed B) Only when severity changes C) After failures are monitored D) As soon as responsibility is assigned

The resulting RPN is calculated after corrective actions have been identified and completed. Only a design and/or process revision can change severity.Responsibility is only part of the corrective action.

Tools of Lean

The tools of lean (e.g. 5S, velocity, spaghetti diagrams, level loading, and process cycle efficiency), are used in every phase of DMAIC.

Value Chain & Porter

Value Chain & Porter 1. Porter states that value chain is at higher operating level 2. Generic value chain involves these components: a) Human Resources b) Technology c) Marketing & Sales d) Inbound and Outbound Logistics e) Operations.

Role of Histograms in Statistical process control errors:

f. Histogram—visual picture of the variation and center of the process.

Design of Experimentation Levels?

2. Levels a. Settings or possible values of a factor in experimental design throughout the progress of experiment b. Levels can be quantified (eg. 3 different temperatures) or be qualitative (high, medium, low)

3 Function of the QFD matrix

3 Function of the QFD matrix 1. Database for product development 2. Basis for planning product/process improvements 3. Opportunities for new or revised product/process introductions.

35. In nominal group technique, how many pieces of paper should each participant receive if there are 40 options to be considered? a. 2 b. 4 c. 6 d. 8

35. D: b. In nominal group technique, each participant should receive eight pieces of paper if there are 40 options to be considered. Each participant will then write one of the options down on each piece of paper, along with its rank (first through eighth). It is typical for each participant to receive eight pieces of paper when there are more than 35 options. When there are from 20 to 35 options, the typical number of papers for each person is six. When there are fewer than 20 options to be considered, it is typical for each member of the group to receive four pieces of paper. Once all of the group members turn in their rankings, the various options are compared, and the most popular are given further consideration."

Cost/Benefit Analysis ---4 Categories of problems on the shop floor:

4 Categories of problems on the shop floor: i. Internal failure costs ii. External failure costs iii. Prevention costs iv. Appraisal costs

5 ways that consumer customer market differs from business market

5 ways that consumer customer market differs from business market: 1) Consumer market has large numbers of customers 2) Most consumer purchases are small in actual dollar amounts 3) Usually transactions are simple purchases 4) Poor consumer knowledge about product 5) Supplier does not share propriety information with consumer

Accuracy

Accuracy 1) Unbiased true value and is reported as the difference between the average of a number of measurements and the true value.

Bottom up Communication

Bottom-up Communication i. Subordinate seeking management feedback or team member communication with executive sponsor. ii. To prevent distortion of the message, top management should encourage open door policy, employee surveys, suggestion boxes,a nd standup meetings. 1. If poorly executed-→ suggestion boxes become complaint boxes and open door policy used for work disruption and pointing fingers. h. Horizontal Communication B

Cause- and- Effect Diagrams

Cause- and- Effect Diagrams: 1) Key process variables are measured with metrics: a) Percent Defective b) Operation Costs c) Elapsed time d) Backlog quantity e) Documentation Errors 2) Fishbone session is divided into 3 parts ------> a) Brainstorming ----Problem identification & brainstorm categories in the diagram b) Prioritizing problem cause via polling c) Development of action plan---narrows to 3 probable causes

A Green Belt noted that parts from a supplier had inconsistent metal thickness from one location on the part to another. He decides to conduct a multi-vari study. The type of multi-vari study that he should run would be: Temporal Positional Cyclical DOE

Correct! A positional study is to assess variance within a single part.

In Analysis of Variance, the F-statistic is a ratio that compares: The variation within treatments to variation caused between the treatments. The variation within treatments to total sum of squares. The variation between treatments to variation within treatments. The variation caused by treatments to total sum of squares

Correct! The F statistic is the ratio of variation caused by the treatment to variation caused by experimental error.

Crosby---first job?

Crosby started as a technician B-47 fire control systems

What made Crosby different from other Quality philosophers?

Crosby was business man who applied quality to running businesses while others were academcians

Cycle Time Reduction

Cycle Time Reduction 1) Amount of time required to complete one transaction of process 2) Reasons for reducing cycle time: a) To please customer b) To reduce internal or external waste c) To increase capacity d) To simplify operation e) To reduce product damage c) to remain competitive 3) Roots of cycle time reduction from industrial engineering principles and Fredrick Taylor.

Interrelationship of DMADV, DMAIC, & IDOV?

DMADV (define, measure, analyze, design, verify)and IDOV (identify, design, optimize, verify) relate to DMAIC and help close the loop on improving the end product/process during the design for Six Sigma phase (DFSS) 5) Improvements can be of two types: improving the existing process or designing a new process altogether. When we have an existing process and we want to improve the process we deploy the DMAIC methodology. While designing a new process or completely revamping the existing process the Design for Six Sigma or IDOV methodology is deployed.

Descriptive (enumerative) study

Descriptive (enumerative) study i. Collecting, organizing, summarizing, and presenting population data. ii. Shows the properties of set of data such as mean, median, mode, dispersion, shape, ect. iii. Graphical tools used (histograms, pie charts, box plots)

Design for Agility

Design for Agility— Products that need to be customized and require fast delivery (eg. Hottubs companies)

Elements of Project Charter:

Elements of Project Charter: i. Problem statement = summary of what needs to be improved ii. Purpose= goals & objectives iii. Benefits iv. Scope = limitations due to time, budget, & resources. v. Results= baseline measures & improvement expectations

Horizontal Communication

Horizontal Communication i. Best for "Flat" organizations. ii. May shortcut te process if used for processes that require management's approval and authorization. iii. Middle and higher management may feel surpassed due to horizontal communication.

Kaizen event vs. Kaizen:

Kaizen event vs. Kaizen: i. Kaizen event= radical change to an existing situation. ii. Kaizen= gradual improvement in the existing situation

Lack of Variability

Lack of Variability A. X-Bar Chart Causes 1) Incorrect calculation of control limits 2) Improvement in process since limits were calculated. 3) Employee cannot be making checks B R- Chart Causes 1) Collecting in each sample a number of measurement from widely differing lots 2) Improvement in process since limits were calculated. C. Corrective Action 1) Check control limits 2) Validate rational sample subgroupings 3) Verify checking procedure, gages, ect 4) Verify proper employee measurement 5) Congratulate someone for improvement

Types of Measurement Scales:

Measurement Scales: 1. Ratio Scale 2. Interval Data Scale 3. Ordinal Data Scale 4. Nominal Data Scale

New Product Term: ---New-To-World Product

New Product Term: ---New-To-World Product Inventions and discoveries like printers

One-Tail Test

One-Tail Test 1) When testing the null hypothesis for whether a sample value is smaller or larger than a population value, then the entire alpha risk is placed on one end of the distribution

Organizational Drivers & Metrics

Organizational Drivers & Metrics 1) Key Drivers a. Derived from business needs, strategy, and provide information about key processes, outputs, & results. b. Selection & use of performance indicators is the main consideration for process improvement . c. Key drivers= customer, product, service, operational, market, supplier, competitive, workforce, cost, financial, governance, and compliance performance.

Paired Comparison Hypothesis Test--- t-test

Paired Comparison Hypothesis Test--- t-test Used to test the difference between 2 sample means when the standard deviations of both samples are unknown

Project Scope

Project Scope 1) Project boundaries that outline the team's activities, 2) Problems with market research/ prototype development/ Financial invenstments outside of scope

Ratio of one black belt per 100 employees can provide ___% cost reduction per year

Ratio of one black belt per 100 employees can provide ___% cost reduction per year 6%

Risk Analysis and Management

Risk Analysis and Management 1) Risk Analysis conducted to inform stakeholder about the project risks and contingencies that develop in mitigating the risks. 2) Risk= Probability of an event or cost of not reaching the goal 3) Must understand the potential consequence of occurrences or non-occurences of project failure can be due to cost, performance, or schedule risks

SIPOC

SIPOC 1) High-level process map that encompasses suppliers, inputs, processes, outputs, and customers 2) Best to limit to 4-7 process steps within a given process block. 3) Used to produce process map for a value streams or product or at the organizational level.

Coifficeint of Determination & Scatter Diagrams

Scatter Diagrams: Coifficeint of Determination.= How well the regression line fits the data.-→ how much of variability in y-values can be explained by the fact that they are related to X-value.

A plot of one variable versus another is a: Scatter Diagram Stem-and-Leaf Plot Control Chart Box Plot

Scatter diagrams plot numerical data of two variables to show the relationship between them. Control charts are time related with probability limits and will not show a relationship between two variables.

Sequential Sampling

Sequential Sampling 1) Similar to multiple sampling plan except Sequential sampling can continue indefinitely 2) Sampling usually ended after the number inspected has exceeded 3 times the sample size 3) Used for costly or destructive testing with sample sizes of one and are based on the probability ratio test

Contribution of Stewart?

Shewart-→ Quality Control Charts a. Graphs used to monitor processes to identify if the process acting in predictable manner verse (special causes) b. Used to compare what processes are doing against the statistical probabilities. c. Special Cause—process acting in predictable manner

Simplex

Simplex Geometric figure that has a number of vertexes (corners) equal to one more than the number of dimensions in the factor space.

Specification vs. Process capability

Specification-→ derived from customer or engineering point of view. Capable Process-→ process variation is significantly lower that the width of the specification (upper limit---lower limit)

Standard Work

Standard Work a. Definition= tool that defines the interaction between man & machine in producing a part. Has 3 components (standard time + standard inventory + standard sequence) b. Training of operators to reduce process variations.

Team Stage---Performing

Team Stage---Performing 1) Member showing maturity, focus on the process, smooth operation, and goal are achieved 2) Group has developed its relationships, structure, and purpose. 3) Feeling from storming may resurface.

Standardized national test scores for the schools in the Northwest district show mixed performance for students coming from various schools in the district. As a response to this, many of the schools in Northwest started a forum where they discuss the strengths and weaknesses of their educational methodologies, and share best practices. Which of the following describes the approach taken by these schools? A) Standardization B) Focus groups C) Brainstorming D) Collaborative benchmarking

This is a good description of collaborative benchmarking.

Types II Error

Types II Error 1) Beta error----> when a false hypothesis is not rejected. 2) "Consumer's Risk" -----> eg. adverse process change 3) Small alpha risk increases the Beta risk

Types of Audits

Types of Audits (internal, external, first/second/third party) 6) Focus on products, systems, suppliers, or regulatory compliance.

Types of Rewards

Types of Rewards 1) Expensive material item s--vacation trips, bonus, patent award, cash/ gift > $1000 2) Intangible Awards--->prestige, satisfaction, pleasure, friendship, learning experience, notoriety, notoriety. 3) Incidental value rewards---> trophy, plaque, certificate, special parking space, name on list, meal with boss, picture on bulletin board, cash less than $1,000

Value of reducing movement and physical space in processes

Value of reducing movement and physical space in processes---> reduces overhead expenses 1) Analysis stage = reveals time lost simply moving from one work site to another. 2) Ways to reduce Cycle Time: a) Consolidate area where task performed ( Best)--> minimizes waiting & unnecessary energy expenditure. for stakeholders in the process b) Grouping employees in multi-function rather than single-function departments reduces wasteful movement An employee group could include representatives from all of the departments required for the process. c) Cross-training allows quick replacement of employees without workflow interruptions.

Variable vs. Attribute Data

Variable vs. Attribute Data 1) Characteristics : a) Variable ---measurable, continuous, "counting data" b) Attribute --Discrete (Good/bad); Countable 2) Types of Data a) Variable --length, volume, time b) Attribute-- No. of defects, scrap items, defectives.

Variables are selected based on these guidelines:

Variables are selected based on these guidelines: a. Key process inputs (KPIV), analyzed to determine the effect of a process. b. Key process output (KPOV) determine process capability and process monitoring using control charting c. DOE and ANOVA also used to find variables.

Variables that are critical for selection for control charts are based on:

Variables that are critical for selection for control charts are based on: a. Importance of customer perception b. Objectivity (counted or measured) c. Clear indicators to suggest whether quality achieved.

What does the SQUARE symbol above on a flowchart mean? Pre-defined process A yes/no decision point A start or end point in the process A connection of 2 process steps

A rectangle with a vertical line at each end of the symbol means that the process was pre-determined outside of this flowchart.

A green belt has been asked to come up with a breakthrough improvement to solve a process constraint. Which of the following process improvement tools or methods should the green belt consider? Kaizen FMEA Kaizen blitz Interrelationship diagraph

Answer= A kaizen blitz is used to obtain breakthrough improvements. Kaizen is a Japanese term for change for improvement or improving processes though small, incremental steps. The purpose of FMEA is to understand the opportunities for failure and the impact of risks in a product or process design. Interrelationship digraphs are used to identify cause-and-effect relationships.

Armand Feigenbaum & Total Quality control

Armand Feigenbaum & Total Quality control 1. All areas of the company must be involved in the quality effort. 2. Quality effort has generally only affected the shop floor people but must extend to all company sections. 3. Products must be made quicker & faster but also sold faster. 4. Quality professional has opportunity to become more than functional specialist. ----opportunity is there to become a true businessman by providing valuable information and direction.

Define Benchmarking

Benchmarking Process of comparing the current project methods or processes with the best practices and using this information to drive improvemnet of all company performance. Standard for comparison may be a competitor within the industry (usually in an unrelated business segments)

Degrees of Freedom

Degrees of Freedom The number of measurements that are independently available for estimating a population parameter

One-Way ANOVA

One-Way ANOVA In one-way ANOVA, the total variation has 2 parts: the variation among treatment means and the variation within treatments.

Operation Level Metrics

Operation Level Metrics 1. Business effectiveness measures a) Relate to the cost and time required to produce products. b) Provide key linkages between detailed process measures and summary business results c) Helps identify key relationships & root causes

"O" phase of IDOV

Optimize phase i. Error-proofing ii. Statistical tolerance iii. Optimize sigma & cost

Define "Process"

Process= Transform inputs -→ outputs ( sequence of steps that uses inputs & procedures and produces a product/service as an output). a. Process must have its beginning and endpoint defined. b. Can have subprocesses c. Plunging step (subprocess) d. Cross-functional processes incur sub-process boundaries defined by the organizational structures, geography, and ect.

Reducing preparation time

Reducing preparation time 1) Preparation time =non-value-added activity or tasks that collect & store materials and resources before a process can begin. a) Examples= Loading CPU programs or collecting resources from warehouse. 2) Strategies to reduce preparation time. a) Keeping all supplies/equipment as close as possible to the workstation to reduce employees's transit time. b) Group employees from different departments in work cells based on the part of the process employee is trained and responsible for ( operation completed w/o moving work-in-progress) . c) Leave equipment on and ready to go even when not in use.

Setting goals with either Short-term or Long-term Results.

Setting goals with either Short-term or Long-term Results: 1) Emphasis on Quarterly stock dividends have led American managers to have short-term outlook . 2) Japanese and Europeans prefer to have smaller, short term profit to ensure the company's long-term growth 3) Reduced Product cycle time for both new 4) Profit Margin needed to operate a business should be optimized for all stakeholder requirements---> Maximum profits are not taken due to internal stakeholder interests------->Must turn down the option for reinvestment, new equipment purchases, wage/salary increases to maintain an optimal level of dividends or investments for stockholders 5)

Shewart Professional Background

Shewart Professional Background 1. Bell South Laboratories entire life 2. Introduced control charts and differentiated special and common cause variation in project to reduce frequency of failures and repairs 3. Control charts used in WWII to improve production in the form of American War Standards Z1.1--Z1.3

Shewart---Awards

Shewart---Awards 1. Holley Medal, ASME 2. Honorary Fellowship of Royla Statistical Society 3. FIRST ASQ Honrary member 4. Rutegars University Professor of Statistical Quality Control

Shewart--Books

Shewart--Books 1. Bell System Techical Journal 2. Economic Control of Quality of Manufactured Product 3. Statistical Method from Viewpoint of Quality Control

Team Member Role

Team Member Roles 1) Training team members 2) Attend team meeting & complete assignments between meetings 3) Actively participating in meeting and encouraging others to participate as well 4) Applying steps in improvement process 5) Benefitting from experience/expertise/ perspectives

The final external customer for an automobile manufacturer is: A) Marketing B) Corporate management C) Buyer of the vehicle D) Dealers

The buyer of the vehicle is the prime target group.

If a particular situation in a company requires the removal of organizational roadblocks and making sure that the team has proper resources, which Six Sigma role is most likely to act? A) Champion B) Process owner C) Change agent D) Black Belt

The main responsibilities of a Champion are to remove roadblocks and ensure the team has needs resources. Process owners can make changes only within their processes. Change agent is another designation for the Black Belt does not include authority to remove roadblocks.

Three Type of Prioritization Matrices

Three Type of Prioritization Matrices 1) Full Analytical Criteria Method -----> Most complex of 3 methods; Requires a set of matrices to form a final matrix. ---> Decision based on numerical values can generally be obtained as a result 2) Consensus Criteria methods--- This process is made to use by combining a tree and matrix diagramming methods to carry out a pair-wise assessment of items resulting to narrowing down to a most preferred or most effectual priority wise sequence of events. 3) Combination ID/ Matrix Method.---> Used to prioritize options but does not numerically value of cause-and-effect relations. In L-shaped matrix, which compares all of the options to each other, the strength values can be developed.

Rational Subgrouping

e. Rational Subgrouping i. Process of putting measurements into meaningful groups to better understand the important sources of variation. ii. Used in process sampling situations when data is collected in real time during process operations. iii. It involves grouping measurements produced under similar conditions, sometimes called short-term variation. This type of grouping assists in understanding the sources of variation between subgroups, sometimes called long-term variation.

For a control chart to be effective in detecting special-cause variation, subgroups should be selected in a way that makes each subgroup as homogeneous as possible and which gives the maximum opportunity for variation to occur between subgroups. This is the key idea of: Process capability Rational subgroups Common-cause variation Kaizen

Correct! Walter A. Shewhart called it rational subgrouping because it is rational to look at the data in such ways. Common-cause variation is variation common to the whole process and is always present. Kaizen is continuing improvement and it is unrelated to the subgrouping on control charts.

An agriculture company is presented with two different methods for testing heavy metals in fresh produce. In order for this company to determine which method is more precise, it runs several tests using both methods and receives comparable results. The company decides to run an F-test to determine which method is more precise. Which of the following can be used as the alternate hypothesis for such test? (Note: σ = standard deviation) HA: σ2 (Method 2) = σ2 (Method 1) HA: σ2 (Method 1) ≠ σ2 (Method 2) HA: σ2 (Method 2) > σ2 (Method 1) H0: σ2 (Method 1) > σ2 (Method 2)

Correct! HA: σ22(Method 2) > σ2 (Method 1) is a correct alternate hypothesis for testing which method is more precise than the other HA: σ2 (Method 2) = σ2 (Method 1) statement describes what the null hypothesis should be for a two tailed test

Drawbacks of Surveys:

2) Drawbacks of Surveys: a. Low return rate b. Doesn't give good overall picture.

3 types of Experimental Objectives

3 types of Experimental Objectives: *** Choosing an experimental design depends on objectives of experiment and number of factors to be investigated: 1) Comparative objective 2) Screening Objective 3) Response Surface Objective

Purpose of Design of Experiments Data matrix & ANOVA

3. Data matrix—table organizing the data into columns for analysis. (Columns= factors and rows= different experiments 4. ANOVA i. Provides accurate results even if the data matrix is small

4 Customer Service Measurements Techniques

4 Customer Service Measurements Techniques 1) Customer visits 2) Customer service measuring quality, service, performance 3) Complaint Analysis: rejects, Pareto analysis 4) Customer service engineer feedback.

4 Metrics for measuring Cycle time

4 Metrics for measuring Cycle time" 1. Existing Cycle times 2. Internal Benchmarkts 3. Expternal Benchmarks 4. Reduction in Cycle times

4 Types of Attribute Control Charts

4 Types of Attribute Control Charts: 1. P-chart 2. NP chart 3. C-chart 4. U-chart

Contribution of Fredrick Taylor & Henry Ford)

4) Separation of work was developed to speed up the process of development and production. (Fredrick Taylor & Henry Ford) a. Quality Control/ Quality Assurance Specialist role was created as result of the new methods of doing business. b. Separation of tasks led people to assume that customer satisfaction was only the duty of QC departments rather than the role of people involved in production or provision of services.

5S--Standardize

5S--Standardize 1) Make 5S activities routine so abnormalities show up 2) Set standards, determine necessary tools, and identify abnormalities 3) Visual control and Visual management (use of color coding, markings, and labels) 4) Determine short-term countermeasures and long-term remedies. 5) Provide equipment markings, maps, and charts

6 Situations where Poke/Yoke is effective

6 Situations where Poke/Yoke is effective 1) SPC difficult to apply 2) Specal cause failure ocurring frequently 3) Mispositioning of compoents 4) Attributes not measurement are important 5) Turnover and training costs are high 6) Vigilance needed in manual operations

A visual workplace provides which of the following? A map of the value stream A method to ensure that mistakes can't occur. Identification of the status of material that you can physically see. Continuous, incremental improvements.

A visual workplace provides information on the status of material.

The success of a design for Six Sigma program is estimated by which of the following? A) Scorecards B) Process sigma C) Response surface methods (RSM) D) Prototype performance

A= CORRECT ! DFSS estimates product sigma using, as much as possible, pencil and paper. The scorecard is the tool for doing this. Process sigma is a metric for scorecards. RSM is a tool for optimization in the improve phase of DMAIC. Prototype performance contributes to the scorecard.

Advanced Quality Planning

Advanced Quality Planning 1) First step in the Plan-Do- Study -Act (PDSA) cycle where the plan is created before any work starts 2) Method of conducting solid planning to ensure efficient use of resources, to avoid surprises, and to create a blueprint or pattern for task completion. 3) Examines parameters (resources, staffing, tools, training, safety) for the project before work starts. 4) Can use fishbone diagram to ensure that all the elements (causes) give desired output.

Advantages of Median Control Charts

Advantages of Median Control Charts 1) Easy to use and requires fewer calculations 2) Shows process variation 3) Shows both median and spread

Advantages of Quality Cost System

Advantages of Quality Cost System 1) Manageable entity and single quality overview 2) Aligns quality with company goals 3) Provides problem prioritization system and a means of measuring change 4) Provides a ways to distribute controllable quality costs of maximum profit 5) Improves the effective use of resources 6) Provides emphasis for doing the job right every time 7) Establishes new product processes

Analysis Stage of DMAIC

Analysis (study process and data for clues to what is going on) 1. Central Limit Theorim 2. Shop audit 3. Experiments 4. Geometric dimensioning and tolerancing (GD & T

Analyze Stage of DMADV

Analyze Stage of DMADV i. Statistical and investigative approaches to finding design priorities with significance and confidence.

Application of DOE

Application of DOE 1) Choosing between alternatives 2) Selecting the key factors affecting the response 3) Response Surface Modeling to: a) Hit a target b) Reduce Variability c) Maximize/ Minimize a response d) Make a process robust e) Seek Multiple Goals

A survey asks for the geographical area code that is used for demographic analysis. This is an example of what type of measurement scale? Ratio Interval Ordinal Nominal

Area codes have numbers that are just labels and are nominal type of data.

Armand Feigenbaum Famous Quailty phrases by him

Armand Feigenbaum --Famous Quailty phrases by him 1. Quality does not travel under an exclusive foreign passport 2. Quailty & costs are partners, not adversaries 3. Failure driven companies ..."If it breaks, we will service it" verses the quality excellence approach..."No defect, no problems, we are moving toward perfect work processes" 4. Quality is everybody's job, but because it is everybody's jobs, it can become nobody's job without proper leadership and organization.

Assumptions that ensure the validity of the tests.

Assumptions that ensure the validity of the tests. a. Measurement system capability ---confirm the measurement aver b. Process Stability--→ establishment of bases lines to detect project drifts & shifts + Setting standard set points for the start and end point of the process. c. Residuals -→estimates of experimental error; should be normally & independently distributed with a mean of zero and constant variance.

At the end of the Define phase—4 essentials must be completed:

At the end of the Define phase—4 essentials must be completed: a. Project charter with problem and goal requirements b. List of measureable customer requirements c. SIPOC map of the process being analyzed d. More detailed map highlighting some suspect area for measurement (project story board + tollgate review)

Average and Range Method

Average and Range Method Total measurement system variability that allows for separation into repeatability, reproducibility and part variation

Which of the following tools is useful in providing link between process inputs and outputs? FMEA chart SIPOC Pareto chart Hypothesis testing

B= CORRECT =SIPOCs represent relationships among inputs, outputs, suppliers, and customers FMEA is for risk assessment and prevention Pareto charts useful for identifying the vital few from the trivial many

Balanced Designs

Balanced Designs a. An experiment where each level of each factor is repeated the same number of times for the set of runs or combinations of levels that make up the experiment.

Benefit of a Properly Performed FMEA

Benefit of a Properly Performed FMEA

Binomial Probability Distribution:

Binomial Probability Distribution: 1) Used to model discrete data (eg. Number of defects, wrong medicines, ect) 2) Must meet the following condition to apply Binomial distribution: a. Applicable only when N > 50 (not good for smaller samples) b. The ratio of sample (n) < 0.1(N)---sample size should be less then 10% of the population 3) Normal Approximation of the Binomial: a. For large values of n, the distributionso fthe count X and the sample proportion p are approximately normal (Central Limit Theorum) b. Can only be used when np > 10 and np (1-p) > 10

Which distribution describes the probability of r occurrences in n trials of an event? Weibull Binomial Exponential Normal

Binomial is the distribution where the probability of r occurrences in n trials of an event. The Weibull distribution displays a wide variety of characteristics. In an exponential distribution it is likely that more observations will occur below the average than above. In a normal distribution the concentration of observations are centered about the average and it is equally likely that the observations will occur above or below the average

Books by Crosby

Books by Crosby 1. Quality is Free 2. Art of getting out your own Sweet Way 3. Quality without tear--Hassle-free management 4. Running things: Art of Making things happpen 5. Quality & Me: lessons from an evolving life.

Business Systems

Business Systems a. Ensures that certain process inputs are int eh right place. b. Used to collect / analyze data from processes -→ process output improvements

CTQ Tree

CTQ Tree 1) Translates the initial customer requirements to numerical or quantified requirements for the product/service---- 2) begins with a general requirement to the specific and will require 2 or 3 levels to transgress from need---> Drivers ---> CTQ ***Exact metrics are not determined at this stage.

Classification of experimental designs: Completely randomized

Classification of experimental designs: Completely randomized ---best when only one factor analyzed

Confounded

Confounded When the effects of 2 factors are not separable,.

Term: Material moves one piece at at-a-time, at rate determined by the customer needs, in sooth and uninterrupted sequence and without work-in-process

Continuous Flow Manufacturing

The average weight and the variance of 100 packaged products is 41 and 4 kg. The distribution of weights is unknown. What is the Hypothesis statistic for testing H0 : µ = 40 vs. H1 : µ ≠ 40 ? A) It is not possible because the significant level is not known. B) Z=5 C) Z=2.5 D) Since the distribution is unknown, we can't calculate the statistic.

Correct! Z= 5 Z= (41- 40) ____________________ [ (SQUARE ROOT 4)/ (SQUARE ROOT 100) Z= 1 ______= (10/ 2) = 5 0.2

Critical Path Method & Crashing an Activity

Critical Path Method & Crashing an Activity: 1) Each activity has a normal cost and time for completion. 2) When an activity is "crashed", the time is reduced and costs increased. **** Crash = to apply more resources to complete an activity in shorter time. 3) Incremental Cost-----> (Incremental Cost) / (Time saved to crash each activity on the critical path) The activity with lowest Incremental cost per time---> Must be crashed first to complete project in shorter period

Data Coding by Truncation of Repetitive Place Values

Data Coding by Truncation of Repetitive Place Values Measurements such as 0.55303, 0.55308, 0.55310----> The digits 0.553 is repeating in all the observations can be recordeds as the last digits expressed as integers

Design for Performance---

Design for Performance--- Achievement of aggressive benchmarks (eg. Aircraft designed to be faster than speed or light or more powerful microchip processing speeds)

The degrees of freedom for a contingency table containing 3 rows and 4 columns is: 7 12 11 6

Df=(r-1)(c-1)=2 * 3 = 6

Interval Data Scale

Interval Data Scale i. Intervals between adjacent scale values are equal with respect to the attribute being measured. (difference between 30 degrees C and 20 Degrees C) ii. Application: a) Addition and subtraction of scale values, b) T-test, c)Standard Deviation, d) Mean

Locational Data

Locational Data 1) Not attribute or variable data 2) Tells about location 3) Eg. Map of USA with sales area or map of all the Walmarts in the USA.

Most valid procedure for collecting customer data ?

Most valid procedure for collecting customer data is to randomly sections a large groups of customers and get compete and accurate data on each one selected. ***Collected data should be objective and designed to shed light on the customer requirements. *****Verify the accuracy and consistency of the data collected

Natural process limits verses Specification limits

Natural process limits verses Specification limits a. Specification limits are the values between which products or services should operate. These limits are usually set by customer requirements.

Negotiations Techniques

Negotiations Techniques a) Most people negotiate based on trial and error b) Should use coercion. 1. Interest -based Bargaining 2. Win-Win negotiations

Pande states that a decision on six sigma may be negative if the following 3 conditions exist?

Pande states that a decision on six sigma may be negative if the following 3 conditions exist? 1. company alread has a strong, effective performance and process improvements in place 2. Current changes are already overshelming the company's people and resources 3. Potential gains aren't sufficient to finance the company's people and resources.

Term: Inventory is delivered to location where it will be consumed

Point of Use Inventory Inventory is delivered to location where it will be consumed

Process Decision Program Chart

Process Decision Program Chart 1) Tree diagram used to illustrate anticipated problems and list possible solutions and can also function as a document to update the project proceeds

Process Elements

Process Elements 1) Business Process Management ----> Methods that have evolved from the basic tenets of quality and continuous improvement to address business objectives and results 2) BPM is focused on understanding, controlling, and improving business processes to create value of all stakeholders.

Process Performance vs Specification

Process Performance vs Specification a. Natural Process Limits i. Natural process limits= +/- 3σ-→ if the natural process limits are outside the specification limits then the process can't meet specifications. ii. Derived from process variation after removal of all special causes and the process reached statistical stability. iii. Green Belts expected to review those out-of-control violations, assign special causes, and recalculate the control limits before firming up the limits of implementation.

How is Process Lead time Calculated?

Process lead time is calculated by dividing the number of items in a process by the number of times the process is completed in an hour. This calculation is known as Little's law. A process can attain a maximum process cycle efficiency of 100% if all the activities in the process add value

Process partial autocorrelation

Process partial autocorrelation Formula for estimating the partial autocorrelation function is , for a given lag of m. r (m)= autocorrelation function . 1) Significance limits for both autocorrelation function & partial autocorrelation function (if the true population of the ACF or PACF is zero) is calculated at the stated significance level. 2) If the true population of the autocorrelation function or partial autocorrelation function is larger, then ----> the significance limit must be determined with the formula for partial autocorrelation (look it up): ---------in which n is the number of observations and k is the ordinate of the normal distribution at the stated significance level.

How can QFD and RPN assist one another?

QFD and RPN assist one another in the following ways: @ Severity, Occurence & Detection determined by internal cross function group but can improved with inclusion of suppliers and customers to provide additional information for a full picture a) Suppliers ---information about frequency of failure and the likelikhood of failure detection b) Customers--information about criticality and failure detection c) Customer service personal who handle customer complaints and Government funded groups (environment, insurance, and consumer) give statistics and failure modes.

Ratio Data Scale

Ratio Data Scale: 1) There is a rational zero point for this scale; Ratios can be equivalent 2) Application: a) Geometric Mean b) Coifficient of Variation c) Multiplication & Division of scale Values

Reasons for Removing Team Members

Reasons for Removing Team Members 1) Team member lacks skills and has very little interest to develop skills 2) Team member unable to keep his/her committments due to involvement in other projects 3) Personality Conflicts

Reducing time spent on relocation and replacement

Reducing time spent on relocation and replacement (NVA) Strategy for reducing time lost due to location issues : a ) standardizing setups. b) Simplify the setup protocol (eg. redesign fixtures) d) Standardize setup procedure for all the members of a product family "

Repair/ Reject Muda

Repair/ Reject Muda 1. Rejects involve scrapping the whole part and waste resources 2. Rejects on a continuous flow line defeats its purpose. 3) Non-conforming product forms filled out by suppliers for rework/reject waste paper 4) Takt time wasted due to line operators and maintenance to fix the rework/repair.

Repeated Trials

Repeated Trials Trials that are conducted to estimate the pure trial-to-trial experimental error so that lack of fit may be judged. ----aka replications.

3 uses of scatter diagrams?

Scatter Diagram a. Used for: i. Root Cause Analysis ii. Estimation of Correlation Coefficient iii. Regression line Calculation to make predictions.

Scatter Diagrams & irrelevant variables

Scatter Diagrams & irrelevant variables Mathmatical relationships can be found with irrelevant variable-→ Must use Engineering judgement to select variables

Sequential sampling

Sequential sampling i. Best for destructive testing and reliability testing where higher cost is involved in testing the unit. ii. Samples tested sequentially until the desired results reached.

Signaling Mechanism & Pole/Yoke

Signaling Mechanism & Pole/Yoke: 1) Color-coded parts 2) tool and fixture templates in place to only accept corect parts 3) Mechanism that detect the insertion of the wrong part. 4) Buzzer/ light signals error requiring immediate action. ***Root cause analysis and corrective action needed befor work resumes.

Special Cause variation: (Assignable causes)

Special Cause variation: (Assignable causes) a. Unusual event that the operator can adjust or remove. b. Process outputs will be unpredictability influenced with random events if the all the special causes are not mitigated.

Stages of Project Management

Stages of Project Management 1) Planning 2) Scheduling 3) Controlling--ensuring that desired results are reached

Stakeholder Analysis

Stakeholder Analysis: 1) As part of the Define phase, attempts to remove/reduce resistance to chance must be made by identifying the stakeholders and planing to involve them in the change process, removal of pitfalls, or alternate solutions. 2) Need a communication plan to keep stakeholders involved----> Should document the perceived scale of commitment or resistance of stakeholders

Standard error of the mean

Standard error of the mean: a. Used to calculate the margin of error (used to find the confidence interval) b. 95% of the time, the sample mean lies within 2SD of the mean (where the true mean should lie +/- 1.96 (Sigma/ square root of n)

5 States of Team Development

Storming i. Understanding of scope & members' roles and responsibilities put to test ii. Team members voice opinions and disagreements slow down the team and not much accomplished.

Taguchi Books

Taguchi Books 1. System of Experimental Designs 2. Introduction to Quality Engineering 3. Off-line Quality Control

Team Dynamics: Rush to accomplishment

Team Dynamics Rush to accomplishment a. Symptoms= incomplete data, inconsistent analysis, trying to reach conclusion faster b. Cause= unrealistics deadline, untrained team members, search for short-term gains. c. Solution: i. Ask for alternative solutions ii. Keep realistic deadlines iii. Team leader requests for data collection, analysis, and statistical significance.

Team Stage--Storming

Team Stage--Storming 1. Members have a. Divided Loyalty b. Think Individually c. Are learning roles d. Confrontations. 2. Conflict on authority issues, vision, values, cultural differences, and personality. 3. Most difficult stage of team dynamics. 4. Solution is use quality improvement process,& tools + proper communication

What kind of plot takes the data points of 12, 13, 18, 18, 22, 22, 29, 29, 31, and 35 to show the minimum 12, maximum 35, median 22, and the quartiles of 18 and 29. Stem-and-leaf Box-and-whisker Weibull Histogram

The box-and-whisker is a graphical plot showing the data set statistics of minimum, maximum, medium, and quartiles.

The most significant factor to be considered in establishing Six Sigma project metrics is: Customer needs Accuracy of measurement Ease of measurement Project timeline

The linkage between project metrics and customer needs ensures that the focus of a project is on addressing customer needs.

Tools used to define the project scope:

Tools used to define the project scope: a. SIPOC b. Brainstorming c. Pareto chart d. Process maps

Types of Control Plans

Types of Control Plans A. Prototype B. Pre- Launch C. Production

"V" phase of IDOV

Validate Phase -IDOV i. Prototype test & validation ii. Assess performance, failure modes, reliability, and risks

Value Flow & Batching

Value Flow & Batching: 1. Batching technique--used in mass production to produce many units of a specific part at a given time to maintain efficiency 2. LEAN requires conversion of batch process to a continuous flow process

W.R Haylip & consumer Surveys

W.R Haylip & consumer Surveys To conduct consumer surveys: 1. Need larger customer population 2. Small Purchase volumes 3. Small Purchase transaction 4. Supper may know more than the customer.

Weibull Plots

Weibull Plots a. Used for reliability data when the underlying distribution is unknown b. Can be used to estimate the shape parameter Beta and mean time between failures (MTBF) or failure rate. c. Can be done manually or via software. d. There is relevance between all these parameters and the product lifecycle.

tem A has a RPN of 120 with severity of 10, occurrence of 4, and detection of 3. Item B has a RPN of 120 with severity of 5, occurrence of 4, and detection of 6. Both items have the same ranking values and have the highest RPN. Which item will you address first? A) Item A because of high severity. B) Item B because of high detection C) Does not matter which one you pick. D) Ignore both because of same occurrence.

When two risks have the same overall score, the risk with the higher severity rating is escalated. Item B has high detection but is not severe and has lower business impact than A.

Drawbacks of X-MR charts

Drawbacks of X-MR charts 1) All interpretation is faulty if the data distribution is not normal 2) Individual charts do not separate piece - to- piece repeatability of the process 3) Variability patterns cannot be assured until 80-100 readings are taken 4) Individual charts are not as sensitive tochanges in theprocess as the X-bar /R chart

Excess Movement of Materials (Waste)

Excess Movement of Materials a. Plants with function-oriented departments require excess material movement. b. Solution: Manufacturing cells with several types of equipment requiring staff with multiple skills. (C-shape cells allow flexibility in staffing layout)

Data collection methods -----(5W1H)

Data collection methods -----(5W1H) a. Method of answering questions (what, where, why, who and how) before collecting data to ensure that data collection is effective. b. Data coding---best used when manual data entry is involved; prevents errors and avoids repetitive recording of numbers.

6 types of Negative conflict results

6 types of Negative conflict results 1) Hostile, impulse to destroy 2) Win-Lose 3) Lose-lose 4) Undesirable consequence 5) Isolation 6) Loss of productivity

Experimental Objectives : Main Effects

6. Main Effects a. Estimate of the effect of a factor independent of any other means. b. Average Main Effects-→ average the results for each level of each factor c. The larger the main effect, the more influence that factor has on the quality characteristic i. Factors with the greatest difference between the "high" and "low" results are factors with the greatest impact on the quality characteristic of interest.

A total of 5,000 pins were manufactured on the machine. 200 pins had a total of 600 defects. 4,800 pins were packed at the end of the line. What are the defects per unit (DPU) and throughput yield for this process? 1.2 DPU and 88% throughput yield 0.12 DPU and 89% throughput yield 0.04 DPU and 96% throughput yield 1 DPU and 96% throughput yield

600/5000 = 0.12 DPU e-0.12= 89% throughput yield.= CORRECT ANSWER Defects per unit (DPU) is determined by total number of defects divided by the total number of products produced in some time period. Throughput yield is determined by 1- % of closure defects * 100.

16. During which phase of response surface analysis is the direction of maximum response identified using the steepest ascent methodology? a. Phase 0 b.Phase 1 c. Phase 2 d. Phase 3

16. B: Phase 1. During Phase 1 of response surface analysis, the direction of maximum response is identified with the steepest ascent methodology. This methodology is also used to define the current operating region. Phase 0 involves the use of screening designs to assemble a basic list of significant factors and create a first-order regression model. P hase 2 is the application of ridge analysis and a second-order model. The intention of Phase 2 of response surface analysis is to identify optimal conditions at the stationary points in a limited region. There is no Phase 3 in response surface analysis."

Three Measures of Dispersion

3 Measures of Dispersion 1) Range 2) Standard Deviation 3) Variance

50. Which of the following increases the power of an estimation of the confidence interval on the mean? a. A sample population with a normal distribution b. A smaller number of samples c. A known standard deviation d. An unknown standard deviation"

"50. C: A known standard deviation. A known standard deviation increases the power of an estimation of the confidence interval on the mean. Indeed, when the standard deviation is known, the z tables may be used to find the confidence interval on the mean; when the standard deviation is unknown, the t tables must be used. The confidence interval on the mean is the percentage of samples that will contain the true population mean. It is assumed that the sample population will follow a normal distribution. When there are more samples, this increases the power of the estimation of the confidence interval on the mean."

"5S

"5S 1) Lean tool derived from a set of Japanese words related to organizational hygiene: organization, purity, cleanliness, discipline, and tidiness. 2) 5S= sort, straighten, shine, standardize, and sustain. a) Improve stage ---> it is a valuable strategy for reducing cycle time that does not add value and that is lost to movement, finding lost materials, and inefficiently using the physical space. 3) 5S may also be used to accelerate inventory processes and to diminish workplace accidents 4) The program is implemented according to a five-level system. a) 1st level= the organization unexposed to Lean management processes. b) 5th level = maximum efficiency and minimum waste have been achieved. ****In Six Sigma, the data collected according to such a rating system often is placed on a radar chart, which indicates the strengths and weaknesses of each department."

"Responses to conflict: Avoidance

"Avoidance= Responses to conflict 1) If a person notices a conflict but fails to bring it up or pursue it, he or she is avoiding the conflict. 2) Least cooperative and assertive way to resolve a conflict. 3) If the conflict is insignificant or the outcome is assured, avoidance may be the best approach. In other words, if there is no chance of getting one's way, it may be better to avoid a conflict. 4) Avoidance may also be the correct response if tempers are already hot over some other issue. It may be preferable to avoid a conflict until the members of the team have settled down. 5) Another time to avoid conflict is when the possible consequences of disagreement vastly outweigh the benefits of resolution. 6) Finally, a team member may be justified in avoiding conflict if he or she feels other members of the team can do a better job of resolving the issue. "

"Confidence interval on mean

"Confidence interval on mean 1) Assumes that the sample represents the entire population and that it has a normal distribution. 2) Assumes that the sample should be constant or stable through time. 3) Measure stage--->, an estimation of confidence interval on mean is used to estimate the process average when a process control cannot be set because of a lack of data. This process average is used for baseline estimates. 4) Analyze stage---->, an estimation of the confidence interval on mean is used to examine the similarities and differences between the sample means taken during various process conditions."

"Consensus-criteria method of creation

"Consensus-criteria method of creation 1) Use consensus-criteria method ---> when there are too many criteria or options or too little time to create a prioritization matrix according to the full analytical method, a ) Initial matrix discarded + team distributes a hundred points across the criteria in accordance with perceived importance. b) Less time-consuming & more subjective method in assessing importance 2) After the criteria have been weighted, they are compared with the options. 3) The final score for each option is calculated by multiplying its score for each criterion by that criterion's weight. ---->The larger the product, the more important the option. *****Both the full analytical and consensus-criteria methods of creating a prioritization matrix make an essentially subjective exercise into something a bit more structured."

"Creation and interpretation of Process Maps

"Creation and interpretation of Process Maps: Goal of process maps---> to spot redundant, unnecessary decisions and tasks and to discover ways to simplify processes. On a process map, a symbol represents every step in a process. 1) Diamonds= decisions : other tasks with rectangles. 2) All decisions should be framed as binary; that is, only two possible answers exist. 3) Shading or shaping will indicate delays or measurement intervals. 4)Glut of decision points.= signs of inefficiency on a process map (like Flowchart)

"Creation of EVOP Strategies

"Creation of Evolutionary operations strategies 1) Assess performance on a cycle-by-cycle basis.----> For the purposes of EVOP, a cycle is defined as an interval in which data has been collected at each point in the process design. 2) In evolutionary operations, several cycles are observed and then the experimental conditions are varied. 3) Phase= Each set of cycles performed with the new operating conditions 4) Mean/ Curvature=. The difference between the edge points and the center point . 5) Limit adjustments to 2-3 factors when implementing an EVOP strategy, Otherwise, the experiment will be too complicated. a) Keep adjustments minor so effects can be observed with precision. b) Best to run experiment twice before estimating error and the effect significance if experiment for 2-3 factors c) If the factor still significant after 3 cycles, the next phase of the strategy should be performed with the conditions centered on the new optimal condition. d) If factors do not appear statistically significant, increasing the range of levels for these factors may be useful. "

"Creation of individual-X charts

"Creation of individual-X charts 1) In an individual-X chart, the plotted statistic is the observation. a) Normal distribution= the centerline is the average. b) Non-normal distribution--->, the centerline is the median of the fitted distribution. 2) The upper control limit is calculated (lookup formula) and the lower control limit is calculated (lookup formula) , where is the process sigma and is the average. 3) When the process is non-normal, the upper control limit is set at the 99.865 % of the fitted curve, and the lower control limit is set at the 0.135 percentile. 4) Selecting a distribution curve can be difficult because the control limits are based on the curve, but the curve cannot be set until it is verified that the process is under statistical control. For this reason, many people prefer to use the EWMA or some other chart to establish process control so a distribution can be assigned to the data."

"Critical-to-quality metrics

"Critical-to-quality metrics 1) Yield = common critical-to-quality metrics or the amount of completed product divided by the amount of product that began the process. a) Obviously, a business would like to have yield equal 1 (or 100%). b) Yield does not distinguish those salvageble pieces of scrap 2)) Scrap Rate = The difference between perfection and reality ----( eg. If a process had a yield of 95%, the scrap rate would be 5%. Yield is a useful metric, although it does not indicate where in the process errors ) 3) Throughput Yeild= average percentage of units with no defects. a) When multiple steps in a process are considered, Six Sigma teams use rolled throughput yield, which measures the expected quality level after several steps."

An exponentially weighted moving average (EWMA) chart

"EWMA charts 1) Control chart (for variables data) ---created selecting a weighting factor, which assigns different values to data depending on its age. 2) Uses data from every available sample to identify extremely small variations in the process. 3) Measure stage--->, charts are used to baseline a process especially if process might be non-normal & rational subgroup size = 1. 4) Control stage--->, EVAP charts are used to control the process, either for the reasons outlined in the measure stage or when it is necessary to detect extremely small shifts in the process. a) Given the same sample size, an exponentially weighted moving average chart can detect shifts from 1/2σ to 2σ faster than a Shewhart chart."

"Fractional factorial designs

"Fractional factorial designs 1) Restricts focus to the interactions between pairs of factors.----> Aliasing --- interactions of more than two factors are represented as a single new actor--allows to get away with this. a) Example-- factorial design with 3 factors will not consider the interaction between factors 1, 2, and 3, but instead will create a new factor (4) representative of this interaction. -->allows the factorial design to be comprehensive with far fewer runs. 2) Confound Data--- third variable that can make it incorrectly appearthat an observed exposure is associated with an outcome.-----Eg. effects of factor 4 cannot be estimated independent of the interactions of factors 1, 2, and 3. 3) Fractional factorial designs are good for quickly identifying factor interactions that should receive more attention in subsequent experiments. "

Goodness-of-fit Tests

"Goodness-of-fit tests 1) Measure the validity of statistical assessment--- indicate whether a chosen statistical test provide an accurate and relevant measure. 2) Used during the measure, analyze, improve, and control stages of DMAIC to confirm a distribution, which enables the team to assume the validity of confidence tests, statistical control charts, hypothesis tests, and other tests. Common goodness-of fit tests: a) Kolmogorov-Smirnov (K-S)- more specific results b) Anderson-Darling c) chi-square-- no software needed

"Identification of key decision points

"Identification of key decision points 1) Process map used to identify the most important decisions made during the process. 2) Any areas that seem to require excessive decision-making, usually a sign of inefficiency. 3) Try to reduce the number of decisions that have to be made every time a process is performed.---> All stakeholders must be aware that changes are not to be made to the process at this time. 4) When measuring the process , all measurements should reflect the process as typically performed. Otherwise, improvement efforts could be targeted incorrectly. The team leader needs to communicate this information to all stakeholders.

Interpretation of process capability index

"Interpretation of process capability index is 1) influenced heavily by the distribution assumptions. a) If the value of Cp (the ratio of tolerance to process variation) is 1,---> then tolerance and process variation are equal. b) If the value of Cp is <1 , then the allowable variation is less than the process variation, which means that the normal amount of variation could be too much. This is a bad situation. c) When Cpk is used, it is possible to obtain information about the process variation and location and their relation to the requirements. ****The statistic Cpm is much like Cpk, but it also considers the relation between process average and a target value. If these values are the same, then Cpm and Cpk are equal. "

"Prediction of benefits

"Prediction of benefits The Six Sigma team must justify their proposed solutions by predicting the benefits to be obtained. 1) Prioritization matrix used to show how proposed solutions are aligned with the most pressing needs of the business. 2) EBIT =A common metric for proposed solutions that requires fixed and variable cost figures. 3) Formula for EBIT EBIT= (Volume) [(Per-unit price) - (Per-unit variable cost) ] - Fixed Cost 4) Use NPV-- to evaluate proposed solution with varying benefits over time, 5) Calculate IRR ---> if the net present value is invested. ****These sophisticated methods allow the team to determine the probabilities of achieving various levels of savings or profit. "

Prioritization matrix

"Prioritization matrix 1) Ranks choices to build consensus in a group setting primarily are used during the define and improve stages of DMAIC. 2) Six Sigma teams typically use one of two methods for creating a prioritization matrix: either the full analytical method or the consensus-criteria method. . 3) Define stage--->,prioritization matrices helps groups pick the projects most important to the organizational objectives and goals. + effective tool for comparing customer desires with process also metrics. ***** As much as possible, a business wants to measure the qualities of a process or product important to clients. 4) Improve stage---> a prioritization matrix confirms the alignment of projects with customer requirements."

"Process metrics

"Process metrics 1) Must be reliable, repeatable, and reproducible to be considered effective illustrations of process characteristics. 2) Must be specific and detailed enough to distinguish between the effects of process alterations and process noise. 3) Metrics should relate to actual targets of business strategy, relate to actual services given to customers and result from cooperation between employees. 4) Before any metrics are applied, employees from several different departments should judge them. 5) Metrics should pertain to values critical to quality, schedule, or cost, as these factors are the three most important variables in any process.

"Process when the historical standard deviation is known

"Process when the historical standard deviation is known 1) If historical standard deviation is known, the process of estimating the confidence interval on the mean begins with calculating the average for n sample units. 2) The confidence interval may then be calculated with the following formula: (see formula) a) Because it is assumed that the samples are taken from a population with a normal distribution., the z values are calculated according to the confidence level. b) If the confidence level is 90%, then alpha= 1 . c) The confidence interval is two-sided: that is, it extends from one side of the mean to the other. Therefore, the α value is divided in two: α/2 = 0.05. The z value, then, can be obtained by referring to a chart of areas under the normal curve.

Project status report

"Project status report a) begins with a current schedule and brief description of the project. b) includes a list of the action items, or the tasks to be completed by assigned team members at particular times. c) Lists of roadblocks, or hindrances to progress not yet resolvable by specific actions. d) list of the data obtained during each phase of the project. e) List outstanding items. ****Project status reports typically are issued once or twice a week and every time one of the DMAIC phases is completed. Copies of the report are sent to the sponsor, team members, and any other relevant stakeholders."

"Responses to conflict: Competition

"Responses to conflict: Competition 1) Competition=conflicting parties do their best to win. a) Dispute settlement and generate excellent work if fair competition. b) . Competition can be as simple as civilized argument during a meeting. 3) Competition as a response to conflict is appropriate : a) when some members of the team are undecided. (A team member also should be ready to compete when an issue is vital to the success of the business. ) b) When conflicts that need a clear resolution ---An objective form of competition settles a dispute for good. ****When competition is used to settle disputes, the team leader should ensure that team members are fair and nice to one another. *****Competition sometimes becomes a habit in a team, which can be detrimental to cooperation in the future." l

Run-test Rules

"Run-test Rules 1) Western Electric created run-test rules as additions to the standard control chart. 2) Purpose tests ---> identify patterns in the plotted points on a control chart using eight run tests 3) All eight of the tests can be used for and individual-X charts. a) The first four run tests can be used for P, U, Np, and C charts. b) Run test 1 ---one subgroup with more than 3 SD from the mean-->process mean shifted. c) Run test 2---> 9 consecutive subgroups on one side of the average. d) Run test 3---> 6 subgroups in a row either increase or decrease. e) Run test 4,---> 14 consecutive subgroups alternate being greater or smaller than the preceding and succeeding subgroups. f) Run test 5,--- two of the 3 consecutive subgroups > 2 SD from mean. g) Run test 6--- 4 / 5 of subgroups in a row > 1 SD from mean h) Run test 7--- 15 consecutive subgroups are within one standard deviation. i) Run test 8----> 8 points in a row are more than one standard deviation from the center.

39. Which of the following autocorrelation functions would indicate the strongest correlation? a. 0.1 b. -0.8 c. 0.9 d. -0.2

39. C: 0.9. An autocorrelation function of 0.9 would indicate the strongest correlation. The range of autocorrelation functions and partial autocorrelation functions extends from -1 to 1. The strength of the correlation is indicated by the distance from zero (that is, the absolute value), regardless of whether the value is on the positive or negative side. Therefore, an autocorrelation function of 0.9 would indicate a stronger correlation than would functions of 0.1, -0.8, and -0.2."

5 Reasons why the Project Charter is Useful?

5 Reasons why the Project Charter is Useful? 1) Team is made aware of project goals & organization and operational boundaries ----->(prevents turf wars and draining energy) ------> Charter also eliminates confusion and defines the subject boundaries in the problem statement 2) Team will remain focused on original goals or the operating scope ----> It identifies areas that should not be addressed and identifies project deliverables 3) Team will work on projects that align with organizational goals- 4) Team champion will support the team goals---> Project charter documents top management's support and commitment. 5) Charter authorizes team to collect data and give access to resources.

6 Characteristics of the Team Process

6 Characteristics of the Team Process ***Team process characterized by: 1) Highly supportive climate 2) Open communciation 3)Commitment 4) Individual achievement 5) Organizational goal achievement 6) Creative problem-solving

6 Metrics for measuring resources

6 Metrics for measuring resources 1. Percent defects 2. Cost of Poor Quality 3. Variation in reduction 4. Process capability studies 5. Return on improvement projects 6. Number of improvement projects.

During the production of yogurt, line operators collect samples from the milk and test its pH before immediately submitting the results to the quality control laboratory. Laboratory technicians then test the pH, protein content, and viscosity of the milk. Which of the following describes the second pH test performed by the laboratory? A)Non-value added B) process performance indicator C) Product release test D) Quality assurance

A= CORRECT! Since the second pH test is immediately performed after the first one, it is an extra inspection step that is not adding any value to the process. Laboratory tests are considered quality control activities, not assurance activities. Product release test is an in-process test (finished product is the yogurt), not a release test.

Which of the following statements best describes why companies find implementing lean to be valuable? A) Lean creates value for the organization by systematically identifying and eliminating waste. B) Lean prevents errors or mistakes from producing waste in the production cycle. C)Lean ensures that processes operate so that products or services meet or exceed customer requirements. D) Lean translates customer requirements into technical specifications to assure outstanding products or services are produced.

A= Correct = Creating value for the organization by systematically identifying and eliminating waste is the value that lean provides. Preventing errors or mistakes from producing waste in the production cycle is the value of error-proofing or poka yoke, one of the lean tools. Translating customer requirements into technical specifications to assure outstanding products or services are produced is the value of quality function deployment. Ensuring that processes operate so that products or services meet or exceed customer requirements is the value of control plans.

ANOVA & F-test

ANOVA & F-test 1) ANOVA compares variation within each subgroup to the variation found between different subgroup found by analyzing the essential differences between the averages of each subgroup. a) Ho: All subgroup averages are equal. 2) F statistic =compares the average variation between subsets, known as the mean square treatment, with the sun of the squares of the residuals, known as the mean square error. a) Assumes subsets have a normal distribution and equal variance. b) Reject H0: if F test's p-value < 0.05.

Adding New Team Members

Adding New Team Members 1. Don't impose an individual onto the team. 2. Allow the team to be part of the selection process where potential candidates are interviewed either individually or collectively. ---> ensures that team is confident they made the right decision.

Appropriate situational Communications

Appropriate Situational Communications 1) Reprimanding employee done privately and verbally 2) Documented letter for repeated offenses 3) Business agreements may be verbally but followed by a letter to ensure that both parties have the same understanding. 4) Strategic initiaitves, such as six sigma should be communicated by the CEO.

Armand Feigenbaum Statement on Quality

Armand Feigenbaum=------Statement on Quality An effective system for integrating the quality development, quality maintenance, and quality improvements of the various groups in an organization so as to enable production and service at the most economical levels allowing for full customer satisfaction.

Balanced Scorecard

Balanced Scorecard a. Measurement system + Management System (translates vision & strategy into action) b. Provides feedback on both internal business processes and external outcomes for continuous improvement. c. Selection of project metrics to ensure customer & business needs are met. d. Metrics (4 perspectives)= financial, customer, internal processes, and employee learning/growth. e. Uses lagging & leading measures across the 4 perspectives i. Lagging measures= measured at end of the event. ii. Leading measures= measures that help reach objectives (measured upstream of the event)

Which of the following is the best reason a company will choose, all else being equal, projects that most positively impact profitability? A) Profit allows an organization to purchase new machinery to meet increasing demand. B) High profits in the present guarantee the long-term success of a company. C) Earning profits allows a company to continue to work toward the goals of management. D) A company with profits can reward shareholders with quarterly dividends.

C= Correct! ----Earning profits allows a company to continue to work toward the goals of management is the correct answer. Profitability ensures an organization can remain an ongoing concern and work toward its goals, regardless of how it chooses to do so. Current profitability is no guarantee of future profitability. The organization must continually adapt to changing market conditions.

Common Cause Variation (Natural variation)

Common Cause Variation (Natural variation) a. Can't be controlled by process operators b. Sources of variation within the process itself. c. Common Cause variation reside within processes within statistical control and can be characterized by the following to estimate predictability. : i. Location (process average) ii. Spread (piece-to-piece variability) iii. Shape (distribution

Control Limit verses Specification limit grid

Control Limit verses Specification limit grid a. First Quadrant— if the process in a state of statistical control (within natural process control limits and follows other applicable rules) and meets specification, the situation is "good" b. Third Quadrant-----if the process in not in a state of statistical control (within natural process control limits and follows other applicable rules) and does not meet specification---→ Greenbelt should stop the process and immediately investigate. c. Fourth Quadrant→ Engineers hesitation to invest time investigating in out of control process-→ Green belts should explain that out-of-control process lacks predictability and ma go out of the specification limit any time and recommend stop

Elements of a visual factory include which of the following? Lines painted on the floor A project charter Failure Modes and Effects Analysis Control plans

Correct! Lines painted on the floor can be used for communicating such as indicating where it is safe to walk.

The current process has a mean of 2.50 and a std deviation of 0.05. A new process has been suggested by research. What sample size is required to detect a process average shift of 0.02 at the 95% confidence level? 16 5 Insufficient information to determine sample size 25

Correct! The correct formula is N = (0.05 (1.96)/0.02)2 = 24.01, round up to 25 0.05 = sigma 0.02 = amount of change you wish to detect 1.96 = z value from table at 95%, remember to use two tail

The process drift in machining operation is said to be normally distributed with a mean of 46.23 seconds and a standard deviation of 3 seconds. What is the probability that the drift of this operation will be less than 52.81 second Where the next measurement is taken? 0.0143 0.9857 0.4857 1.00

Correct! 0.9857 represents the area under the curve to the left when Z is 2.19, and represents probability of an occurrence that is less than the value Z. 0.0143 represents the area under the curve to the right when Z is 2.19 and represents probability of an occurrence that is greater than the value Z (This is the compliment of rational 4, the candidate must understand the difference when calculating z and using the z tables to determine the area under the curve).

Correlation Coefficient

Correlation Coefficient Number between -1 and +1 that indicates the degree of linear relationship between two sets of numbers--Zero= no relatinship

6. Which of the following conflict-response strategies would be most appropriate when a group is fragile? a. Collaboration b. Competition c. Avoidance d. Accommodation

D "6. D: Accommodation. When a group is fragile, the most appropriate conflict response strategy would be accommodation. Accommodation is the temporary sacrifice of personal desires in the name of group consensus. If a group is in danger of falling apart, the best way to handle a conflict may be to temporarily put aside differences in order to make progress in other areas. Collaboration or competition may be too risky for a fragile group, and avoidance of the conflict only jeopardizes the long-term health of the group by failing to resolve the underlying issues.

"DMADV and DMAIC

DMADV and DMAIC are both Six Sigma methodologies, but they have different applications. 1) DMADV a) new products and processes (b/c includes a design phase) + bringing an existing process up to sufficient quality standards. b) If a process needs to be created from scratch or subjected to a comprehensive overhaul. 2) DMAIC= a) Suited for gradual improvements of existing processe"gradual improvements of existing processes and products. b) appropriate for bringing processes and products into alignment with customer requirements and quality standards.

Describe the Critical-to-Schedule Metrics

Describe the Critical-to-Schedule Metrics 1) Cycle time= most common metric----aka. delivery time, order processing time, or downtime. 2) Requires distinction between NVA and VA activities to improve Cycle time 3) Cycle time is secondary in importance to cost or quality.

Duties of Champion

Duties of Champion i. Resource allocation ii. Determine project selection criteria iii. Approval of completed projects iv. Removal of project barriers

Excess Motion (source of Waste)

Excess Motion a. Can be reduced by having experienced staff and staff with similar functions. b. Use of Kaizen work teams to improve work areas. c. Reasons for excess motion: i. Poor workplace layout ii. Awkward positioning of supplies/ equipment

Experiment

Experiment Test undertaken to make an improvement in a process or to learn previously unknown information

How to go about removing team members?

How to go about removing team members? 1)Both Team and manager should have a frank discussion with the individual 2) Conversation should focus on: a. Expectations b. What's at stake c. What's not happening that needs to happen or what is happening that should not be happening.

Intermediate Customers

Intermediate Customers 1) Those who purchase and then resell /modify/ assemble product for sale to end-user 2) Quality training & consulting firms 3) University and commercial bookstores 4) Foreign distributors /Transport companies/ Wholesalers 5) Retailers

Interpretation of X-chart

Interpretation of X-chart 1) Search & remove for special-cause variation after X chart is completed----> To allow for the future capability of the process to be predicted by eliminating any out-of-control points from the analysis. 2) Study special cause variation in designed experiments by looking for non-random behavior or trends in the data. using run-test rules 3) After confirming that process is Once a process has been observed for a sufficient time and is found to be in control, calculating the process capability relative to requirements is possible. However, it is not possible to predict the capability of a process not in statistical control."

Interpretation of autocorrelation function

Interpretation of autocorrelation charts When interpreting an autocorrelation chart---> false correlation is possible Sources of False Correlation: a) Low lag & autocorrelation significant only at adjacent data points----Solution= increase the time between data point collection. b) Sampling from several different streams in a process. c) Large autocorrelations for smaller lags influence larger lags. ( e.g., if a strong correlation exists b/w observation #4 & # 5 or another strong correlation exists b/w observation 5 & 6 --->observation 4 & 6 appear falsely correlated----> Solution = diminish using the partial autocorrelation function. *****In any case, autocorrelation functions and partial autocorrelation functions range from -1 to 1. The farther away the value is from zero, the stronger the correlation. "

What are the 3 levels of needs in Kano Model?

Kano model graphically portrays the three levels of need: expected, normal, and exciting.

Latin Square Designs

Latin Square Designs 1) An experiment design that can be used to control the random variation of two factors. The design is arranged with an equal number of rows and columns, so that all combinations of possible values for the two variables can be tested multiple times. This design is used to reduce the effect of random or nuisance factors. 2) Field marks: Treatments are assigned at random within rows and columns, with each treatment once per row and once per column 3) The Latin square design applies when there are repeated exposures/treatments and two other factors. This design avoids the excessive numbers required for full three way ANOVA. 4) Originally applied in agriculture when two sources of non-homogeneity were the two direction of the field and the square is the plot of ground.

Lessons Learned

Lessons Learned 1) Formal and documented critique that analysize what went well or wrong to use in future projects. 2) Conducted by committee of qualified persons 3) C overs all phases of Development Topics: a) Project tracking, recognition of project team efforts, project team performance, cooperation amongst team members, quality level of end product/service, effectiveness of corrective actions, and adequacy of resources.

Master Black Belts

Master Black Belts 1) Teachers who mentor black belts and review their projects 2) Selection Criteria= Quantitative skills and teaching ability a) Teaching ability is evaluated by the number of people they have mentored or classroom environment experience. b) Must have active black belts with many projects.

Negative Team Dynamics

Negative Team Dynamics 1) Overbearing Members 2) Dominant Members 3) Floundering 4) Reluctant participants 5) Unquestioned acceptance of opinions & facts 6) Group Think 7) Feuding 8) Rush to accomplishment 9) Attribution 10) Discounts 11) Digressions & Tangents

4 Organizational Goals & Six Sigma Projects

Organizational Goals & Six Sigma Projects 1) Hoshin Planning 2) Selection and use of metrics ---→reflects the performance and health of the organizational processes. 3) System= must consider the effect of proposed changes to other process within the system. 4) Suboptimization= operating at less than its best.

Image displayed next to each question A Six Sigma Manager is conducting a closure meeting with the project sponsors to ensure agreement that the project is completed. Which of the following would the Manager likely create an output of the project closure? Project metrics Gantt chart Project archives Project charter

Project archives= answer= Project specific documents need to be archived at the end of the project and need to be accessible for relevant stakeholders.

Importance of Project documentation

Project documentation 1) Project documentation is controlled and updated during the lifecycle of the improvement project. 2) Failure to provide project documentation-→ miscommunication.

Purpose of Design of Experiments

Purpose of Design of Experiments a. Objective of a designed experiment to generate knowledge about product/service. b. Find the effect that a set of independent variables has on a set of dependent variables. c. Control factors= independent variables that the experimenter controls (the rest are called noise factors)_ d. Focus on how the outcomes depends on the variation. e. May involve designing a set of experiments to identify optimal conditions

Random sampling

Random sampling i. Every item has equal probability of being picked ii. Lot being sampled has to be homogenous for random sampling 1. Random samples are used in population sampling situations when reviewing historical or batch data. 2. The key to random sampling is that each unit in the population has an equal probability of being selected in the sample. 3. Using random sampling protects against bias being introduced in the sampling process, and hence, it helps in obtaining a representative sample. iii. In general, random samples are taken by assigning a number to each unit in the population and using a random number table or Minitab to generate the sample list. Absent knowledge about the factors for stratification for a population, a random sample is a useful first step in obtaining samples

Randomized Trials

Randomized Trials Frees an experiment from the environment and eliminates biases. This technique avoids the undue influences of systematic changes that are known or unknown

Residuals

Residuals the difference between experimental responses and predicted model values

Risk Mitigation & Verification

Risk Mitigation & Verification Function= Provides team with the worst-case scenario and what they should do by assessing the status of existing risks, determining whether new risks identified, and by checking if the identified risks were realized. a. Initiated after identification of risks (reduce schedule delays, cost savings, product quality, or customer satisfaction) b. Performed throughout the project's lifecycle. c. Risk mitigation plans should be reviewed regularly (weekly or monthly) 1.

Run chart

Run chart a. Best tool for showing the stability of a process: i. Oscillation = process lacks steadiness ii. Trend= upward/downward pattern due to tool wear, gradual temperature changes, or loosing of fixture. iii. Mixtures=process points appearing on either side of the chart with nothing close to centerline→ mix-up between two different machines, two different operators, or two lots of materials. iv. Clustering=measurement problems

Interpretation of Gage R& R

Several points lie outside the control limits in the sample= samples cover the spread of the process specification. -→ Must verify if it is a recording error or variation due to special causes. h. Best to have the points within the control limits i. Spread of measurement points & outliners will display whether an appraiser is consistently measuring higher or lower than others. 14) Sources of Measurement Variation:

Term: Techniques for rapid changeovers of production machinery. Long-term objective is zero setup time so that changeovers do not interfere with continuous flow. Ten minutes is the common intial objective

Single Minute Exchange of Dies (SMED) Techniques for rapid changeovers of production machinery. Long-term objective is zero setup time so that changeovers do not interfere with continuous flow. Ten minutes is the common intial objective

Steps in Process Analysis:

Steps in Process Analysis: 1) Fishbone diagram used to categorize the causes of process variation 2) Documentation of the causes/ideas from brainstorming session. 3) Convergence tools (NGT, multivoting) are used to condense the list of ideas into those mostly likely to have impact. 4) Affinity diagram used to organize large number ideas into their natural relationships

Steps in the Benchmarking process:

Steps in the Benchmarking process: a. Flowchart of current process (ensures understanding of how current process work before benchmarking) b. ID the areas for improvement c. Brainstorming d. Investigate how others (internal & external) perform similar processes e. Plan for application (list of required resources, timelines, and location of new processes plus training needed to implement changes) f. Pilot test ideas (aids detection of defects) g. Initiate new processes h. Evaluate new processes.

Summary of 8 events occuring in Deming's Chain reaction (Delavigne and Robertson)

Summary 8 of events occuring in Deming's Chain reaction (Delavigne and Robertson) 1. Quality and productivity rise 2. Costs decrease 3. Time required for development and production reduced 4. Management begins to know their cost, "they have a system" 5. Increased division of labor and specialization occurs 6. Near-term future is more predictable 7. Standard of living rises 8. System has a future and can provide "jobs and more jobs"

Team Dynamics & Discounts

Team Dynamics & Discounts Discounts a. Symptoms= Sarcasm, low team morale, members' opinions ignored, members don't listen to each other b. Solution= encourage respect, discipline, and clarification of opinions.

Team Leadership : Forming

Team Leadership Team leadership varies depending on the matureing of the team and the state the is at based on the leader's perception. 1) Forming-→ Directing is the leadership style a. Close supervision b. Leader identifies opportunities for developing skills to meet team goals c. Leader listens to feedback from team and encourage team members

Team Leadership: Storming & Performing

Team Leadership: Storming & Performing Storming-→ Coaching style of leadership a. Closer supervision with directive behavior b. Increased listening to solicit team feedback 4) Performing-→ Delegating leadership style with performance reviews

Team Problems & Solution----Dominant Participants

Team Problems & Solution----Dominant Participants Examples/ Solution 1) Members interrupt each other---> Promote equal participation 2) Members dominate discussion ---> Structure the discussion

Team Problems & Solution---Floundering Examples and Solution

Team Problems & Solution---Floundering Examples/ Solution: 1) Unclear team direction ---> Leader provides clarity 2) Postponed decision----> Ask how can we proceed? 3) Overwhelmed members---> Revisit team purpose

Test Statistic

Test Statistic 1) Calculated value that is compared with the critical value (p-value) when testing the null hypothesis.

Theory of Constraints terms---Inventory, Throughput, & Operational Expenses

Theory of Constraints terms---Inventory, Throughput, & Operational Expenses 1. Inventory= (aka sold patents or sold investments)---money that is stuck in the system with the intention of being sold 2. Throughput- "rate at which the system generates money via sales; incoming money"--finished goods must be sold to get money 3. Operational expenses--" money going out; money spent to turn inventory into throughput"---includes depreciation, lubricating oil, scrap, carrying cost, ect.

Theory of Constraints terms--Key Principles

Theory of Constraints terms--Key Principles 1. Many key principles, according to Goldratt, have significance because of the management approach used in adopting organizations. The principles include: 1) Processes/organizations as chains--->If processes % organizations function as chains/ flows, possible to find & strengthen the weakest links --weak links can be present a) Between the different steps b) In activities in a process c)Between diverse organizations within a supply chain. 2) Local versus system optima---> Interdependence and variation cause the optimum performance of a system as a whole to be different from the sum of all the local optima. In other words, an organization that maximizes the output of every machine will not perform as well as one that ensures optimization of the flow of materials and value created through its linked set of activities. 3) Cause and effect---> All systems operate in an environment of cause and effect. Capturing the essence of cause and effect within the system and identifying measurements that emulate these relationships are the keys to optimizing system performance 4) Physical versus policy constraints----> Most of the constraints faced in systems originate from policies, not physical things. a) Unlike the easily identifiable and repairable physical constraints, Policy constraints (e.g behavior patterns, attitudes, lack of information, and assumptions) hard to identify & more damaging than physical constraints--> Example or Policy Constraint=The belief that producing in large batches is optimal can hinder in implementation TOC. 5) Total system impact---->All organizations are system of interdependent activities, each with its own level and type of variability In order to optimize performance, management needs to understand and focus on the total system impact of a decision or event, not just

Process Performance Measures: Calculating the Yield of Sigma Process.

To calculate yield of Sigma process, must determine the area under the normal distribution curve that is + or - 3 standard deviations and gives a yield of 0.99973 3. If the process variation is reduced and sigma equals 0.002 and there is + or - 6 sigmas between the tolerance limits, the process is 6sigma process. 4. Defect is 1- 0.9999966-→ Defect level= 0.0000034 or 3.4 ppm 5. The Six Sigma approach is concerned with the number of sigmas that will fit between the process mean and the nearest specification limit. A process that fits 6 sigmas between a mean and its nearest specification limit in the short term will experience a 1.5 shift that will result in only 4.5 sigmas fitting.

Two calculation used to identify how capable a process is:

Two calculation used to identify how capable a process is: i. Capability index (Cp) ii. Process performance (Cpk)

Two types of quantitative data:

Two types of quantitative data: a. Continuous (variable) data-→ measurement on continuous scale and have infinite number of values(temperature, length, weight) b. Discrete (attributes) data-→ result from counting the occurrence of events (eg. Number of bubbles)

Types of Attribute Control Charts

Types of Attribute Control Charts P-charts i. Control charts dealing with the proportion or fraction of defective product within the group under review ii. Pieces either conform or are rejected-→ Rejected parts are expressed as a portion of the sample size. iii. Best used for binary data (each item is one of the 2 categories) iv. Also used for comparisons across conditions rather than over time. v. A p control chart is used to look at variation in yes/no type attributes data. There are only two possible outcomes: either the item is defective or it is not defective. The p control chart is used to determine if the fraction of defective items in a group of items is consistent over time.

Types of Experimental objectives:

Types of Experimental objectives: i. Comparative -conclude if the factor is significant ii. Screening -select a few important main effects iii. Response Surface ---find optimal settings and weak points of processes. iv. Mixture—determines the best proportions

Variation in Process capability

Variation in process capability can be a. Physical or Mechanical (tool, machine, equipment, environment, maintenance) b. Procedural (workload, operator, accuracy, legibility) c. Two types: Special & Common Cause

Voice of Internal Customer

Voice of Internal Customer---Survey can establish a communication tool in gathering data such as: 1) Stat of company--Employee's perception of the state of the company 2)State of quality-- Whether quality efforts are worthwhile 3) State of Process--are there improvements 4) Reacting to policies 5) Rating of company satisfaction ---Good place to work? 6) Rating of job satisfaction.

Who develops the Project charter?

Who develops the Project charter? Can be Developed by EITHER: 1) Top Management OR Team 2) Either way, Top Management's endorsement is critical for giving the team the support and direction it needs. Why Top Management's endorsement important? ---> 1) Teams need to know what top management expects of them. 2) Team must have the authority, permission, and blessing of each level of management to operate, conduct research, implement changes to achieve results

You can minimize constraints and work more efficiently toward accomplishing your goals by working through these 3 steps:

You can minimize constraints and work more efficiently toward accomplishing your goals by working through these steps: a. Identify the constraint. b. Manage the constraint. c. Evaluate performance.

Formal and Informal Communications

Formal and Informal Communications 1) Formal communication= official company sanctioned methods of communicating to employees. 2) Informal communications----Links the company to grapevine, rumor mill, ect.---> clarification about the latest policy can be made (tranfer or retirment)

Function of Flowcharts:

Function of Flowcharts: a. Illustration of machine or process that needs improvement b. Monitoring a process over time c. Training of new operators & supervisors d. Visual standard to ensure that thing are running correctly.

Gage R&R for destructive testing----Types of Challenges:

Gage R&R for destructive testing --- the part or sample is altered or destroyed during testing. Gage R&R for destructive testing----Types of Challenges: 1. One-sided specification 2. Skewed distribution 3. Fully automated equipment with no or minimal appraiser interaction 4. Destructive testing 5. New product introduction where only a few units are available 6. Destructive testing 7. New product introduction where only a few units are available 8. Multiple station comparison 9. Equipment that requires resetting or calibration after every measurement 10. Incomplete GR&R (units shipped during the study due to urgency)

How are sample size chosen in Rational subgrouping?

How are sample size chosen in Rational subgrouping? 1) Samples size is influenced by A) Desire to permit minim chance of variation WITHIN subgroups b) Large sized subgroup has greater chances for process change WITHIN subgroup.

How are samples selected in Rational Subgrouping?

How are samples selected in Rational Subgrouping? 1) Choose sample size frequency such that if a special cause occurs a ) between-subgroup variation is maximal b) within-subgroup variation is minimal.

Improve Stage of DMAIC

Improve (study the data to change the process for improvement) 1. Organizational development 2. Variation reduction 3. Problem solving 4. Brainstorming 5. FMEA

Juran--Basics for Success in Quality

Juran--Basics for Success in Quality *** Prime basic belief is that America had improved but needed to improve at revolutionary rate by doing: 1. CEO's must serve on quality council 2. Specific quality improvement goals must be in the business plan and include: a. Means to measure quality results against goals b. Review of results against goals c. Reward for superior quality control 3. Responsibility for improvements must be assigned to individuals 4. People must be trained for quality management & improvement 5. Workforce must be empowered to participate in the improvement process.

MX-MR charts

MX-MR charts i. An individuals and moving range (X-MR) chart is a pair of control charts for processes with a subgroup size of one. ii. Used to determine if a process is stable and predictable, iii. It creates a picture of how the system changes over time. iv. The individual (X) chart displays individual measurements. The moving range (MR) chart shows variability between one data point and the next. Individuals and moving range charts are also used to monitor the effects of process improvement theories

Machine Capability

Machine Capability 1) Short-term capability measurement that is used to determine the inherent process (machine variation) by EXCLUDING elements like batch-to-batch, stream-to-stream, and time-to-time variation and by MINIMIZING measurement factors (operator & equipment), piece-to-piece variation, and within piece variation. 2) Control charts can be used to determine machine capability if the process is free of drift, depending on how the information will be used for improvement

Matrix Diagram

Matrix Diagram 1) Used to show the relationship between objectives and methods, results and causes, tasks and people, ect. 2) Must determine the strength of the relationships. 3) Result of two or more tree diagrams can be meshed into matrix

"Operational procedures

Operational procedures 1) Operational procedures uses humans to execute operational procedures so more errors (similar to engineering process control, except that machines execute operational procedures). 2) Transactional processes= operational procedures ( eg. standardizing the process). best and cheapest option a) Example of operational procedures--training bank tellers process deposits according to the same steps every time or separating orders by type. Many businesses streamline their responses to orders by dividing orders into several key categories. 3) Another way to prevent deviation with operational procedures ---->establish processes for diverting resources during urgent needs. 4) Operational procedures require careful observation and should be documented on process maps, flowcharts, or written summaries."

Theory of Constraints terms--Bottleneck

1. Bottleneck Resources= resources whose capacity is equal or less than the demand place upon it. a) To relive bottleneck---> offload the material to relieve bottle neck or can make bottleneck work only on needed parts. b) Beware of lost production at bottleneck, due to poor quality or rejects. If the bottleneck stops for 1 hour, you actually lost 1 hour of total plant capacity. 2) Nonbottleneck---> Resource whose capacity is greater than demand----->A non-bottleneck machine should not work faster than needed for immediate production, because that will just pile up inventory. If you have excess production capacity, you must waste it rather than use it.

3 reasons why management may get misleading information

3 reasons why management may get misleading information: 1) In vertical communication, subordinates may withhold or screen out negative information resulting in only positive to be reported. 2) Tendency to tell a supervisor, especially an authoritarian type, what he or she wants to hear, shielding the negative---> concealed errors and avoidable delays and costs. 3) Incumbent manager or leader is not always surrounded by allies---> competition and rivalry.

5 Marketplace responses to the firm's and 8 events underdoing Deming's chain reaction

5 Marketplace responses to the firm's and 8 events underdoing Deming's chain reaction 1. Customer gets reduced prices 2. Increased cooperation 3. New products and services given to customer 4. Higher levels of customer satisfaction 5. Reduced competition for market share. ** Chain reaction of good things can occur through Deming's philosophy.

5S--Scrub

5S--Scrub 1) Includes ways to continue keeping things clean 2) Commitment to be responsible for all working conditions 3) Perform Root cause analysis and remedy machinery and equipment problems 4) Complete training on basics of equipment maintenance 5) Divide each area into zones and assign individual responsibility 6) Rotate difficult or unpleasant jobs or implement 3,5, and 10 minute 5S activities 7) Inspection checklists and perform white glove inspections.

7 Key Project Management Elements

7 Key Project Management Elements 1) ID schedule time limits 2) Allocation of functional responsibilities 3)Establishing continuous reporting methods 4) Selecting Applicable tradeoff methodologies 5) Measuring Accomplishments against plans 6) Identifying problems and applying corrective action 7) Improving capabilities for future project.

Appraisal Cost Examples

Appraisal Cost Examples 1) Inspections of Prototype, shipping& receiving 2) Test Equipment maintenance 3) In-process inspections 4) Equipment Calibrations 5) Expense Reviews 6) Lab testing

Binomial Distribution

Binomial Distribution Basic assumptions: Discrete distribution Number of trials are fixed in advance Just two outcomes for each trial Trials are independent All trials have the same probability of occurrence Uses include: Estimating the probabilities of an outcome in any set of success or failure trials Sampling for attributes (acceptance sampling) Number of defective items in a batch size of n Number of items in a batch Number of items demanded from an inventory

DFSS --- Systemic Design 4 main design phases

DFSS---- Systemic Design: a. German design process methodology that uses a rational approach and structure to produce valid solutions b. 4 main phases of Systemic Design: 1) Task Clarification ---> collect information , fomulate concepts, identify needs 2) Conceptual Design--->identification of essential problems 3) Embodiment design---->design concepts, layouts, refinements 4)Detail Design--->Finalize drawings & concepts and then generate documentation. *****Abstract concept is turned into concrete item shown by drawing. Synthesis needs the act of combining parts/element sto make a new form

Which of the following is a component of an attribute (data) sampling plan? Unit is inspected by its proximity to specifications Inspection method is more sophisticated Items are classified as Percent Defective Smaller sample size can be utilized

Defective or not defective is an example of attribute data, not variable data. The utilization of a smaller sample size is a component of variable sampling plan. Each unit can be weighted by its proximity to specifications in a variable sampling , whereas for attribute data all defectives are equal. Inspection method is a component of variable sampling plan because each item has a measurement, as opposed to attribute data where go/no-go gages are employed.

Define phase of DMAIC has 2 purposes:

Define phase of DMAIC has 2 purposes: a. Define the project management process b. Define the problem/issue to be work on by the GBS team

A continuous improvement team has come together to brainstorm ideas for a new product line. Which of the following techniques will likely reduce the effectiveness of the brainstorming session? Discussing ideas as they are presented Avoiding judgment on ideas Recording all ideas Letting people be freewheeling

Discussion can hinder the free flow of ideas. Discussion should come after many thoughts are generated.

Ensuring Data Accuracy & Integrity

Ensuring Data Accuracy & Integrity Data should not be removed from a set without an appropriate statistical test or logic. Generally, data should be recorded in time sequence. Unnecessary rounding should be avoided. If done, should be late in the process. Screen the data to remove entry errors. Avoid emotional bias. Record measurements of items that change over time as quickly as possible after manufacture and again after the stabilization period. Each important classification identification should be recorded alongside the data. (Ex. Time, machine, operator, gage, lab, material, conditions, etc).

Examples of processes suitable for monitoring with a u-chart include:

Examples of processes suitable for monitoring with a u-chart include: Monitoring the number of nonconformities per lot of raw material received where the lot size varies Monitoring the number of new infections in a hospital per day Monitoring the number of accidents for delivery trucks per day

Hypothesis Tests for Means F-test

Hypothesis Tests for Means----F-test a. The F-test is designed to test if two population variances are equal. It does this by comparing the ratio of two variances. So, if the variances are equal, the ratio of the variances will be 1. b. This ratio of sample variances will be test statistic used. If the null hypothesis is false, then we will reject the null hypothesis that the ratio was equal to 1 and our assumption that they were equal.

Importance of design of data collection & feedback systems requires decisions on:

Importance of design of data collection & feedback systems requires decisions on: i. Points where data should be collected (control chart) ii. Plan for improvement (if stable processes found to be incapable, then need designed experiments)

List the modern motivational theories that support the concept of teams

List the modern motivational theories that support the concept of teams 1) Maslow's higher level of human needs 2) McGregor's Theory Y---recognizes the individual worth 3) Herzberg's theory of motivation

Matrix Diagrams

Matrix Diagrams 1) Used to collect data on customer defects or complaints and then create Pareto chart or select a project--> Must note the number of occurences per fixed time

Two Types of Value Stream Maps?

Most value stream maps have two versions: a current state and a future state. 1) Future state map---- depicts the process after planned improvements have been made. (less complicated than the current state). 2) Current state value map --- examined for steps that do not non- value added.steps to be eliminated or minimized."

Nested Experiments

Nested Experiments 1) Experimental design is which all trials are not fully randomized, usually due to logical reason for taking the action. 2) Example----> technicians may be nested within labs. As long as each technician stays with the same lab, the techs are nested. It is not often that techs travel to different labs just to make the design balanced.

New Product Term:--New Category entries

New Product Term:--New Category entries Company product not new to the world but new to the company

Objective of process capability studies:

Objective of process capability studies: i. To determine if the process is in statistical control ii. To determine if the process is capable of meeting specifications. e. If the process is not capable-→ take action to improve capability. f. If the process is capable→review the percent nonconformance outside the specification limits.

PERT----Expected Time between Events 6 key points

PERT----Expected Time between Events: 1) Node = End/ Starting Pt for a group of activities = circle with event number inside----Two or more activities that merge at a common nodal point. 2) Time between events = arrow with number indicating time duration---Fig. >0 3) Expected Time between Events = = [(Optimistic time) + (4 *Most likely time) + (Pessimistic time) ] / 6 4) Burst Point = more than one task emanates from a common point. Fig 0 < 5) Projects not on the critical path may be delayed by an amount equal to the slack time without delaying the completion of the project 6) Task that end late may delay the project but also can modify the remaining tasks' critical path. 7) Critical Path- path with the longest time

Risk Prioritization Number/ Matrix

Risk Prioritization Number/ Matrix 1) FMEA generates RPN associated with failure mode in the design PROCESS. Higher RPN, the more serious the failures that needs to be addressed in the design phase of the early engineering efforts. 2) FMEA evalates failures in 3 ways: severity, occurence, & detection. 3) Helps determine the potentially highest-risk items to aid project team in prioritizing the items to work on most aggressively. Aid in resource allocation in that higher RPN are addressed first and lower RPN addressed only if resources available.

Second- Order

Second- Order 1) Refers to the power ot which one or more factors appear in model 2) Second-order models can account for cuverature and interaction. B12 (X1 * X2) is an example of 2nd order, representing interaction between X1 and X2

Strategic Benchmarking

Strategic Benchmarking 1) Examines how companies compete and is rarely industry-focused. 2) It moves across industries to find winning strategies

System FMEA

System FMEA 1. Deals with systems, components and the interaction between systems and the elements of the system---via a balanced approach to performance, costs, and effectiveness. 2. Inputs derived from Quality Function Deployment---Voice of Customer a) QFD translates the "voice of the customer" (VOC) into functional requirements for the product. b) QFD helps transform customer needs into engineering characteristics for a product or service, prioritizing each product or service characteristic while simultaneously setting development targets for a product or service. c) Through a planning matrix named the "House of Quality" (HoQ), QFD transforms customers wants into designs, manufacturing processes and production control requirements. The transformation of customer requirements will help develop part characteristics, process requirements and product standards necessary for the product development phase 3. Potential cause of failure can be gleaned from: a) Warranty claims b) Customer Complaints c) Field Service Data d) Reliability Data e) Feasibility Studies.

Virtual Teams

Virtual Teams 1) Works best if all the team members are familiar with each other 2) Key benefit= reduced coasts, real-time data sharing & updating. 3) Drawbacks a. Slowing of the progression of normal team building b. Miscommunication c. Lack of true commitment and buy-in

Process Capability (Learning)

Process Capability (Learning) a. Specification limits and process variation both reflect the voice of customer b. Process capability measured by capability index (Cp) and process performance (Cpk)

PFMEA & Product Classifications

PFMEA & Product Classifications: Characteristics of a Product can be Classified as: 1) Critical---> shall comply with safety/ government regulation or service requirements 2) Significant---> customer & supplier quality features 3) Key---> characteristics that provide prompt feedback on the product aiding in corrective action process ***AIAG FMEA has 2 design columns---one for prevention and one for detection a) Detection ranking for control used for RPN calculations b) AIAG FMEA---> consideration is for internal and external customers with 2 sides listed in the severity ranking tables.---> Emphasis on recommended action placed on reducing high severity rankings followed by high RPN

If a company produces 100 compressors and 5 compressors have 2 defects each, what is the defects / unit (DPU) produced? A) 0.10 B) 5 C) 95 D) 0.01

(5 compressors * 2 defect each) / 100 produced = 0.10

If a company produces 100 compressors and 5 compressors have 2 defects each, what is the defects / unit (DPU) produced? asq/learn_check.gif 0.10 5 95 0.01

(5 compressors * 2 defect each) / 100 produced = 0.10 Defects per unit is determined by total number of defects divided by the total number of products produced in some time period.

Functions of Checklists:

Functions of Checklists: a. Precategorize potential outcomes for data collection b. Allows identification of patterns c. Aids in keeping focused on continual improvement and may foster changes

Fundamental purpose of establishing teams?

Fundamental purpose of establishing teams? 1. To improve internal & external company efficiencies via team effort to improve quality, methods, & productives

Gantt Chart

Gantt Chart 1) Graphical display of projects activities and their duration + project milestones+ task relation between predessor and successor task 2) Quick look for schedule conflicts of tasks 3) Assessment of key resources of the plan and the project performance against the plan.

Tools for Business performance measures: Process Performance Measures:

Process Performance Measures: a. Defects per unit (DPU) b. Defect per million opportunities (DPMO) c. Rolled throughput yield (RYT) d. Sigma Levels e. Process Capability Indices

Define RPN

RPN a. (Severity) (Occurrence) (Detection)= RPN b. Criticality= (Severity) (Occurrence) c. Cutoff score is assigned after RPN assigned. RPN is not applicable to another organization. i. Too low Cutoff-→ must use resources to eliminate several risks ii. Too high cutoff→ failure to address important risks d. Management must agree on the cutoff

Regression Analysis

Regression Analysis 1) Identifies when independent variables are influenced by one or more dependent variables 2)Measure Stage----> Evaluates the degree to which a measurement system is linear 3) Analyze Stage---->Used to explore connections between metrics and process factors. 4) Improve Stage----> Used to connections after improvement have been made

n the hypothesis test for means, if the test statistic is in the reject region, then: Accept the null hypothesis. Reject the mean. Do not reject the null hypothesis. Reject the null hypothesis.

Reject the null hypothesis.

Repeatability and Reproducibility

Repeatability and Reproducibility 1) Assuming gate accuracy and sensitivity is assured--------->

Run Charts--Control Charts for Variables

d. Run Charts-→ for single point data

Problem Statement

Problem Statement 1) Details the issues that team aims to improve 2) Should include a baseline measure at the start of the project as well an anticipated completed date, 3) If the baseline measures differ from the team's assumptions, clarification is needed. or more data

"The Goal" reminds readers that there are 3 basic measures to be used in the evaluation of a system---List them

"The Goal" reminds readers that there are 3 basic measures to be used in the evaluation of a system---List them 1. Throughput 2. Inventory 3. Operational expenses These measures are more reflective of the true system that impact: a) Machine Efficiency b) Equipment Utilization c) Downtime d) Balanced Plants

Theory of constraints

"Theory of constraints 1) Approach to improving processes by focusing on their most problematic areas ( constraints). 2), Theory of constraints encourages businesses to improve continuously by diagnosing weaknesses and addressing them. a) No end to work -----=>once one area improved, must be address another constraints. b) Exploiting the constraint= term for develop effective strategies for maximizing the potential of the constrained area.

Assume two events A={1,2} and B={3,4,5} in rolling a fair dice with six possible outcomes S={1,2,3,4,5,6}. Select the best option. Events A and B are mutually exclusive and dependent Events A and B are independent. Events A and B are exclusive but not dependent. Events A and B are dependent but not exclusive.

...Events A and B are mutually exclusive and dependent

DFSS vs. DMAIC?

1) Unlike DMAIC, DFSS emphasizes defect prevention rather than reduction. 2) Also, DFSS is more concerned with the creation of new, high-quality processes than with the incremental improvement of old processes. 3) IDOV is the methodology most commonly associated with design for Six Sigma, or DFSS.

"Value stream mapping

1) Value stream mapping is a communication tool for Six Sigma professionals. It depicts the flow of materials and information through an organization. 2) Value stream maps are effective when they identify waste and redundancy in processes. Each activity in a process should be represented with a shape on the map. 3) each activity should be accompanied by the following information: cycle time, down time, in-process inventory, path of information, and path of materials.

22. Which type of human error is typically limited to a particular task? a. Willful b. Inadvertent c. Technique d. Selective

22. C: Technique. Technique error is typically limited to a particular task. Six Sigma experts identify three categories of human error: technique, inadvertent, and willful. Technique errors are the result of a lack of comprehension or poor training. It is more likely that technique errors will occur on difficult tasks. Inadvertent errors are slightly different, because they occur by accident even when an employee is experienced and understands the task. It is impossible to eliminate entirely inadvertent errors so long as there are human operators. A willful error is made intentionally by an employee. The best way to reduce willful errors is to maintain high morale and incentivize high performance."

A Type I error in hypothesis testing

A Type I error in hypothesis testing is the probability of rejecting the null hypothesis when it is true. In this example, it is the probability that research will show that the knee joint replacement is causing nerve damage when it is not. Instead of accepting the null hypothesis of no damage, the researchers make a mistake of rejecting it and concluding that the replacement does cause damage.

Formal Surveys

A formal survey a) Gather information from a more representative sample of customers than focus groups, b) Like focus groups, surveys are often returned by the same highly motivated customers (very unhappy customers) c) Surveys are more effective when they ask customers for specific information about their preferences."

A Green Belt wishes to determine the primary pattern (positional, cyclical or temporal pattern) of variation in a process or product. Which of the following techniques would be most useful? Full Factorial Multi-Vari Analysis of Variance Orthogonal Array

A multi-vari design is used to identify variation related to positional, cyclical and temporal factors in the process/product. ANOVA does not break analysis down into positional, cyclical and temporal contributions.

AIAG Reference: Variation in Sample parts

AIAG Reference: Variation in Sample parts = the selection of sample parts from the process to represent the entire production operating range when : i. Independent estimate of process variation is not available ii. Determination of process direction and continued suitability of the measurement system for process control.

Which communication method is most effective when handling sensitive issues? An email A luncheon meeting In person A team meeting

Any sensitive communication issue must be handled privately and in person. Luncheon meetings are good for bottom up communication. Team meetings are good for horizontal communication Email lacks the ability to communicate emotion and does not allow for two-way communications. Close

Armand Feigenbaum---8 TQC Principles

Armand Feigenbaum---8 TQC Principles: 1. TQC is company wide process, all functions are involved 2. Quaility is what customer says it is 3. Quality and production costs are in partnership.....higher quality will equate with lower costs. 4. Both individual and team zeal are required 5. Quality is way of managing, providing a continuous and relentless emphasis on quality through leadership. 6. As a ethic, all of management must be involved in quality, not just the specialist. 7. Requires continuous improvement, the use of new and existing technologies 8. It is most cost-effective, least capital intensive route to productivity, and is implemented with both customers and suppliers.

"Contributions of Armand V. Feigenbaum to quality control"

Armand V. Feigenbaum of total quality management. Four key actions businesses needed to take in order to implement TQM programs: 1) establishing standards; 2) measuring conformance to these standards; 3) resolving issues hindering conformance; and 4) continuously planning for improvement. Feigenbaum was one of the first quality specialists to insist that quality standards be assessed from the perspective of the qualities for which a customer would be willing to pay.

Attributes of sample data: Shape of sample

Attributes of sample data: b. Shape of sample i. Smooth curve that serves as umbrella covering the tops of bars in the histogram ii. Shape: symmetry, skewness, and kurtosis. iii. A fundamental task in many statistical analyses is to characterize the location and variability of a data set. A further characterization of the data includes skewness and kurtosis.

Theory of Constraint terms-- Balanced Plants

Balanced Plants & TOC: 1. According to TOC, it is good if non-constraint operations have a greater capacity and therefore have some nonproductive time during the workday 2. If this is not the case, the plant is "too balanced" and will not operate effectively---->. In other words, the plant should be unbalanced with greater capacity at all operations except at the designed constraint. Then the rest of the plant can be properly scheduled and raw materials fed into the process based on that schedule 3. If the plant is balanced, the constraint will appear to move around during the day. a) To properly operate, the plant must be unbalanced to the extent that the constraint is obvious and consistently located. b) Non-constraint should never be fed with work just to keep it busy. That does not increase throughput and only serves to add unneeded work-in-progress (WIP) inventory 4) Central idea of TOC----> Make flow through the bottleneck equal

Tools for Business performance measures: Balanced Scorecard

Balanced Scorecard a. Financial aspect of measurement is restricted to past events b. Examines 4 areas of performance (finance, customer, internal processes, and learning & growth) c. Aids organization to line up with the vision & goals in a "Balanced" way.

Basic Component of Control Plan

Basic Component of Control Plan 1) Information related to error prevention and detection efforts, which are derived from FMEA and DOE 2) Inputs that need to monitored and controlled 3) Basic information about the specification ----objective range within which the characteristic could tolerably fall. 4) Indicates ow the characteristics will be measured and the frequency & quantity of samples collected. 5) Reaction rules---response protocols when the characteristic does not meet the specifications.

Batching

Batching 1) Batch size is traditionally set to optimize the time of a single step of production against the time of setting up for production---> Local optimization does not relate to overall optimization. 2) If the machine in question is a non-bottleneck, then you have excesses capacity and excess time. a) Saving setup time by optimizing batch size, is an illusion---> Time for each batch is added at each step in the process. a) Half the batch size means it gets through the entire process twice as fast. ----> This means that smaller batch sizes can mean faster time to market, cutting lead times, and being more responsive.

Benchmarking

Benchmarking 1) Process of looking at one system and applying another system's concepts 2) Can be done internally or externally 3) An important aspect of initial project definition is to set KPIs that track performance across a number of areas such finance, operations, and human resources. Project teams can set KPI figures that compare against previous company projects (internal goals) or those of external competitors.

Benchmarking Sequences

Benchmarking Sequences 1) Determine Current Practice ---> Select problem area, Id. key performance factors, understand your own and others' processes, select performance criterial based on needs and priorities. 2) Id. Best Practices ----> Measure performance in the organization. find internal and external organization to benchmark, determine the leader in each criteria area 3) Analyze Best Practices -----> Visit the benchmark partner to collect data & info than evaluate/ compare current practices with benchmark 4) Model Best Practices ---> Drive improvement changes to advance current performance levels, extend performance breakthroughs within the organization, incorporate new information in decision-making, and share results with benchmark partner.

5 ways that Benchmarking is a Continuous Improvement Process

Benchmarking as a Continuous Improvement Process: 1) Measures the most relevant, specific attributes of its own products/services/ practices/ stratgies 2) Uses information to improve its own performance 2) Determines how those companies achieved their superior performance 3) Aims to perform at the level of the benchmarked process 4) Continuosly repeats itself

Benefits from control charting

Benefits from control charting 1) Can be used as a predictive tool when changes are required prior to the production of an out-of-tolerance material 2) Allow continuous monitoring of improvement efforts to see if the efforts were effective

Benefits of Teamwork to Team Members

Benefits of Teamwork to Team Members 1) Greater understanding of work issues 2) Chance for creativity and sharing ideas 3) Strengthening of working relationships 4) Chance to learn new skills 5) Full Support of project from upper management 6) Satisfaction of solving chronic problems--->more customers, revenues, & less expenses.

Best Approach to Management and Why?

Best Approach to Management and Why? Participative management style is the best approach because: a) Workforce of highly-educated workers desire to be part of the decision-making process b) Employees get motivated when challenged ****American comapanies in the area of team participation.

Best measurement of project's activity?

Best measurement of project's activity? 1) Project Time Line-= most visible yardstick of measurement of project activities 2) Quality Perspective----> both early & late projects have the opportunity for poor quality compared to on-time projects. ----> Early then possible corner-cutting and if late, then what errors created the delay.

Bias

Bias 1) Difference between output of measurement system and true value------->Lack of bias= Accuracy 2)**Know the formula for Bias 3) Bias is reported as a percent of process variation or percent of tolerance.-----> Negative bias (underestimated value); Positive bias (over-estimated bias) 4) Statistical significance of bias is tested using the t-distribution-----> increase the number of measurements increases the discriminatory power of the t-test.

Who invented 6 sigma?

Bill Smith's Yeild Theory--Motorola--invented 6 sigms

If two processes have the same value of Cp but process one has a Cpk value of 1.33 and process two has a Cpk value of 0.67, what can be said about these processes? The first is in control, the second isn't. The first is better centered than the second. The second is better centered than the first. The first is not capable, the second is.

Both processes have the same width but, based on the Cpk values given, the first is centered and the second isn't. (4 sigma to the nearest spec for the first, but only 2 sigma for the second.) The second process has a spec limit at 2 sigma from the average, while the first has the spec limit at 4 sigma from the average. A value of 0.67 identifies a process that is not capable.

Box- Behnken

Box- Behnken When full 2nd order polynomial models are to be used in response surface studies of 3 or more factors. Very efficient; highly factional, 3-level factorial design

Brainstorming

Brainstorming 1) Uninhibited technique for generation of creative idea when the best solution is not obvious a) Encourage free-wheeling, participation of everyone, and selection of proper meeting place b) Avoid ending brainstorming session too early and criticizing ideas 2) Commonly combined with fish-bone diagram

For activity X: Optimistic Time = 2 Most Likely Time = 4 Pessimistic Time = 5 What will be the estimated standard deviation for activity X? 0.25 3.83 0.16 0.50

CORRECT ! Estimated standard deviation means (Pessimistic Time - Optimistic Time / 6)= 0.5 Estimated duration means (Optimistic Time +4Most Likely + Pessimistic Time)/6= 3.63 Incorrect calculation (Pessimistic Time - Optimistic Time) / 12= 0.25 Incorrect calculation (Pessimistic Time - Most Likely)/6 = 0.16

A team needs a tool to illustrate anticipated problems and possible solutions. Which of the following tools would be the best tool for this team's need? Force field analysis Matrix diagram Process Decision Program Chart (PDPC) Prioritization matrix

CORRECT = A Process Decision Program Chart (PDPC) is a network diagram that is used to illustrate anticipated problems and identify alternative paths. A matrix diagram is used to identify relationships between two sets of elements. It is not useful in contingency planning. A force field analysis separates driving forces from restraining forces. It is used to assess the viability of a future state. Prioritization matrix is used to decide the relative importance of items in a list. It is not useful in contingency planning.

Calculating Process Performance verse Specification

Calculating Process Performance verse Specification 1) Process Capability-------->How do you know if your process is capable? Process Capability Pp measures the process spread vs the specification spread. In other words, how distributed the outcome of your process is vs what the requirements are. a) Note: use Pp & Ppk when you are initially setting up your process. After a process has reached statistical control, use Cp & Cpk. b) Process has 2 specifications; [(Limit (LSL) , (USL)] The difference between the two is the specification spread; sometimes referred to as the Voice of the Client. c)Process spread =distance b/w the highest and lowest value ---> referred to as the Voice of the Process. 2) Can A Process Meet Specifications? The answer is in the amount of variation in your process. If your process spread is greater than the specification spread, then the answer is no. However, if the process spread is less than the specification spread, then process variation is low enough for it to fit. 3) Calculating Process Capability (Pp) Pp = (USL - LSL) / 6* s : where s the standard deviation, or the 'fatness' or dispersion of the bell curve. 4) What is a 'Good' Process Capability (Pp) Number?====> Six Sigma wants a Pp of above 1.5 because that would reflect a process with less than 3.4 DPMO - the definition of 6 Sigma quality. 5) How do we come to that? Well, we want to have 6 sigmas (standard deviations) between the mean of the process and the LSL. Since a normal distribution is symmetric, that means we also want 6 sigmas between the mean and the USL. That's a total of 12 sigmas between the USL and LSL. In other words, USL - LSL should = 12 for us to reach 6 σ quality standards of 3.4 DPMO. See how that is reflected in the equation Pp = (USL - LSL) / 6* s ? Let's replace (USL - LSL) with 12: Pp = (USL - LSL) / 6* s = 12 σ / 6 * s = 2 σ / s Is the Process Acceptable? Ppk (Capability) Ppk is another performance index that measures how close the current process mean's proximity is to the specification limits. In other words, does this process deliver acceptable results? Process Mean close to USL If your Process Mean (central tendency) is closer to the USL, use: Ppk = [ USL - x(bar) ] / 3 s, where x(bar) is the Process Mean. Process Mean close to LSL If your Process Mean (central tendency) is closer to the LSL, use: Ppk = [x(bar) - LSL ] / 3 s, where x(bar) is the Process Mean. Interpreting Ppk Scores A Ppk of 1 means that there is "half of a bell curve" between the center of the process and the nearest specification limit. That means your process is completely centered. Pp, Ppk In Relation to Z Scores Ppk can be determined by diving the Z score by three. A z score is the same as a standard score; the number of standard deviations above the mean. Process Mean close to USL If your Process Mean (central tendency) is closer to the USL, use: Ppk = [ USL - x(bar) ] / 3 s, where x(bar) is the Process Mean. Process Mean close to LSL If your Process Mean (central tendency) is closer to the LSL, use: Ppk = [x(bar) - LSL ] / 3 s, where x(bar) is the Process Mean. Interpreting Ppk Scores A Ppk of 1 means that there is "half of a bell curve" between the center of the process and the nearest specification limit. That means your process is completely centered. Pp, Ppk In Relation to Z Scores Ppk can be determined by diving the Z score by three. A z score is the same as a standard score; the number of standard deviations above the mean.

Charts used to calculate the mean and standard deviation.

Charts used to calculate the mean and standard deviation. i. Histogram ii. Run charts iii. Control Charts iv. Scatter Diagram ***Any process can be monitored using SPC techniques. If you have some control of the process, you then make changes to improve the variation (range) or the move the target (average)

Define Checklists

Check Sheets 1) Tool for reviewing process during the execution phase. (can be basis for other analytical tools and can be incorporated into the attribute process behavior charts)

Classification of experimental designs: Fractional Factorials

Classification of experimental designs: Fractional Factorials ---- combination of factors and levels needed to run while resulting a close estimate to the full factorial

Classification of experimental designs: Latin square/ Youden square ---Investigates the primary factor while controlling the interactions and affects other variables. i. Latin Square is not a Factorial design

Classification of experimental designs: Latin square/ Youden square ---Investigates the primary factor while controlling the interactions and affects other variables. i. Latin Square is not a Factorial design

Classification of experimental designs: Mixture designs

Classification of experimental designs: Mixture designs---factor levels expressed as percentages. The property of interest depends on the proportions of the mixture components and not the amount.

Classification of experimental designs: Randomized Blocks-

Classification of experimental designs: Randomized Blocks---single factor when material or environment can be blocked.

Classification of experimental designs: Response surface

Classification of experimental designs: Response surface---contour diagrams or maps of how the factors influence the response and direction for optimization of variable settings

Collecting Customer Data

Collecting Customer Data 1) Business operations and process levels of business are data sources for collecting Customer Data---> Must consider the levels where the customers impact the business A) Business Level --->Shareholders & Top Managmenet are the customers---> Quarterly/ Annual Measurements conducted using ------>Financial Analysis Tools to collect data, such as stock price, market share, revenues, earnings, ROI, and ROA B) Operational level ---> Internal & External Customers are the primary customers---> Daily/ Weekly Measurements using---> Lean/ 6sigma Tools, operational analysis, or industrial engineering to collect data on overall process measures with focus on customer satisfaction and internal operational efficiency C) Process Level---> Only External Customers (suppliers, employees)---Measurements (hours or sec) depend on production rate---> Lean & 6sigma tools used to collect data on key process variables ***Employees are customer and should be surveyed regualarly on pay, training, job satisfaction, advancement fairness, company's interest in well-being, and treatment (respect or dignity)

Collinear

Collinear Occurs when 2 variables are totally corelated. One variable must be eliminated from the analysis of valid results.

Common Instruments to Gather Data

Common Instruments to Gather Data 1) Surveys --Questionare that can be self-administered or via interviews 2) Focus Groups-- 3 to 12 people explore topics for 1-2 hrs. 3) Face-2-Face Interviews -- 30 to 60 min. individual interviews 4) Satisfaction/ Complaint Cards 5) Dissatisfaction sources --complaints, claims, refunds, returns, repeat services, litigation, warranty work, misshipments, replacement, downgrades 6) Competitive Shopper-- CEO's may call their own offices to measure the ease of customer access.

9 Common features of Standards

Common features of Standards 1) Besst, easiest, and safest way to do a job 2) Preserves know-how and expertise 3) Provide a way to measure performance 4) Correct standards show the relationship between cause and effect 5) Basis for maintenance and improvement 6) Provides visual signs of how to do a job 7) Basis for training and autditing 8) Means to prevent recurrence of erors 9 ) Minimizes variability

Contributions of Kaoru Ishikawa

Contributions of Kaoru Ishikawa and Genichi Taguchi to quality control 1) Main promoters of total quality control in Japan, 2) Ishikawa Diagram ( Fish Bone) 3) Developing the quality circle, in which work groups rather than lone employees resolved problems. ****One of his most popular ideas was that employees should be involved as much as possible in quality management issues.

Control Stage of DMAIC

Control (monitor the system for gains) 1. Control plan 2. Mistake-proofing 3. Process behavior charts 4. Update lessons learned.

Conversion of Batching to continuous flow layout

Conversion of Batching to continuous flow layout 1. Continuous Flow Layout= production steps for single piece flow, without weakness in process are arranged as follows: a) Arranged in Sequence b) Arranged as U- shaped c) Arranged in a cell ** Continuous Flow layout requires complete reliability of operator and work of each station.

A team is tasked with deciding which option should be selected among several competing alternatives. Which of the following is the best tool for this team's need? Matrix diagram Process Decision Program Chart (PDPC) Force field analysis Prioritization matrix

Correct Answer = Prioritization matrix A Prioritization Matrix is used to decide the relative importance among options by assigning weights to each option and then evaluating them against a set of criteria. A matrix diagram will effectively uncover relationships between two groups of elements, but will not in and of itself lead to prioritization. A tree diagram's primary function is to break big ideas or issues into more detailed components for easier comprehension and action. Close

The implementation of a pull system is being considered. Which of the following is a requirement for a pull system? There is consistent demand for the item. The part is always in stock when needed. Demand for the item varies greatly. The item is rarely needed.

Correct! A pull system should be used when there is consistent demand for the item. Pull systems should be used when shortages have a significant impact on service levels. Pull systems should not be used when there is high variability in demand.

In the hypothesis testing of a normal distribution, what type of parametric test would be used when comparing one sample average to a historical average or target? z-statistic t-test Weibull Distribution Chi Square

Correct! A t-test is the correct test to use for comparing one sample average to a historical one in a normal distribution. Weibull Distribution is for reliability. The z-statistic is used for area under the curve. Chi Square is not for normal distributions.

When designing a cause and effect matrix a team should do which of the following? Number the scales for the process input and process output variables from 1 to 10 inclusively. Select a member to reduce the process input and process output variables to a reasonable number using his/her subject matter expertise. Use some deciding method to help the team identify the key process input and key process output variables for the row and column headers. Place all the brainstormed process input and process output variables in the row and headers respectively.

Correct! The first step is to brainstorm all the process input and output variables. After discussions with the team members, a multi-vote or other deciding process should be used to narrow the lists to the key candidates. Because this is a team, it is better to use all of the brainpower to help narrow the choices versus selecting a single member to make the final decision. If you place all the brainstormed inputs & output not rows and columns-----You are likely to get more input and output variables than are feasible to use in the cause and effect matrix. Completing it would become cumbersome and would discourage the team with the paperwork.

The first five measurements taken in sequence from a process are: 10 14 12 13 21 What is the average moving range? 3 2.75 3.75 -2.75

Correct! The moving range values are 4, 2, 1, 8. These average to 15/4 = 3.75.

All of the sample sizes are equal. Which of the following charts is the most appropriate to substitute for a p chart? I chart np chart c chart u chart

Correct! The np chart is appropriate when the statistic in question is the number of nonconforming items out of the total number of items inspected and the subgroup size is constant. The c chart is appropriate only when the statistic of interest is the number of occurrences of an attribute for a given area of opportunity, where the area of opportunity remains constant for all subgroups.

The number of imperfections on rolls of steel is being monitored on a u chart. Each day, the number of imperfections found and the number of rolls inspected is recorded. The data so far are as follows: Imperfections: Day 1 (5) + Day 2 ( 10) Rolls: Day 1 (2) + Day 2 (3)

Correct! ū is the sum of the imperfections divided by the sum of the rolls inspected. Sum or imperfections= 5 + 10= 15 Sum of the rolls= 2+ 3= 5 U-bar = 15/ 5= 3

Two machines are to be compared. A sample of 100 pieces of manufactured product is taken from each machine and their measurements recorded each shift. The most appropriate control chart to compare the results from the two machines over time is an: I chart MR chart Xbar and R chart Xbar and s chart

Correct! An Xbar and s chart is appropriate for the large subgroups. The part of the chart will compare the averages, and the s part of the chart will compare the standard deviations. An I chart is designed for individual measurements from a process in time (or space) order. The MR chart monitors the moving range (spread) between individual measurements. With such large subgroups (100), the R (range) values are not useful.

If a team is looking at ways to eliminate, combine, simplify, or change steps in a process, what type of analysis are they performing? Cost of poor quality analysis Critical-to-quality analysis Cycle time analysis Importance performance analysis

Correct!The four basic principles of cycle time analysis are: eliminate, combine, simplify, and change. mportance performance analysis studies customers' perceptions about both the importance and the performance of products or services. Critical-to-quality (CTQ) analysis is a way of studying the flowchart of a process to identify quality features or characteristics most important to the customer and to find problems Cost-of-poor-quality analysis is a way of studying a process's flowchart to identify potential problems

Review Graphs involving CPK

Cpk is a measure of the distance between the center of the distribution and the nearest specification limit. Clearly that is less than 3σ=.009. Therefore Cpk is less than 1. Cp is the ratio of the tolerance to the natural process limits. It appears from the chart that the tolerance is approximately .026(4.566-4.54), the process spread is 3x.009=.018. Therefore Cp>1.

f the 6σ spread for a process is equal to six and the process average is equal to sixteen, what should the lower specification limit be set at to ensure less than .135% of the process output is rejected? 13 12 15 30

Cpk must be greater than 1 to get the desired percent defects rate. sing the formula ZL = (process average minus Lower Spec Limit) divided by σ. LSL=process average-σZLUsing the normal distribution table Prob(z At 13 ZL is (16-13)/1=3. Using the standard normal table, this puts the probability of defects just at .135%. It needs to be lower than that.

Given an upper spec limit of 15.5mg, a process average of 10.35mg, and a standard deviation of 1.291mg, which of the following statements is true? The Cpk is 1.33. The process will produce .135% defective outside of the specification limit. The process will produce .26% of its output outside the spec limit. The specification limit is the same as the process limit.

Cpk= [USL - (X-BAR)] ___________ (3 * SIGMA) Cpk = (15.5 - 10.35) ------------------ = 1.33 (3* 0.1291)

Cumulative frequency distribution

Cumulative frequency distribution a. Provide insight into how often a certain phenomenon (feature) is below a certain value. b. Statistical technique can be used to see how likely an event like a flood is going to happen again in the future, based on how often it happened in the past.

Current Controls for FMEA

Current Controls for FMEA a. Various types of controls (failure detection, failure prevention, SPC, alarms, mistake-proofing, AQL)

Customer Feedback

Customer Feedback 1) Process of finding out customer's wants, needs, & perspective. 2) Live customer interaction during the use of product/service is the best method of customer feedback

Customer Identification

Customer Identification 1) Customer defines quality and sets expectations. 2) Customers---> may be external to the process (purchasers) or internal department/ receiver of the next operation. 3) Primary customer= highest impact on the process a) Customers that make us money b) Customers that contribute to the bulk of revenues

Customer Loyalty

Customer Loyalty 1) Accounts for the majority of the sale and profit growth 2)Customer retention leads to repeat sales 3) Credit card industry-->Break even point= 6 yrs due to bad debt costs and high marketing. 4) Trust, good service, and act of execution lead to to customer loyality.

DFSS Roadmap & Tollgates / Process

DFSS Roadmap & Tollgates / Process: 1) Stopping point within the flow of phases a) A thorough assessment of deliverables b) A thorough review of the project management plans for the next phase 2) Checklists---Summary statements of tools and best practices required to fulfill gate deliverable 3)Scorecards---Summary statements from specific application of tools and best practice

Data collection methods --- Reasons for data collection?

Data collection methods ---Reasons for data collection a. Legal/ regulatory b. Contractual requirements c. Analysis, improvement, & knowledge management

Deciding between u-chart and p-chart

Deciding between u-chart and p-chart With Attribute data, decide on what type of distribution the data follows. Binomial data takes on two values, usually "good" or "bad". If the sample size is constant, use an np-chart. If the sample size changes, use a p-chart. Poisson data is a count of infrequent events, usually defects. If the sample size is constant, use a c-chart. If the sample size changes, use a u-chart

Defect Elimination & Detection Techniques used in TPM (total productive maintenance)

Defect Elimination & Detection Techniques used in TPM (total productive maintenance) 1. Poka-yoka (error prevention) 2. Source inspection (error detection & correction) 3. Self-check (Proof-reading and correction of errors) 4. Successive Checks (checks by next process and to correct the next process)

Define Gantt Chart

Define Gantt Chart 1) Bar chart that only indicates an ambiguous description of how the project, as a system, reacts to changes

Define Pull Value

Define Pull Value 1. System where products are created in response to an estimated sales forecast and not based on customer demand

Deming's Contributions?

Deming a. Promoted changes in management attitudes and structure b. List of "Fourteen Points" 1. Drive out fear + remove barriers amongst staff+ eliminate the staff rating or merit system + On-the-job training.

Detection Methods used in Design FMEA

Detection Methods used in Design FMEA 1. Design intent comes from one of the following: a) QFD b) Feasibility study c) Risk-benefit Analysis. 2) Potential Causes of Failure are related to materials/ processes/ costs 3) Detection methods used with DFMEA: a) Simulation b) Mathematical modeling c) Prototype testing d) Design for Experiments e) Verification testing f) Product testing g) Tolerance Stack-up studies

FMEA---Detection Rating

Detection rating-------Ability to detect the occurrence of failure mode (Scale 1-10, higher number=low dectectability)

In an effort to streamline their processes, the manager of a dry cleaning chain has developed a SIPOC diagram. Which of the following components would be considered an input to the dry cleaning process? A) Detergent and cleaning instructions B) The washing of the clothes C) The detergent vendor D) Clean clothes and invoices

Detergent and cleaning instructions are considered inputs to the dry cleaning process.

Develop Win-Win Relationships

Develop Win-Win Relationships---> via negotiation sessions to build trust between the 2 parties, provide feedback and reinforce commitment.

Developing Sampling Plans

Developing Sampling Plans: **Sample plan depends on purpose and customer requirements for the study 1) If the process is currently running and is in control-----> use control chart to calculate the process capability indices. 2) If it is project proposal for a new process---> use a pilot run to estimate process capability ----> Drawback= estimated process variability is less than the process variability of an ongoing process. 3) If trying to improve the process---> use Design of Experiments to get the optimum values of the process variables which yield the lowest process variation

Difference between Risk Analysis and Risk Management

Difference between Risk Analysis and Risk Management" Risk Analysis is part of larger methodology called Risk Management. Risk management is more inclusive process of dealing with risk and is divided into 4 phases (risk planing, Risk Assessment, Risk Handling, & Risk Monitoring )

Difference between Statistical Process Control (SPC) and Statistical Quality Control?

Difference between Statistical Process Control (SPC) and Statistical Quality Control? Statistical Process Control (SPC) a. Application of statistical techniques for measuring and analyzing the variations in the process. Statistical Quality Control (SQC) a. Use of statistical techniques to measure & improve quality.

Team Dynamics: Digressions & Tangents

Digressions & Tangents a. Symptoms= distractions, inefficient meetings, off-topic discussions, unfruitful meetings b. Causes= major organizational changes and lack of focus from leadership. c. Solution= redirect discussions, restate meeting ground rules, and enforce compliance to agenda items and time allotment.

Batching Disadvantages

Disadvantages of Batching: 1) Batching (reduces waste)= groups of identical tasks performed simultaneously 2) Six Sigma = batching increases overall cycle time but does create delays at the beginning & end of task. 3) Shorter time requirement if the process tasks are divided into batches, 4) Batching= product or service may be ready to move on for quite a long time, but may have to wait until the other products or services have been processed.

Discuss the identification of process metrics during the Measure Stage

Discuss the identification of process metrics during the Measure Stage: 1) Factors critical to schedule (CTS) ----> impact project completion date 2) Factors critical to quality (CTQ) ---->Direct effect on the desired characteristics of the product/ service. 3) Factors Critical to Cost (CTC)----> impact materials, labor, delivery, overhead, inventory and/or cost to the consumer of the good or service. 4) Metrics should be customer-focused and transparent; that is they should be objective 5) Metrics need to repeatable and reproducible

Document Archieving

Document Archieving 1) Final state of project and includes a) Test data b) Traceabilty of materials c ) Key project variables d) Consideration of duplicate copies at different sites e) Use of medium with a life longer than record retention period

Downward Flow of Communications & typical types of information

Downward Flow of Communications 1) The flow of information from team leaders/ managers to team members/ peers 2) Type of Information : a) Staff and team instruction s b) Rationale for instructions to team members c) Policies and procedure of the company d) Team member performance feedback 3) Key disadvantage= Filtering of information or distortion as it is passed down.

Drawback of Control Charts over SPC

Drawback of Control Charts over SPC 1) Given the expenses of SPC, one may be tempted to control chart everything-----since control charts allow the process to be stopped when the characteristic changes plus it eliminates the need to determine which one characteristic is more important than the other BUT------Doing this is very time consuming + operator will be drowned in charting everything and may overlook any process changes

Educational techniques promoted by Deming?

Educational techniques promoted by Deming? Deming promoted: 1. Parable of the red beads 2. PDSA cycle 3. Concept of 94% management causes verse 6% special causes

Effect of R&R on process cability:

Effect of R&R on process cability: a. Higher gage R&R, the higher the error in C(p) assessment., and greater increases in capability. b. Example: If the observed C(p)=1 and Gage R&R = 50%, then actual C(p)= 1.23

Effective leader & Communication roles

Effective leader & Communication roles Effective leaders will operate as: 1) Monitor of external information from contacts/peers/ experts 2) Monitor of internal information 3) Disseminator or distributor of information 4) Spokesperson to outsiders 5) Decision maker using gathered information.

Effects and Confounding

Effects and Confounding a. Certain situations where the effects of factor variables after interaction can't be differentiated from each other. Their contribution to the result on the response variable is intertwined and cannot be separated.

Examples of Signal- mechanisms for error proofing

Examples of Signal- mechanisms for error proofing 1) Gas cap attached to car 2) Automatic shut-off value at gas pump 3) Microwave stops when door open 4) Product drawing on the cash register 5) Seatbelt buzzer when belt is not worn 6) Car keys ground symmetrical to allow two way insertion.

Examples of Special cause variation:

Examples of Special cause variation: a. Changes in quality b. Operators with varying skill levels c. Changes to process settings d. Environmental variations.

Experimental Objective ----Screening Objective

Experimental Objective ----Screening Objective Experiment that is used to screen out the few important main effects from the less important ones------> Fractional Factorial Design

External Failure Costs

External Failure Costs 1) Penalties 2) Pricing Errors 3) Liability suits 4) Loss of market share 5) Customer complaint visits 6) Recalls 7) Cost of restocking/repair/ rework 8) Premium Freight

F-distribution

F-distribution 1) The F distribution is the distribution of the ratio of two estimates of variance. 2) It is used to compute probability values in the ANOVA analysis of variance. 3). Used in Hypothesis testing (to test whether variances of 2 or more populations are equal)

FMEA

FMEA Definition= systematic approach to evaluating a process/product to determine how the product/process could fail and the failure effects.(Front-end tool) i. Risk Priority Number (RPN) aid in prioritizing the focus and then the actions are identified to reduce the RPN. ii. RPN= severity x occurrence x detection iii. Used to address design change, introduction of new equipment, large changes in staff or assembly process, c. FMEA evolved informally with changes to how risks were weighted/categorizes/ calculated. (not a one-time tool and requires continued review & update of the failure modes)

FMEA is used to do which of the following? Prove to management that extra resources are required Simulate the time and cost due to failure Indicate estimation of an activity Get an indication of the risk involved in an existing process

FMEA is used to prevent failures and their effects by identifying the risks involved in an existing process.

Factors affecting sample size include:

Factors affecting sample size include: a. How confident (accurate) the analyst wants to be that the sample will provide a good estimate of the true population mean. The more confidence required, the greater the sample size needed. b. How close (precision) to the "truth" the analyst wants to be. For more precision, a greater sample size is needed. c. How much variation exists or is estimated to exist in the population. If there is more variation, a greater sample size is needed.

Factors critical to schedule (CTS)

Factors critical to schedule (CTS) 1) Identification of process metrics= key tasks of the measure stage ---. need few process metrics get information about schedule, cost, or quality of a process. 2) Factors critical to schedule (CTS) --->directly impact completion date of the process. 2) Factors critical to quality (CTQ)----> direct effect on the desired characteristics of the product or service. 3) Factors critical to cost (CTC) ----> impact on materials, labor, delivery, overhead, inventory, and/or the cost to the consumer of the good or service. ***Metrics should be customer-focused and transparent (aka should provide immediate objective data that can be analyzed). ***Metrics need to be obtained with measurement systems that are repeatable and reproducible.

Failure Mode and Effects Analysis

Failure Mode and Effects Analysis a. Evolved informally as product developers thought about possible ways that a product could fail. b. Identification of failure modes in system, design, and process with evaluation of how risks were weighted and calculated. c. FORD used FMEA approach to design to safeguard against safety and regulatory-related failure modes. d. QS-9000= standard expectations for automotive suppliers

Failure mode

Failure mode The specific manner or way by which a failure occurs in terms of failure of the item (being a part or (sub) system) function under investigation; it may generally describe the way the failure occurs. It shall at least clearly describe a (end) failure state of the item (or function in case of a Functional FMEA) under consideration. It is the result of the failure mechanism (cause of the failure mode). For example; a fully fractured axle, a deformed axle or a fully open or fully closed electrical contact are each a separate failure mod

Field intelligence

Field intelligence a) Information obtained by employees through their normal interactions with customers. Employee who talks or works with customers will pick up valuable information. b) The key for a business is to establish and maintain channels for this information so field intelligence can reach the higher levels of the organization. c) Some businesses establish mechanisms for gathering extra field intelligence i.e., video cameras in to study customer behavior.

Define Final Report

Final Report Definition----->Report Card on performance for completion of objectives, comparison of actual benefts and cost with budgets, and measure of major activity completion data verses milestones

Finalizing a project charter

Finalizing a project charter 1) Project charter includes appointment for an initial meeting, at which the team members will discuss and consent to the scope, deliverables, and provisional schedule to be included in the charter. 2) mMeeting allows team members to become acquainted with one another if they are not already and to voice any initial concerns or ideas related to the project. 3) Team then maps the top level of the process to be improved which is entered into a revised charter, before sponsor approval. ---approval allows any initial misgivings can be addressed without much loss of resources.

Flow Chart

Flow Chart 1) identifes complications in a process that can lengthen cycle time or cause adverse variations. 2) Improve stage, flowcharts can be used to diagram possible improvements and changes. 3) Control stage, flowcharts can be used to map out processes that have been revised and improved.

Flowcharts Vs. Process Maps

Flowcharts Vs. Process Maps a. Flowcharts show each step in process, decision points, inputs & outputs b. Process maps provide additional information (costs, setup time, cycle time, inventory, types of defects, probability of defects and other information) c. Both expose problems and OFI

Focus Groups

Focus Groups 1) Limited to consumer- product relations 2) Used to improve the depth and accuracy of the customer responses and provides the most accuracy 3) Focus group planning includes: a. Identify the goals of the sessions b. Develop actual/prototype products c. Determine how the sessions will be conducted (brainstorming, survey, question-response, fishbone)

Harmonic Mean

Harmonic Mean A kind of average. To find the harmonic mean of a set of n numbers, add the reciprocals of the numbers in the set, divide the sum by n, then take the reciprocal of the result. The harmonic mean of {a1, a2, a3, a4, . . ., an} is given below. Harmonic Mean = N/(1/a1+1/a2+1/a3+1/a4+.......+1/aN) Step 1: Find the Harmonic Mean of 1 and 100 H= 2 _____________ = 2 / [101/ 100] = 2/ [1.01] [ 1/1] + [1/ 100] H ≈ 1.98019802

A Green Belt is calculating a confidence interval at a 95% level of confidence. She should reject the null hypothesis if the calculated p-value is: Less than .05 Between .1 and .5 Between .5 - .95 Greater than .95

If you calculate a p-value at .05 at 95% confidence, you should reject the null hypothesis If the p-value is between .1 and .5, you should not reject the null hypothesis at a 95% confidence level

When t0 use Inter-relationship Digraphs/ Relations Diagram

Inter-relationship Digraph ====Visual display that maps out the cause and effect links among complex, multivariable problems or desired outcomes. It helps the team to analyze the natural links between different aspects of a complex situation. A Inter-relationship Digraphs used for: a) When trying to understand links between ideas or cause-and-effect relationships, such as when trying to identify an area of greatest impact for improvement. b) When a complex issue is being analyzed for causes. c) When a complex solution is being implemented. d) After generating an affinity diagram, cause-and-effect diagram or tree diagram, to more completely explore the relations of ideas.

Inventory/ Muda

Inventory/ Muda 1. Inventory ---(eg. work-in-process, parts, raw materials, supplies, finished good)---------does not add value to the product and idle inventory at risk for damage, deterioration, & obselense

Is the Process Stable?

Is the Process Stable? 1) Experimental runs should have control runs which are done at the "standard" process setpoints, or at least at some identifiable operating conditions. 2) Experiment should start and end with such runs. 3) Plot of the outcomes of these control runs will indicate if the underlying process itself drifted or shifted during the experiment. 4) Best to have stable process---> If not stable, thenmust account for instability in analysis of the experiment.

Kaizen & Flow

Kaizen & Flow 1) . The goal of kaizen is to create continuous flow (improvement) through the organization rather than improving limited areas of a process. 2) When a process has good flow, materials and information do not wait between activities. 3) Businesses tend to create inefficiencies when they focus only on flow in one or two areas. Instead, flow should be considered from the perspective of the entire business.

Kaoru Ishikawa & Concept "Next operation as Customer"

Kaoru Ishikawa & Concept "Next operation as Customer"" 1. Operator concerned about their own defects were considered spies when they traveled to the next department to view the original work. 2. Departments were defensive when outsiders toured-------Solution= Remove Sectionalism and concept "Next operation as customer"----The concept of internal customers in which every operation is both a receiver and a provider.

Kaoru Ishikawa---- 6 main characteristics that make CWQC different.

Kaoru Ishikawa---- 6 main characteristics that make CWQC different. *** To differentiate b/w Japaneese style and Western style of TQC, he called Japanese style- Company-wide Quality Control. 1. More education & training in quality control 2. Quality circles are only 20% of the CWQC activities 3. QC audits 4. All members participate in the company (top to bottom participation in the entire product life cycle and humanistic respect for workers with acknowledgement of their contributions and input) 5. Using 7 tools and advanced statistical methods 6. Nationwide quality control promotion activities

Kaoru Ishikawa----Training Tapes subjects

Kaoru Ishikawa----Training Tapes subjects 1. Elimination of selctionalism 2. Humanistic management of worker 3. Empowerment & Training of Workers 4. Operation as customer.

Kaoru Ishikawa---Quality Circle

Kaoru Ishikawa---Quality Circle 1. 1st concept adopted by West 2. Quality circle= bottom up approach---originally they were study groups that workers formed to study quality concepts published in the book "Quality Control for Foreman" 3. Circles solve problems on continuous basis. and membership can change depending on project.

Kaoru Ishikawa---Statement on Quality

Kaoru Ishikawa---Statement on Quality To practice quality contro lis to develop, desing, produce, and service a quality product that is most economical, useful, and always satisfactory to the customer

Founders of the Balanced Scorecard

Kaplan & Norton i. Founders of the Balanced Scorecard ---developed as new approach to strategic management.

Key challenge in implementation of Six sigma?

Key challenge in implementation of Six sigma? Assigning high skilled problem solvers who are valuable but need to pulled from other areas of the organization.

The development of a formal communication plan is mainly justified in which situation? Every time a team is initiated When the team has strong internal conflicts Large projects or teams with geographical barriers Before selecting the project

Large projects or teams with geographical barriers are the situations that require a formal communication plan because of their inherent complexity.

Latin Square Design is restricted by 2 conditions

Latin Square Design is restricted by 2 conditions: 1) Number of rows, columns, and treatments must be the same 2) There should be no expected interactions between row and column factors, since these cannot be measured. If there are , the sensitivity of the experiment is reduced

Leader's 6 key Duties

Leader's Duties 1) Communication hub & liaison with management 2) Administrative duties---meeting/ scheduling 3) Work with, not over participants 4) Be encouraging/ supporting ---esp. of mistakes and participation 5) Be a good listener & show concern 6) Ensure individual needs/ expectations are considered

Limitations of Quality Cost System

Limitations of Quality Cost System 1) Quality cost measurements don't solve quality problems 2) Quality cost reports don't suggest specific actions 3) quality cost are prone to short-term mismanagement 4) Difficult to match effots with accomplishments 5) Omission of important costs from quality cost reports 6) Inappropriate costs may be included in cost reports 7) Quality cost may have measurement errors

Line graphs

Line graphs Graphical display if either discrete or continuous characteristics of the product or service changing

Linearity & Bias

Linearity & Bias a. Linearity Study used when equipment is accurate at one of measurement but varies in accuracy at other times. i. Tells how biased the measuring equipment is compared to the "master"

Scatter Diagrams--Linearity and Implications

Linearity and Implications 1) Judges the possibility that bias error found in particular system of measurement will be present throughout the equipment's entire operating range. 2) Measure Stage---> Assessing the accuracy of a measurement system within the range of values likely to be observed during the process. 3) Procedure for analysis of linearity ---> Operating range is examined in multiple parts and then reference value is obtained---> An average of the ranges is calculated after repeated measurements taken----> Bias is calculated by subtracting the average from reference value. ----> Coiffiecient of determination of greater than 70% will mean that measurements are adequate.

Design FMEA Vs. Process FMEA: List potential effects of failure

List potential effects of failure a. Design FMEA i. Subject identified and problem defined to keep focused on idea. Facilitator explains the rules ii. Effect = ability of the component to perform its intended function due to failure mode b. Process FMEA i. Potential effect at interim and effects are identified. Effect= impact on process outcome & quality due to failure mode.

List the reasons why teams have been successful in companies.

List the reasons why teams have been successful in companies. 1. Team procedures allow teams to communicate and express their creativity 2. If management has created teams in the company, it means that management is open to listening to employees and value their ideas

"Listed information & Control Plans

Listed information & Control Plans a) A control plan provides six pieces of information for every characteristic: 1) Specification----defines what is considered acceptable & unacceptable for a characteristic. (expressed as an objective value). a) Example=the specification for an oven temperature of 400 degrees in order to function properly. 2) Measurement technique----the equipment and protocol for assessing the specifications 3) Sample size--- number of measurements required at every collection time. 4) Sample frequency-- interval between each successive data collection. 5) Analytical tool---analyzing the measurements (e.g. monitoring for adverse trends using control charts). 6) Reaction protocols----- on a control plan will be the directions for employees in the event that a characteristic requires adjustment."

Locational Data

Locational data-→ used to identify where the data is coming from or to locate the defect which cannot be found with discrete or continuoud data

Short-term Vs. Long-term Capability Traits of Long-term capability index (CP)

Long-term capability indices (PP and PPK) The same capability indices that you calculate for short-term variation can also be calculated for long-term, or total, variation. To differentiate them from their short-term counterparts, these long-term capability indices are called PP and PPK. (The P stands for "performance.") The only difference in their formulas is that you use σLT in place of σST. These long-term capability indices are important because no process or characteristic operates in just the short term. Every process extends out over time to create long-term performance.

Given that a process' natural variation is ±3σ and the process is centered within the specification limits, what would be the defects, in parts per million, if the specification limits were set at ±4σ? 32ppm 1350ppm 64ppm 135ppm

Look up the answer using a z table. We need to find the percent above and below 4 standard deviations. The z table number for 4.0 is 0.000032. Multiply this number by 2 because there are 2 tails. This gives .000064. Change to ppm by multiplying by 1,000,000. = 64ppm

A Green Belt wishes to identify sources of variation. She finds that there are several sources of variation that need to be categorized. The most appropriate tool for categorizing possible causes of variation would be a: DOE Multi-Vari study Binary Logistic Regression FMEA

MULT-VAR!

Manipulating response and calculating weights in phase 2

Manipulating response and calculating weights in phase 2 1) If desirability function used to minimize the response, --->must have specified target value and an UPPER boundary to calculate the desired value 2) If desirability function used to maximize the response,---> must have specified target value and a LOWER boundary to calculate desired value 3) If desirability function used to achieve a target value,----> must define target value & upper and lower boundaries a) If the response falls b/w upper boundary & target---> calculate desirability function in cases where the intention is to minimize the response. b) If the response falls b/w the lower boundary & target, ---> calculate desirability function in cases where the intention is to maximize the response. The values for the s and t weights depend on the relative emphasis on the target and the boundary. a) Value of target and boundary are equal-->, then the s and t weights = 1. b) Value of target more important than boundary, then s and t weights can be between 1 and 10. c) If the boundary is more important, the s and t weights can be between 0.1 and 1.

6 Skills of Master Black Belt

Master Black Belt a. Skills: 1) Training in Executive communication + teaching/coaching/ mentoring black belts, green belts, 2) . Change management 3) . 6 Sigma methods & statistics 4) Basic financial tools 5) . Project management 6. Risk assessment

Measles Chart

Measles Chart 1) Best for large products and lots of opportunities for improvement. 2) Used for data with specific locations (eg. Rental car diagram with checklist)

Measure (collect data from process) Stage

Measure (collect data from process) 1. Management commitment ----PDSA 2. Identify a data collection plan 3. Measurement system analysis (MSA) 4. Collect Data (check sheets, histograms, Pareto diagram, scatter diagram) 5. Identify variability -variations 6. Benchmarks 7. Cost of quality

Objectives of "Measure Step"

Measure Stage----> primary concern is to create an accurate map of the process as currently performed. 1) Create a comprehensive process level map of processes with participation of expert employees ----group defines and clearly describes all of the activities they aim to improve. (more detailed and comprehensive version of the top-level process map created in the define stage 2) Participation of expert employees is important especially because employees may have a vastly different conception of process sequence than their superiors. 3) Employees will have streamlined / modified a process without notifying management. Judging the beneficiality of customizations made by employees is unnecessary and potentially harmful.

Measurement Scales---Interval

Measurement Scales---Interval ) Central location = Arithmetic Mean 2) Dispersion of Standard Deviation 3) Significance Test= F-test, t-test, Correlation Analysis 4) Data is arranged in order and differences can be found. However, there is no inherent starting point and ratios are meaningless.

Mechanical Screening Devices & Poke/ Yoke

Mechanical Screening Devices & Poke/ Yoke: 1) Application can be based on length, width, height, and weight. 2) Cash registers at fast food outless with descriptions or schematics of the product produced. 3) Barcode ---> elimination of data entry error and saves time.

Median Control Charts:

Median Control Charts: i. Plots the median of the sample rather than the average. ii. Often used when outliers are expected. iii. All data points in the sample are plotted and user connects the middle point in successive samples. iv. Paper gage is used to detect ranges outside the control limits. v. If gauge cant cover all the points, then range exceeds the control limits.

Define Method

Method a. Unwritten process that must be followed consistently. b. Used when there is no specific internal procedure for a particular activity and is not required to written by any standard.

Method of conducting Brainstorm Session:

Method of conducting Brainstorm Session: i. Subject identified and problem defined to keep focused on idea. Facilitator explains the rules ii. Best to keep the subject open even if team members are unfamiliar with topic. iii. Round Robin discussion + evaluation/criticism of others' ideas iv. Ideas are recorded-→ Quantity is more important than quality.

Methods to collect customer feedback:

Methods to collect customer feedback: a. Voice of Customer b. Surveys c. Quality Function deployment d. Interviews e. Focus Groups

Milestones Reporting

Milestones Reporting 1) Milestones = key points in which plans are to completed 2) Intermediate Milestones= refocus on longer objectives and updates project's current status 3) Project team leader presents the milestones to the management -----includes project status relative to milestone, upcoming roadblock to timely completion, and plans to handle the roadblocks 4) After being approved, milestones are not subject to negotiation or change

Which of the following is a benefit of project risk analysis? More efforts Fewer project deliverables Greater probability of project success Project completion time delayed

Mitigated risk increases the chances of project success.

Mixture Experiments

Mixture Experiments Experiments in which the variables are experssed as proportions of the whole and sun of 1.0

The most useful "measure of central tendency" for nominal measurement scale data is: Geometric mean Median Mean Mode

Mode Mode is the "measure of central tendency" for nominal scale. An example of a nominal scale is marital status: single, married, divorced.

How is the status of processes monitored?

Monitoring the status of processes: The status of processes can be monitored by averaging the measurement values in subgroups ( using central limit theorem) using SPC control charts.

Multi-vari Studies

Multi-vari Studies 1) A multi-vari chart is a tool that graphically displays patterns of variation. It is used to identify possible Xs or families of variation, such as variation within a subgroup, between subgroups, or over time 2) Unlike Stem-and-leaf charts, histograms, and frequency distributions which are snapshots of process variation and tell you nothing about the source of variation, the multivari tool shows you variation (the enemy!!!) by separating it into three possible sources (within-piece, piece-to-piece, and time-to-time.) 3) Used to study 3 types of variation: a) Variation within subgroup b) Variation between subgroups c) Variation over time 4) Used to also investigate the stability or consistency of a process. 5) Interpretation of Muti-vari Studies: a) Length of line =Variation within samples b) Vertical position of lines = variation from sample to sample c) Line connecting the midpoint values helps to identify trends.

Multi-variate Studies

Multi-variate Studies 1. Tool for analyzing the 3 types of variation: a. Cyclical (lot-to-lot or part-to-part variation) b. Temporal (shift-to-shift; changes over time) c. Positional (with-in part variation) 2. Minimizes variation by identifying areas in which there is excess or deficient variation. 3. Tool for investigating the stability or consistency of process. 4. Aids in identification of the source or location of variation within the process.

PERT Network Planning Rules

Network Planning Rules 1) Must complete current activity before starting new one 2) Arrows= logical precendency (length and compass direction of arrow meaningless) 3) Must have unique event numbers 4) Any 2 events may be directly connected only by one activity 5) Network must start at single event and end at the single event. 6) Event numbers must be unique 7) Net

New Product Term---Repositionings

New Product Term---Repositionings Product targeted fro new use because the original prupose too narrow--eg. baking soda for baking and for cleaning.

New Product Term--Additions to Product lines

New Product Term--Additions to Product lines New products that extensions of the existing products--eg Coke and Diet Coke

Nominal Data Scale

Nominal Data Scale i. Values on the scale have no "numeric" meaning in the way that ususally applies with numbers and no ordering scheme (eg. Color-coded wires) ii. Application: Counting, Mode, chi-square

Nominal Group Technique

Nominal Group Technique 1) Concept used to prevent peer or social pressures from influencing the generation of ideas 2) Facilitator allows 60-90 min for problem solving session without influencing the problem solving process 3) Main advantage----> encourages independent thinking

Nominal Group Technique (NGT)

Nominal Group Technique (NGT) a. Type of brainstorming technique with limited team vocal interaction. b. Best used to encourage equal participation in discussion, esp. when there is dominance by some group members c. Subject identified and problem defined to keep focused on idea. Facilitator explains the rules. 10-15 minutes to write down ideas and ideas are posted in a space for all to read. Round-Robin format of reading the ideas aloud but no judgment can be passed. Idea submissions are kept anonymous if the topic is controversial. d. Consolidation=expansion and clarification o d ideas to eliminate redundancy.

The weakest and simplest scale of measurement is called: Nominal scale Ratio scale Interval scale Ordinal scale

Nominal scale is only the presence /absence of an attribute. Nominal data can only be counted values. Example: 17 blue, 13 yellow and 5 green. In order of increasing statistical power: Nominal, Ordinal, Interval and Ratio.

Normal Probability plots

Normal Probability plots a. Used to test whether random data come from normal distributed data b. Used to test the normality before doing further analysis. c. Will produce a straight line when manually plotting data on probability graph paper d. Tests used to check normality of random data: i. Empirical cumulative distribution function tests: 1. Anderson-Darling test for normality (most widely used with statistical software) 2. Kolmogorov-Smirnov test for normality ii. Correlation-based tests 1. Ryan-Jointer test (correlation-based test) 2. Shapiro-Wilk test (Correlation based test) 8) Weibull Plots

What are the 3 way in which Norming takes place ...?

Norming takes place in three ways 1) Norming begins ss storming is overcome---> team more relaxed & stable, less conflicts to throw the team off focus. 2) Norming begins when team makes routine---> schneduled team meetings provide orientation & predictability. 3) Norming cultivated via team-building activities

Given a process in statistical control with = 20 and =3 calculate Cpk if the specifications are 19±6. The subgroup size is 3. 5.00 0.9416 0.19 1.32

Note first that the average is over the target so we will calculate Cpk using the USL. For a subgroup size of 3, d2=1.693. Sigma = (R-bar ) / (d2) = 3 / 1.693 = 1.77 Cpk= [USL -(X-bar)] / (3 * Sigma) = (25-20) / ( 3 * 1.77) = 0.9416

Np Chart/ Control Chart

Np Chart/ Control Chart 1) Assess attribute data to measure the number of times a condition exists in each sample, the condition may occur only once and the sample size is consistent. 2) Depicts a stable process 3) Measure Stage---used to guess the process baseline (variable control chart is more commonly used) 4) Improve Stage---used to find the number of errors in the process sample.---but the small error rate make Np chart ineffective

A gage study is conducted over two working days, and there will be a total of 48 weight measurements gathered for the study on one calibrated scale. There are two different settings options for the scale display (kg or lbs). Two employees will evaluate six different products in random order. How many measurements will each employee take on each product? 2 8 1 4

Number of days and scale settings are not relevant to the number of measurements, so the actual number of measurements per product = 48 / 2 employees / 6 products = 4 measurements per product

One-tail Test

One-tail Test a. Level of alpha risk determines the level of confidence (1- alpha). This risk factor used to determine the critical value of the test statistic, which is compared to calculated value. b. This test is used to test whether the sample value is larger or smaller than the population value.

Optimization

Optimization 1) Involves finding the treatment combinations that give the most desired response. 2) Can be "maximation" ( eg. product yield) or "minimization" (in the case of impurities)

A survey asks for a rating of service from poor, fair, good, very good, or excellent. This is an example of what type of measurement scale? Ordinal Ratio Interval Nominal

Ordinal data are when a numeric or non-numeric and have order. Ordinal data is most useful in rankings and ratings.

Out-of- Control Charts

Out-of- Control Charts: 1) "Out-of-control" Process means that special cause variation is present is either the average chart or range chart or btoh. 2) Process out-of-control is detected on control chart by having any points outside the control limits or by un-natural pattern of variability. 3) Because there are two components in every control chart, 4 possible conditions could occur: A. Process out of control : 1. Average Range (out of control, in-control) 2. Average Range (In control, Out-of-control) 3. Average Range (out-of-control, out-of-control) B Process In-Control 4. Average Range (In-control, In-control)

Overproduction/ Muda

Overproduction/ Muda" 1. Overproduction due to making more, making product too early, or making product later than is needed by next process or customer are all waste---> eg. storage space, extra raw materials, extra transportation and scheduling costs 2. Just-in-time movement--producing early is bad because parts need to available at a certain location/time/time according to the customer's schedule

Difference in NP, P, C, & U control chart?

P-chart (fraction non-conforming) C-chart (number of defects) U-chart (non-conformities per unit) p charts For discrete attribute data, use the p chart. Recall that discrete attribute data results when you categorize or bucket each instance you are measuring. For example, you might track defective and non-defective components in a manufacturing process. This chart plots the proportion ("p") of the data falling into the relevant category over time. np charts When each data point is based on the same sample size, a special version of the p chart can be used. The np chart follows the same principle as the p chart, but actually plots the number of instances in a category over time rather than the proportion in the category. The name "np" derives from the convention of using "n" to refer to sample size. By multiplying sample size by proportion (n x p) you get the actual number in a category. c charts The c chart is similar to the np chart, in that it requires equal sample sizes for each data point. For example, in evaluating errors on loan applications, you would use this chart if you sampled the same number of applications each week. But instead of plotting the proportion of data in a certain category, as does the np chart, the c chart plots count data, such as number of errors. As with the other control charts, special cause tests check for outliers and process shifts. u charts The u chart is a more general version of the c chart for use when the data points do not come from equal-sized samples. For instance, if you review all loan applications each week, and the number submitted differs on a weekly basis, you could still count errors and plot the number of errors by week over time. Because of the difference in sample sizes, the control limits will not be constant for each data point. Thus while the same special cause tests apply as for other charts, the outlier test checks specifically for whether a given data point is outside its own control limits.

Paired Comparison

Paired Comparison 1) Basis of a technique for treating data so as to ignore sample-to-sample variability and focus more clearly on variability cause by a specific factor effect. 2) Only the differences in response for each sample are tested becasue sample-to-sample differences are irrelevant.

Paired T-test

Paired T-test The two-sample t-test is used to determine if two population means are equal. The data may either be paired or not paired. For paired t test, the data is dependent, i.e. there is a one-to-one correspondence between the values in the two samples. For example, same subject measured before & after a process change, or same subject measured at different times. For unpaired t test, the sample sizes for the two samples may or may not be equal.

Perceptual Maps--Steps for describing how to determine the right questions to help quantify and prioritize customer needs

Perceptual Maps------Steps for describing how to determine the right questions to help quantify and prioritize customer needs 1. Conduct brainstorming sessions to identify a wishlist of features and/or problem resolutions. 2. Rank the brainstorming session items and consider the highest ranking items for possible customer survey questions. 3. Construct the set of questions, being careful not to bias responses on customer satisfaction or customer importance with the wording.

Tools for Business performance measures: Performance compared to established goals

Performance to established goals a. Goals are measured periodically. b. Cost performance index i. Ex. Budgeted cost vs. actual cost. ii. Project performance measured in dollars. c. Schedule performance index-------Measure of the project's efficiency to schedule as expressed in the rato of earned value to planned value.

Plackett-Burman Designs

Plackett-Burman Designs 1) This is a special category of two level fractional factorial designs, proposed by R. L. Plackett and J. P. Burman, where only a few specifically chosen runs are performed to investigate just the main effects (i.e. no interactions). 2) Used for screening experiments. and are economical 3) Run number is a multiple of 4 rather than a power of 12. 4) Two-level , non-geometric design where each interaction effect is counfounded with exactly one main effect. 5) Two-factor interaction will be partially confounded with each of the other main effects in the study ---> thus non-geometric design are "main effect designs", when there is reason to believe any interactions are of little practical importance.

When creating a detailed process map for a plastic molding process, which of the following are necessary process inputs? Resin material Solid weight Inspection report Part weight

Plastic resin is a normal input to a plastic molding process that is necessary to carry out the transformation to a molded part.

Point Estimation

Point Estimation A point estimate of a population parameter is a single value of a statistic. For example, the sample mean x is a point estimate of the population mean μ. Similarly, the sample proportion p is a point estimate of the population proportion P. Interval estimate.

Prevention Cost Examples

Prevention Cost Examples 1) Capability Studies 2) Forecasting 3) Equipment Repair 4) Vendor surveys or evaluations 5) Quality Design 6) House Keeping 7) Time and Motion Studies 8) Pilot Projects 9) Controlled Storage

Prioritization Technique

Prioritization Technique 1) Aid in decision-making amongst options. 2) Team consensus used to compare criteria which the options are measured and relative importance of each of the criteria. 3) Ranking of options against criteria with desirable numbers being larger.

Problem Solving

Problem Solving 1) Scientific method 2) PDSA cycle 3) 8 discipline Approach

Project Metrics

Project Metrics 1) Project measures commonly lined with project: a. Schedule Performance Index= Percent of work accomplished on time b. Cost Performance Index= percent of work accomplished on budget. c. Quality, cycle time, & availability of resources

Project Planning Tools

Project Planning Tools: Use to determine project timeline, required resources and estimating costs. 1) PERT 2) GANNT 3) CRITICAL PATH METHODS *** Work-breakdown structure identifies detailed activities for the plan and enables estimation of project costs.

Purpose of Project Risk Analysis

Project Risk Analysis Purpose of Project Risk Analysis ---identify potential risks, associated impacts, and potential mitigation plans.

Push Production

Push Production: 1. Most mass production in push production. 2. Goal is to maximize machine efficiency with maximum amount of "in-process" inventory. 3. Customer's demand initiates activities 4. Each operation produces parts as needed in response to a downstream signal (No inventory kept) 5. Minimal Work-in-progress enables continuous flow based on LEAN 6. Problems in any area are corrected immediately and process are stopped till solution found.

Quality Circles

Quality Circles Structure--Team of highly motivated individuals on a voluntary basis. 1) Japanese-derive means of allowing & encouraging people on the production floor to participate in decision that will improve quality and/or reduce manufacturing costs 2) Quality is not the only factor considered in this circles----> a) Cost Savings must be considered to determine if an idea is worth of implementation. b) can also function as a forum for exhanging ideas /solutions

Questioning Technique

Questioning Technique 1) Open-ended questions allow discussion and probing 2) Prepare questions in advance of interview 3) Avoid leading questions 4) Phrase questions in a positive way

Reasons why people may be skeptical of long-term success of teams

Reasons why people may be skeptical of long-term success of teams 1) Most traditional American companies don't have a management style that is conducive to team formation 2) Management fails to comprehend the value of teams.

Which of the following is a direct benefit of a properly executed FMEA? A) Increased safety stock B) Decreased product functionality and robustness C) Reduced impact of risks due to defects D) Increased business process problems

Reduced impact of risks due to defects is a benefit of a proper FMEA. FMEA generally does not focus on inventory metrics. Even if it did, it is more likely that safety stock would be reduced, not increased. A benefit of FMEA would be improved product functionality and robustness.Reduced business process problems are a direct benefit of FMEA.

Relationship Diagram

Relationship Diagram 1) Displays the degree of relationship between variables, causes, and effects 2) Relationship expressed as either weak or strong and uses weighted score used for prioritization. 3) Matrix that shows the relationships between causes & effects or the connectween process inputs and desireable process outputsl 4) "traffic jam" is an unpredictable event that is beyond the caterrer's control

Design of Experimentation Replication

Replication a. Observations or measurement to increase precision, reduce measurement errors, and balance unknown factors. b. Repetition of the set of all treatment combinations to be compared in experiment for sake of achieving reliability.

Reproducibility

Reproducibility 1) Reproducibility of a singe gage is checked by comparin resuls of different operators taken at different times --->will affect both accuaracy and precision of reported Values

Required Sample size

Required Sample size a. The sample size needed for hypothesis testing depends on: i. Type I and Type II risk ii. Minimum value to be detected between the population means iii. Variation in the characteristic being measured.

Resolution II

Resolution II 1) Experiment where some of the main effects are confounded.

Resolution III

Resolution III A fractional factorial design in which no main effects are confounded with each other, but the main effects and two factor interaction effects are confounded

RESPONSE SURFACE ANALYSIS

Response Surface Analysis: Response surface analysis is an OFF0 LINE-optimization technique (COMPARE WITH EVOP). Usually, 2 factors are studied; but 3 or more can be studied. With response surface analysis, we run a series of full factorial experiments and map the response to generate mathematical equations that describe how factors affect the response.

Response Surface Methodology

Response Surface Methodology 1) Graph of a system response plotted aginst one or more system factors. 2) Response surface methodology employs experimental design to discover the "shape" of the response surface and then uses geometric concepts to take advantage of the relationships discovered.

Risk Assessment & FMEA

Risk Assessment & FMEA Risk Assemsnet = comibintion of the probability of a event/ failure to the system's operators/ users/ enviroment. Two measures of Failure: a) Severity of Failure b) Proability of Failure

Risk verification

Risk verification= process of ensuring that the risk mitigation activities prevent risk from occurring a. Examples (numeric password & limit of 3 false attempts to prevent unauthorized access to bank account is the mitigation. Verification is the attempt to access the bank account with invalid password)

Run Chart Decision Rules

Run Chart Decision Rules *** Signals for Effective Change: a) Shift---> 6 or more consecutive points above or below the median b) Trend---> 5 or more increasing or decreasing consecutive points c) Runs---> are there too many (mixture) or too few (called clustering) for just common cause variation? *** A run is a series of consecutive points that all lie on the same side of the line. d) Astronomical---a completely differnet value

SERVICE FMEA

SERVICE FMEA/ Healthcare FMEA 1) Focused on investigating failure modes (tasks, errors, mistakes) caused by system or process deficiencies before the first service. a) Covers non-manufacturing aspects 2) Involvement of financial services, legal, hospitalilty services, government, educational institutions, & healthcare 3) Input come from: a) QFD b) Benchmarking c)Marketing Research d) Focus Groups

SPC and associated costs

SPC and associated costs 1) Cost of selection of variable or attribute to monitor 2 ) Cost setting up control charts and data collection system 3) Cost of training operators 4) Cost of investigation and correction when data values fall outside the specification limits.

Interpretation of Scatter Diagrams

Scatter Diagram b. Find the cause-and-effect relation or the correlation between variables. c. Linear relationship must be present to estimate correlation Coiffeent (Pearson's correlation) i. No correlation if the data are spread out with no inclination to right or left ii. X-axis= independent variable iii. Results: Positive correlation/ Negative Correlation/ No Correlation/ Non-linear Correlation. ****Association and causality are not the same thing e. The closer, r, is to +1 or -1, the stronger the association and there is a higher likelihood that variables are related.

Scorecards

Scorecards= snapshot of organization's performance a) Performance measurement tool that break performance down into a series of categories, each of which is assigned a few top-level metrics. b) Balanced scorecard,---- financial, customer, internal, and innovation. i) Customer metrics ----> satisfaction, ii) Internal metrics ---> Company's quality performance iii) The innovation ---"learning and growth," =---->se metrics of how much the business is working to stay current and improve its processes, products, and services. "

Screening Experiment

Screening Experiment **** Screening experiments are the ultimate fractional factorial experiments. These experiments assume that all interactions, even two-way interactions, are not significant. 1) Technique to discover most probable important factors in a experimental system---- They literally screen the factors, or variables, in the process and determine which are the critical variables that affect the process output. 2) Most screening experiments employ two-level designs. 3) Beware that results of screening experiments----> if factor is not highly significant, then it does not mean it is insignificant There are two major families of screening experiments: Drs. Plackett and Burman developed the original family of screening experiments matrices in the 1940s. Dr. Taguchi adapted the Plackett-Burman screening designs. He modified the Plackett-Burman design approach so that the experimenter could assume that interactions are not significant, yet could test for some two-way interactions at the same time.

Should I use an S chart or an R chart?

Should I use an S chart or an R chart? Both S charts and R charts measure subgroup variability. The S chart uses the standard deviation to represent the spread in the data and the R chart uses the range. When to use the S chart Use the S chart when your subgroup sizes are 9 or greater. S charts use all the data to calculate the subgroup process standard deviations. You should consider using S charts for processes with a high rate of production or when data collection is quick and inexpensive. When to use the R chart Use the R chart when your subgroup sizes are 8 or less.

Significance of Six Sigma (Motorola and then expanded by GE):

Significance of Six Sigma (Motorola and then expanded by GE): a. Continual Improvement = use of problem-solving techniques and quick deployment to implement improvements and then use processes as breakthrough system b. Collecting data on process c. Reducing variation and waste→ greater customer satisfaction d. SIPOC identification of process: processes, input, output, and customer e. If the root cause for an accident/ project rejection, then the root cause can be traced back to many small changes that occurred whether within the organization or the supplier

Simple Linear Correlation Coeifficent-

Simple Linear Correlation Coeifficent- a. Indicates the strength and direction of the relationship between dependent and independent variable. b. Strength of the coefficient= how close the variable is to +1 or -1. c. Rho= population correlation coefficient d. Simple Regression= straight-line that best fits a series of ordered pairs (x,y) e. Assumes atht the data point are within a straight line whose equation is y= B0 +(B1) (X1)

Simple linear regression

Simple linear regression 1) orient data points around a single straight line. ( y = mx + b, y = dependent variable, m = slope, x = independent variable, and b (y-intercept) is the point along the y-axis where x = 0 ) . 2) In Six Sigma, however, it is more common to see the formula expressed[ Y0= B0 + B(1)X + error] can be used to explore the connections between metrics and process factors. a) Improve stage, regression analysis is useful for confirming these connections after improvements have been implemented. ****A scatter diagram often is interpreted with a simple linear regression analysis. If more than one factor influences the value of the independent variable, then multiple regression is necessary."

Standard Work

Standard Work 1) Standards ====> Operation of a plant depend on the use of policies, procedures and work instructions 2) If things go wrong in Gemba, the look for the root cause, implement corrective action , and change work procedures 3) In Japan, Standards are used to control processes, NOT WORKERS---and to describe a process that is the easiest and safest for workers and most cost-effective and productive for companies.

Term: Precise description of each work activity, specifying cycle time, takt time, work sequence, and minimum inventory of parts needed to conduct activity

Standard Work Precise description of each work activity, specifying cycle time, takt time, work sequence, and minimum inventory of parts needed to conduct activity

Standardized Work sheets & Issues to be considered

Standardized Work sheets: 1) Combines the 3 elements-----materials, worker, and machines---in work environment 2) Issues to be considered in standardized work sheets: a) Resource Availability b) Defect prevention & Deterrance of operational mistakes c) Tools or process improvements d) Safe workplace concepts e) Machines arrangement and minimization of transport f) Worker ideas valued g) Installation of autonomous systems h) Optimizing inventory

Steps for Process Capability Studies Measuring Process Capability

Steps for Process Capability Studies Measuring Process Capability a. Construct histogram using the original readings (not the averages) from the control chart. i. Normality (normally distributed data)= histogram with most points grouped around a single peak and fairly symmetric tails on each side ii. Can conclude that data is drawn from normal population. iii. More data used, greater significance

Stratified Sampling

Stratified Sampling Basic assumption of sampling----> sample is selected from a homogenous lot ----> Hetergenous lot---eg. car parts in a pile but made by different machines, different condition, or different lines 1) Stratified Sampling-----> Selection of random samples from each group/ process that are different from other similar groups or processes 2) Results in a mix of samples that can be biased if the proportion of samples does not reflect the relative frequency of the groups. -----> Importance? a) User must be made aware of possibility of stratified groups b) Data reports must state that the observations are relevant only to the sample drawn and may not necessarily reflect the overall system.

Student T-test

Student T-test 1) Applies to samples drawn from a normally distributed population. 2) Used fro making inferences about a population mean when the population variance is unknown and the sample size is small. 3) t-distribution is never wrong for any sample size----> Sample size 30 is the crossover point between the t and Z test. 4) Works well for bell-shaped distribution.

If the upper and lower specification limits are set at ±6σ, the process capability ratio (Cp) is 2.0. If another process capability study is performed and is determined that σ hasn't changed but the process capability index (Cpk) is now 1.5, the process has a mean that can move even more in the direction it is going without producing more than 3.4 ppm defective. is producing approximately 6.8 ppm defective. is producing the same % defective as when the original study was performed. is producing approximately 3.4 ppm defective.

Substitute 6σ for USL and solve. It is 1.5 σ, so the upper spec is now at (1.5+3=4.5) σ. Look up 4.5 z in the standard normal table. It is 3.4 ppm. Cpk= [USL - (X-BAR)] ___________ = 1.5 (sigma) (3 * SIGMA)

Supplier-Input-Process-Ouput-Customers (SIPOC) Diagram

Supplier-Input-Process-Ouput-Customers (SIPOC) Diagram 1) First , define the process and boundary + ID process outputs (data, services) + internal/external customer that receive outputs 2) External suppliers= provide process inputs (materials, purchased parts, contracted services, electrical power) 3) Internal suppliers= processes inside the company that provide inputs.

System of Business Process Performance Metrics

System of Business Process Performance Metrics 1. Effective business process management requires integrated system of metrics to reach 6sigma 2. Peason states a system of metrics should link all 3 levels of the enterprise---process, operations, & business with KPOV at each level of process. 3. System should monitor and control each metric at each level as well as find the key linkages of KPIV and KPOV in entire system.

Systemic Design---Process Structure of Modern German Design Thinking

Systemic Design---Process Structure of Modern German Design Thinking 1. Determine design requirements 2. Select appropriate process elements 3. Step-by-step transforms qualitative items into quantitative items. 4. Deliberate combination of elements of differing complexities is used.

Taguchi--Concept of Robustness

Taguchi--Concept of Robustness 1. Robustness derived from consistency ---products with robustenss are resistant to variations from process that are either ill-controlled or uncontrolled. 2. Building parts to target is key to scucess. 3. Produce designs that can be made consistently

A methodology used to overcome the "Five Sigma Wall" is which of the following? A) As-is value stream map B) DMAIC C) DFSS D) The seven basic quality tools

The "Five Sigma Wall" refers to a "fatal" roadblock to DMAIC. An as-is value stream map is a tool that evaluates lead time. In Six Sigma methodology, DFSS is the next step when DMAIC fails to achieve the desired performance. In other words, if the current process cannot be improved, the design of the process and/or the product needs improvement. The seven basic quality tools are tools for performance in the one- to three-sigma range.

The x-bar control charts makes use of what statistical concept? Specification limits Response Surface Methodology Box-Cox Transformation The Central Limit Theorem

The Central Limit Theorem is the concept that states that the average values of samples drawn from any statistical universe, regardless of the shape of the distribution of the population of that universe, will tend toward a normal distribution as the sample size grows. Specification limits, USL and LSL, are defined based on customer requirements analysis. Box-Cox Transformation is a technique used to normalize data by accomplishing a mathematical "transformation" of each data point, and is not the driving force behind the x-bar chart methodology. Response Surface Methodology is an experimental design methodology that allows experimenters to study curvature in experimental factors, and is not the driving force behind the x-bar chart methodology.

Which of the following control charts doesn't work based on the central limit theorem? asq/learn_x.gif U Chart X-bar Chart X Chart P Chart

The central limit theorem is based on the average or summation statistic of independent samples, the X chart shows the individual values though. U statistic is an average of n Poisson samples. P statistic is an average of n Bernoulli samples.

There have been many quality management systems developed in the past 100 years. Of the following list, which quality management tool is the most recent addition? A) FMEA B) TQM C) SPC D)Balanced scorecard

The concept of the balanced scorecard was developed by Kaplan and Norton in the mid-1990s.

A detailed process map is normally initiated during which stage of DMAIC to define the current process? Analyze asq/learn_check.gif Measure Improve Control

The details of the process are needed in the measure phase. This is where the detailed process map is initiated.

Which diagram depicts the most efficient sequence of events possible for a project? Activity network diagram Tree diagram Affinity diagram Fishbone diagram

The primary function of an activity network diagram is to determine the most efficient sequence of events for a project. A tree diagram is a useful input to an activity network diagram, but is not in itself sufficient to plot the most efficient project path. In affinity diagram helps to group ideas or issues into logical categories, but will not determine the most efficient way to pursue their implementation.

When using statistical methods for data analysis, the most important step is to first: Determine the shape of the distribution. Determine the standard deviation for the distribution. Determine if the process is in statistical control. Determine the average for the distribution.

The shape of the distribution is key, since it determines what statistics are used to describe the distribution. Not all distributions use the same descriptive statistics (Weibull distributions, exponential distributions, etc. do not use the standard deviation).

Theory of Constraints (Dr. Goldratt)

Theory of Constraints (Dr. Goldratt) 1) The theory says that every system, no matter how well it performs, has at least one constraint that limits its performance---the system's weakest link. You use the theory by identifying your constraint and restructuring the way that you work so that you can overcome it.

Three process ( Ppk, Pp, & Cpm) all require stability of processes.

Three process ( Ppk, Pp, & Cpm) all require stability of processes. i. Cp & Cpk are better measures because they reflect capability derived from common cause variation. ii. Pp & Ppk= can be applied to data collected from incoming inspection material to obtain an indication of process capability at the supplier end were the SPC data is unavailable. iii. Process data containing special cause variation-→ Pp & Ppk can be used for information about the processes as long as the data follows normal distribution pattern. iv. Pp & Ppk can be used in situation where SPC-implemented suppliers are not willing to share their process data in time sequence with customers or when suppliers have not implemented SPC in their processes.

Given a runout tolerance of .003in max, a process average of .0025in, and a calculated standard deviation (σ) of .0002in, what can be said about this process? The process is not in control. The process is capable. The process is not stable The process is not capable.

To answer this question Cpk needs to be determined using the formula: (spec limit - process average) / 3σ. For this problem: Cpk = .83 so process is capable To be capable the value of Cpk must be at least 1.00. To answer this question Cpk needs to be determined using the formula: (spec limit - process average) / 3σ. For this problem: Cpk = .83

Cost/Benefit Analysis & Total Cost Curve

Total Cost Curve i. Starting at the top of the traditional economic model of quality of conformance, we find the total cost curve. It represents the cost of producing your product. This curve, much like a smiley-face, dips as you institute and experience the new costs of a quality assurance program and then swings back up as the costs of quality assurance continue a climb higher than would be considered optimal.

Total Productive Maintenance

Total Productive Maintenance a. Preventative maintenance ensures reliable equipment (ex. Oil change, tightening loose parts, monitoring visual/audio failure) b. TMP programs pairs maintenance technicians with line workers (trained for basic repairs & maintenance) to reduce machine breakdowns c. Operators are trained to recognize symptoms of failure to facilitate faster reporting of failures. d. Mature TPM uses overall equipment effectiveness (product of equipment availability + performance + quality of output)

Traditional Cost Concepts

Traditional Cost Concepts 1) Financial Reports used to compare actual costs with budgeted costs. 2)

Traits of Value-added process steps:

Traits of Value-added process steps: i. Customer recognizes value ii. Transformation of product iii. It is done the first time.

5 States of Team Development --Transitioning

Transitioning -----Disbanding of team and team returns to forming stage.

Transport/ Muda

Transport/ Muda 1. Poor plant layout, poor cell design 2. Batch processing 3. Long lead times 4. Large storage areas 5. Scheduling problems

If a team had a listing of multiple concerns and wanted to understand the cause and effect of each, the best tool for the team to use for this activity would be which of the following? A) Tree diagram B) Interrelationship digraph C) Activity network diagram D) Affinity diagram

Tree Diagram

Tree Diagram/ Systematic Diagram--4 key uses

Tree Diagram / Systematic Diagram 1) Systematic method to outline all of the details needed to complete a given objective. ---> same cause in an affinity or fishbone diagram can be filled into tree diagram and can be done after brainstorming session. 2) Similar to value analysis 3) Used for : a) To develop element for a new product b) Show the relationship of a production process c) Create new ideas in problem solving d) Outline the stops to implement a project.

Trends

Trends A. X-Bar Chart Causes 1) Deterioration of Machine 2) Tired Operator 3) Tool Wear B R- Chart Causes 1) Improvement or Deterioration of Operator Will 2) Tired Operator 3) Change in incoming material quality C. Corrective Action 1) Repair or use alternate machine if available 2) Discuss operation with operator to find cause 3) Rotate operator 4) Change, repair, sharpen tool 5) Investigate material

Two Sample t-test

Two Sample t-test This test is used when comparing the means of: 1) Two random independent samples are drawn, n1 and n2 2) Each population exhibit normal distribution 3) Equal standard deviations assumed for each population The degrees of freedom (dF) = n1 + n2 - 2

Two Types of Benchmarking

Two Types of Benchmarking 5) Internal Benchmarking-→ different departments assess each other's processes and take the best from each process to improve their own process. 6) External Benchmarking-→ within the same industry (easier because removes the competitive aspects) or with an organization from another industry.

Two types of studies used for drawing statistical conclusions:

Two types of studies used for drawing statistical conclusions: a. Descriptive (enumerative) study b. Analytical (inferential) Study

Types of Sampling Methods:

Types of Sampling Methods: a. Random sampling b. Sequential sampling c. Stratified sampling d. Rational Subgrouping

Types of Teams

Types of Teams 1) Six Sigma teams 2) Improvement Teams 3) Process Improvement Teams

Use of Metrics & Business plan

Use of Metrics & Business plan: 1. Measures are developed to measure achievement of organizational goals and must be implemented 2. Each team assigned a portion of metrics to achieve. 3. Strategic plan will have strategic goals which are then deployed into smaller tactical goals

Value Chain—

Value Chain— Creating and building a series of value-building activities in the company. The concept of value chain was introduced by M.E. Porter in 1985. It involves constructing a chain of activities within the specific industry, such as shipping, that add value to the operations of the company. For example, activities within operations and logistics must add up to create value in the company's primary role.

Variable Width Box Plot

Variable Width Box Plot The other common variant that shows more information than the standard fixed width box plot is called the variable width box plot (like the plot at the top of this page). But this is only valuable if comparing more than one box plot since the width is irrelevant is just displaying one box plot. One common convention is to make the width of the boxes for group of data is proportional to the square roots of the number of observations in a given sample.

Term: Placement in plain view of all tools, parts, production activities, and indicators of production system performance, such that the system status is easily understood

Visual Control Placement in plain view of all tools, parts, production activities, and indicators of production system performance, such that the system status is easily understood

Visual Factory

Visual Factory a. Common source of process variation=Failing to follow instructions in detail (esp. mix-model schedules or cross-trained staff that flex into various workstations) b. Lines, signs, and labels reduce variation

Walter A. Shewhart

Walter A. Shewhart, 1) Combined economics, engineering, and statistics in his work. 2) Known as a top engineer at Western Electric and Bell Telephone Laboratories and as the author of Economic Control of Quality of Manufactured Product. "

3 key facts about Walter Stewart?

Walter Shewart a. ASQ first honorary member b. Father of statistical quality control c. Developed Control charts

An agriculture company is presented with two different methods for testing heavy metals in fresh produce. In order for this company to determine which method is more precise, it runs several tests using both methods and receives comparable results. The company decides to run an F-test to determine which method is more precise. What type of F-test should be used? Double-ended test One or two tail test will give the same result. One-tailed test Two-tailed test

We use a one-tailed test in this case because the only information we are interested in is whether Method 1 is more precise than Method 2.

The distribution plot of fatigue life using median ranks represents which of the following? Poisson distribution Weibull distribution Normal distribution Exponential distribution

Weibull distributions can be used for many distribution shapes, and are frequently used for median ranking. Normal distribution plots involve a bell Shape using the actual data or averages. Exponential distribution includes average time between failures, but not median ranks. A Poisson distribution is a discrete distribution for the probability of occurrence p on each trial. It does not involve median ranks. Close

What is the Objective of the House of Quality Methodology?

What is the Objective of the House of Quality Methodology? 1) Voice of Customer is carried from deign to manufacturing by using the "how's" from the house of quality as the "whats" for the house of product design House of Quality (customer attributes VS. Engineering Characteristics ) -----------> Parts Deployment (Engineering Characteristics vs. Parts Characteristics) -------> Process Planning (Parts Characteristics vs. Key Process Operations) ----> Production Planning (Key Process Characteristics vs. Production Requirements) 2) House of quality is a kind of conceptual map that provides the means for inter-functional planning

When is DMADV used?

When is DMADV used? 1. Product being developed 2. Product/ Process exists but fails to meet customer specifications or six sigma level

When seeking to reduce variation, MSA should be evaluated for 2 reasons:

When seeking to reduce variation, MSA should be evaluated for 2 reasons: a. All data from the process is filtered through measurement system b. It often represents the most cost-effective way to reduce variation. 8) MSA systems commonly have 40-50% chance of having errors of process specification.

Win-win negotiations-- 4 steps

Win-win negotiations-- 4 steps ***Best approach for dealing with business relationships 1) Establish win-win plans 2) Develop win-win relationships 3) Form win-win agreements 4) Perform win-win maintenance.

13. In gauge repeatability and reproducibility analysis, what percentage of total process variation is acceptable? a. 10% or less b. 15% or less c. 20% or less d. 30% or less"

a "13. A: 10% or less. In gauge repeatability and reproducibility analysis, 10% or less total process variation is acceptable. Variation from 10 to 30% is considered problematic, and any variation over 30% is unacceptable. A gauge repeatability and reproducibility analysis may result in a statistic expressed as a percentage of total process variation or a percentage of tolerance. The range of acceptable percentages is the same when the statistic generated is a percentage of tolerance. Gauge repeatability and reproducibility analysis indicates the degree to which a measurement system avoids common- and special-cause variation.

Define and Interpret Cp

c. Cp= measures a process's ability to meet a specification of client. 1. Cp of more than 2 implies that a specification spread is significantly larger than a process spread. A large specification limit allows for fewer errors to be made during a manufacturing or a business process. For example, if a machine produces snow shovels of a large specification spread, the likelihood of producing shovels that do not meet customer requirements are smaller than one with a small specification spread.

21. Which of the following would be considered a value-added activity? a. Design b. Delivery c. Marketing d. Manufacturing

d "21. D: Manufacturing. Manufacturing is considered a value-added activity. Value-added activities are those that create value in a product or service from the perspective of the customer. There are certain processes that are necessary but that do not add value for the customer. These are known as business-value-added activities. Design, delivery, and marketing are classic examples of business-value-added activities, because they do not directly add value for the customer, but they are a necessary part of the production process.

You are a full-time Scrum Master on an agile project team. A team member becomes sick in the middle of a sprint in which release delivery was planned. Which action is most appropriate in this situation? Ask other team members to work extra hours Deliver what you can within the sprint Start development yourself for the remaining work of the sick team member Ask sick team member to work from home

he right approach is to deliver what you can within the sprint and discuss the same with customer Asking other team members to work extra hours AND THE REMAINING CHOICES is a counterproductive action in the long term and it will go against the agile principle of maintaining a sustainable pace

Cost/Benefit Analysis & Optimum Level of Quality Line

iii. Optimum Level of Quality Line The optimum level of quality for a business falls precisely at the point where the two curves, cost of quality assurance and the cost of nonconformance, intersect. Draw a vertical line on the traditional economic model of quality of conformance graph that passes through the intersection of the two curves -- cost of quality assurance and the cost of nonconformance. Note that the optimum level of quality line intersects the total cost curve at its lowest possible point.

Robust Design

Robust Design Term that means a response variable is considered robuts or immune to input variables that may be difficult or impossible to control.

Sampling Plan

Sampling Plan 1) Guides when and how to conduct the sample 2) Ensures that process, product, and customer requirements are met. 3) Randomness= every part that could be measured has equal chance of actually being selected by the operator 4) Kruger's grocery chain asks customer during their Saturday shopping if they would fill out surveys. In return, they offer them in-store savings. Customers are chosen randomly and the overall sample numbers depends on customer compliance. What is this type of sampling referred to? 5) Judgment Sampling: ---sampling methodology that targets pre-selected experts. a. Key Takeaway: Judgment is a sampling method that selects a sample for research based on their expertise or knowledge. By selecting weekend managers at Kroger's stores, the company is wishing to gain information on response to door promotions of those specifically knowledgeable to weekend sales and customer traffic.

Scatter Diagrams

Scatter Diagrams 1) Correlation originates from the following: a) Cause-effect Pattern b) Relationship between one cause and another cause 2) Not all scatter diagrams have linear relationship 3) Regression line ---" Best Fit Line"---> must analyze the scatter diagram before making a decision in correlation statistics.

Sequence or process for individual audits:

Sequence or process for individual audits: a. Audit planning ----->Auditor's preparation phase (familiarity with audited item, audit scheduling) b. Audit performance-→ evidence of performance gathered via interviews, observations, and ect.------>Reconciliation to ensure the validity of findings (ie. Noncompliances or nonconformances) + OFI c. Audit reporting-→ summarization of audit scope, activities, and results + corrective actions + preventative actions (OFI) d. Audit closures--→ Followup activities to ensure action plan is implemented and effective in eliminating associated causes

Taguchi---Design of Experiments

Taguchi--Design of Experiments 1. Unlike other statistical techniques that tell what has happened, Taguchi tools tell how to make something happen 2. His quality concepts are more technical in nature and made for technicians. (Juran's QC aimed at management and not technicians) 3. Unlike Juran, Taguchi does not call on internal revolution and feels that management's role should be limited to simply provide training and promote use of QC throughout organization.

Process of Team Formation

Team Formation 1) 7 is the ideas team size 2) Members with complementary skills to achieve team objectives. 3) Team size should be driven by the size and scope of project. 4) Team includes stakeholders and subject matter experts. 5) Interaction between team members improved if there's diversity. 6) When outsiders (non-stakeholders) are included in team, outsiders asks questions that were never explored by the team. 7) Six Sigma teams are generally corss-functions t address issues from every angle.

Team Problems & Solution---Overbearing Participants

Team Problems & Solution---Overbearing Participants 1) A member has excess influence ---> Reinforce team concept 2) Member with legitimate authority ---Ask the expert to lead the group 3) Member is an "expert"---> Private discussion with expert.

In a team environment, who is primarily responsible for identifying and resolving negative behavior? Note-taker Manager Leader Champion

The Leader is responsible for creating favorable conditions that will enable a team to reach its purpose In a Six Sigma team, the team leader must assume all roles that a facilitator would perform.

When using the DMADV methodology, in the final phase you should do which of the following? A) Verify design performance and its ability to meet customer needs. B) Visit customers to ensure designs are performing as intended. C) Establish metrics to monitor progress of the design. D) Voice all concerns still remaining after the product has been designed.

The correct order of steps is define, measure, analyze, design, and verify.

Pull Value & Throughput Reductions

Pull Value & Throughput Reductions: 1. 50% reduction in cycle time of product development 2. 75% reduction in cycle time of order processing 3. 90% reduction in cycle time of physical production.

Recurring Cycles

Recurring Cycles: A. X-Bar Chart Causes 1) Physical Environments ----Temperature, Humidity 2) Tired Operator 3) Regular rotation of machine/operator B R- Chart Causes 1) Scheduled maintencance 2) Tired Operator 3) Tool Wear C. Corrective Action 1) If environment is controllable, adjust it 2) Service equipment 3) Rotate operators 4) Evaluate machine maintenance 5) Replace, sharpen, or repair tool

"Redefining process flow

Redefining process flow 1) Processes improvement via redefining process flows via the following methods: a) A process simplified or optimized. b) Remove redundant steps & apply level-loading techniques. c) reducing set-up times. ***Whenever process flows are redefined, assessing the impact on quality is important & can use critical-to-quality metrics (measure stage ) or benchmarking. are useful. a) Benchmarking = systematic review of similar or dissimilar processes in another organization. ----> use for inspiration rather than mapping out & copying competitor's process

Relational Matrices

Relational Matrices 1) Cause-and-effect matrix--->tool that can aid with the prioritization of key process input variables 2) Matrix lists key process output variables horizontally and key input process variables vertically. 3) Numbers are assigned to the output variables. 4) A number is entered for the effect that each input variable has on the output variable 5) Process input Variable-----> product of the process output priority multiplied by the effect value. -------used to compare and determine which input variables have the greatest effect on the output variables

Required Sample Size

Required Sample Size 1) Ideally, the alpha and beta error desired must be determined before the sample size is selected. 2) Sample Size needed depends on : a) Desired type I (a) or Type II (beta) risk b) Minimum value to be detected between the population means c) Variation in the characteristic being measured.

Resolution V

Resolution V Fractional factorial design in which no confounding of main effects and two factor interactions occurs. However, two factor interactions may be confounded with 3 factor and higher interactions

Response Variable

Response Variable (aka. Output or dependent variable) 1) Variable that shows the observed results of an experimental treatment

"11. What is the name for the amount of completed product divided by the original amount of product? a. Scrap rate b. Throughput yield c. Yield d. Rolled throughput yield

"11. C: Yield. Yield is the amount of completed product divided by the original amount of product. This is one of the more popular critical-to-quality metrics. The ideal yield is one (or 100%). Scrap rate, meanwhile, is the percentage of materials not ultimately used in products. Throughput yield is the average percentage of completed units with no defects. Rolled throughput yield, finally, is the quality level that can be anticipated after several steps in the process have been completed.

Affinity Diagrams

Affinity Diagrams 1) Best for finding responses to open-ended questions 2) Must first brainstorm for a list of responses which is documented. Responses are then grouped. Generate titles for each group (okay to have same responses duplicated into different groups)

5 steps to system improvement based on theory of constraints:

5 steps to system improvement based on theory of constraints: a. Identify -→ find processes that limit the effectiveness of the system b. Exploit-→improve rate of constraining process c. Subordinate d. Elevate-→ investment in new technology/equipment to improve the sytsem rate e. Repeat-→ repeat steps with the new constaint

8 symptoms of Groupthink

8 symptoms of Groupthink 1) Invulnerability --> feeling that group is above criticism 2) Collective rationalization ---> refusing to accept contradictory data or to consider alternatives 3) Belief in inherent group morality---> group feels that they are inherently right and oabe the reproach of outsiders 4) Out- group stereotypes --> refusing to look realistically at other groups 5) Self- censorship= refusing to communicate personal concerns to group 6) Illusion of unanimity = accepting consensus prematurely, without testing its completeness 7) Direct pressure on dissenters ---> refusing to tolerate a member who suggests the group may be wrong 8) Self- appointed mindguards --> protecting the group from hearing disturbing ideas from outsiders.

Alias

Alias Occurs when 2 factor effects are confuses with each other

Poisson Distribution:

B. Poisson Distribution: 1) Discrete data only 2) DPU= defects per unit

Go/no go gage.=

Go/no go gage.= This gage simply tells you if the part passes or it fails. There are only two possible outcomes.

Goal Statement

Goal Statement 1) Goals should be completed in 120-160 day period with a 50% reduction in the metrics selected.

Goldratt's Definition of Theory of Constraints

Goldratt's Definition of Theory of Constraints: Goldratt describes the Theory of Constraints as intuitive framework for managing based on the desire to continually improve a company. Using Theory of Constraints, a definition of the goals of the company are established along with metrics for critical measures.

Importance of Management Support for Teams

Importance of Management Support for Teams 1. Management should be knowledgeable on the concept of teams 2. Teams should be kept informed of things that aid in reaching their goals 3. Leaders, facilitators, and team member all should be trained in 6 sigma 4. Must prioritize team's project schedules and solutions.

Interrelationship Diagrams

Interrelationship Diagrams 1) Identify the cause-and-effect relationships. 2) Draw arrows if there is a relationship between categories. 3) Driver= the note that has the most arrows= key cause of the problem; source of a metric for determining project success.

Father of Japanese Quality Control Efforts?

Kaoru Ishikawa

Kaoru Ishikawa---Books

Kaoru Ishikawa---Books 1. Author of the 1st japaneese book to describe TQC in Guide to Quality Control 2. What is Total Quality Control? The Japanese Way

Metrics Selection----Primary Metrics

Metrics Selection----Primary Metrics 1) Primary metrics that are considered in project come from several sources (suppliers, customers, internal process) 2) These basic metrics include quality, cycle time, cost, value, and labor

Natural Work Team Organizations

Natural Work Team Organizations **Area Supervisor + Facilitators + Outside members/specialists 1. Leadership is given to area supervisor 2. Team members mostly come from area supervisor's workforce 3. Outside members from specialist organizations can either be contributing members or active members 4. Facilitator Roles a) Team activity coordination b) Oversight of team progress c) Document Results d) Training team members

5 States of Team Development ---Norming

Norming i. Start to function as team ii. Some accomplishments due to conflicts getting resolved and agreement on ideas to move forward iii. Trust and sharing of ideas.

Sequential Experiments

Sequential Experiments Experiments are done one after another

Test for Means, Variations & Proportions:

Test for Means, Variations & Proportions: 1. Continuous Data—large samples ---normal distribution used to find the CI 2. Continuous Data- small samples----Samples less than 30 use T-distribution test. 3. Confidence Intervals for Variation---- CI are not symmetrical around the average and so must use Chi Square to find the CI. 4. CI for Proportions--- normal distribution used to find the CI

Time-Keeper Role

Time-Keeper Role 1) Optional responsibility---can be assigned to the facilitator

Work Instructions

Work Instructions 1) Procedures describe the process at general level, while work instructions provide details and step-by-step sequence of activities. 2) Flow charts may be used with work-instructions to show relationships of process steps. 3) Controlled copies of work instructions are kept in the area where the activities are performed. 4) Level of detail in the work instruction should be appropriate for background, experience, and skill of the personnel. 5) Wording and terminology should match that used by those doing the work.

A Six Sigma team is focusing on reducing cycle time for an order. Which of the following best describes consequential metrics? Unplanned downtime Labor hours/order Frees up storage floor space Number of backlog orders

hink of consequential metrics like negative side effects. If you try to reduce cycle time of an order, then it may happen that labor hours per order are increased. Cycle time is linked with duration and Labor hours are linked with efforts. For example - Cycle time per order is 5 days and Labor hours are 40 hours per order. If you reduce cycle time per order to 4 days, it doesn't mean that Labor hours will be 32 hours per order. While achieving 4 days cycle time performance, there may be negative side effects on Labor hours per order. Due to excessive pressure more rework, or more overtime may result to increase Labor hours/order.

Evaluation Phase of Brainstorming

iEvaluation phase (ideas evaluated for applicability or usefulness) 1. Facilitator reviews the ideas in sequence 2. Stress the importance of avoiding idea-stopping thoughts/behaviors before starting brainstorming. 3. Categorize or group the ideas generate before evaluating them.

Types of Attribute Control Charts: C-chart

Types of Attribute Control Charts: C chart In statistical quality control, the c-chart is a type of control chart used to monitor "count"-type data, typically total number of nonconformities per unit.[1] It is also occasionally used to monitor the total number of events occurring in a given unit of time. The c-chart differs from the p-chart in that it accounts for the possibility of more than one nonconformity per inspection unit, and that (unlike the p-chart and u-chart) it requires a fixed sample size. The p-chart models "pass"/"fail"-type inspection only, while the c-chart (and u-chart) give the ability to distinguish between (for example) 2 items which fail inspection because of one fault each and the same two items failing inspection with 5 faults each; in the former case, the p-chart will show two non-conformant items, while the c-chart will show 10 faults.

Value Stream & Competitive Advantage

Value Stream & Competitive Advantage 1. Competitive advantage is gained by performing activities better or at lower cost than competitors. 2.Value stream in lean manufacturing involves analysis of a single product stream for : a) reduction of waste b) reduction in cycle time c) improvement in quality.

"Basic objectives of the improve stage

"Basic objectives of the improve stage 1) Occurs when all of the hard work of the preceding sections finally is applied to processes. 2) Key Objectives: a) Set the new process operating conditions based on the experimentation and analysis of the measure and analyze phases. b) Identify & address the failure modes for the new processes. i) Assessing /predicting the benefits of the proposed solution before making massive changes. c) Implement & confirm process improvements, the moment when the predictions developed during the preceding sections finally are tested against reality.

SIPOC (suppliers, inputs, process, outputs, and processes) analysis

"SIPOC 1) Define stage ---> when the Six Sigma team tries to identify each top-level process, as well as its stakeholders. 2) Suppliers =persons who contribute inputs to the process. 3) Inputs = knowledge, resources, and information required to produce the desired output. 4) A process =any task that translates inputs into outputs. 5) Outputs are the deliverables, or the products of the process. Finally, the customers are all of those parties that receive the deliverables (outputs). ***SIPOC is initiated with the creation of a flowchart or process map. Typically, the first category to be identified is the outputs. From there, locating the customers, inputs, and suppliers will be easier. "

"Schedule of Project Charter

"Schedule of Project Charter 1) Includes the date on which the charter was approved by the sponsor as well as the start and completion dates. 2) Phase-gate Review = Indicates the dates on which each phase of the DMAIC schedule will be complete. Each of the phases of the DMAIC schedule should be subdivided into a set of activities. 3) Critical path or program evaluation and review technique (PERT) analysis to set the target dates. 4) The schedule typically is expressed as a Gantt chart, in which each phase of the project is defined, and the prerequisites and required resources are listed.

Two types of Factorial designs in Experiments

"Two types of Factorial designs in Experiments 1) Fractional Factorial designs (analyze & improve stages ) a) Analyze stage= fractional factorial designs are used to identify process drivers and sources of variation. b) Improve stage=, fractional factorial designs are used along with center points to estimate the effects of curvature. 2) Complete factorial design a) Estimates the characteristics & interactions of all the factors in an experiment (+ ) analyzes the interactions of more than two factors at a time. Key Difference= fractional factorial design only looks at the interactions b/w 2 factors at a time but ignores "high-order interactions" (Interactions between more than two factors) which allows it to work faster

"U charts

"U charts 1) Type of control charts designed to handle attributes data. a) depicts the percentage of samples that have a particular condition in situations where sample sizes may vary and each sample may have more than one occurrence of the condition. b) Measure stage--- attributes data used to estimate process baselines( better to use a variable control chart for this purpose) c) . Improve stage----, U charts occasionally are used to establish target figures, though the relative paucity of errors (see the formulas) . In a U chart, the plotted statistic is , where nj is the sample size of group j. The centerline is , where m is the number of groups in the analysis. The upper control limits are and the lower control limits are , where is the average percent. "

Work breakdown structures Defined

"Work breakdown structures 1) Diagram that breaks the problem into manageable subparts or processes when a problem seems overwhelming, 2) Most useful during the define and analyze stages of DMAIC. a) Define stage----> work breakdown structures combined with Pareto analysis to organize problems before solutions are applied. ---Aids in eliminating less important areas of a project to ensure adherence to a schedule. b) Analyze stage---->,work breakdown structures organize all of the issues and complaints to be handled during the improve stage. ---> Suggests that different solutions should be applied to different parts of a general problem.

"Creation and interpretation of Nonparametric tests of equal means

"Creation and interpretation of nonparametric test on the equality of means: 1) H0: Medians of Population 1's & Population 2's are equal 2) H1:Medians of Population 1's & Population 2's are NOT equal (Median is preferable to mean in these tests because it indicates central tendency regardless of distribution--Nonparametic tests are distribution-free) . a) Use P-value to determine the probability of Type II error can be assessed. b) Reject Ho: -- calculated statistic > critical value of the test statistic. c) Accept H0: ---Calculated statistic < the critical value of the test statistic,. ***Rejection does not imply confirmation of the null hypothesis.

Creation of Affinity Diagram

"Creation of Affinity Diagram 1) Teams begin by identifying the target issue or problem. 2) Each member of the team list a few issues that impact this target. It is important to collect as many ideas as possible. 3) Categorization of the ideas collected into a general list (Disgard Redundant items ) 4) If argument about the logic of sorting, team leader should administer this part of the process calmly and proactively. 5) Once the categories have been labeled and filled, the group should come up with header cards for each title. The titles may be useful for nominal group technique or in the creation of a prioritization matrix. 6) If the process successful, then the main ideas listed on the header cards will indicate the most important drivers to be addressed for the achievement of objectives and goals. ****Note that affinity diagrams do not require objective information; instead, these diagrams are tools for organizing the subjective impressions of team members."

Customer service and complaint management

"Customer service and complaint management : 1) Information about customers comes from routine customer service and complaint management. 2) Executives in Six Sigma businesses commonly meet with customers personally. 3) Some businesses establish programs in which every employee spends a certain amount of time dealing with customers-----> prevents employees from pursuing goals that are distant from the concerns of the customer. ***Six Sigma businesses pay special attention to customer complaints. ---->Any reports of error or malfunction are assessed and organized so pervasive problems can be noted + makes it easy for customers to complain and for these complaints to be heard. ****Complaints are an excellent source of ideas for new Six Sigma projects.

"Deliverables of Project Charter

"Deliverables 1) A deliverable is any fundable benefit that enables the overall success or failure of the project to be assessed. 2) Establishing targets for deliverables allows the project to be judged on its own terms. 3) Common mistake-- listing aspects of the project's method as deliverables---Do not include data or statistical process controls obtained during the project as a deliverable. 4) Best to place deliverables in financial terms.

Documentation at the close of the control stage

"Documentation at the close of the control stage 1) Project Report summarizes final results of the Six Sigma project a) Begins with the project charter. b) Summary of the results from each DMAIC stage. c) Objective conclusions---supported with raw data & analysis---- indicate whether the short- and long-term goals were met --- found in report's appendix d) Outline of project's expenditures & cost -savings (+) an outline of control plan wh/ maintains project's changes in place. e) Recommendations or suggestions for future projects

Effective change agents and issues that diminish buy-in from stakeholders

"Effective change agents and issues that diminish buy-in from stakeholders 1) To be effective change agents, Sigma team must first acknowledge the importance of having all employees on board with changes. 2) Change agents take conscious steps to increase buy-in, commitment, and participation. a) Buy-in may be lacking if the goals of the project are unclear. b) Explain how the proposed solutions will benefit them personally to ensure employees are more likely to participate and conform. c) If employees believe the project team has prejudices and preconceived solutions or if the project team lack regular communication with the employees, the employees will lose faith in the project. d) Finally, buy-in will be reduced if the project's goals are too diffuse. Having a number of small projects with single targets is better than one giant project with several targets."

Elimination of NVA activities

"Elimination 1) Analyze stage of DMAIC, the Six Sigma team will be alert to NVA activities that expand cycle time b/c creating nothing valuable for customters ( eg. requiring authorizations for routine task--NVA activity that inc. cycle time) 2) Checklists requiring employees to complete paperwork at every step in a process.are unnecessary and time-consuming."

"Full analytical method of creation

"Full analytical method of creation: 1) List options and the criteria required to evaluate those options. 2) It Phrasing the criteria as targets such as lower cost or greater employee satisfaction is important 3) First matrix = comparisons of every criterion with every other criterion. ***Six Sigma teams uses a numerical rating system in which 10 (most importan), 1 the average level of importance, and 1/10 the least importance. 4) Second matris=, each option then is evaluated with respect to each criterion. Each option receives a final score compiled of his or scores on the weighted criteria. *****Most statistical software programs provide a summary matrix in which all of the options are ranked."

"Gantt charts

"Gantt charts 1) Identify the critical path of each dependent task in a particular project. 2) Includes everything that has to be done to maintain the project's timing. 3) Any process not on the critical path may be slightly delayed without extending the overall duration of the project. These activities are said to have built-in slack time. 4) At the beginning of a Six Sigma project, distinguishing the tasks that are on the critical path is important. 5) Gantt charts typically is restricted to the define stage of DMAIC. ---> identify those tasks that, if truncated, would lead to an overall cycle time reduction. + place all project activities on the schedule. 6) May be used during the improve stage to confirm the reduction of critical path cycle time.

"Interaction plots

"Interaction plots 1) Ilustrate the interrelationships of three parameters (two factors and one response) . 2) Analyze Stage----Interaction plots useful for evaluating the results of designed experiments and multiple regression. a) x-axis = plot variables & Y-axis= responses b) Levels of interaction in the variable = The distinct , unparallel lines on the plot= interaction exists . f) If the plot variables exhibit no interaction---> parallel lines ( means that both plot variables will produce similar trends when combined with the response variable)

"P charts

"P charts a) Type of control chart primarily used for attributes data. b) Measures the percentage of samples with a particular characteristic in situations where sample size may vary and the sample unit either will have the characteristic or not have the characteristic. Measure stage---->, Estimate the process baseline from the attributes data. (best to use variables control chart) Improve stage---->, P charts used to estimate the number of errors. ****Like Np charts, however, they are not ideal for this purpose because of the low number of errors. P charts can be generated by Minitab, Excel, and other statistical software programs. "

Process decision program charts

"Process decision program charts 1) Break a process down into its component tasks, with special emphasis on potential problems and solutions. 2) Brainstorming Tool protocols for handling crises that emerge during processes. 3) Analyze stage---> effective for identifying the underlying causes of recurrent problems. 4) Improve stage--->, effective at linking possible problems with probable solutions, so contingency plans can reinforce process. 5) Process decision program charts structure: a) Process name (top) b) Steps required to complete the task are found. below this top line the chart. c) Order of the steps from right to left. Below each step, one or two sub-steps will list some potential problems. Provisional solutions should accompany each of these possible problems."

Process for Gauge repeatability and reproducibility analysis

"Process for Gauge repeatability and reproducibility analysis 1) When analyzing gauge repeatability and reproducibility (R & R), using samples obtained during actual operations is essential. 2) Each appraiser should receive the samples in a random order, although someone other than the appraiser should be able to identify the samples. 3) In a standard gauge (R & R),---> 3 appraisers & 10 samples:---> each appraiser measures each sample 3 times. a) Regardless of the arrangement of the test ----multiple trials for each sample need to test for repeatability. b)Large sample size is preferable but analysis is costly (limiting factor)--->. Again, it is essential to obtain a representative sample so all the potential sources of variation in the process are evident."

Project statement, scope, and deliverables of Project Charter

"Project statement, scope, and deliverables Project charter = documents project details and can be revised; hallmarks of the DMAIC structure, and Six Sigma itself, is a willingness to rethink first principles at any time. 1st step in project charter= Clarify the problem and why the project necessary a) Includes deliverable (actual output and in a manner approved by the financial department of the organization. b) Must confirm the reasonableness of the estimate with the finance department before writing it into the charter. c) Should include a reasonable estimation of the project scope, based on the data collected.

36. When a batch sample has upper and lower specifications, which statistic is used in the creation of a process performance index? a. Pp b. Ppk c. Ppl d. Cp

"36. A: Pp. When a batch sample has upper and lower specifications, the Pp statistic is used in the creation of a process performance index. If the batch sample has either an upper or a lower specification but not both, the Ppk statistic may be used. If the distributions are not normal, the Cp statistic is used to calculate the process performance indices. "

"48. Which statistical distribution is appropriate for continuous data with neither an upper nor a lower boundary? a. Lognormal b. Weibull c. Exponential d. Normal

"48. D: Normal. A normal distribution is appropriate for continuous data with neither an upper nor a lower boundary. Continuous data is obtained through measurement. A lognormal or Weibull distribution is appropriate for sets of continuous data with a fixed lower boundary but no upper boundary. In most lognormal and Weibull distributions, the lower boundary is zero. An exponential distribution is appropriate for continuous data sets in which the values are relatively consistent.

Six Sigma Team

"A Six Sigma team 1) work to minimize the chances and significance of human error in processes by dividing human error into three categories: willful, technique, and inadvertent. 2) Willful errors are made on purpose by disgruntled employees can be difficult to predict and stop--Solution=keep employees happy. 3) Technique errors are limited to a particular task/ employee and occur because poorly trained employees, poor understanding of processes, or due to lack of employee's awareness. 4) Inadvertent errors (generally rare) are the normal variations that occur in any human endeavor. ----Complete Error elimination requires that person stop performing the task. 5) Ways to eliminate these errors: a) Automating the process b) Conform the workspace to the physical dimensions of the person so movements are more natural.

Cause-and-effect diagrams

"Cause-and-effect diagrams 1) Depict the reasons for a particular even or lists the causes and effects listed in distinct boxes with arrows pointing from causes to effects. a) A cause can have more than one effect, and an effect can have more than one cause. 2) Analyze stage---->, they are used to create innovative lists of possible process factors to be explored further in a designed experiment. 3) Improve stage---->, cause-and-effect diagrams are used to create lists of possible failure modes that must be considered when crafting a solution.

Common problems and solutions for teams

"Common problems and solutions for teams 1) Define the team roles early in the project to reduce team conflicts.(leader's role) 2) Alternatively, have team members to settle into roles as they become familiar with one another --at the risk of possible conflict 3) Team leader responsible for keeping members are not held accountable for their actions. 4) Teams should meet only when necessary but should stay in regular communication throughout a project. 5) Team leader must prevent meeting from being hijacked by members who digress, go off on tangents, or rudely reject the ideas of others. 6) Ensure sufficient number of experts on the team to prevent "Groupthink"---- team member are reluctant to disagree or do not have enough experience to come up with alternate solutions.

Contour plots

"Contour plots 1) Groups of curves assigned a constant value according to a fitted response. a) Each curve' s path relates to values that have been separated at regular intervals. b) Any additional factors are placed on the chart according to their mean or some other value. 2) Improve Stage ---Contour plots used in response surface analysis to estimate the maximum and minimum responses associated with particular ranges of data. a) If only first-order main effects----> parallel lines , on the contour plot, separate from one another by equal distances. b) Curved Contour lines= interactions occur between the responses

"Creation and interpretation of Goodness-of-Fit tests

"Creation and interpretation & Goodness of Fit 1) Distributional fit = best tested using minimum 200 data points 2) Process out of statistical control--- no distribution will fit its data points. 3) Kolmogorov-Smirnov test a) Actual data points vary slightly from the curve that best charts their values. ---This difference should decrease as the number of data points rises. b) this test checks if an inverse relationship exists. 4) The Anderson-Darling test a) Emphasizes data from extreme ends of the distribution. a) Normal distribution= the mean should be approximately the value of the line as it passes 50th percentile. b) Excess data in tails of the distribution---> excess points above the confidence interval ( left side) and below the confidence interval ( right of curve). c) Goodness-of-fit tests H0: Data does follow the distribution in question. ***** Larger p value indicates better fit.

"Creation and interpretation of Fishbone diagram

"Creation and interpretation of Fishbone Diagram 1) 1st step=make a provisional list of the possible relationships between the process and the outcome. 2) Phrasing the target in terms of a problem is better than phrasing the problem in terms of a goal. 3) All of the factors that influence a main problem will branch off from that main problem. For some problems, beginning with either the four Ps (people, plants, policy, and procedures) or the 5Ms and E (measurement, material, methods, machines, manpower, and environment) will be appropriate. 4) The main causes should be connected to the problem, and subordinate clauses should be connected to the main causes. 5) In especially detailed diagrams, several hierarchical levels of causes may exist. 6) Cause-and-effect diagrams are primarily brainstorming tools, not final assessments, so it is important to include many different possible answers. 7)Once all of the causes and effects have been outlined, the group should examine the diagram to see whether certain categories exhibit a preponderance of causes. These categories may require further subdividing, or they may become targets for problem-solving efforts.

Creation of Np charts

"Creation of Np chart 1) Gather sample data at the same point in each process and make sure that every sample should has the same number of units. 2) Plotted statistic = the number of items in the sample that have the particular characteristic. 3) Centerline of the chart is calculated with the following equation: ( lookup equation) , in which m is the number of groups involved in the analysis. 4) Lookup equation for lower & upper control limits 5) In each of the control limit calculations, the sample size is n and the average count is . The value for is calculated . "

Critical-to-cost metrics

"Critical-to-cost metrics 1) Identifies areas of a process that significantly raise the expense --- should include the typical cost of a task, increased cost of errors in the performance of task, and likelihood of an error should be included in the metric. Examples = the metric for how long it will take to redo or repair a product made incorrectly + the cost of keeping materials on hand if lag time present in the process . 2) The effects of errors on cost tend to multiply as the product moves farther down the path.--- mistake design phase will be cheaper to resolve than manufacturing phase error. 3) Cost of quality = The amount of money required to align a product or service with quality baselines i "

"Inventory

"Inventory 1) In lean methodology, inventory and work-in-progress are setbacks on progress---> strives to respond quickly to customer orders without relying on the storage (little inventory) or works-in-progress. 3) A general habit involves finishing work in progress before handling new orders, which slows down production. 3) Lean emphasizes other strategies for improving velocity. ( ie. increase the number of completions per hour or eliminate redundant or unnecessary process steps) 4) Velocity improved by decreasing errors, waiting time, or decreasing need for personnel or moving resources and indirectly increases value.

"Swim lane" Mapping

"Swim lane" Mapping a. process mapping involving multiple departments or functions. b. Key difference from typical process map= process blocks are arranged aligned with the department or functions that perform a given process step. c. Same components as basic flowchart (process, decision points, inputs & outputs)

4 Stages of the QFD Cycle:

4 Stages of the QFD Cycle: i. translating customer input and competitor analysis into product/ service (basic design elements) ii. Translate product/service features into specifications and measures. iii. Translate product/service specifications and measures into process design features. iv. Translate process design features into process performance specifications & measures.

"I" phase of IDOV

6) IDOV a. Method used for testing and validating business process design. Identify Phase (links design to voice of customer) i. Establish business case ii. Roles & Responsibilities iii. Milestones. iv. Find the technical requirements ation.

7 Benefits of the QFD Process

7 Benefits of the QFD Process 1) Creates customer-driven environment 2) Reduces the cycle time for new products 3) Uses concurrent engineering methods 4) Reduces design to manufacture costs (fewer changes) 5) Increases communication across cross-functional teams 6) Creates data for proper documentation of engineering knowlege 7) Establishes priority requirements and improve quality.

A Green Belt determines that there are several potential sources of process variation. If the engineer wishes to categorize related causes into families to reveal the largest casual factors, the most appropriate tool to use is: FMEA Binary Logistic Regression Multi-Vari study DOE

ANSWER= MULTI-VAR!! An FMEA is useful for identifying potential failures. It does not identify potential sources of process variation. A Design of Experiment (DOE) is used for designing and conducting experiments to determine independent and response variables.

Attributes Chart:

Attributes Chart: a. Larger subgroup sizes (50-200) required to get detectable process shifts. b. Subgroup sizes should not vary more than 20% c. Recalculate control limits if the sample size changes.

Benchmarking Companies compare their own performance against 4 things

Benchmarking Companies compare their own performance against 4 things: 1) Best in class company performance 2) Companies recognized as industry leaders 3) Company's toughest competitors 4) Any known process that is superior to the company's

Benefit of Project Risk Analysis

Benefit of Project Risk Analysis include determining the impact severity if risk occurs and the probability that risk will occur. 5) Project risk analysis done early in planning stage of the project life cycle.

Box-and-whisker plot

Box-and-whisker plot a. Pictorial vies of minimum, maximum, median, and inter-quarantile range in one graph b. Better than distribution plots because easier to fine the outliners and gives more information. c. Normally distributed data= center line located in the middle c. Skewed data (unequal whisker or presence of outliner data outside the whiskers) d. Structure: i. Quarantile (4) set of data denoted as Q1, Q2, Q3, Q4 ii. Q2- median

Control Chart's Role in Maintenance of Statistical Control:

Control Chart's Role in Maintenance of Statistical Control: i. Expected range of variation will be the established standard against which other samples are compared. ii. Provides traceable evidence in detection of process changes in 3 ways: 1. Collection -→ Run the process and collect the data for plotting on graph/chart. 2. Control-→ calculate control limits to support analysis and establish process variability 3. Capability-→ ability of the process to meet customer specifications

Control Charts

Control Charts Advantage over Control Charts---> Calculated control limits facilitates the ability to detect special cause variation

A company conducted an on-site supplier workshop. In one exercise the suppliers were instructed to walk through the factory and write down examples of locations of excess inventory, unnecessary movement of material, and unneeded employee movements. The information gathered is an example of which of the following? Kaizen blitz Defects Audit findings Muda

Correct!---Muda translates to "waste" and has several categories. In this case, waste of material, waste of transportation, and wasted motion. A kaizen blitz is an offshoot of kaizen, in which an intense focus is on one activity and implements significant improvements Remaining choices are not relevant

Cost/Benefit Analysis: Costs Due to Nonconformance Curve

Costs Due to Nonconformance Curve i. The cost of nonconformance curve represents the costs or inherent losses associated with not having a quality assurance program. These costs are highest at first, demonstrating the high cost of not conforming to a quality assurance program. Then, as you move to the right, the curve drops steadily, representing the costs dropping as you institute a quality assurance program in the production process. The curve showing the drop in quality assurance, therefore, swings from high (no quality assurance program = costs to the business) to low (fewer to no costs with a quality assurance program).

Creative Phase of Brainstorming

Creative phase (generation of ideas) 1. No criticism or distraction allowed to ensure generating many ideas and to keep open mind 2. Can use either facilitation or free-wheeling (equally effective) 3. Wild ideas are welcome 4. Quantity is more important than Quality 5. Hitchhike—build on precious ideas

Customer Requirements

Customer Requirements

DFSs & Stage Gate Process Clarification

DFSs & Stage Gate Process Clarification 1. Stage gate process--used to screen and pass projects as they progress through development stages and fulfill the project requirement 2. Gate= management review of the project at each stage to decide whether to kill or keep project going sot aht project team efforts are not diluted and company resources are not overloaded.

Function of Flow Chart?

Flowchart used to design/ map the process

Measurement Scales---Nominal

Measurement Scales---Nominal 1) Central location = Mode 2) Dispersion of Information only 3) Significance Test= Chi-Square 4) Data consists of names or categories without any ordering scheme

Methods used to identify the process/project's customers:

Methods used to identify the process/project's customers: a. Brainstorming b. SIPOC c. Marketing analysis data d. Tracking a product/service to delivery

Multiboidin & the method

Multi-voting. a. Typically used with NGT but can also be used alone or with brainstorming technique Method= consolidated ideas are prioritized according to their level of significance to the problem. Facilitator must restate the object and refocus team to list from an ASQ perspective to prevent multiple ideas with many checkmarks. End result is 5 ideas selected either via weighted (ranking) or weighed approach (rating

Multiple Regression ---1st order and higher order

Multiple Regression ---1st order and higher order 1) First Order---->value of the dependent variable is influenced by each factor by itself as well as combination of 2 factors----> Produces a straight line over small regions so best used for only targeted data (flexing of the plan is due to impact of interacting factors) 2) Higher-Order -----> Includes squares and cubes of the value which produce a response surface with peaks and valleys-----> best for defining the area AROUND the stationary point and for evaluating how current operating parameters influence the response.

2. Experimental Objectives ANOVA , Alpha risk, Beta risk

Must use ANOVA to check if the perceived difference of "high" and "low" results are statistically significant difference in the dependent variable (and not due to errors) d. Alpha-risk= probability that analysis will show that there is a difference e. Higher the power of the experiment, the lower the beta risk f. Higher number of replications or a larger sample size-→ reduced beta risk or more precise experiment.

Process Capability for Attributes Data

Process Capability for Attributes Data a. Discrete (Attribute Data)--→ Binomial process capability= process capability is simply the average of proportion defective which is then converted to process sigma Z. b. Percent defective= percentage of parts sampled during the final inspection that are defective i. Percent Defective (100 X Average P) ii. Process Z= capability index computed from the average P by finding the value from the standard normal distribution such that the area to the right of the value is the average P c. Chi-Square Distribution—used to calculate the confidence intervals for capability indices (calculation is not important) *** If the data is normal but not stable-→ use Pp and Ppk. **** If the data are normal and stable-→ use Cp and Cpk

Process Decision Program Charts

Process Decision Program Charts 1) Used to chart the course of events that will take us from start point to final complex goal. ---> possible uncetainity of achieving intermediate events which could derail us from project completion . 2) Application a) New/ Unique/ complex Problems or those with sequence of difficult and challenging steps b) Can be used to create contingencies and counter problems available to the team.

Process Improvement Teams

Process Improvement Teams 1. Employees from more than one department look into the flow of materials & semi-finished good needed to streamline processes.

Process Mapping

Process Mapping 1) Key advantage = provides visual presentation of the process being described with the use of symbols rather than the clutter of words 2) Used to outline new procedures and review old procedures for viability and throughness. 3) Multiple Flowchart styles ----> person-to-person, action-to-action, and conceptual

Process Metrics

Process Metrics 1. Information ---data from production, people, and machinery ---that operators & supervisors need to run normal operations 2. Central focus of the MAIC phses of 6 sigma after the improvement project defined.

Define Process Modeling

Process Modeling 1) Definition of Process= set of interrelated or interacting activities that transforms inputs into outputs. 2) Use of shapes & symbols facilitates understanding of a process. 3) Process map documentation= document with process information sent to suppliers & customers.

Process capability, Common Cause , Special Cause Variation.

Process capability= ability of the process to meet the expected specifications a. Every process has variation due to common causes and special causes, both internal and external to the process. b. Common Cause variation (random process variations) = influence process capability c. Special Cause variation (Assignable causes)= removed before estimating process capability.

A process that crosses through multiple departments in a company can present challenges for improvement efforts because: A) It slows the progression of normal team-building B) Cross-functional teams dilute the knowledge of the group C) Management support is more difficult to obtain. D) Two or more areas may think they own the process

Processes that travel through multiple departments (such as product design, which can go through Marketing, Engineering, and Manufacturing, for example) may have each department believing they own the process, and have final decision authority on changes. Cross functional teams are preferred as they help approach a problem from multiple angles.

Project Scope

Project Scope 1) Function= ensures common understanding of team's goals 2) Based on the problem statement

Provide 6 examples of Flow Chart Application

Provide 6 examples of Flow Chart Application 1) Purchasing--->processing purchase orders, placing actual purchases, vendor contract negotiations. 2) Manufacturing---> Processing returned goods, handling internal rejections, production processes, training new operators. 3) Sales----> Making sales call, taking order information, advertising sequences. 4) Administration---->Correspondence flow, processing times, correcting mistakes, handling mail, typing letters, hiring employees. 5) Maintenance---> p.m. scheduling and work order processing 6) Laboratory---> Delivery of samples, testing steps, selecting new equipment, mapping work flow

Random Sampling

Random Sampling Definition---every unit in the population has the same chance of being selected. 1) Best for time or economic constraints. 2) Requires samples to be representative of the lot and not just the product 3) Sampling without randomness make the plan ineffective. ----Sampling sequence must be random as well

Risk Management Phases & Definitions ---Risk Assessment

Risk Management Phases & Definitions ---Risk Assessment Definition-----> Identify/ Analyze Project Risks to increase the ability of meeting cost, performance, and schedule requirements Inputs: "R- SALE" ---Review of plans, Similar systems, Assessments, Lessons learned, Expert Interviews

Six Sigma Structure options---Process Owner

Six Sigma Structure options---Process Owner Champion of Sponsor

Six Sigma Structure---Team Member

Six Sigma Structure---Team Member Green Belt, Associate with team training, Associate with process knowlege

Six Sigma Structure options--Six Sigma Management

Six Sigma Structure--Six Sigma Management Six Sigma Manager, Master Black Belt, Six Sigma Director

Six Sigma Teams

Six Sigma Teams Structure= 8-12 members with black belt or master black belt support. 1) Not all companies have the same Six sigma team structure 2) Very close to description of ad hoc teams but includes master black support

Specification limits verses Control limits:

Specification limits verses Control limits: a. Control limits are calculated from process data. They represent how your process actually performs. Specification limits are defined by your customer and represent the desired performance of your process. b. Specification limits and control limits are used for different purposes. Control limits let you assess whether your process is stable. Specification limits allow you to assess how capable your process is of meeting customer requirements.

"Sponsors

Sponsors 1) Provide guidance. & funding to the project, 2) Gives the team members access to resources, and inform other managers about the project's existence and intentions. 3) Creates enthusiasm and support for the project, so the project will receive any possible assistance from other employees 4) Select sponsors from the departments most relevant to the project. --->avoid selecting sponsors who are already involved in several other projects. 5) If the project participants are drawn from a number of different departments,---> draw the sponsors from the ranks of those managers who oversee multiple departments. to ensure project team receives all the required support.

Team Dynamics: Unquestioned acceptance of opinions & facts

Team Dynamics & Unquestioned acceptance of opinions & facts a. Symptoms= Members present information without backing up data or anlysis b. Cause= organizational culture or lack of management by facts c. Countermeasures= Team leaders request for data analysis and conclusions that are statistically valid. Assumptions behind analysis are questioned.

Team Life Characteristics

Team Life Characteristics Build (Forming/ Storming) --->Develop Phase (Norming) ---> Optimize (performing

Team Life Characteristics--Build Phase

Team Life Characteristics--Build Phase (Forming/Storming) 1. Group is uncertain & lacks cohesivenss---> Hard to reach a consensus 2. Leadership= high task/ high relationship style

Team Problems & Solution----Opinions as Facts

Team Problems & Solution----Opinions as Facts 1) Members present opinion as fact --> Ask for support data 2) Members make unfounded assumptions --> Question opinions & assumptions 3) Self assurance seen as unquestionable --> Groupthink

Team Problems & Solution----Shy Members

Team Problems & Solution----Shy Members 1) Members reluctant to speak up ---> structure group participation . 2) Fear of making mistake ---> direct the conversation their way.

The X-bar & R-charts chart is advantageous in the following situations:[2]

The X-bar & R-charts chart is advantageous in the following situations: i. The sample size is relatively small (say, n ≤ 10—X bar and s charts are typically used for larger sample sizes) ii. The sample size is constant iii. Humans must perform the calculations for the chart

Company that created LEAN?

Toyota-→ LEAN

Two reasons why the use of quality circles is declining

Two reasons why the use of quality circles is declining 1. Quality Circle has a Japanese connotation 2. Most circle projects tend to employee selected, while most team efforts are management selected but team directed.

Variable Data/ Continuous

Variable Data Any real number----4.69, -1.4, --->measurable data that tell how long, what volume, or how much? 1) Measured data is more precise and more informative than counted Data but is expensive to collect.

What is most important for Improvement Teams?

What is most important for Improvement Teams? Diversity is most important for improvement teams because a single person can't remove a problem and multiple perspectives are beneficial.

Define the 3 types of actives mapped in the value stream map

1) Value-added activities= create value for the customer. 2) Business-value-added (BVA) activities do not create value for the customer, but are necessary anyway (Type 1 waste) ---->. examples of BVA activities are marketing, design, and delivery. 3) Non-value-added (NVA) activities create value for neither the customer nor the business. ----> Six Sigma project goal is to eliminate NVA activities

History of Design of Experiments

1. History of DOE a. Structured, organized method that is used to determine the relationship between different factors (X's) affecting a process & output of that process (Y). b. Use of DOE in agriculture

20. In response surface analysis, which of the following values for s and t weights would indicate that the upper and lower boundaries are more important than the target? a. -0.3 b. 0 c. 1 d. .7

20. D: 7. In response surface analysis, values of .7 for the s and t weights would indicate that the upper and lower boundaries are more important than the target. In Phase 2 of response surface analysis, the s and t weights are based on the relationship between the target and the boundary. When the target and the boundary have equal value, the s and t weights are 1. When the target is more important than the boundary, the s and t weights are between 1 and 10. When the boundary is more important than the target, the s and t weights are between 0.1 and 1

Analysis of customer data

3) Analysis of customer data is always categorized as observational studies.--→ with observational studies, high correlation doesn't imply cause & effect. Best to compare internal predictive metrics with customer feedback data.

41. Which Six Sigma methodology is more appropriate for existing processes? a. DMADV b. IDOV c. DMAIC d. DFSS

41. C: DMAIC. The Six Sigma methodology of DMAIC (define, measure, analyze, improve, and control) is most appropriate for handling existing processes. It is geared toward gradual improvement. DMADV (define, measure, analyze, design, and verify), on the other hand, includes a design phase, during which new products can be developed. On occasion, DMADV is used to give existing products a large-scale remodeling. IDOV (identification, design, optimization, and validation) is the primary methodology of DFSS (design for Six Sigma). The main difference between DFSS and DMAIC is that the former attempts to prevent rather than reduce defects. "

5 Advantages of Multi-vari Studies

5 Advantages of Multi-vari Studies 1) Can dramatize the variation within the piece (positional) 2) Can dramatize the variation from piece to piece (cyclical) 3) Help track any time-related changes (Temporal) 4) Minimizes variation by identifying areas to look for excessive variation 5) Identifies areas not to look for excessive variation.

5 Benefits of using VOCS

5 Benefits of using VOCS (collection of customer needs and perceptions) 1) make decisions on products/services 2) Id. product features and specifications 3) Focus on improvement plan 4) Develop baseline metrics on customer satisfaction 5) ID customer satisfaction drivers.

5S Methods

5S Methods a. Definition= process impacted by its environment are improved by changes in the general state of work area ( access to hand tools, cleanliess, lighting, housekeeping status

Green Belt

6) Green Belt a. Full-time position that requires training in 6 Sigma, basic statistical tools& process improvement, lean enterprise synergy, and presentation skills. b. First level of certification where they are trained for two weeks in all the essential methodologies. Upon returning to their previous jobs, these Green Belt certified professionals are likely to have additional responsibilities integrating what they have learned in their work processes.

Experimental Design & Interaction Effects

7. Interaction Effects a. DOE approach screens large number of factors with highly fractional experiments. b. Interactions occur when effect of one input factor on the output depends on the level of another input factor.

Calculate the FMEA RPN from the following data: Probability of occurrence = 6 Severity = 4 Detection = 3 A) Cannot be determined with information provided. B) 13 C) 27 D) 72

72 is the correct answer based on the formula for RPN, which is RPN= [occurrence * severity * detection]= (4) (3) (6)

A CP value less than 1 means?

A CP value less than 1 means that the process is wider than the specification, with defects spilling out over the edges. A CP value greater than 1 means that the effective width of the process variation is less than the required specification, with fewer defects occurring.

Which of the activities in this process flow diagram that must be non-value added?

Activities D and E are part of a rework loop. Activity D is an inspection activity and activity E must be a re-work activity for products to pass activity D. If activities A, B, and C are performed correctly, activities D and E don't add any value.

Activity Network Diagrams

Activity Network Diagrams 1) Methodology that includes program evaluation and review PERT, critical path methods, node/ activity on node diagram (AON), precedence diagrams, and other network diagrams 2) Lot of PERT and CPM techniques in usage---to help monitor, schedule, modify, and reciew projects.

Advancing order of attainment of benchmarks by Juran

Advancing order of attainment of benchmarks by Juran: 1) Customer Specification 2) Actual Customer Desire 3) Current Competition 4) Best in Related Industries 5) Best in the World.

Which of the following is a key difference between an interrelationship digraph and an affinity diagram? A) The interrelationship diagraph uses a flowchart for relationships while the affinity diagram uses a matrix for relationships. B) The interrelationship digraph uses open ended questions while the affinity diagram starts with closed end questions. C) Interrelationship diagraphs should be used with seven or more concerns while the affinity diagram should be used with fewer than seven concerns. D) The affinity diagram arranges ideas into groups, while the interrelationship digraph intends to specify the ways the ideas influence one another.

Affinity diagrams organize ideas into their natural relationships. Interrelation diagrams organize by cause-and-effect relationships.

Analysis of Variance Method

Analysis of Variance Method Most accurate method for quantifying repeatability and reproducibility that still allows the variability of interaction between appraisers and the parts to be determined

Analytical (inferential) Study

Analytical (inferential) Study i. Involves making inferences, hypothesis testing, and making predictions. ii. Data taken from sample used to make estimates or inferences about the population which the sample was drawn. iii. Tools used include hypothesis testing and scatter diagrams to determine the relationships between variables and make predictions using regressions equations.

Analyzing Process Capability

Analyzing Process Capability 1) Predictable pattern of statistically stable behavior where the chance causes of variation are compared with engineering specifications---esp. to judge the suitability of the process 2) Capable Process-------->Bell-shaped spread that is narrower than the tolerance range. 3) Three steps in process capability study: a) Planning for Data Collection b) Collecting Data c) Plotting and analyzing data

Which of the following was one of Shewhart's contributions to the foundation of the Six Sigma methodology and tools? A) He experimented, identified, and documented the concept of the "Hawthorne effect." B) He created and used control charts, and he introduced the concept of statistical quality control. C) He was the first to describe and define the Six Sigma DMAIC approach to continuous improvement. D) He first promoted the idea that any process variation from a desired target value was a loss to society.

Answer=Control Chart= Shewart While he worked at the Hawthorne plant, the so-called "Hawthorne effect" was not documented by Shewhart. The reality of the "Hawthorne effect" is also still debated.

Data that consists of classifications rather than measurements is referred to as: Variable Continuous Interval Attribute

Attributes are classifications, such as good/bad, big/small, white/black, etc. Interval is a measurement scale involving the difference between any two successive points is equal (i.e., temperature).

Attributes of sample data: Sample shape & Kurtosis

Attributes of sample data: a. Kurtosis is a measure of whether the data are peaked or flat relative to a normal distribution. That is, data sets with high kurtosis tend to have a distinct peak near the mean, decline rather rapidly, and have heavy tails. Data sets with low kurtosis tend to have a flat top near the mean rather than a sharp peak. A uniform distribution would be the extreme case. b. The histogram is an effective graphical technique for showing both the skewness and kurtosis of data set. vii. The skewness for a normal distribution is zero, and any symmetric data should have a skewness near zero. Negative values for the skewness indicate data that are skewed left and positive values for the skewness indicate data that are skewed right. By skewed left, we mean that the left tail is long relative to the right tail.

Attributes of sample data: Skewness of the sample shape

Attributes of sample data: iv. Skewness is a measure of symmetry, or more precisely, the lack of symmetry. A distribution, or data set, is symmetric if it looks the same to the left and right of the center point.

Attributes of sample data: Variation

Attributes of sample data: Variation (dispersion or spread of sample) i. Dispersion of data is quantified with either the sample range or sample standard deviation.

Attributes of sample data:

Attributes of sample data: a. Variation (dispersion or spread of sample) b. Shape of sample c. Center of sample d. Normal distribution.

Which of the following factors is considered in the assessment of risk? Outsourcing Potential for financial loss Relocation of facilities Insurance premiums

B= CORRECT

Define Balance ScoreCard

Balance Score Card---management concept that helps managers to monitor their results in key areas

Balanced Design

Balanced Design A Fractional factorial design in which an equal number of trials (at every level) is conducted for each factor

Baldrige Award

Baldrige - award by US. Congress used to raise awareness of quality management and to recognize companies that have successfully implemented quality management. 1. Named after the Secretary of Commerce 2. Award is managed by the US Commerce Department's Nation

Benefits of Batching

Benefits of Batching: 1) Good strategy for efficient processing materials or performing activities setup time the equipment is significant a) Cheaper to make a large batch of soup b) Shipping of small items---buying bulk is cheaper

3 Benefits of DFMEA:

Benefits of DFMEA: i. Objective analysis ii. Synergy (new thought or potential failures identified) iii. Higher chance of finding failure modes and their associated effects.

Benefits of Process FMEA:

Benefits of PFMEA: i. Better customer satisfaction ii. Improved employee morale iii. Reduced bottom-line costs

Causes of non-linearity:

Causes of non-linearity: i. Instrument not calibrated properly at both the high and low end of the operating range ii. Error in on e or more of the master part measurements iii. Worn instruments iv. Characteristics of the instrument design. c. Average percent bias has P-value= 0-→higher significance of bias issues with the instrument. (Report also gives a breakdown of percent bias at the measurement points covered during the study)

Control Charts for Variable Data

Control Charts for Variable Data 1) Line graphs with dynamic (Changing ) process behavior 2) Requires 25 rational subgroups of subgroup size 4 or larger to calculate the UCL and LCL but do require periodic subgroups to continue to monitor the process.

In an analysis of DOE results, which of the following graphic techniques would best show the dependency effects of a range of input variables against the response variable? Histogram Run-sequence plot of residuals Interaction plot Scatter plot of residuals

Correct! Interaction plots show interaction effects of a range of input variables against the response variable. A run-sequence plot would show trends in data. Scatter plots show relationships between two variables; DOEs often have many more than two variables. Histograms show frequency distribution and would be best used to identify if there were outliers and shifts in response variation.

For a time-ordered variables control chart, what is the primary reason why subgroups should be kept as small as possible? To save time. To have the minimum opportunity for variation within a subgroup. So ranges could be used instead of standard deviations. So control limits may be narrower.

Correct!To minimize the variation within the subgroup. The time-ordered variables control chart is most effective if the variation occurs between subgroups rather than within a subgroup. Ranges are used because they are easier to calculate by hand, but the standard deviation is a better measure of spread since it uses all of the data. When subgroups sizes are smaller, the control limits are wider.

Crosby credited his business training to this man

Crosby credited his business training to this man" Harold Gennen

Two people connected Quaility with management

Crosby---14 step approach to quality improvement Deming---14 obligations of top management

Auditing

D. Auditing 1) Independent assessment of processes and/or projects against established plans & goals 2) "Quality Auditing" -checking process in HCOs with quality management systems (AS9100, ISO/TS 16949) or those applied for quality awards (Baldridge) wledge, emergency procedures, & cleanliness.

A total of 4000 closures were manufactured on machine #7. 200 closures had one defect each. A total of 3800 closures were packed. What are the defects per unit (DPU) and throughput yield? 1 DPU and 99% throughput yield 0.0526 DPU and 94.7% throughput yield 0.05 DPU and 95% throughput yield 1.05 DPU and 95% throughput yield

DPU is determined by 200/4000 = 0.05 and throughput yield is determined by 1.00 - 0.05 = .95 x 100 = 95%

Data Coding by Substitution

Data Coding by Substitution 1) Consider a dimensional inspection procedure in which the specification is : [(nominal) +/- (deviation from nominal)] 2) Measurement Resolution = 1.8 inch 3) Inspectors record the Standard Deviations 4) Data can be coded as integers expressing the number of 1/8 increments deviating form nominal. 5) Example of data entry (32 - 3/8 inches) crammed onto check sheet----> Recommended Solution-= Either make the check sheet blocks bigger at the risk of having fewer samples and plot points per page

Difference between Theory of Constainsts & Kanban

Difference between Theory of Constainsts & Kanban: 1. Toyota's Kanban is a way to prevent excess production, but most people think that preventing production is a waste. It is the opposite: using excess production is a waste. 2. A common mistake is trying to keep all the workers busy. Labor is a cost, it might seem intuitive that you should keep them busy. However, if they are busy on a non-bottleneck function, and that prevents them from getting to a bottleneck function quickly, it can decrease the total output. It may be worth having some people sitting, doing nothing, so that when they are needed for the critical function they are immediately available. Idle time has been considered a waste in the past, but that is an illusion. Making an employee work and profiting from that work are two different things. 3. There are two big lessons: a) If you design capacity to exactly meet the need, when you have a sequence of dependent steps, then any failure, and flaw anywhere will compound along the assembly line. To keep production stable, later steps may need higher production capacity than earlier steps. b) Measuring the productivity of a part of the process is flawed because you can meet the required production speed in one place, but it does not matter if this does not contribute to making a sale. The only thing that matters is the final output that is sold.

Duties: of Process Owner

Duties: of Process Owner i. Select team members ii. Provide process knowledge iii. Review process management iv. Approve/ support management changes v. Implement change and ensures that improvement are sustained. vi. Resource allocation

Establishing Win-Win Plans

Establishing Win-Win Plans" Key objective of Win-win planning-->to help the other party to achieve its goals which in turn results in the negotiator achieving its goals. 1. Advance planning for the negotiator to determine his goals & objectives. 2. Must consider the goals of the opposition and seek alternate solutions to areas of disagreement.

Everyday Examples of Poke/ Yoke

Everyday Examples of Poke/ Yoke

Flow Charts

Flow Charts 1) Process map used to depict the procedures. 2) Often is the starting point for process improvement fo six sigma teams.

Force-Field analysis

Force-Field analysis 1) Method: a. Define the future state and brainstorm to create two columns (restraining & driving forces) b. NGT used to rank the items in the two columns. (aim to reduce the restraining forces) c. Team consensus provides guidance on how to proceed

Fractional

Fractional Term for fewer experiments than the full design calls for.

Gant Charts (Bar Charts) --4 DisAdvantages

Gant Charts (Bar Charts) --4 DisAdvantages 1) Inter-dependencies of activities (shown in PERT & CPM) not shown in Gantt 2) No way to indicate variation in the expected activity times 3) Activity details not indicated 4) Little predictive value to this presentation of data

The FMEA has the following RPNs. Which item should be addressed first? Item with RPN of 75 Item with RPN of 30 Item with RPN of 156 Item with RPN of 210

Generally the highest RPN is prioritized.

Green Belts

Green Belts 1) Not in full-time process improvements position (unlike black belts) 2) Basic Skills Mastery --->core statistical tools demonstrated in financial or customer benefits projects. 3) May be black belts in training and have less experience than full black belts 4) Operates under the guidance of black belts

Group think

Group think 1) Occurs when highly cohesive group publically agree with suggested courses of action while pricately having doubt about them 2) Strong group loyalty prevents members from evaluating or criticizing other's ideas--> risk of poor decision

Benefit of Written Procedures

Having written procedures for business and manufacturing process helps drive consistency.

Inventory (source of Waste)

Inventory a. Tempting time to let inventory level to increase is when the business cycle is in recovery b. In times of recovery, the best strategy is to synchronize production to increase with actual demand.

Lean Processes & Value

Lean states that : 1) value is the set of qualities or characteristics for which a customer is willing to pay. 2) The goal of lean is to increase value and diminish waste. Value always is assessed from the perspective of the customer.

Lean Processes & velocity

Lean states: 1) Velocity is the speed with which processes are completed. The ultimate measure of velocity is the interval between a customer's order and the delivery of the good or service. 2) In lean methodology, velocity is sometimes called flow. The goal of lean is to reduce process lead times and enable the business to respond quickly and agilely to customer requests

Marketplace Response is an Performance Measure

Marketplace Response is an Performance Measure: 1) Marketplace response is an organizational performance measure 2) Ability to respond to competitors quality , technology, product, design, safe are all important

Measures of Central Tendency

Measures of Central Tendency Different ways of characterizing the central value of a collection of data includes-------> Mean, Mode, Median Central Limit Theorem----> Probability distribution of the sample means approach normal distribution as the number of sample sizes increases, provided that they are simple random samples of uniform size.---> used for small sample sizes and when true distribution is unknown

Measuring Project's Success or Failure

Measuring Project's Success or Failure 1) Were the specified goals & objectives achieved 2) Within deadlines 3) At or below cost constraints 4) Utilization of allocated resources *** A project that accomplishes a large feat would still be a success irrespective of delays, over-budget, or failing to meet stated goals

Which of the following yield metrics most accurately reflects quality throughout a process? A) Final yield B) Throughput yield C) Rolled throughput yield D) First time yield

Rolled throughput yield is calculated based on the throughput yield or defects per opportunity at each step in the process. It considers all opportunities for defects. Final yield only looks at the defect-free units produced at the end of the process. Besides not including all of the opportunities for defects, it also ignores the rework that occurs throughout the process. First time yield only looks at the number of defect-free units produced and not all of the opportunities for defects contained in a unit. Throughput yield only measures the defects per opportunity at a specific step in the process and not all the steps in the process.

When the mean < median < mode the data in the distribution is: Skewed Right Positively skewed Symmetric data about the median Skewed left

Skewed left distributions have a mean < median < mode. Symmetrical (normal) distributions have mean=mode=mean. Positively skewed is similar to skewed right, In a skewed right distribution we have; mean>median>mode.

An organization is organized by function. An improvement team has concluded that the process they are working on involves several of these functions. Who would be the best person to address this situation? Team Leader Team Member Team Sponsor Team Facilitator

The Team Sponsor person must be senior enough to get the commitment of different functions. The Team Facilitator's responsibility is to facilitate a team once it has been formed, not to work through the politics of getting the right people on the team. The team leader is usually not at a high enough level to persuade various functions to participate.

Traditional Methods of obtaining customer information

Traditional Methods of obtaining customer information 1) Customer Scorecards 2) Customer or Supplier audits 3) Data mining 4) Data Warehouses 5) Targeted and multi-level surveys 6) Targeted and multi-level interviews

What is the interpretation of run tests?

What is the interpretation of run tests? 1) Run Test (4,7,8) ----> tell information about sampling errors 2) Runt Test (1,2,3,5,6) ------> identify shift in the process mean 3) Run Test #4------> suggest that samples have been taken from multi-stream process 4) Run-test #7----> tell sample stratification exists 5

Womack's 5 guiding principles

Womack's 5 guiding principles ** "Lean Thinking : Banish Waste and Create Wealth" --book that tells how to convert mass

Term: Layout of machines or business processes of different types, performing different operations in tight sequence, typically a U or L shape, to permit single-piece flow and flexible deployment of human effort

Work Cell Layout of machines or business processes of different types, performing different operations in tight sequence, typically a U or L shape, to permit single-piece flow and flexible deployment of human effort

Define Stage of DMAIC

a. Define (identification of issues causing decreased customer satisfaction 1. Management commitment -PDSA 2. SIPOC (suppliers, input, process, output, customer) a. Purpose= "At- a-glance" perspective for process or process improvement efforts b. Application: i. Identification of boundaries for the process or process improvement efforts ii. Understanding the scope of the process or process measurement efforts. iii. Determining key customers (internal & external) iv. Linking SIPOC maps to understand "upstream" and "downstream" process. 3. Systems thinking 4. Process identification 5. Flowchart 6. Project Management

Which of the following is the best tool for discovering relationships between two sets of elements? Process Decision Program Chart (PDPC) Force field analysis Prioritization matrix Matrix diagram

Answer= Matrix Diagram= A matrix diagram is typically used to discover and illustrate relationships between two groups of items. A Process Decision Program Chart attempts to understand risk and drive contingency plans. Force field analysis compares impediments and aids to project success, but not the relationships between these 2 groups A prioritization matrix will rank order a list of ideas, but is not aimed at understanding relationships outside of this limited context.

For a range control chart with a sample size of 2, the lower control limit is zero and the 3σ upper control limit is 4.5. The last point plotted on the chart was 5.2. This is an indication of what? The process is in control. The sample size should be changed. The process variation has increased. The process mean has shifted.

Correct! The process variation has increased. An out of control signal on the range chart shows the width of the process distribution has shifted.The range of the process has changed more than we would expect due to common causes. You need at least 2 samples to get a range measurement and increasing the sample size would not decrease the range. It is time to investigate the cause of the out of control point.

Which of the following is the option that best fits the sequence for implementing 5S? Remove unneeded items, arrange items, develop standardization, clean, and sustain the improvements. Clean, develop standardization, remove unnecessary items, arrange items, and sustain the improvements. Clean, develop standardization, arrange items, remove unnecessary items, and sustain the improvements. Remove unneeded items, arrange items, clean, develop standardization, and sustain the improvements.

Correct! The proper order is: remove unneeded items, arrange items, clean, develop standardization, and sustain the improvements Cleaning should happen before standardization is developed.

Which of the following describes the three components of standard work? Standard deviation, standard inventory, and standard sequence. Standard deviation, standard operating procedure, and standard sequence. Standard time, standard inventory, and standard sequence. Standard time, standard operating procedure, and standard sequence.

Correct! The three components of standard work are standard time, standard inventory, and standard sequence. A standard operating procedure may document the steps for standard work, but it is not a component of standard work. Standard deviation is a statistical concept and not a component of standard work.

The optimize phase in certain DFSS roadmaps most nearly refers to the desire to do which of the following? A) Test the design and record information for design improvements. B) Emphasize quality attributes. C) Link design to voice of the customer. D) Balance quality and cost.

D= CORRECT! In the fourth phase, optimize, the team achieves balance between quality and cost. The identify phase tasks link the design to the voice of the customer. The validate phase consists of testing and recording information for design improvements. The design phase tasks emphasize CTQ variables and attributes.

When working on improving an existing problem or process, which of the following processes would you use? A)DFSS B) DMADV C) DMAIC D) SIPOC

DMAIC is the process that works to improve an existing problem or process. SIPOC is a tool used in the define phase of DMAIC DMADV is used when there a process does not yet exist. DFSS is part of the new product design of DMADV

DOE Checklist

DOE Checklist 1) Define the objective of experiment 2) Principle experimenter should learn as many facts about the process before brainstorming 3) Brainstorm a list of independent and dependent variable and determine if the factors can be controlled or measured. 4) Assign levels to each independent variables in light of all avaiable knowledge. 5) Select standard DOE plan or develop one by conultation 6)Draw conclusions---> verify by replicating experiments

Defect Correction (source of Waste)

Defect Correction a. Causes of Defects: i. Poor equipment maintenance ii. Poor system quality iii. Poor training iv. Poor Product design.

Define Value Flow

Define Value Flow 1. Production activities should be in continuous flow with: a) No wasted motions b) No batches c) No process weaknesses d) No flexibility to meet present needs 2. Work of people/functions/departments must be readjusted to make the value stream flow and to create value for customer.

Deming's statement on quality

Deming's statement on quality: He was the founder of the 3rd wave of the industrial revolution.

Describe Critical-to-Quality Metrics

Describe Critical-to-Quality Metrics Definition= amount of money required to align a product/service with quality baselines 1) Yeild--amount of completed product divided by the amount of product that began the process.---> Does not aid in location of process errors or tell which parts can be salvaged 2) If the yeild= 95%, then scrap rate is 5% 3) Rolled Throughput ---average % of units with no defects---measures the expected quality level after several steps ----better than yeild

Design for Efficiency

Design for Efficiency---- Designed to consume the least resources

Design for Security

Design for Security --- Protection of intellectual property and user privacy against viruses, identity theft, and product misuse.

Design for Testability -

Design for Testability - Tests performed early in the production cycle rather than rely on functional tests of the finished product.

When first established, team members go through a period of team growth often referred to as forming. This stage typically includes which of the following behavior by team members? Arguing among members, even when they agree on the real issue Questioning the wisdom of the person who selected the team members for the project Attempts to define the task and decide how it will be accomplished. Establishing and maintaining team ground rules and boundaries

Attempts to define the task and decide how it will be accomplished is a typical and normal behavior observed in the Forming stage. Questioning the wisdom of the person who selected the team members for the project is behavior typically associated with the second stage of team growth, Storming. Establishing and maintaining team ground rules and boundaries is behavior typically associated with the third stage of team growth, Norming. Arguing among members, even when they agree on the real issue is behavior typically associated with the second stage of team growth, Storming

Autocorrelation charts

Autocorrelation charts 1) determine the degree that current data depends on previously gathered data by automatically examining multiple observations of a particular characteristic while looking for possible correlations. a) Measure Stage=these charts may be used to gather information about processes, including their effects on baseline data b) Analyze Stage= charts are used to investigate the regression residuals, namely to test for independence. c) Control Stage=autocorrelation charts are used to develop a strategy that takes into account a process' serial dependence." 2) Like scatter diagram (SD) except that SD identifies correlations b/w different characteristics.

Design of Experimentation (DOE)

Design of Experimentation ***Design of experiments (DOE) is a powerful tool that can be used in a variety of experimental situations. DOE allows for multiple input factors to be manipulated determining their effect on a desired output (response). By manipulating multiple inputs at the same time, DOE can identify important interactions that may be missed when experimenting with one factor at a time. All possible combinations can be investigated (full factorial) or only a portion of the possible combinations (fractional factorial). Fractional factorials will not be discussed here. 1) The arrangement in which an experimental program is to be conducted and the selection of the levels of one or more factors or factor combinations to be included in the experiment. 2) Factor levels are accessed in a balanced full or fractional factorial design. The term SDE (Statistical design of experiment) is also widely used.

2. What is one major problem with obtaining information about customer satisfaction from comment cards? a. Participants must be compensated. b. The most pleased and displeased customers are overrepresented. c. Responses are often vague. d. The expense is high.

B "2. B: The most pleased and displeased customers are overrepresented. One major problem with obtaining information about customer satisfaction from comment cards is that the most pleased and displeased customers are overrepresented. That is, customers who have extreme opinions, whether positive or negative, will be the most motivated to comment. Businesses that use comment cards may find them to be a valuable source of specific information, but should avoid assuming that commenters are representative of the larger body of customers. Some of the advantages of comment cards are that they are relatively inexpensive and tend to elicit detailed feedback. Also, participation by customers is voluntary and does not require compensation. "

Drawing Valid Statistical Conclusions

Drawing Valid Statistical Conclusions 1) Statistics used to draw valid conclusion after the analysis of representative samples from a homogenous population.

8 Building Steps of Using Balanced Scorecard

Building Steps of Using Balanced Scorecard: 1. Scope of balanced scorecard is defined. 2. Facilitator gathers information for the scorecard via interviews wit senior management 3. Facilitator distributes the information at an executive workshop designated to develop a draft of the balanced scorecard measures 4. Facilitator generates a new report and rough draft scorecards 5. Second workshop is held with senior and other management levels. Draft is refined and objectives are provided for proposed measures. 6. Third worksh0p finalizes the vision, objectives, and measures 7. New task team devel0ps an implementation plan 8. Periodic reviews of the balance scorecard are conducted.

Business Process VS. Organization Function

Business Process VS. Organization Function: 1) BPM addresses the problem of unclear functional relationships leading to failure of business processes by applying a project management and matrix organization approach to production 2) Derived from Military/ Industry Sector 3) May rely on intentional sub-optimiztion of a local function to improve overall business outcome 4) Six Sigma black belt role is to service as project management for overall business improvement projects----> a) Bridge the gaps b) Eliminate Confusion c) Identify global business improvement opportunities verse local functional ones.

Control Plans Basic Components

"Basic components of Control Plans: 1) list all of the information-- related to error prevention and detection efforts. ---that is based on on designed experiments and FMEA a) FMEA assigns risk priority numbers to the most important failure modes ( aka processes and inputs that need to be monitored and controlled) . 2) Specification on Control chart= is the objective range within which the characteristic tolerably can fall. 3) Methods for measuring the characteristic & how many measurements will be included in each sample. 4) Frequency of collecting samples & analytical tool used to evaluate the samples. 5) Reaction rules = the response protocols when the characteristic does not meet the specifications."

"Contributions of Joseph M. Juran to quality control J

"Contributions of Joseph M. Juran to quality control 1) Emphasized that quality control should be considered customer perspective. 2) Management in quality improvement process.

8 types Positive Conflict Results

8 types of Positive Conflict Results 1) Combined desire to unite and improve 2) Win-win situation 3) Creative ideas brought forth 4) Better understanding of tasks or problems 5) Better understanding of other's views 6) Wider selection of alternatives 7) Increased employee interest and participation 8) Increased motivation and energy.

AIAG Reference: Product Controls Situations

AIAG Reference: a. Product Controls Situations i. Assessment of the measurement system is based on the percent GR&R to tolerance ii. Measurement result & decision criteria determine the "conformance or nonconformance to the feature specification"

Steps for Process Capability Studies

B. Steps for Process Capability Studies 1) Measurement System Verification 2) Identification of rational sample subgroups for the plotting a control chart 3) Process Stability 4) Measuring Process Capability

Calculate the RPNs and prioritize them for taking actions. Given: Severity-Occurence-Detection A= 8-5-4 B= 6-2-7 C= 3-7-10 A) A-C-B B) C-A-B C) B-C-A D ) A-B-C

C-A-B is the correct sequence. RPN for C is 210, RPN for A is 160, and RPN for B is 84. Generally the item with the highest RPN is given higher priority for taking actions.

Classification of experimental designs: Factorials

Classification of experimental designs: Factorials ---best when investigating several factors at two or more levels and interaction is necessary.

Confidence Interval

Confidence Interval: 1) Assumes normal distribution and a stable, unchanging population 2) Measure stage---> used to estimate process average when a process control cannot be set because of lack of data. ---> Process average used for baseline estimates 3) Analyze Stage---> Used to examine similarities or differences between samples means taken during various process conditions

Control Chart Methods

Control Chart Methods 1) Mathematical Model for variability study using average and range and requires 2-3 replications by 2-3 appraisers on 1- parts.

Use of Control Charts

Control Charts: a. Primary purpose is to detect the special cause i. Used to attain the state of statistical control, monitor a process, and determine process capability. ii. Can monitor process stability. iii. Other techniques are used to reduce variation after the control chart detects the variation. iv. Control limits can be estimated at +/- 3 standard deviations from the men

In forming a team how many unique teams of 3 people from a department of 5 could be selected containing a leader, a facilitator and a member? 10 60 6 20

Correct Answer= 20 Permutation of 5 ______________ Permutation of 3 5*4*3* 2*1 120 _______ = ____ = 20 3* 2* 1 6

You are in the medical device business and the cost of a type 1 error in your experiment is high since human lives are at stake. Which combination below gives you the lowest probability of a Type 1 error? α = 0.95, β = 0.90 α = 0.10, β = 0.01 α = 0.95, β = 0.05 α = 0.01, β = 0.10

Correct! α, The probability of a type one error is 1%.

A Green Belt noted that parts from a supplier had inconsistent metal hardness from month to month shipments. He decided to conduct a multi-vari study. The type of multi-vari study that he should run would be: DOE Positional Temporal Cyclical

Correct! A temporal study is to assess variance over time

What is the systematic error that contributes to the difference between a population mean of measurements and an accepted reference value called? Bias Consumer risk Producer risk Skew

Correct! Bias is systematic error and not random error that is difficult to control in measurements. Skew is a measure of symmetry of a distribution. Consumer risk is the probability of accepting a bad lot.Producer risk is the probability of not accepting a good lot.

In contingency tables, the null hypothesis is that the probabilities for each outcome are independent is based on using which of the following measurements? Standard Deviations Variances Means Proportions

Correct! Proportions Attribute data, probabilities are determined using Chi-square.

Design FMEA

Design FMEA a. Focus= design is producible in a cost-effective manner b. Used for initial design or when significant changes c. Conducted by cross-functional team which includes downstream stakeholders and engineering design team.

Design for Usability—

Design for Usability— Ability of users to use product for its prescribed purpose (aka Validation)

C charts

C charts 1) Control chart appropriate for attributes data that tracks the number of events in a sample with predetermined size. 2) Hallmark of C chart= measured event may occur multiple time in each unit of the sample. a) For instance, a C chart may be used to track errors in a particular process, with the knowledge that several errors might occur in a single iteration of the process. 3) Measure stage--->, C charts used to estimate process baselines with attributes data. IT may be better, however, to use a variables control chart for this purpose. 4) Improve stage--->,Possible to have a role for the C chart---> Usually by this point in the process the number of errors will be too small for the C chart to be the most effective.

C. Hypothesis Tests for Means 1. Z-test

C. Hypothesis Tests for Means 1. Z-test a. Statistical test for which the distribution of the test statistic under the null hypothesis can be approximated by a normal distribution. b. If the sample size is less than 30, then standard deviation is used to estimate the population standard deviation. c. Test statistic, Z, is compared with a critical value, which is based on alpha (significance level) for a one-tail or two-tail test.

Which of the following quality management tools would best help a team to organize and summarize a natural grouping of a large number of ideas and issues? A) Activity network diagram B) Interrelationship diagraph C) Affinity diagram D) Tree diagram

C= Correct ! An affinity diagram helps teams logically group many brainstorming concepts into higher level buckets for further analysis. An interrelationship diagraph allows for analysis of cause-and-effect relationship, but does not create a hierarchical grouping of ideas per se. An activity network diagram is used to determine the most efficient sequence of work for a project. While it often employs Post-It notes, as does affinity diagramming, it is not focused primarily upon grouping ideas and issues into rational subgroups. The end product of an affinity diagram development exercise may take a tree diagram form, but with a tree diagram buckets are first defined, then brainstorming or data collection fills in the detail.

FMEAC

FMEAC= "C" for critical analysis; type of FMEA that includes provisions for assessing and charting the probability of a failure mode occurring verses the severity of the consequence in realizing the failure mode.

Factors to consider when documenting a process:

Factors to consider when documenting a process: a. Effect on quality b. Risk of customer dissatisfaction c. Statutory/ regulatory requirement d. Economic of risk e. Effectiveness & efficiency f. Competence of personnel g. Complexity of personnel

Flowchart

Flowchart 1) Picture of separate steps of a process in sequential order, including material or services entering/leaving, people involved, timeline for each step, and process measurement 2) Process: a. Define process to be diagrammed b. Define the parameters of the process c. Brainstorms & documents the activities (ignore sequence) d. Arrange the activities in sequence and draw arrows to show flow of process.

In which stage of team maturity do team members have a clear focus on the purpose of the team, and a sense of satisfaction from the team's achievements? Performing Norming Forming Storming

In the Performing stage results motivate the team and it is focused. While the team begins to focus in the Norming stage, it has no substantial result yet.

Least Square Method:

Least Square Method: a. Mathematical procedure to identify the linear equation that best fits a set of ordered pairs by finding values for the slope and y-interecept. b. Goal of this method= reduce the total squared errors between the values y and y-bar c. Standard error of estimate = tells the accuracy for y verse x; measures the amount of dispersion of observed data around the regression line i. Low values= data points are very close to line and small percentage or error

Poke-Yoke (Error-proofing)

Poke-Yoke (Error-proofing) a. Error-proofing method b. Mistake-proofing is a Lean manufacturing mechanism that helps operators prevent and detect human errors as they occur c. Examples (electrical sockets, shut-off valves, fixtures preventing loading wrong orientation, envelope window to allow view of address, car alarms)

Six Sigma Structure options--Team Leader

Six Sigma Structure options--Team Leader Black Belt Green Belt Trained supervisor/ Facilitators

Takt Time

Takt Time 1) Term (first used by Toyota) defines a time element equal to demand rate or the pace of the work 2) CFM or one-piece flow line---> time allowed for each line operation is limited. 3) Line is ideally balanced so that each operator can perform their work in the allotted time.

Team Problems & Solution-Risky Shift

Team Problems & Solution-Risky Shift Suggestions for expansive and expensive remedies---> Ask "if this were my own personal money, would I still spend it"

Types of Attribute study involving ranking of scores:

Types of Attribute study involving ranking of scores: 1. Paper corrections 2. Tasting coffee/ tea/ wine

Types of Error in Hypothesis Testing

Types of Error in Hypothesis Testing a. Type I Error i. Null hypothesis is incorrectly rejected ii. Alpha error= producer's risk (eg. Incoming goods were good but incorrectly labeled defective) b. Type II Error i. Failure to reject the null hypothesis ii. Beta error (defective product labeled as good)

What is an MR (moving range) chart?

What is an MR (moving range) chart? An MR chart plots the moving range over time to monitor process variation for individual observations. Use the MR chart to monitor process variation when it is difficult or impossible to group measurements into subgroups. This occurs when measurements are expensive, production volume is low, or products have a long cycle time. When data are collected as individual observations, you cannot calculate the standard deviation for each subgroup. The moving range is an alternative way to calculate process variation by computing the ranges of two or more consecutive observations.

An operator is expected to assemble certain parts without error. Each assembly may be different depending on the customer order. Which of the following is the best example of error-proofing for this situation? When the operator begins putting together an assembly, he or she is given a list containing only the parts for that assembly. Standardized work instructions are written and posted at the operations. These contain instructions for all the assemblies. When the operator begins putting together an assembly, only the bins containing the parts for that assembly light up. Each operator is trained on which parts go into each assembly being made available to customers.

Correct!---Light-up bins that contain only the parts needed for assembly will make it easier to know which parts go with which assembly. A list still allows for confusion as to which parts are in which bins; therefore, errors could still be made. Written and posted standardized work instructions still allows for confusion as to which parts are in which bins; therefore, errors could still be made. Although training is always helpful, it does not prevent errors from occurring.

Business process management is based on the premise of which of the following? A) Each process in an organization should be optimized to its own highest level of performance to achieve company goals. B) Process ownership is delegated to those actually doing the work in a process. C) Achieving organizational goals is often driven by the coordination of a complex system of cross-functional processes. D) Managing the quality of tangible process outputs is more important than managing the quality of service delivery.

C= Correct! Achieving organizational goals is often driven by the coordination of a complex system of cross-functional processes is the correct answer, suggesting that monitoring of inputs, outputs, and process feedback are necessary to achieve organizational goals. Optimizing a process to its highest level of performance without regard for impact on other processes can sub optimize the organization as a whole. While staff doing the work have responsibilities for quality, process ownership typically resides at the executive or management level. Managing the quality of process outputs and related service delivery will vary in importance depending upon customer requirements and organizational goals.

What is C (p)?

CP is a measure of short-term process or characteristic capability. Use only the short-term standard deviation to calculate its value. Using a long-term standard deviation in its calculation gives you incorrect results.

Calculating process velocity (aka take time)

Calculating process velocity step 1= Categorize tasks value-added, non-value-added but necessary (NVA type 1), or non-value-added and unnecessary (NVA type 2). Step 2= Measure the physical distance between the sites of each successive task. Step 3= Control chart used to predict the average time to complete each process task. and to estimate the customer / employee's wait-time before beginning each successive step in the process. Step 4= Control chart estimates the number of items in each queue. Step 5 = Calculate the total number of items in the process (Work in progress) and (Cycle time) or the No. of times the process is completed each hour by adding the averages of all these measurements Lead Time = Cycle Time * Work-in-Progress Or Lead Time = WIP/Throughput Velocity = (number of value-added steps) /(process lead time) "

Central Limit Theorem and Sampling Distribution of the Mean 3 key statements

Central Limit Theorem and Sampling Distribution of the Mean 3 key statements: a. Mean of the sample distribution of means is equal to the mean of the population from which the samples were drawn. b. Variance of the sampling distribution of means is equal to the variance of the population from which the samples were drawn divided by the size of the samples. c. If the original population was distributed normal, the sampling distribution of means will be normal. If the original population is not normally distributed, the sampling distribution of means will increasingly approximate a normal distribution as the sample size increases.

Champions

Champions 1) Upper-level managers who control & allocate resources fro process improvements & black belt development /training 2) Trained in core concepts and deployment strategies of 6 Sigma 3) May lead the 6 sigma teams 4) Works with Black belts to keep senior managers informed of the deployment strategies 5) Conduct project reviews

Chi- Square Test

Chi- Square Test Applied in two Ways: 1) Comparing variances when the population variance is known 2) Comparing observed and expected frequencies of test outcomes when there is no defined population variance (attribute data) 3) Unlike Z and t distribution, the tails of the chi-square distribution are non-symetricals

Confidence Interval on Proportion

Confidence Interval on Proportion 1) Percentage of samples in the confidence interval will include the true error rate 2) Example---> 90% error rate means that 9 out 10 will have included the true error rate. 3) CI of the mean decreases as the sample increases--->As more samples are obtained, fewer values are required to create confidence interval. 4) If standard deviation Known, use z-table---> If unknown, then use T-tables 5) Process stability cannot be determined by the fact that a particular sample falls within the confidence interval. ----> Must confirm process stability with statistical control chart.

The statistical technique that allows estimates about population parameters with a known level of certainty is called: Confidence Intervals Action Limits Specification Limits Control Limits

Confidence Intervals are a statistical technique that allows estimates about population parameters with a known level of certainty. Control limits are calculated from the process to identify assignable causes and verify common cause variation. Specification limits are limits set by the customer or industry standards. Action limits are limits provided to the operator to enact countermeasures as appropriate

Which of the following situations illustrates the lean concept of "pull"? Nothing is produced by the upstream operation until the downstream operation communicates it is ready. The upstream operation sets the pace for the downstream operation's production. The upstream operation produces to the schedule provided by the downstream operation. The upstream operation's production is synchronized to provide the downstream operation just-in-time

Correct!---The pull comes from the downstream operation signaling that it needs what is being produced upstream. The upstream operation sets the pace for the downstream operation's production is rather a push situation which typically leads to large amounts of work-in-process inventory and increased costs. Here the upstream operation is producing to an anticipated need. This often leads to a push situation such as excess inventory and costs when downstream disruptions occur. Here the upstream operation is anticipating the downstream need instead of responding to a clear signal.

A hospital pharmacy has a large number of unfilled prescriptions. In lean categories of waste this is considered which of the following? Kanban Overproduction Lost creativity Waiting

Correct!--If people are unavailable to fill the prescriptions and shipments are delayed, this is an example of the lean category of waiting. Kanban is a system to replenish supplies, not a lean waste.

Using a template to eliminate the possibility of error at a drilling step would be an example of which of the following? 5S Kanban Poka-yoke Standard work

Correct!--Poka-yoke refers to error-proofing an operation or step. Kanban is a resupply procedure. 5S is a system for keeping an area tidy. Standard work refers to keeping a repetitive task the same through good work instructions and training.

Why can items such as bins, floors, benches and racks be used as part of a visual factor? The items can be kept clean and orderly. These items can be used to communicate information. Items such as bins and benches can be arranged by order of operation. Items such as floors, bins benches and racks can be used for storage.

Correct!These items can be used to communicate information as part of a visual factory. Visual factory is used to communicate, not store things.

All of the points fall within the control limits and no unusual patterns are present. The control limits define the extent of: Special-cause variation Assignable-cause variation Common-cause variation Engineering tolerances

Correct!W. Edwards Deming called the variation inherent in the process as common-cause variation because it is common to the whole process. Walter A. Shewhart referred to points out of the control limits or unusual patterns as being due to assignable-cause variation.

Correlation vs. causation

Correlation vs. causation a. Causation= produces effect or gives rise to action. b. No such things as absolute cause for an event, the identification of which satisfies and completes all inquiry. c. If two variables are found to be associated, that does not mean than a cause-and-effect relationship exists between the 2 variables.

Cost of Poor Quality

Cost of Poor Quality: 1) Costs associated with providing poor quality products/services 2) 4 categories of Cost: a) Failure Costs ----> 2 types : I) Internal Failure Costs = costs associated with defects found BEFORE customer gets the product II) External Failure Costs = costs associated with defect found after customer receives the product/service b) Appraisal Costs = Cost incurred to determine the degree of conformance to quality requirements c) Prevention Costs = Cost incurred to keep failure and appraisial costs to a minimum

Cost/Benefit Analysis & Cost of Quality Assurance Curve

Cost of Quality Assurance Curve a. The cost of quality assurance curve, also on the traditional economic model of quality of conformance, swings from low to high. It represents the cost increases you incur as spending on quality assurance increases. It starts with low or no costs with no quality assurance program and rises to reflect the increasing costs of the quality control program. Because it is the opposite trajectory of the costs due to nonconformance curve, these two curves intersect forming an important juncture.

CP = PPK meaning?

Cp = Ppk---> Diagnosis = Your process is operating at its entitlement level of variation. Solution= Continue to monitor the capability of your process. Redesign your process to improve its entitlement level of performance.

Creating PERT Analysis

Creating PERT Analysis 1st step (PERT) analysis = isolate activities on the critical path using any number of charts, such as an activity network diagram. a) In PERT analysis, assume that activity times follow a β distribution. ---> The estimated duration of each activity calculated with the formula , in which a is an optimistic estimate of the time, b is a pessimistic estimate of the time, and m is the most likely time. i) The calculation for estimated standard deviation for each activity is calculated with . ii) The central limit theorem suggests that the total time for all tasks should be in a normal distribution. iii) Adding up the times for the tasks on the critical path yields a measure of total time. iv) The upper predicted limit for total time is equal to the total time + 1.96 × standard deviation. The lower predicted limit for total time is equal to the total time - 1.96 × standard deviation. These are good estimates for the best and worst possible durations."

Creating a C chart

Creating a C chart 1) Selecting an appropriate sample from a specific part of the process at the same time. 2) C chart's Hallmark= each element of the sample may have more than one of the featured attributes. a) For instance, if the purpose of the C chart is to measure errors during a particular part of production, each instance of that process may contain more than one error. b) C chart 's plotted statistic = number of occurrences of a particular event during the sample. c) Lookup these Formulas of Centerline of the chart is determined with the formula "", where m is the number of groups used in the analysis. The upper control limit of the chart is calculated UCL, and the lower control limit is calculated LCL. For both control limit formulae, is the average count and n is the sample size. "

Work breakdown structure--- Creation and interpretation

Creation and interpretation 1) A work breakdown structure is created first by breaking a problem into components which are then broken down into categories. 2) Categories represent the different ways the particular problem can occur. 3) Goal of work breakdown structure= to reduce complex problems to a collection of discrete and manageable issues. ----creates a map for problem solving that can be used by members of different departments. 4) Create material for other data-organization charts such as Pareto charts.

Creation of Exponentially weighted moving-average chart

Creation of Exponentially weighted moving-average chart a) Key 1st step= setting the value for λ (lambda), the weighting factor. 1) To detect small shifts,---> set t value close to 0.2; 2) To detect large shifts, set values between 0.2 & 0.4. b) The plotted statistic for an EWMA is calculated: z (t) = λ(1) ( X-bar (t) ) + [(1 - λ(1) ] [z (t- 1)) , where t is a particular time, λ is the value of the weighting factor, is the subgroup average for the present subgroup at t, and the value of z at t0 is either the overall average or a target. c) Formula for calculating control limits : (see Formula) where z0 is the target value or process mean value, n is the size of the subgroup, m is the number of functions being analyzed, and d2 is a function of n."

Control plans Definition

Control plans 1) Summary of the detection and/or prevention strategies used to control processes or materials. 2) Control stage of DMAIC--- used to create a record of the strategy used to control the key process variables. 3) A control plan generally is compiled from the results of designed experiments and FMEA a) FMEA= shows the most important sources of failure to control. b) The characteristics to be monitored and controlled will be listed in left side of the control chart. c) For each characteristic, there will be a specification, measurement technique, sample size, sample frequency, analytical tool, and reaction protocol."

Conversion of Attribute Data to Variable Data

Conversion of Attribute Data to Variable Data 1) When collecting data, there are opportunities for some types of data to be either variable or discrete. 2) Consider costs when deciding to collect which type of data. ---> Measuring instruments are most costly for variable data 3) Variable data requires storing of individual value and computations of the mean, st. deviation, and other population estimates 4) Attribute data requires minimal counts of each categoty so little data storage space needed. 5) Manual Collection ---> Variable Data requires more skill than attribute data.

Converting from Cpk to a Z-score

Converting from Cpk to a Z-score With a normally distributed set of data an approximation of Z from Cpk and vice versa can be made in substituting and solving within these two formulas. What is the Cpk approximated from a 6 sigma performance? Substituting the Z formula in Cpk (USL) and plugging in 6 for Z from the given data gives: Relating Cpk to sigma (Z) Cpk of 2 corresponds to approximately 6 sigma (short term) performance or 4.5 sigma (long term). The Z in the above formula refers to a short term sigma, short term Z.

control chart is to be kept on the temperature of a process recorded each day at noon. Which of the following is the most appropriate control chart to use? u chart p chart c chart I chart

Correct ! I-chart---The data points come one at a time (one per day). There is no need to wait to accumulate data for a subgroup. remaining are attribute charts and cannot plot temperature

The finished control plan should include which of the following? Action instructions, characteristic, machine and inspectors name Readings, process capability, subgroup size and control item Graphical representation, supplier, special characteristics and test equipment A flow chart, gages, product name, sample size and specified reactions

Correct! A control plan should include a flowchart or graphical representation, gages or test equipment, sample sizes and specified reactions or troubleshooting solutions.

Choose the most appropriate statement that applies to the term kaizen. A three- to five-day event. A way to provide quicker implementation of results. A way to obtain breakthrough improvement. Small continuous improvements.

Correct! Kaizen is a philosophy of small, gradual continuous improvements.

During an experimental run, setting a single level to a single factor is called? Bias Screening Treatment Balance

Correct! Setting a single level to a single factor is a treatment. Balance is when all combinations have the same number of observations Bias is systematic error. Screening is a lower resolution experiment used to reduce a large number of possible factors down to the critical few.

Total Preventative Maintenance is used to watch for which of the following? Symptoms of machine failure. Overproduction. Ensuring quality checks are performed. Signs a part will go out of specification

Correct! TPM watches for visible or audible symptoms of machine failure SPC is used to detect problems before parts go out of specification. Overproduction is an element of non-value added activities

Which of the following is a visual comparison of the effect of each of the factors by displaying the means of all factors at each factor level? DOE FMEA Multi-vari study Binary logistic regression

Correct! The example above defines multi-vari studies.

Total Productive maintenance is used to do which of the following? Prevent quality failures Correct machine failures Reduce machine downtime Improve product quality

Correct! The purpose of TPM is to reduce machine downtime.

Customer Surveys Themes

Customer Surveys Themes: 1) To determine what quality is 2) Find out what competitors are doing 3) Define quality performance measures for sue 4) Identify factors to give a competitive edge 5) Identify urgent problems

Why would a Six Sigma Green Belt create a control plan after improvements are implemented? To identify potential risks in a process. A control plan is required during a Six Sigma project. To systematically identify and resolve out of control conditions. To identify failures before they occur.

Correct! A control plan is used identify when out of control conditions occur. Feedback for distractor 2 SPC can detect a potential problem before it occurs.

When using a factorial design of experiment with k-factors (each factor at two levels), what tool can be utilized to detect an aberrant value (very low probability of occurrence)? Youden plot Yates' algorithm Half-normal plot Response curve

Correct! A half-normal plot can be used to detect an aberrant value in this case. The Youden plot shows the systematic differences. The Yates' algorithm is a rapid method for obtaining estimates of main effects and interactions for two-level factorials. A response curve is utilized particularly in factorial experiment with two factors to show presence or absence of interaction.

What assumption can be made if an organization is effectively using lean thinking and applying lean tools to reduce waste throughout the value stream? It can be considered to be a traditional manufacturing business. It can be considered to be a house of quality. It can be considered to be a lean enterprise organization. It can be considered to have a system of profound knowledge.

Correct! A lean enterprise organization effectively uses lean thinking and applies lean tools to reduce waste throughout the value stream and offer value to its customers House of quality is the first diagram used in quality function deployment (QFD) to plan the design or improvement of a product or service. It combines several matrices and charts into one massive diagram.

A team is faced with a clear need to rapidly improve a production process and upper management has challenged the team to come up with a workable solution within a 48-hour timeframe. This would be an appropriate scenario for applying the concept of which of the following? Trend analysis Kaizen blitz Poka-yoke PDCA cycle

Correct! Kaizen blitz----A rapid, time-limited solution is the key to this question and this approach The PDCA cycle is a useful continual improvement tool, but it does not meet the timeframe requirements of this situation. Poka-yoke refers to mistake-proofing. While the term does apply, it is not the best solution of the possible answers. A trend analysis involves collecting information to identify trends. The above situation requires a more rapid approach.

Each day the number of rejected bulbs out of a sample of 100 bulbs is recorded as a subgroup. To take advantage of the equal subgroup sizes, the most appropriate control chart is: u np c p

Correct! The np chart is the most appropriate chart when the statistic in question is the number of nonconforming items (rejected bulbs) out of the total number of items inspected and the subgroup size is constant (here set at 100). The p chart could be used because the statistic in question is the proportion of rejected bulbs out of the total number of items inspected.

A firm that manufactures stereo units has specifications set for the clarity of sound produced by the finished unit. Each day between 100 and 200 units are inspected, and that sample size is recorded, along with the number of rejected items. The proportion of rejected items is calculated each day. Which control chart is most appropriate to use? u c p np

Correct! The p chart is appropriate when the statistic in question is the proportion of nonconforming items (rejected items) out of the normal number of items inspected, which here varies between 100 and 200. The u chart is appropriate only when the statistic of interest is the ratio of the number of occurrences of an attribute to the area of opportunity. The c chart is appropriate only when the statistic of interest is the ratio of the number of occurrences of an attribute for a given area of opportunity, where the area of opportunity remains constant for all subgroups. The np chart is appropriate only when the statistic in question is the number of nonconforming items out of the total number of items inspected and the subgroup size is constant.

30. During which stage of DMAIC is it most useful to calculate process velocity? a. Analyze b. Define c. Control d. Improve

"30. A: Analyze. It is most useful to calculate process velocity during the analyze stage of DMAIC. Process velocity is the rate at which a particular phase of the process adds value. Obviously, the higher the process velocity, the better. This metric is most useful during the analyze stage of DMAIC because it can be used to prioritize methods for improving cycle time. Velocity is typically calculated by dividing the number of value-added steps by the process lead time, which is the number of items in the process divided by the number of process completions per hour. Of course, as with any metric of quality, process velocity is somewhat subjective. "

45. How are decisions represented in the ANSI set of flowchart symbols? a. Circles b. Squares c. Rectangles d. Diamonds

45. D: Diamonds. In the ANSI (American National Standards Institute) set of flowchart symbols, decisions are represented with diamonds. There are other symbols to represent different types of tasks, but many simple flowcharts will simply use the diamond for decisions and rectangles for all other tasks in the process. An excessive number of decisions on a flowchart is a common symptom of inefficiency."

6 Categories of conflicts

6 Categories of conflicts 1) Intrapersonal ---within the individual 2) Inter- personal-- between 2 people 3) Intra-group--within a group 4) Inter- group---between groups 5) Inter-departmental ---between departments 6) Inter-company ---between companies.

Drum-Buffer- Rope

Drum-Buffer- Rope a. Drum-buffer-rope is the Theory of Constraints production application. b. Named after the 3 essential elements of the solution; the drum or constraint or weakest link, the buffer or material release duration, and the rope or release timing. c. Aim is to protect the weakest link in the system, and therefore the system as a whole, against process dependency and variation and thus maximize the systems overall effectiveness. d. The outcome is a robust and dependable process that will allow us to produce more, with less inventory, less rework/defects, and better on-time delivery always.

Efficiency

Efficiency 1) A concept from R.A. Fisher. He considered one estimator more efficient than another if it had a smaller variance. 2) Percentage Efficiency= (variance of minimum estimator) / * 100 (variance of an estimator in questions)

Experimental Objective ----Response Surface Objective

Experimental Objective ----Response Surface Method (RSM) Objective 1) Experiment that is used to estimate interaction and quadratic effects, and therefore, give an idea of the shape of the response surface under investigation 2) RSM is used to: a) Find improved or optimal process settings. b) Troubleshoot process problems and weak points c) Make a product/ process more robust against external influences.

Experimental Objectives

Experimental Objectives a. Objectives derived from questions or enterprise goals. b. Factors to consider in setting objectives: i. Requires decision to give affordable and time available design. ii. Start with screening design, whose purse to determine the variables and levels for future study. c. Confirmation tests determine if the test is successfully identifies better process conditions

Exponential Distribution

Exponential Distribution 1) Basic assumptions: a) Family of distributions characterized by its m b) Distribution of time between independent events occurring at a constant rate c) Mean is the inverse of the Poisson distribution d) Shape can be used to describe failure rates that are constant as a function of usage Uses include probabilistic assessments of: Mean time between failure (MTBF) Arrival times Time, distance or space between occurrences of the events of interest Queuing or wait-line theories

Failure mechanism

Failure cause and/or mechanism Defects in requirements, design, process, quality control, handling or part application, which are the underlying cause or sequence of causes that initiate a process (mechanism) that leads to a failure mode over a certain time. A failure mode may have more causes. For example; "fatigue or corrosion of a structural beam" or "fretting corrosion in an electrical contact" is a failure mechanism and in itself (likely) not a failure mode. The related failure mode (end state) is a "full fracture of structural beam" or "an open electrical contact". The initial cause might have been "Improper application of corrosion protection layer (paint)" and /or "(abnormal) vibration input from another (possibly failed) system".

The typical balanced scorecard measures organizational performance in which of the following categories? Financial, Output, Process, Input Strategic, Financial, Customer, Learning Supplier, Process, Customer, Financial Financial, Customer, Process, Learning

Financial, Customer, Process, Learning are the categories (dimensions) found on the typical balanced scorecard developed by Kaplan and Norton.

Forms of Communication

Forms of Communication 1) Oral Communication ----telephone, face-to-face, internet 2) Written Communication----emails/ reports/ computer message---may be considered as one-way channels 3) Face-to-face --Two-way communication b/c immediate feedback. 4) Non-verbal Signals *** Speaking and writing are important for leadership ability *** Use of Interpersonal space varies between cultures

"Team formation

Four stages of Team formation 1) Forming stage=, the members of the team are getting to know one another, and on their best behavior---->the team has not yet begun working seriously enough to inspire much conflict. 2) Storming,----> the team begins its work in earnest, and differences between the members begin to emerge. During this stage team members will begin to assume different roles. The team leader must exercise control during this stage so conflicts can be resolved beneficially. 3) Norming stage--->,Agreement on normative behavior ----> the team members settle their differences and settle on compromises that make effective work possible. 4) Performing stage--->, the team operates at an optimal level because members have learned to work together."

Fractional Factorial Design

Fractional Factorial Design= balanced experimental design with fewer than all combinations of all levels of all factors. a. Used for quick exploratory tests where interactions insignificant and many test needed rapidly.

CPK greater than 1.33 indicates?

Generally, a CPK greater than 1.33 indicates that a process or characteristic is capable in the short term. Values less than 1.33 tell you that the variation is either too wide compared to the specification or that the location of the variation is offset from the center of the specification. It may be a combination of both width and location.

Which of the following represents the primary disadvantage of focusing exclusively on hard-dollar savings when assessing Six Sigma project benefits? asq/learn_x.gif Requires more effort to calculate hard-dollar benefits than other benefits Hard-dollar benefits are not considered to be tangible benefits Hard-dollar benefit reduction does little to reduce "hidden factory" costs Hard-dollar benefits are not a business priority

Hard dollar benefits are the easiest to qualify, because they are typically measured only at the end of the process and are usually stated in a company's financial books/statements. However, a 'hidden factory" includes all costs of waste and rework embedded within the process execution steps. They may be harder to identify and quantify in terms of hard dollar costs, and may require more sophisticated analysis to translate into savings that appear in a company's financial statements. It is typically more difficult to capture soft-dollar benefits (such as Cost of Poor Quality) than hard-dollar benefits, which tend to be visible within the budget.

Histograms Shapes------Truncated or heart-cut.

Histograms Shapes---Truncated or heart-cut. 1) The truncated distribution looks like a normal distribution with the tails cut off. 2) The supplier might be producing a normal distribution of material and then relying on inspection to separate what is within specification limits from what is out of spec. The resulting shipments to the customer from inside the specifications are the heart cut.

Histograms Shapes---Normal Distribution

Histograms---Normal Distribution The number of columns in a histogram should have the square root of the sample size -------eg. 100 samples will show 10 bars 1) Normal. A common pattern is the bell-shaped curve known as the "normal distribution." 2) In a normal distribution, points are as likely to occur on one side of the average as on the other. ***Be aware, however, that other distributions look similar to the normal distribution. Statistical calculations must be used to prove a normal distribution.

Horizontal Communications

Horizontal Communications Information sharing across same levels of the organization----eg. engineers sharing with production planning group

Define Hoshin Planning

Hoshin Planning a. Planning process that develops the company's vision for next 5 years. b. Project proposals are required to include précises statement of the problems c. Sigma Team→problems with extensive data analysis and improvements d. Lean Team-→ process improvement not involving techniques. e. DFSS-→ new product designs.

Hypothesis Tests for Means One-Way Anova

Hypothesis Tests for Means-----One-Way Anova a. The one-way analysis of variance (ANOVA) is used to determine whether there are any significant differences between the means of two or more independent (unrelated) groups (although you tend to only see it used when there are a minimum of three, rather than two groups). b. Required conditions that must be met to conduct one-way ANOVA: i. Normally distributed population of interest ii. Samples are independent for each other iii. Each population have the same variance.

Identifying Characteristics to be measured in a process capability study should meet 3 requirements

Identifying Characteristics to be measured in a process capability study should meet 3 requirements: 1) Characteristic should be indicative of a key fctor in the quality of product or process 2) Possible to adjust the value of the characteristic 3) Operating condition that affect the measured characteristic should be defined and controlled. ***When multiple product dimensions, best to select a few key dimension for manageable method of evaluating process capability. -----> may be dtermined by the history of the part and the parameter that has been the most difficult to control OR by problem created in the higher levels of assembly. ***Customer purchase order requirements or industry standards may also determine the characteristics that are required to be measured

Identifying Specification/ Tolerances

Identifying Specification/ Tolerances: 1) Specification/ Tolerances of a process are determined by : a) Customer Requirements b) Industry Standards c) Organization's Engineering Department 2) Process Capability Study------> used to demonstrate that the process is centered within the specification limits and that the process variation predicts the process is capable of producing parts within the tolerance requirements 3) If process is found to be not capable, the then process capability results are used to improve the process 4) If the specifications/ Tolerance too high------> Reevaluate these parameters OR perform 100% inspection of the process, unless inspection testing is destructive.

If CP = CPK and PP = PPK, then interpretation?

If CP = CPK and PP = PPK, then interpretation? Diagnosis= Overall, your process or characteristic is centered within its specifications. Solution= As needed, focus on reducing the long-term variation in your process or characteristic while maintaining on-center performance.

If CP = PP and CPK = PPK, then interpretation?

If CP = PP and CPK = PPK, then interpretation? Diagnosis= our process or characteristic suffers from a consistent offset in its center location. Solution= Focus on correcting the set point of your process or characteristic until it's centered.

Impacted Parties

Impacted Parties 1) Those who don't purchase the good/service but impacted @Communities/ Civic groups/ City Governments @ Vocational Schools/ Colleges /service companies who hire students @ Suppliers

The distribution where the concentration of observations is centered about the average and it is equally likely that the observations will occur above or below the average is called: Poisson Normal Binomial Exponential

In a normal distribution the concentration of observations are about the average and it is equally likely that the observations will occur above or below the average.

A portion of units obtained from the same location in all containers, racks, bins, or dunnage illustrates which of the following? Stratified sampling Sampling bias Consumer's risk Producer's risk

In sampling bias, procedures set up for sampling can deteriorate into biases. In this example, the sample is not random, which results in bias. Consumer's risk is the probability that a "bad" lot will be accepted by the sampling plan. Producer's risk is the probability that a good lot is rejected by the sampling plan. Stratified sampling is sampling that is deliberately stratified to obtain sample proportions from each true lot. In this example, you are looking for the term that illustrates what would happen without rigorous procedures for stratified sampling.

Interpretation of Confidence Interval

Interpretation of Confidence Interval 1) An estimation of the confidence interval on the mean indicates the percentage of samples that will contain the true population mean, μ (mu). a) 97% confidence limit on the mean =an expect that 97% of samples the confidence interval will include μ. c) More samples, the more accurate the mean. d) Confidence interval and the number of samples are inversely proportionate. --especially useful with the SD is known. e) When the standard deviation is unknown, the t tables must be used. When the standard deviation is known, the z tables may be used.

Interpretation of Quality Function Deployment Matrix

Interpretation of Quality Function Deployment Matrix QFD= Process for planning new or redesigned products/services. 1) Matrix formation conducted by filling customer requirements, which is developed from analysis of the voice of the customer. a. Two Categories of Customer Requirements: i. Output Requirements = final product's features ii. Service Requirements= how the customer expects to be treated during the process. 7. Two types of Co-relationships: a) Positive co-relationship indicates both technical requirements can be improved at the same time. b) Negative co-relationship→ improving one technical requirement worsens the other.

Interpreting Process Capability

Interpreting Process Capability b. Process reached Six Sigma quality-→ Cp= 2 and Cpk= 1.5 c. Process is capable-→ Cp & Cpk ≥ 1.33 d. Process barely meets spefication & will have a 0.27 percent defective units-→Cp or Cpk value= 1 e. Process producing units outside the engineering specifications-→ Cp or Cpk < 1 f. Specification is loose or identifies an opportunity to move to a less expensive process-→ Cp or Cpk > 3

An experiment measures the temperature of a chemical reaction in degrees Fahrenheit. The results in this experiment are example of what type of measurement scale? asq/learn_check.gif Interval Nominal Ordinal Ratio

Interval data have the property that the "intervals" between the numbers are meaningful and can be compared. This is the case with the Fahrenheit scale for temperature.

Benefit of using Interviews

Interviews ---Higher response rate than written surveys and more accurate data. source of a metric for determining project success. Q. Interviews 1) Higher response rate than written surveys and more accurate data. R. Matrix Diagram 1) Allows illustration and discovery of relationships between two groups. Each square in matrix is examined for relationship between items in the row or column.

Lean Concepts & Tools--Inventory

Inventory ----Lean Concepts & Tools-- Inventory leads to cost increases because: a. Storage space expenses b. Quality deterioration c. Delays in design changes d. Money used on inventory could be used elsewhere e. Late detection of quality problems→ defective products.

3 Components of Juran's version of quality management

Juran's quality management had three main components. 1) Quality control and the use of statistical control methods for the mitigation of problems. 2) Quality improvement and the use of the breakthrough sequence for resolving persistent problems. 3) Quality planning (esp. annual quality reviews )

Juran---Awards

Juran---Awards 1. Edwards Medal, ASQ 2. Brumbaugh Award ASQ 3. ASQ Honorary Member 4. Second Order Medal of the Sacred Treasure 5. Plus 30 other awards and fellowships.

Juran---Statement on Quality

Juran---Statement on Quality Adopt a revolutionary rate of improvement in quality, making quality improvement by the thousands, year after year

Juran--professional experiences

Juran--professional experiences 1. Inspection position at Western Electric's Hawthorne Plant in Chicago 2. 50 years in research, consulting, writing, teaching 3. Taught with the American Management Association

Kaizen

Kaizen 1) INCREMENTAL changes 2) Umbrella terms for: a) Productivity b) Total quality control c) Zero Defects d) Just-in-time production e) Suggestion Systems

Define Kaizen Events

Kaizen Events i. Used to provide quicker implementation results in Lean ii. Use of cross-functional teams to review options iii. If staff can't take 3-5 days to improve process constraint, then either the problem is unimportant or cultural adjustment needed.

Kano Analysis

Kano Model Analysis --- useful technique for deciding which features you want to include in a product or service. It helps you break away from a profit-minimizing mindset that says you've got to have as many features as possible in a product, and helps you think more subtly about the features you include.

Key information that can be expected from the use of control chart

Key information that can be expected from the use of control chart 1) Average level of the quality characteristics 2) Basic variability of the quality characteristics 3) Consistency of performance. ***Benefits from control charting are derived from both attributes and variable charts. ***If the control chart shows that the process is in control and within specification limits, then cost relating to inspection can be eliminated.

Key point to consider when using Attribute Charts

Key point to consider when using Attribute Charts : 1) Normally the subgroup size is greater than 50 (p-charts) 2) Average number of defects/ defectiveness is equal or greater than 4 or 5 3) If the actual p-chart subgroup size varies by more than +/- 20% from the average subgroup size, the data point must either be discarded or the control limits calculated for the individual point 4) Most sensitive attribute chart= p-chart 5) Most sensitive and expensive chart is X-bar/R chart 6) Defects and defectiveness plotted in attribute charts are often categorized in Pareto fashion to determine the vital few. ------> Fundamental change required to reduce the defects.

Types of Variation in Measurements: Linearity -

Linearity - Accuracy of measurement at various measurement points of measuring range in the equipment.

Pareto Diagrams

Pareto Diagrams 1) Snapshots of customer defects or rejects can be displayed for a selected time interval

Resolution I

Resolution I 1) Experiment in which tests are conducted, adjusting one factor at a time, hoping for the best. 2) This experiment is not is not statistically sound

Purpose of Design of Experiments Response Variables

Response Variables i. Variables that show the observed results of an experimental treatment ii. Outcome of the experiment as a result of controlling levels, interactions, and number of factors

Which of the following is the sampling method if each item is treated as a sample of size one, and a decision to accept, reject, or continue sampling is made after inspecting each item? Multiple sampling Continuous sampling Sequential sampling Double sampling

Sequential sampling is a procedure of testing items in sequence and continually making the decisions on accept or reject and whether to continue after each sample. In multiple sampling, more than two samples can be taken to reach the decision of acceptance or rejection. In this example, only one sample is being taken at a time Continuous sampling often begins with 100 percent inspection until a predetermined number of units are accepted. Sampling then takes place until a set number of defects are found and the original sampling technique resumes. In double sampling, two samples can be taken to reach the decision of acceptance or rejection of the lot or draw another sample.

There are 2 defective products in a production lot of 10. An inspector randomly selected 3 of them without replacement. What is the probability that all selected products are good? 3/10 46/120 56/120 2/10

The correct calculation is (8/10) x (7/9) x (6/8) = 336/720 = 56/120

Tollgate Review:

Tollgate Review: 1. Formal review conducted at the end of the DMAIC 2. Purpose of that review is for the Champion, Sponsor, leadership council to review the team's progress and give a formal go-ahead for the next phase 3. Conducted in joint meetings with the entire team and the oversight group

Two phases of Brainstorming :

Two phases of Brainstorming : 1) Creative Phase of Brainstorming 2) Evaluation Phase .

Waiting (source of Waste)

Waiting a. Reasons for increased waiting: i. Delayed shipments ii. Long setup times iii. Missing people b. Waiting leads to wasted resources and demoralization of staff. c. Improved with cross-training staff.

Waiting/ Muda

Waiting/ Muda: 1. Operator is idle due to machine downtime, lack of parts, unwarranted monitoring activities or line stoppages. 2. Maintenance operator waiting at a tool bin for parts 3. Long changeover times, uneven scheduling of work, and long/unnecessary meetings

Weibull Distribution

Weibull Distribution Basic assumptions: Family of distributions Can be used to describe many types of data Fits many common distributions (normal, exponential and lognormal) The differing factors are the scale and shape parameters Uses include: Lifetime distributions Reliability applications Failure probabilities that vary over time Can describe burn-in, random, and wear-out phases of a life cycle (bathtub curve)

Win-Win Negotiations

Win-Win Negotiations 1. Concept that both sides emerge with a successful deal 2. If negotiation is competitive game with one winner and one loser--> risk that loser won't play again or will not help in implementing agreement.

Image displayed next to each question A soda company is developing a new recipe for grape-flavored soda, and would like to get some feedback on the new product without catching the attention of its competitors. Which of the following would be an appropriate method for collecting consumer feedback? A) Product promotions at major sport events B) Public mail surveys C) Focus groups D) Online customer surveys

C= Focus Groups Online surveys & Public mail surveys will not preserve the secrecy of the product and is usually not appropriate at product development stages.

18. In an analysis of variance, how is the F statistic used? a. To compare the mean square treatment with the mean square error b. To estimate the process average c. To find the variation within each subgroup d. To find the variation between different subgroups"

18. A: To compare the mean square treatment with the mean square error. In an analysis of variance, the F statistic is used to compare the mean square treatment with the mean square error. The mean square treatment is the average variation between the subsets, while the mean square error is the sum of the squares of the residuals. In order to trust the results of the F statistic, one must assume that the subsets have a normal distribution and unequal variance. The variation within each subgroup is calculated by taking repeated samples from the subgroup. The variation between different subgroups is found by comparing the averages of each subgroup.

Tools for Business performance measures: Process Performance.

3) Process Performance. a. Process Performance= measure of how the process is executing against some established goals or statistical measure.

5 areas of risk for most projects>

5 areas of risk for most projects> 1) Technical Performance 2) Supportability risks 3) Cost risks 4) Schedule Risks 5) Environmental Risk

6 Short-comings of classical approach to experimentation

6 Short-comings of classical approach to experimentation 1) Need too many experiments to study all input factors 2) May not reveal the optimum combination of all variables 3) Interaction between factors cannot be determined 4) Conclusions may be misleading 5) Non-statistical experiments are often inconclusive 6) Wasted time on studying wrong variables or having too much/ too little data

9 Benefits of Properly functioning Team

9 Benefits of Properly functioning Team 1. Improved employee morale 2. Remove areas of conflict 3. Develop Creative skills of members 4. Improve communication and leadership skills of members 5. Develop problem-solving skills 6. Improve attitude of management & team members 7. Indicates to team members that mgmt listens 8. Demonstrates that employees has good ideas 9. Improve management/employee relationships

A revision in the budget is required due to development. Since the original budget approval. What does the Six Sigma Manager need to do first? Conduct Budget Review Conduct Management Review Create Budget Report Create Status Report

A budget review involves revising budgets, either upward or downward, based on developments since the original budget approval. A management review provides management with the status of the project, an opportunity to review the project charter and project team mission, and a chance to discuss management activities likely to have an impact on the progress of the team, etc. A budget report is only used in monitoring actual expenditures versus budgeted expenditures.

Confidence Interval for Variation

Confidence Interval for Variation 1) Based on Chi-square Distribution-----> distribution of a sum of the squares of k independent standard normal random variables. 2) The mean of a Chi Square distribution is its degrees of freedom. Chi Square distributions are positively skewed, with the degree of skew decreasing with increasing degrees of freedom. As the degrees of freedom increases, the Chi Square distribution approaches a normal distribution.

A process adds value in the value chain if it satisfies which of the following? A) It is done right the first time, the customer is willing to pay for it, and it changes the thing in the process. B) It is done right the first time, the customer is willing to pay for it, and it does not change the thing passing through the process. C) It is done right the first time, the customer does not care, and it changes the thing in the process. D) It reduces error, the customer is willing to pay for it, and it changes the thing in the process.

A= CORRECT !--A process is value added if it meets all three criteria: 1. done right the first time (no rework), 2. the customer is willing to pay for it, and 3. the process changes the thing that passes through the process. A process that reduces error means it was not done right the first time and has errors that need rework, so it is not a value-added process. f the thing passing through the process is not changed then it is not required at all and is not a value-added process.

What are the two optional stages in a project-driven team? asq/learn_check.gif Adjourning and recognition Forming and storming Storming and norming Performing and adjourning

Adjourning and recognition are additional stages in a project-driven team since project teams have a beginning and an end.

Affinity Diagrams--3 uses

Affinity Diagrams 1) Good tool to take complex customer information and to organize it into logical categories. 2) This technique can be used to organize potential creative solutions 3) Best used in the "Problem Definition" stage of Six Sigma

Defined Scope Brainstorming vs. open scope brainstorming.

After team decides on the project or subject, then it is important to define the problem to aid in keeping focused on the project/ topic. If the subject if completely unfamiliar to team, then keep the scope of the brainstorming ides open to get a large variety of ideas. Examples of Defined Scope-- low atendence to meeting, , failed menu for Christmas Examples of open brainsotrming= global waminig or unemployment

Which of the following hypothesis test can be used to determine if the variance calculated from machine #1 data on Shift A is the same as on machine #1 during shift B? t-Test F-Test Chi-Square Test z-Test

An F-test compares the variance from one population (Shift A) to the variance from another population (Shift B). The t-Test compares sample means. A z-Test is a test for two means. Chi-Square tests are for normality.

Armand Feigenbaum 7 Key awards

Armand Feigenbaum==7 Key awards 1. Honorary ASQ member 2. E. Jack Lancaster award, ASQ 3. 2- time president of ASQ 4. Founding Chairman, International Academy of Quality 5. Fellow, American Association for the Advancement of Science.

3. Which distribution is appropriate for a continuous set of data with a fixed lower boundary but no upper boundary? a. Johnson b. exponential c. normal d. Logormal

B- my answer but incorrect ...look up difference between a, b, d "3. D: Lognormal. A lognormal distribution is appropriate for a continuous set of data with a fixed lower boundary but no upper boundary. In most cases, the lower boundary on a lognormal distribution is zero. These distributions can be tested with a goodness-of-fit test. A Johnson distribution is more appropriate for continuous data that for whatever reason is inappropriate for a normal or exponential distribution. An exponential distribution is appropriate for any set of continuous data, though these distributions are most often used for frequency data. A normal distribution is appropriate for a set of continuous data with neither an upper nor a lower boundary. The normal distribution follows the pattern of the classic bell curve." "

Twenty-one randomly selected managers in the healthcare industry were sampled with regard to their average bonus. A 95% confidence interval was calculated to be ($1,131,356, $4,824,123). Which of the following interpretations is correct? 95% of the sampled bonus values fell between $1,131,356 and $4,824,123. There is a 95% confident that the sample mean of managers' bonuses falls in the interval $1,131,356 and $4,824,123. Within the population of Managers in the healthcare industry, 95% of managers will have a bonus that fall in the interval $1,131,356 and $4,824,123. If we calculate the confidence interval 100 times (one of them is $1,131,356 and $4,824,123), almost 95 of them contain the population average for Managers' bonuses.

Based on sample evidence, we are 95% confident that the true, but unknown, population average for Managers' bonuses in the healthcare industry is between $1,131,356 and $4,824,123.This is an accurate interpretation of a confidence interval.

An agile project team decided to use the expert judgment project estimation technique based on three points estimate in an agile environment. Which factors should be considered? A) Planning Poker B) Regression Analysis C) Best Case, Worst Case and Most Likely Scenarios D) Lead time, throughout and Work In Process (WIP)

Best Case, Worst Case and Most Likely Scenarios are the 3 points that are used in the formula W + (4M) + 6 Regression Analysis calculates projected times for an activity based on historical records from previous projects. The result is an activity estimate based on measures like time per line of code. Planning Poker is an expert judgment project estimation technique based on estimation from team members and building consequences on estimation of user story.

A standard has a known value of 15.500 mm and is measured twenty times obtaining an average of 15.401 mm. If the tolerance is +/- 0.25 then what is the percent bias? 0.396 0.0253 0.198 0.0505

Bias 15.500-15.401 = .099 Note: Absolute value of Bias result % bias=(.099*100)/.500=19.8%

Design of Experimentation Block

Block a. Portion of the experimental material or environment that is common to itself and distinct from other portions. b. Mitigates the effect of variable that we are trying to avoid via grouping and eliminate the effect of special cause variation by the grouping experiments in batches of test runs.. i. Randomized block design—use to nullify the by grouping a selected factor that is the source of variability ****In the statistical theory of the design of experiments, blocking is the arranging of experimental units in groups (blocks) that are similar to one another. Typically, a blocking factor is a source of variability that is not of primary interest to the experimenter. An example of a blocking factor might be the sex of a patient; by blocking on sex, this source of variability is controlled for, thus leading to greater accuracy.

Business Case (Financial Impact) Section of the Project Charter

Business Case (Financial Impact) Section of the Project Charter 1) Summarizes the strategic reasons of the project---generally about quality, cost, or deliver of product with financial justification 2) Basic Activities a) Design a new product /process b) Redesign new process / existing process

8. How is takt time calculated? a. Available time divided by demand b. Overall process time minus time required for a particular task c. Demand divided by the amount of time available d. Time required for a task divided by demand"

C "8. C: Demand divided by the amount of time available. Takt time is calculated by dividing demand by the amount of time available. This value, which is also known as target process time, should be posted at each workstation during the process of level loading. The takt time is the maximum amount of time a process can take without slowing down the overall completion of the task."

"Parameters of statistical distribution

"Parameters of statistical distribution 1) The four primary parameters of a statistical distribution are central tendency, skewness, standard deviation, and kurtosis. 2) Central tendency = general trend indicated by the data. 3) Symmetrical distribution ( eg. normal distribution) ---->, the mean is the best estimate of the central tendency; No skew 4) Asymmetrical distribution, the median is a more accurate site of the central tendency. ( Skew exists) 5) Skewness of a distribution is essentially the distance between the average and the mode, or the most-represented data value. 6) Standard deviation = average variation of data points from the mean. or suggests how close to the mean each data point likely will be. 7)Kurtosis = sharpness of the distribution's peak. In a normal distribution, the kurtosis is 1. A distribution with a sharper peak will have a higher value for the kurtosis."

Constraint Management

Constraint Management: 1. First introduced by Goldratt in book "The Goal"---speaks of the trials of a plant managers as he struggles to manage his plant & marriage. 2. Constraint Management-- removing the bottlenecks that limit production or throughput in the process. 3. For ongoing improvement, management must find the system constraint that is limiting production or throughput. 4. Process Chart or Value Stream Map can be used to find the system constraint.

Creation of process capability:

Creation of process capability: 1) Must use to determine the stability of the process before calculating process capability 2) The capability indices for a normal distribution are calculated , , and . In these equations, is the grand average, T is the process target, and σx is process sigma. If the subgroup size is 1, process sigma is calculated according to the moving-range statistic. If the subgroup size is greater than 1, the process sigma is calculated with the subgroup sigma statistic. The capability indices for non-normal distributions are calculated with ( see formula) , , and . In these equations, ordinate 0.99865 and ordinate0.00135 are the z values for the non-normal distribution curve at 99.865 and 0.135. Also, Znormal,p and Znormal, 1 - p are the z values of the normal cumulative distribution curve at the p and 1 - p percentage points, respectively."

Critical Path Method--5 Unique Features

Critical Path Method--5 Unique Features 1) Emphasis is on activities 2) Time & Cost factors for each activity are considered. 3) Only activities on the critical path are contemplated. 4) Activities with the lowest crash cost (per incremental time salving) are selected first. 5) As an activity is crashed, it is possible for new critical path to develop

DFMEA & Product Classifications

DFMEA & Product Classifications: Characteristics of a Product can be Classified as: 1) Critical---> shall comply with safety/ government regulation or service requirements 2) Significant---> customer & supplier quality features 3) Key---> characteristics that provide prompt feedback on the product aiding in corrective action process ***AIAG FMEA has 2 design columns---one for prevention and one for detection a) Detection ranking for control used for RPN calculations b)Emphasis for recommended actions is placed on reducing those with high severity ranking followed by those with high RPN numbers

DFSS Roadmaps & Denominator Management

DFSS Roadmaps & Denominator Management 1. Denominator Management= a short-term method of making the desired return on investment by altering factors in the ROI equation a) Can Reduce the Denominator (Investment)--eg. downsizing, restructuring, process redesign, cost containment, or investments in people/ materials/ resources b) Increase Numerator---Increasing sale by introducing more new products--40% of profits are from sales 2. ROI= Net income/ Investment

DMADV

DMADV a. DMAIC is used, when the problem you want to solve is for a process that is already existing but not meeting the expected levels of performance. DMAIC is used when you want to correct an existing process that has not been performing up to the expectations. b. Alternatively, DMADV is used when you want to set up/design a whole new process, that should be meeting the desired level of performance from the beginning. DMADV may also be used when there is a need to redesign an existing product or process, rather than constantly correcting the process. The soul of DMADV is in understanding the customer's requirements and the CTQs which are mostly expressed qualitatively.

Deciding on the Type of Control Chart to use when working with variable data

Deciding on the Type of Control Chart to use when working with variable data First decide what type of data you're dealing with. Variable data takes on a measurable, numeric value. There are many possible values. Attribute data consists of categories. There are only a few (usually two) discrete values. With variable data, decide how large subgroups are. If the subgroup size is one, then use an Individual measurements chart, with or without a moving R chart. If the subgroup size is from two to ten (or possibly twelve), then use an X and R chart. If the subgroup size is over ten (or twelve), then use an X and S chart.

Process Performance Measures: Defects per unit (DPU) Vs. DPMO?

Defects per unit (DPU) = (Total defects) / (Total products produced a given time period) Defect per million opportunities (DPMO) i. DPO= DPU / (Total number of opportunities) ii. DPMO= DPO X 10^6

Describe the process performance indices, including their application to the measure stage of DMAIC

Describe the process perfoRmance indices, including their application to the measure stage of DMAIC 1) Process performanc index---> tells whether a particular batch of material will be satisfactory to customer 2) Key difference between process performance index and process capability---> Pp limited to single batch. 3) Process performance index used to create process baseline estimates for uncontrolled processes------> requires large sample size. 4) Statistical control is preferred over Process performance index but can't be done if process lacks statistical control or insufficent data.

Describe the use of enumerative statistics in process baseline estimation

Describe the use of enumerative statistics in process baseline estimation 1) In estimation of a process baseline, enumerative statistics used to evaluate random samples from populations. 2) Enumerative statistics can determine whether samples represent the population, meaning that they were drawn without bias. 3) Confience level intervals of 95% used. 4) Drawback of Enumerative Statistics----> they are drawn from static, unchanging population.--but Six Sigma needs information from dynamic process and uses analytical statistics.

Design Selection Guidelines

Design Selection Guidelines **Choice of design depends on the amount of resources and degree of control over making wrong decisions that experimentar desires. 1) If there are one factor----> Comparative objective = 1-factor randomized design. 2) If there are 2-4 factors, ------> a) Comparative Design = Randomized Block Design b) Screening Objective = Full or Fractional Factorial c) Response Surface objective = Central composite or Box-Behnken 3) if there are 5 or more -----> a) Comparative Design = Randomized Block Design b) Screening Objective = Fractional Factorial or Plackett-Burman c) Response Surface objective = Screen first to reduce the number of factors.

Design for Compliance

Design for Compliance--- Regulations must be met before product is marketed (eg. Design process that must achieve specific product performance capabilities).... *** 1. If the DFSS initiative is operating in a highly regulated environment, cost-benefit can be derived by the penalties and opportunity costs of noncompliance

Design for Extended Functionality—

Design for Extended Functionality— products initially designed for single purpose may have features extended for additional functions (eg. Computers)

Design for Manufacturing/Producibility / Assembly---

Design for Manufacturing/Producibility / Assembly--- The earlier the manufacturing staff are involved in design process, the more producible the design. Tolerance design (minor design changes to reduce costs and easier to produce)

Developing an SIPOC Diagram

Developing an SIPOC Diagram: 1) Team will create process map that will be posted on the meeting room wall 2) 4-5 key steps in a process. 3) Process outputs---product/ service of this process. 4) Customers of the process outputs (end users) 5) Process inputs / Materials 6)Suppliers of the process 7) Identification of preliminary customer requirements 8) Involvement of stakeholders, team leader, and champion for verification.

Devise a sampling plan for obtaining a representative moisture sample from an open top railcar of coal, where the contents are known to have changed and also stratified due to vibration and exposure during the 10 days of shipment. A) Draw a slipstream sample by taking a representative slice of the bulk material as the car is being unloaded being certain to capture all of the generated dust. B) Draw a quota sample with one sample off the top and from five more standardized depths in the railcar. C) Receive a certificate of analysis from the supplier of a sample properly collected from the entire load at the time of loading knowing that the coal got wet and dust blew off the open railcar. D) Draw a convenience sample from multiple points that can be easily accessed.

Drawing a slipstream sample by taking a representative slice of the bulk material as the car is being unloaded being certain to capture all of the generated dust samples the entire railcar following arrival and allows assessment of moisture and changes in material from initial shipment. The product will have changed during transit in terms of moisture content or in other properties such as changes from particles blown off the car during shipment. Therefore, the analyses of the initial product will not represent the received product, regardless how careful and accurately the analyses are done by the supplier. Drawing a convenience sample from multiple points that can be easily accessed is a non-probabilistic sample. Drawing a quota sample with one sample off the top and from five more standardized depths in the railcar. is a non-probabilistic sample because the sampler has no prior knowledge of the stratification, has no equally spaced distribution plan, and likely has skewed the data by taking the sample from the top, but apparently not from the very bottom.

Duties of Green Belt

Duties of Green Belt i. Support 6 sigma projects with high ROI ii. Follow DMAIC process and apply statistics iii. Identify and report barriers hindering the success of the project. iv. Share lessons learned with extended team v. Consult/ inputs to Black Belt

EVOP (Evolutionary Operation)

EVOP (Evolutionary Operation) EVOP (evolutionary operations) is an online optimization technique. Usually, two factors are studied using small, step changes in factor levels to incrementally explore the operating bounds of the process. 1) The way sequential experimental designs can be made to adapt to system behavior by learning from present results and predicting future treatments for better response. 2) Small response environments may be made via large sample sizes. 3) The experimental risk, however, is quite low because the trials are conducted in the near vicinity of an already satisfactory process.

Employee Training

Employee Training 1) Employee training is vital for maintaining improvements--- should include employees from different departments and levels of the organization. 2) Employees should learn the updated processes, key process factors and inputs, & why the process changes will improve his or her work. 3) Provide a little context to give employees a sense of the rationale for changes 4) Training sessions should be pitched to the level of the audience so all employees will understand the content. 5) All training sessions should conclude with some sort of evaluation in order to gauge employee comprehension. "

Engineering process control (EPC)

Engineering process control (EPC) 1) Mechanical system for automatically adjusting input in reponse to process variations ( can have multiple inputs and outputs) a) Example---A refrigerator is equipped with a basic engineering process control: when the temperature inside rises too far from the desired level, a cooling engine lowers the temperature. b) Don't use statistical process controls when using engineering process control devices because EPC System's constant adjustment will influence the data. ****Data collected from engineering process control devices is not independent.

Ensuring Data Accuracy and Integrity

Ensuring Data Accuracy and Integrity Bad Data corrupts decision-making process---Preventions methods: 1) Avoid emotional bias relative to targets or tolerance when counting/ measuring/ recording 2) Time-sequenced data---> Record the order of its capture 3) Avoid Rounding (reduces measurement sensitivity) & Calculate average to at least one more decimal than individual readings 4) Use objective statistical tests to identify outliners rather than guessing. 5) Sampling plan is selected on the basis of analyst's specific needs and experiance. 6) Screen & filter data to detect & remove data entry erros (eg. transposition and magnitude shifts due to misplaced decimal point) 7) To apply statistics which assumes normal population, determine whether the expected dispersion can be represented by at least 8-10 resolution increments-----> If no, then default statistic may be the count of observations which do or don't meet specification criteria. 8) Make sure that your data has labels

"Equality-of-variance tests

Equality-of-variance tests 1) Determines if similar degree of variation exists in subsets of data. 2)Establishes equal variance which is needed to conduct analysis of variance 3) Analyze & Improve Stages ---used to assess observed data from analysis of variance. 4) Regression Analysis --equality-of-variance tests may be used after regression analysis to assess the residuals. 5) Barlett (MC used test for equality of variance)= best test if regression residuals show normal distribution a) Compares equality of treatment variances with the possibility that one variance is unequal to others H0: All Subsets have equal variances H1: At least one of the subsets has unequal variance. b) Found in Levene & Minitab. --> Best to use Levene test if non-normality highly possible. *** To assess the variance at each position, must have more than one sample from each test condition. If non-normal variance exists, the variable in question is transformed to mitigate its effect.

Excess Processing (source of Waste)

Excess Processing a. Type of waste that is difficult to recognize. b. Eg. (Rust prevention (greasing) of parts before painting is non-value added because customer unwilling to pay for degreasing )

Kano Model & Exciting Needs

Exciting Needs = These are features and properties that make a supplier a leader in the market. a. The highest level of customer expectations, as described by Kano, is termed the wow level qualities, properties, or attributes. b. These expectations are also known as the delighters or exciters because they go well beyond anything the customer might imagine and ask for. Their absence does nothing to hurt a possible sale, but their presence improves the likelihood of purchase. c. Wows not only excite customers to make on-the-spot purchases but make them return for future purchases. These are unspoken ways of knocking the customer's socks off.

Executive Sponsors

Executive Sponsors 1) Key element in the black belt program 2) Sets the direction & priorities for the organizaiton 3) Executive team == leaders that will communicate, lead/direct company objectives for successful deployment of six sigma project.

Kano Model & Expected Model

Expected Needs = Fully satisfying the customer at this level simply gets a supplier into the market. The entry level expectations are the must level qualities, properties, or attributes. a. These expectations are also known as the dissatisfiers because by themselves they cannot fully satisfy a customer. However, failure to provide these basic expectations will cause dissatisfaction. b. Examples include attributes relative to safety, latest generation automotive components such as a self-starter, and the use of all new parts if a product is offered for sale as previously unused or new. The musts include customer assumptions, expected qualities, expected functions, and other unspoken expectations.

Design of Experimentation Experimental Design

Experimental Design a. Formal design or patterns that includes responses, factors, levels, blocks, treatments and the use of planned grouping, randomization, and replication. 6. Experimental Error a. Variation in response variable, when levels and factors are held constant. b. Experimental error must be s subtracted to determine

Experimental Objective ----Comparative Objective

Experimental Objective ----Comparative Objective **Experiment that is used to make a conclusion about which factor is significant---and then need to find a comparative design solution when there are several factors,

F-test

F-test F test - test of whether two samples are drawn from different populations have the same standard deviation, with specified confidence level. Samples may be of different sizes. The F-Test used in the hypothesis testing of variances as in ANOVA. The F-Test assumes a normally distribution, as well as Bartlett's Test. The samples should exhibit normal conditions within each set of experiments (or trials). Levene's Test is similar, but is used when analyzing data that is not from (or can not be assumed) a normal distribution. This test can be used for any continuous distribution to compare variances. The value of F represents the ratio of two variances, and comparing the F-test value to the F-critical value is used to make a decision on the null hypothesis. It is used to compare: 1 Sample Variance to a Target 2 Sample Variances >2 Variances use ANOVA In ANOVA, the value of F is the ratio of treatment variance to the error variance.

FMEA Process Steps---Risk Assemssment & RPN

FMEA Process Steps---Risk Assemssment & RPN: 1. P= probability that failure mode will occur is ranked on index of 1 (least likely) to 10 (high risk) 2. S= Severity ranked on 1 (low risk) to 10 (high risk) 3. D= Design Weakness----> effectiveness of current control in place to identify failures prior to release to production which is ranked 1-10 4. RPN = P x S x D Actions are based on

Feigenbaum

Feigenbaum a. Originated Total Quality Control b. Describes the two primary components of quality costs as the cost of quality disconnects (failure costs) and the costs of quality investment (prevention and appraisal costs). c. Three steps to quality -→ Quality leadership , modern quality technology, & organizational commitment

Feigenbaum : Nine M's of quality:

Feigenbaum was also known for his Nine M's of quality: 1) markets, management, men, 2) money, motivation, 3) materials, machines, modern information sources, 4) mounting product requirements. (By mounting, Feigenbaum means consistently improving).

First-order model of multiple regression

First-order model of multiple regression 1) Multiple regression used if multiple factors influence a dependent variable of if there ae interactions between these multiple factors. I a) 1st -order model= dependent variable ' s value is based on the influence of each factor by itself plus possible any combination of two factors. b) Assume that errors in the model are the same for all factors and combinations of factors. c) First-order multiple regression graph = straight line over small regions & useful when only targeted data needed. *** Linear regression ( different from MR) = the plane curved or twisted caused by the influence of interacting factors."

Genichi Taguchi Contribution

Genichi Taguchi 1) Advocate of quality management in Japan. 2) Design of Experiments, which included the first signal-to-noise measurement. 3) Taguchi was employed by Japan's Ministry of Public Health and Welfare, where he promoted the use of statistics in quality management.

Geometric Distribution

Geometric Distribution Basic assumptions: Discrete distribution Just two outcomes for each trial Trials are independent All trials have the same probability of occurrence Waiting time until the first occurrence Uses include: Number of failures before the first success in a sequence of trials with probability of success p for each trial Number of items inspected before finding the first defective item - for example, the number of interviews performed before finding the first acceptable candidate

Graphical Method: Tally

Graphical method=Tally a. Used to count defect quality by type, class. or category b. Provides visual idea of distribution shape c. Tally mark concentration and spread indicate distribution shape. i. Tally marks of 5 are crossed out as a group ii. Isolated groups of tall marks= uneven distribution.

Grouping of customers into segments driven by customer requirements into the following categories:

Grouping of customers into segments driven by customer requirements into the following categories: a. Internal and External b. Age groups (esp. consumer products) c. Geographic location (climate, language, ethnicity, and ect) d. Industry type. *****When project changes are proposed, must consult with external/internal customers and represent customer concerns.

Guidelines for using Rational Subgrouping

Guidelines for using Rational Subgrouping 2. Processes must not be out of control to use control chart. 3. If processes are out of control, then use run chart. 4. Selection should result in groups that are homogenous. 5. First subgroup should reflect product all produced 6. Most uselful information comes from smaller groups (less than 25) 7. Attributes control charges are based on binomial distributions and require 50 or more samples within subgroups.

Two Fundamental Rules of Rational Subgrouping

Here are a few rules to consider for rational subgrouping. 1) Rule 1: Minimize the variation within a subgroup Form the subgroups so that there is the minimum chance for variation to occur. Can you overdo this? Yes, it is possible. But most of the time, you should strive to minimize the within subgroup variation. 2) Rule 2: Maximize the opportunity for variation to occur between subgroups This means that if there is an opportunity for two things to be different, be sure to put them in separate subgroups. Go back to our three subgrouping plans. We have already ruled out Plan C because it blends the streams thus putting four things that could be different into the same subgroup. Rule 2 is violated. What about Plan A and Plan B? Plan A takes four parts from the same machine and forms a subgroup. So, each subgroup contains parts from the same machine. Plan B takes one part from each machine and forms a subgroup. Note that we are mixing things that could be different - after all they are produced by different machines. We are violating rule 2 with Plan B. This seems to leave Plan A as the best subgrouping plan.

Higher-order models of multiple regression

Higher-order models of multiple regression 1) Used for complex analysis of multiple factors influence a dependent variable, a) Can include squares and cubes of the values that produce a response surface with definable peaks and valleys. b) Use only if exhaustive experiments already have been performed on the main effects. c) Useful for mapping smaller regions, esp peaks, valleys, and minimaxes (intersections between the minimum for one factor and the maximum for another). d) Good for defining the area that surrounds a stationary point. ****In Six Sigma, these models often are used to evaluate how current operating parameters influence the response."

Highly disciplined process that focuses on developing and delivering near-perfect product and services consistently

Highly disciplined process that focuses on developing and delivering near-perfect product and services consistently Six Sigma

Histograms Shapes----Double-peaked or bimodal.

Histograms Shapes---Double-peaked or bimodal. The bimodal distribution looks like the back of a two-humped camel. The outcomes of two processes with different distributions are combined in one set of data. For example, a distribution of production data from a two-shift operation might be bimodal, if each shift produces a different distribution of results. Stratification often reveals this problem.

Histograms Shapes---Skewed distribution

Histograms Shapes---Skewed distribution 1) The skewed distribution is asymmetrical because a natural limit prevents outcomes on one side. 2) The distribution's peak is off center toward the limit and a tail stretches away from it. ****For example, a distribution of analyses of a very pure product would be skewed, because the product cannot be more than 100 percent pure. Other examples of natural limits are holes that cannot be smaller than the diameter of the drill bit or call-handling times that cannot be less than zero. These distributions are called right - or left-skewed according to the direction of the tail.

Hypothesis Tests for Means Two-mean, Equal Variance T-test

Hypothesis Tests for Means Two-mean, Equal Variance T-test a. Tests are between two sample means and sigma are unknown but considered equal. 5. Two-mean, Unequal Variance T-test a. Tests between two samples means with unknown sigmas that are not equal.

Improvement through FMEA via 4-step process

Improvement through FMEA via 4 step process ** FMEA considered tedious and only good for quality audits. Bongio suggests 4-step process: 1. Measure current FMEA performance for a baseline. 2. Agree on a projected FMEA performance level 3. Develop a work plan to close the gaps 4. Implement the recommended methods to reach target

Improvements in Customer Service

Improvements in Customer Service include 1) Listen to customers needs 2) Service strategy should be customer focus 3) Set performance standards ---needed for measures and results 4) Select the right employees and properly train employees 5) Recognize & reward *** Must also get info from non-customers.

portion of units obtained from the same location in all containers, racks, bins, or dunnage illustrates which of the following? Producer's risk Sampling bias Consumer's risk Stratified sampling

In sampling bias, procedures set up for sampling can deteriorate into biases. In this example, the sample is not random, which results in bias. Stratified sampling is sampling that is deliberately stratified to obtain sample proportions from each true lot. In this example, you are looking for the term that illustrates what would happen without rigorous procedures for stratified sampling. Consumer's risk is the probability that a "bad" lot will be accepted by the sampling plan. Producer's risk is the probability that a good lot is rejected by the sampling plan.

Types of Attribute Control Charts: U=chart

In statistical quality control, the u-chart is a type of control chart used to monitor "count"-type data where the sample size is greater than one, typically the average number of nonconformities per unit. The u-chart differs from the c-chart in that it accounts for the possibility that the number or size of inspection units for which nonconformities are to be counted may vary. Larger samples may be an economic necessity or may be necessary to increase the area of opportunity in order to track very low nonconformity levels.[1] As with the c-chart, the Poisson distribution is the basis for the chart and requires the same assumptions.

Which of the following is a reason that market share is a key concern of management? A) Increasing market share brings significant additional risk to delivery of existing products and services. B) Increasing market share may bring economies of scale as fixed costs do not increase in proportion to variable costs. C) Decreasing market share improves the ratio of fixed costs to variable costs. D) Market share is a reliable indicator of a product or service's capability to address management's needs.

Increasing market share may bring economies of scale as fixed costs do not increase in proportion to variable costs, is the correct answer. Market share indicates the capability of a company's products/services/prices to address the needs of the customer. Increasing market share brings additional risk, but it is considered modest as the means of delivery and deployment have been tested and found effective. Decreasing market share worsens the ratio of fixed costs to variable costs, the opposite of gaining economies of scale.

Interaction

Interaction 1) Occurs when the effect of one input factor on the output depends upon the level of another input factor. 2) Often, interactions are lost with fractional factorial experiments. 3) Preferred DOE approach examines (screens) a large number of factors with highly fractional experiments. Interactions are then explored or additional levels examined once the suspected factors have been reduced. often, a full factorial or three level fractional factorial trial is used in the follow-up experiment.

Interest-based Bargaining

Interest-based Bargaining 1) Same as Win-Win 2) Different from traditional collective bargaining where the outcome of the negotiations creates only winners and losers.

Internal Customers

Internal Customers 1) Anyone in company affected by the product/service as it is being generated. 2) Aim to create a product/service that is convenient for internal consumption---> proper training, equipment, specific instructions, and standardized practices 3) Remove sectionalism--> enable employees of all departments to contribute to problem-solving 4) Management practices relate to employee satisfaction which impacts customer satisfaction. 5)Staff members (design/ purchasers/ engineers) must consider themselves as service providers to prevent conflicts with line management & employees

Internal Failure Costs

Internal Failure Costs 1) Employee Turnover 2) Late Time cards 3) Overpayments 4) Rework/Redesign/ Retyping letters/ Rework 5) Sorting/ Scrap 6) Excess inventory/ material handling/ interest expense/ travel expense

Which of the following is the best statement describing the difference between an external customer and an internal customer? A) Internal customers are within an organization, and external customers receive a product or service from the organization. B) External customers are the purchaser, and internal customers are merchants. C) Internal customers are suppliers, and external customers are employees. C) Both internal and external customers are within the organization.

Internal customers are within an organization, and external customers receive a product or service from the organization provides the definition for internal and external customers.

Internal employee communications for customer satisfaction can be improved via 7 tools

Internal employee communications for customer satisfaction can be improved via 7 tools: 1) Company newsletters ---corporate news/ basic info. 2) Story Boards--memos, letter, projects 3) Team meetings--business news/ new event announcements 4) Posting customer letters of appreciation/ dissatisfaction 5) Staff meetings 6) Display of goals/ progress charts 7) Quality awards from customers.

Interpretation of P-chart

Interpretation Of a P chart, 1) If all data points lie between the upper and lower control limits---> process in statistical control (+) variations due to common causes signals that future performance can be predicted. 2) If data points outside of the control limits,--> special causes of variation & must look for possible sources of special-cause variation and then measure them with designed experiments. 3) If P chart contains some points outside the control limits----> it is possible to predict the future process capability by simply removing the out-of-control points.

Iterative Approach to DOE

Iterative Approach to DOE 1) Approach Experimental Design---> more times experiment repeated, the more complete the answer. ---"Iterative Approach" is more economical because it if logical to move thru stages of experimentation, each stage supplying a different kind of answer.

Juran & 3 dimension of measuring process quality

Juran & 3 dimension of measuring process quality 1) Effectiveness= how well the output meets the customer needs. 2) Efficiency = the ability to be effective , at the lowest cost. 3) Adaptability= the ability to remain effective and efficient in the face of change ***These measurements address the need for business processes to provide value to both customer (effectiveness) and shareholders (efficiency), now and in the future (adaptability) . Six Sigma tries to manage ENTIRE business process the maximize the business process goals for the OVERALL business.

Which one of the following in the Juran Trilogy included evaluating actual quality performance? A) Quality improvement B) Quality planning C) Quality assurance D) Quality control

Juran was one of the first to write about the cost of poor quality.[11] This was illustrated by his "Juran trilogy", an approach to cross-functional management, which is composed of three managerial processes: quality planning, quality control, and quality improvement. Without change, there will be a constant waste, during change there will be increased costs, but after the improvement, margins will be higher and the increased costs get recouped. D= Quality control includes evaluating actual performance, comparing actual performance, and acting on the difference. Quality planning involves establishing goals, identifying customers and their needs, and developing features, processes, and controls to meet the customer needs. Quality improvement involves proving the need, establishing the infrastructure, identifying the improvement projects, and establishing teams and controls.

Define Kaizen

Kaizen (Japanese "Continuous Improvement") → Process control that can be enhanced via Kaizen events that reduce non-value added activities. The result is more productive and permits a closer process focus by all those involved. i. Kaizen is derived from a Japanese term for continuous improvement in manufacturing and business standards and practices. Kaizen is comprised of five main elements: teamwork, personal discipline, improved morale, quality circles, and improvement suggestions. The goal of Kaizen is to standardize and improve manufacturing and business activities continuously. ii. An important distinction between value-added and non-value-added activities is that value-added activities are measurable contributors to performance. ***Key Takeaway: In business and manufacturing, value-added activities are those activities that contribute to the performance of a business unit whether operations, productions, or sales. For example, offering sign-on bonuses to customers is considered a value-added activity if it leads to a 5% increase in conversion to sales for a company.

Kaizen Blitz & its 7 Performance Metrics

Kaizen Blitz 1) Most Kaizen activities are generally long-term but a Kaizen blitz is a short-term Kaizen activities 2) Kaizen Blitz---> uses cross-functional volunteers for 3-5 days to create a rapid change in the workplace on a project basis. 3) Requires a Facilitator 4) Performance metrics used to evaluate Kaizen Blitz are the following: a) Floor space Saved b) Safer Work Environment c)I mprovement ideas d) More line flexibility e) Increased quality levels f) Improved work flow g) Reduced quality levels

Kanban

Kanban (Pull System) 1) Signal ( usually a card) to internal process to provide some product. Only upon receiving a kanban card will an operator produce more good---> aims to reduce WIP & finished goods inventory and simplify paperwork 2) Provides indication of: a) Part number b) Location/ delivery time & frequency c) Bar Codes d) Quantity e) Color of shelves at destination. 3) Efforts are made to eliminate causes of machine downtime and sources of production errors------> because a stoppage will distress entire production system. 4) Applicable to repetitive production plants, but not in one-of- a kind production operations. 5) Designed to provide product to customer in shortest time possible.

Kano Analysis model involves two dimensions:

Kano Analysis model involves two dimensions: • Achievement (the horizontal axis) which runs from the supplier didn't do it at all to the supplier did it very well. • Satisfaction (the vertical axis) that goes from total dissatisfaction with the product or service to total satisfaction with the product or service. • The figure below portrays the three levels of need: expected, normal, and exciting.

Kaoru Ishikawa---Awards

Kaoru Ishikawa---Awards 1. Deming Price (1952) 2. Nihon Keizai Press Prize 3. ASQ's Grant Award 4. Shewart Medal---1st Japanese to get this award 5. Establi

Key Guidelines for resolving conflicts

Key Guidelines for resolving conflicts 1) Determine importance of issue to all those involved and if the issue can be discussed----> select private meeting place 2) Let parties make open comments and guide them toward to clear problem definition. (same as ethical decision-making framework)

3 Key steps in achieving the ideal design:

Key steps in achieving the ideal design: a. Link customer requirements to features of products and processes (Use Quality function Deployment) b. Design product/process that will give desired quality at the lowest cost. c. Use design tools that will result

Lean Concepts & Tools--Lean Enterprise

Lean Concepts & Tools--Lean Enterprise a. Organization that effectively uses lean thinking and applies lean tools to reduce waste throughout the value stream and offere value to the customers b. Requires examination of the flow of information and materials

A project sponsor asked a Manager of an agile team to collect the lessons learned. Which of the following is the best statement that applies to this situation? The manager should collect the lessons learned throughout the project. The manager should collect the lessons learned only at the end of the project The manager should collect the lessons learned only when the project performance is not good The manager should collect the lessons learned when the projects go well

Lessons learned should be captured throughout the project when the details are fresh and team members can remember the most detail. Also this allows lessons learned early in the project to inform later phases.

Level

Level 1) A given factor or a specific setting an input factor 2) Four level of a heat treatment may be 100, 120, 140, and 160 degrees fariegnheight.

Linearity

Linearity 1) Found by obtaining reference part measurement values throughout the operating range of the instrument and plotting the bias against the reference values. 2) Default procedure for determining linearity is to measure 10 parts 5 times. 3) % linearity = slope -----> Linearity is equal to the slope multiplied by the process variation (process spread or tolerance)

Linking Projects to Organizational Goals:

Linking Projects to Organizational Goals: a. Projection selection Team (set the criteria for project selection and team assignments): 1. Master Black Belts 2. Black Belts 3. Champions 4. Executive supporters b. Project selection criteria= key element to furthering organizational goals

List 4 reasons that Pareto diagram are used

List 4 reasons that Pareto diagram are used: 1) Analyze a problem from a new perspective 2) Focus attention on problems in priority order 3) Compare Data changes during different time periods 4)Basis for construction of a cumulative line ***Cumulative lines help answer questions like "What defect classes constitute 70% of all defect?"

Listening Strategies

Listening Strategies 1) Project Managers spend 45% of their time being a listener 2) Active listening---helping find the source of problem or meaning 3) Put the message sender at ease 4) Remove distractions and be patient with your response.

Lognormal Distribution

Lognormal Distribution Basic assumptions: Asymmetrical and positively skewed distribution that is constrained by zero. Distribution can exhibit many pdf shapes Describes data that has a large range of values Can be characterized by m and s Uses include simulations of: Distribution of wealth Machine downtimes Duration of time Phenomenon that has a positive skew (tails to the right)

Measurement Correlation

Measurement Correlation 1) Comparison of measurement values from one measurement system with the corresponding values reported by one or more different measurement systems 2) Both measurement system and standard have variation in their measured value. ---eg. calibration of instruments 3) Proficiency Testing ---"Round Robin Testing" ------Measurement system or device is compared against the mean and St. deviation of other similar devices---->can find components of variation due to artifacts and instruments , especially if no national reference standards are present for specific range/ condition being measured. 4) Can also be a mean comparison of values obtained using different measurement methods used to measure different properties------eg. correlation of hardness and method strength, temperature & linear expansion of item being heated, or weight and piece count of small parts

Measurement Scales---Ordinal Data

Measurement Scales---Ordinal Data ) Central location = Median 2) Dispersion of Percentages 3) Significance Test= Sign or Run Test 4) Data arranged in some order but difference between values cannot be determined or are meaningless

Methods used to measure whether a project achieved its intent

Methods used to measure whether a project achieved its intent: a. Project charter= best tool to measure project completion and its established goals/scope/ objectives b. Review of project charter and documented project results is sufficient for closing project. c. Project sponsor may request an external audit d. Analysis of project measures.

Milestones/ Deliverables

Milestones/ Deliverables 1) Initial Team Project should be at 120 days. 2) Only half of the project would include define and measure stages 3) Lower success if the initial project exceeds 160 days

"In businesses that apply the theory of constraints, which element of a process receives immediate attention? a. The most problematic b. The most important c. The most efficient d. The most complicated

Most Problematic "1. A: The most problematic. In businesses that apply the theory of constraints, the most problematic element of a process receives immediate attention. Indeed, the most problematic area is known as the constraint. The focus of improvement will be reducing the constraint without diminishing performance in any other area of the process. Once the targeted constraint has been diminished so that is no longer the most problematic component of th"

Multi-voting

Multi-voting 1) Helps to narrow down the items worthy of immediate attention when there are too many items for a team to work on within a given time frame---Problem solving teams can only address 2-3 items at time. 2) Used to select among the choices on a list with most prioritized items ----usually after brainstorming session a) List can have either ideas for improvement-----OR---potential causes for a problem b) Brainstorming can be used to segregate potential causes into a cause-and-effect diagram.

Nelson Funnel Experiment

Nelson Funnel Experiment 1) While the intent of making small changes to a process may be to make improvements, these perturbations generally increase the variation of the results. To improve a process requires understanding the source of the variation and often controlled experiments to identify process improvements. 2) Equipment operators often 'tweak' the process with the intent to improve yield or throughput. When these adjustment are made without effective feedback system they generally simply increase the variation and chance of failures. 3) No adjustments------> Common cause variation of the system. ---->if we meddle with the process we generally increase the variation. There is a 50% chance that the next drop would have been naturally to the left of the target, so adding a funnel movement would increase the miss to the left. 4) Adjusting to the Target ------> Another cause of this type of variation is managers tampering with worker performance without investing in systemic process changes. Exhorting employees to hit the target more often increases the incidence of meddling with the process. 5) Adusting for " Windage" --->Another way teams increase the variability of a system is to adjust for 'windage'. Windage is the compensation for expected direction of variation. It tends to over compensate and broaden the variation. Examples of the windage adjustments include the US tax policy or corporate right sizing efforts. For item creation windage adjustment occur when using uncorrelated readings to make adjustments for a lot of material, for example. 5) Drift/ Using last run as a guide----This occurs when one operator trains the next operator, who then trains the next, and so on. Another source is not correcting for wear or accumulation often due to ineffective feedback of the process performance.

Obstacles for Conversion of batch process to a continuous flow process

Obstacles for Conversion of batch process to a continuous flow process: 1. We always done it in batches 2. We live in world of departments and functions 3. Lot-based production plant 4. Lack of tools for quick change-over 5. High momentum machinery that is not flexible.

One Sample t-test

One Sample t-test This test compares a sample to a known population mean, historical mean, or targeted mean. The population standard deviation is unknown and the data must satisfy normality assumptions. Given: n = sample size Degrees of freedom (dF) = n-1 Most statistical software will allow a variety of options to be examined from how large a sample must be to detect a certain size difference given a desired level of Power (= 1 - Beta Risk). You can also select various levels of Significance or Alpha Risk. For a given difference that you are interested in, the amount of samples required increases if you want to reduce Beta Risk (which seems logical). However, gathering more samples has a cost and that is the job of the GB/BB to balance getting the most info to get more Power and highest Confidence Level without too much cost or tying

Optimum Quality Costs

Optimum Quality Costs 1) Most companies spend inadequate amount money on prevention activities ----> Low prevention costs but high failure costs or Failure costs are greater than appraisal costs. 2) Relationship between failure costs (internal & external) ----> consider changes in planning or product design. 3) Appraisal measure that are initially taken---> cause internal failures to increase and external failures to decrease

Organizational performance goals and corresponding measurement are obtained the following 4 areas

Organizational performance goals and corresponding measurement are obtained the following 4 areas 1. Profit 2. Cycle time 3. Marketplace response 4. Resources.

Owners and Stakeholder & SIPOC

Owners and Stakeholder & SIPOC 1) Stakeholders ---stockholders, customers, suppliers, company management, employees, community, society 2) SIPOC only explains the supplier-process-customer relationship but this is only one of the many relationships in a business process. 3) Process owner oversight of the process while GB or BB are project managers working acrsos multiple processes ot identify stakeholders 4) Each stakeholder is both a supplier and customer, forming many closed loop processes that must be managed, controlled, balanced, and optimized for business to thrive. 5) Communication within the entire stakeholder community is channeled through internal company procesess ***Objective = improve individual process configurations to improve the overall outcome for all stakeholders.

PDSA

PDSA 1. Deming's unhappiness with Japanese PDCA created the PDSA 2. Key difference is that PDSA is a team-oriented, problem solving technique. 3. Team objective= improve the output and input of every stage. 4. Team ideally made of people in one area or stage of the plant. 5. Advancement happens when team takes material learned from previous session and makes a fresh start.

PFMEA & Potential Cause of Failure

PFMEA & Potential Cause of Failure **PFMEA should not relay on product design changes to correct process weakness Potential Causes: 1) Little/ missing/ wrong elements of labor 2) machine 3) methods 4) measurement 5) environment

A Six Sigma project team has very limited time with an executive champion in an Analyze phase review. Which of the following tools represents the best combination of quantifiable data and visual display to justify the team's selection of Critical Xs from Potential Xs? A) Analysis-of-Variance (ANOVA) table B) Pareto chart C) Cause-and-Effect diagram D) Quality Functional Deployment matrix

Pareto chart combines both quantifiable data with visual display. It is a bar chart, where the bars depict the frequency of different data categories. The bars are arranged in order (left to right) by frequency. Many times the Pareto Chart will also include a line graph that shows the cumulative frequencies of all data types. ANOVA will indeed produce quantifiable justification for separating Critical Xs from Potential Xs, but the table will not display the results visually unless a separate plot such as a Main Effects Plot is employed. he Cause-and-Effect diagram is a visual display and may separate Critical Xs from Potential Xs, but does not typically include any quantifiable data.

First ASQ president

Philip Crosby

Power of Test H0: u = u

Power of Test H0: u = u 1) The value of Beta risk is large if the mu is close to mu-zero to indicate that larger differences are easier to detect. 2) A shift in the mean away from the null increase the probability of detection. In general, as alpha increases, beta decreases and the power of (1- beta) increases. 3) A gain in power can be obtained by accepting a lower level of protection from alpha error 4) Increasing the sample size makes it possible to decrease both alpha and beta and increase power.

Prioritization Matrices

Prioritization Matrices 1) Original Japanese matrix data-analysis tool that would arrange data into a matrix diagram allowing a large array of numbers to be seen at one time. 2) Degree of correlation would be entered into the cell 3) Very complex and not easy to use---> Heavy statistics calculations but computer optional

Process Capability--5 Applications

Process Capability Applications 1) Evaluation of new equipment 2) Reviewing tolerances based on the inherent variability 3) Assigning equipment to product 4) Routine process performance audits 5) Effects of adjustments during processing

Process FMEA

Process FMEA a. Assess the impact of process on producing the desired outcome b. Similar to DFMEA, PFMEA identifies potential failures, the likeihood of occurrence, and the impact of the occurrence. c. Key Difference from DFMEA is the PFMEA is focused only on process changes and not on design changes

Project Team Member Duties

Project Team Member Duties: i. Charter and project scope definition ii. Project meeting input + brainstorming iii. Data collection iv. DMAIC process v. Provide input to green & black belts

QFD Matrix ---Foundation , Right-sided Wall, and other elements

QFD Matrix ---Foundation , Right-sided Wall, and other elements 1) Foundation ---Benchmarking and target values---> indicates "how much" for each of the measures or target values to include ---OR---objective measures for competitive products and technical importance of each factor. 2) Right-hand wall---> Customer competitive assessment and other factors affected the customer---> competition comparison shows the graphically the relative weights between our Primer and the competition. 3) Elements in the house are customized---->easiest method to look at each area of the house separately.

6 Quality Cost Pitfalls

Quality Cost Pitfalls 1) Perfectionalism in numbers---> lengthy debates over precision of quality cost figures may result in delays in moving the initiative forward with lot of total savings 2) Other data pitfalls ----> Presentation to managers should stimulate discussion on improvement proposal and not data validity 3) Inclusion of non-quality costs ---> management must decide exclusion or inclusion of pure product quality waste 4) Implications of reducing quality cost to zero ----> Quality Cost presentation must regonize that zero defects can't be feasible. 5) Reducing Quality costs but increasing total company costs ------->May need to ensure that reduction in quality costs will not increase total costs. 6) Understatement of Quality Costs----->most commonly due to dealing only quality costs in excess of some normal standard. Prevention is to challenge the standard.

Risk Management Phases & Definitions ---Risk Handling

Risk Management Phases & Definitions ---Risk Handling Definition-----> Identify, evaluate , select, and implement actions to fix risk at the lowest possible level given project constraints and goals Inputs: " AM CAT" ---Avoidance, Mitigation, Control, Acceptance, Transference.

SFMEA & Potential Cause of Failure

SFMEA & Potential Cause of Failure 1) LABOR 2) MACHINES 3) METHODS 4) MATERIALS 5) MEASUREMENT 6) ENVIRONMENT

SIPOC Diagram

SIPOC Diagram Close of SIPOC model will show Six Sigma is based on the concept of closed-loop business system: 1) Any change in process output will be related to one or more changes in supplier, inputs, and processes (SIPS) 2) if all SIPS are stable, then output is stable 3) A change in "O" means that one or more SIPS must be changed (a) If the O's do change---> changes in SIP may be used to predict and control changes in O's (b) If the O's DO NOT change---> SIPs being changes are robust and may process process savings opportunities. 4) Apparent violations of this rule indicate that the process model is incomplete (an SIP or O parameter is missing...a chance to build organization wisdom) *** Closed-Loop relationships between SIPs and O's provide a methods to define process correlations and possible cause-effect relationships.

Which of the following tools is used to identify a process? A) SIPOC B)DMAIC C) SPC D) QFD

SIPOC is a tool used to identify a process and all associated elements. SIPOC is an acronym for suppliers, input, process, output, customer DMAIC is a strategy used to improve a process. It is not used to identify the process. SPC, also referred to as Statistical process control is used to control quality through the application of statistical techniques. It is not used to identify a process. QFD also known as Quality Function Deployment is a process in itself. It is used to deploy customer requirements into the product development and production phase

Sequence of 5S tool:

Sequence of 5S tool: (Sort, Set-in-order, Shine, standardize, sustain): a. Sort---eliminate unused items b. Set-in-Order--→arrange required & rarely used items for ease of accessibility. c. Shine—cleaning of work area leads to better measurement results, better movement , and accident prevention. d. Standardize-→ Checklists, instructions, & standards e. Sustain---(Most difficult)-→ employees should be empowered to take ownership of their work areas.

Shewart Cycle

Shewart Cycle ---Design cycle using the PCDA 1. Design product 2. Test product in lab and production 3. Put product on market 4. Test product in service through market research 5. Redesign product in light of consumer reaction.

Subgrouping Schemes-

Subgrouping Schemes *** One needs to remember that control charts are really a study of the variation in your process. And the variation displayed on the control chart depends on how you subgroup your data - which may or may not be the variation you would like to study. 1) When order of production is used as basis for subgrouping, two approaches are possible: 1) Rule of First subgroup= all products produced at one time (follows the rule of selection of rational subgrouping by permitting minimum chance of variation within subgroup and maximum variation from subgroup to subgroup) -----> Result? Change in process average after one subgroup is taken and corrected before the next subgroup and change is NOT reflected in the control chart 2) Product intended to representative of all production over a given period of time---> Result? Product accumulates at the point of production, with a random sample chosen from all product made since the last samples

System Variation

System Variation 1) 85% of all process problems due to common cause variation . 2) Fundamental changes in the process are done to remove common cause varation ----> like changing equipment, machinery, methods, materials, or other factors. 3) Management exhorts the worker to "do the right thing the first time" but are really tampering with the system and causing unpredictable results. 4) Nelson Funnel--------> Adjustment or Tampering? --- >shows how a stable system is inappropriately adjusted; show what happens if people react to individual unit in a stable process a) Statistical stability of a process indicates that if left unaltered, the process will continue to produce outcomes that show a stable pattern of variation. b) Nelson's funnel experiment examines four strategies for adjustment. They are Rule 1: No adjustment is made. The process is left to run. Rule 2: Every time a marble lands away from the target, the funnel is moved from its current position an equal distance from the target, but in the opposite direction. Rule 3: The funnel is moved back to being centered over the target and is then moved from the target in the opposite direction as the marble missed the target and an equal distance. Rule 4: Try to repeat the last performance by moving the funnel over the landing point of the last drop.

Systemic Design--Main Conceptual phase steps

Systemic Design--Main Conceptual phase steps: 1. Clarify task + Id. essential problems 2. Establish function structures 3. Find solutions via brainstorming & intuition 4. Combine solution principles & select qualitatively 5. Firm up concept variants: preliminary calculation and layouts 6. Evaluate concept variants.

Team Diversity

Team Diversity 1. Productive teams are sensitive to each others' viewpoints 2. Diversity in the orientation of individual members achieves optimum performance. a) Creative members helpful for product design, optimism, or humor---> b) Task-oriented members c) Organized people

Team Dynamics Attribution

Team Dynamics Attribution a. Casual references, preference for emotional judgments, and members don't seek explainations b. Cause= unrealistic deadline, untrained team members, search for short-term gains. c. Solution= Devils' advocate approach, provide analysis for conclusions, Team leaders challenge assumptions.

Team Dynamics: Dominant Members

Team Dynamics & Dominant Members a. Symptoms= Chaotic discussions with a few dominant voicing opinions b. Cause= Dominant member interrupt others c. Countermeasures i. Round robin for opinions to support equal participation. ii. Effective moderation by team leader

Team Dynamics: Feuding

Team Dynamics & Feuding a. Poor conflict resolution and lack of mutual respect between team members + lack of reinforcement of group rules of team. b. Solution : i. Restate the main focus of the team objective ii. Stress mutual respect & discipline iii. Confront adversaries in private (not public)

Team Dynamics: Overbearing Members

Team Dynamics & Overbearing Members a. Symptoms= Few individuals interacting while most of the team is in listening mode b. Causes= Team with few influential members (investors, senior management, founders, inventors, shareholders, subject mater experts) intimidate the other members c. Counter- measures i. Influential team leader reinforcement of opinions with round-robin voicing of opinions ii. Nominal Group Technique iii. Informal meeting setup by keeping experts as extended team members

Team Resources

Team Resources 1) Team charger establishes the team expectations aobut resources. 2) Management must optimize resources available to teams

Team Size

Team Size 1) Teams over 20 tend to loose active participation 2) Teams less than 4 don't generate enough ideas 3) Change Management idea---> Get better buy-in if the team members are part of the decision-making process 4) Key challenge is how to keep team size reasonable but till have participation from customers 5) Best to get input from larger group as the solution set is developed 6) Communication plan is used to gain commitment from entire operations department.

Team Stages & Dynamics

Team Stages & Dynamics 1) Projects can fail due to lack of teamwork or lack of understanding of roles & responsibilities. 2) Cross-functional teams are at risk of failing to complete projects successfully because of differing approaches and territorial behavior. Managers concerned about productivity must understand and address those differences in leadership and cultures in approaches to processes.

Team Stages---Adjourning

Team Stages---Adjourning Six sigma team disbands at the end of the project

A six sigma team has considered "current cost of poor quality" in a Six Sigma project charter. Which of the following common components of the project charter is most relevant? A) Problem statement B) Scope C) Objectives D) The Business Case

The Business Case component focus is on the financial impact and importance of the project. The current cost of poor quality explains the same. The problem statement component focus is on what's wrong with process. Current cost of poor quality is a measurement and doesn't indicate what's wrong with process.

The X-charts & S-charts is advantageous in the following situations:

The X-charts & S-charts is advantageous in the following situations: 13. The sample size is relatively large (say, n > 10— and R charts are typically used for smaller sample sizes) 14. The sample size is variable 15. Computers can be used to ease the burden of calculation

A supervisor has asked his Green Belt to compare the average daily scrap value from the first shift to the average from the second shift to determine if there is a difference of more than $1.00 with 95% confidence. The Average daily scrap for both shifts has been $25.76 for the past year with a day to day standard deviation of $7.07 for the past year with a normal distribution. The supervisor believes the two shifts are contributing about the same amount of scrap but wants the answer next week so he can appraise his team leaders. What do you tell him? It will take 14 Days to get the data the supervisor wants You can give him the results in 7 days with 56% confidence You can produce the results in 8 days Based on the confidence interval wanted and the significant difference of $1.00, a sample of 192 days pf production would be required which is not practical.

The one sided Z score for α = 0.05 = 1.96 Sample = n = (Z2 σ2)/((1.0)2) = (1.96)2 * (7.07)2 / (1.0)2 = 192

ABC Company is receiving many customer complaints due to external defects reported in delivered software. External defect density is 2.2 defects/project total size. Most of the external defects reported are from ERP enhancement projects. Which of the following best describes the problem statement? Reduce customer complaints in general. Build a testing team in EPR enhancement projects to work on testing before delivery to reduce external defect density of 2.2 defects/project total size. Clients are dissatisfied with software quality. Our metrics baseline shows mean external defect density of 2.2 defects/project total size with 80% of defects from ERP enhancement projects. Clients are dissatisfied with software quality. Our metrics baseline shows mean external defect density of 2.2 defects/project total size with 80% of defects from ERP enhancement projects. Code Review needs to be automated.

The problem statement is a summation of what requires improvement and it focuses on the symptoms.

The Percent of Study Variation (%SV) for Total Gage R&R leads to which of the following conclusions for this measurement system? Unacceptable Acceptable for most applications No conclusion should be drawn from % of Study Variation Acceptable in some cases depending on the application

The results of a gage study are shown in the table below. Unacceptable data!! % of Study variation is 31.93%. If it is greater than 30%, the measurement system is unacceptable and should be improved Acceptable for most applications applies when the % of study variation is less than 10%, which is acceptable. Graph given: Source StdDev (SD) The results of a gage study are shown in the table below. Source StdDev (SD) -->Study Var---->(6 × SD) ---->%Study Var ----> (%SV) ---->%Tolerance (SV/Toler) Total Gage R&R --> 0.35559 -->2.13354 31.93--> 10.67 Repeatability: 0.27648-->1.65888--> 24.83-->8.29 Reproducibility: 0.22362--> 1.34169 20.08-->6.71 Operator =0.22362--> 1.34169--> 20.08--> 6.71 Part-To-Part = 1.05525 --->6.33151 94.76 --> 31.66 Total Variation 1.11355 6.68132 100.00 33.41

Theory of Constraints terms--5 Step Method

Theory of Constraints terms--5 Step Method 1) Identify system constraints that limit limit throughput or progress towards goals 2) Exploit the System's Constraints by taking advantage of the existing capacity at the constraint-- ( often wasted by making & selling the wrong products, and by improper policies and procedures for scheduling/ controlling the constraint) a) Manage non-constraints so that resources are provided to feed the constraints. 3)Subordinate everything else to the above decisions--->Constraint may have a limit, so look for ways to reduce the effects of the constraint, or look to expand the capacities of the constraint----management directs its efforts toward improving the performance of the constraining task or activity and any other task or activity that directly affects the constraining task or activity 4) Elevate the system's constraints-->Eliminate the problems of the constraint by either adding capacity so that there will be an increase in the overall output of the constraints activity or otherwise change the status of the original resources so that the dominating a) This differs from Step 2 in that added output comes from additional purchased capacity (eg. buying second machine/tool or implementing new technology) 5) Return to Step 1 and look for new constraints. ---> As a result of the focusing process, the improvement of the original constraining task or activity may cause a different task to become a constraining task or activity. Inertia could blind management to additional steps necessary to improve the system's output now limited by a new constraint.

Three Types of Checksheets

Three Types of Checksheets 1) Location or concentration diagram:----> When you rent a car, you probably receive a document with the sketch of the car which allows you to circle any damages, dents or scratches on the car with a corresponding mark on the diagram. 2) Graphical or Distribution check sheet: --------->Using the graphical form, the person collecting the data is able to visualize the distribution of the data. For example, the number of people in line at the registration desk at 15 minute intervals could be counted to determine the staffing needs and the size of the waiting room. 3) Tabular check sheet or tally sheet-------> : The tally sheet is commonly used to collect data on quality problems and to determine the frequency of events. For example, the tally sheet is useful for understanding the reasons patients are arriving late for appointments, causes for delays in getting the lab results back, etc. It is also useful in determining frequency of occurrence, such as number of people in the line for blood tests at 6:00 am, 6:15 am, etc., to understand staffing needs.

Tools used for tracking & evaluating projects' s phases:

Tools used for tracking & evaluating projects' s phases: a. Project Charter b. Project documentation c. Storyboards i. Best tool for conveying changes within the project ii. Before & After pictures (aka current and future states) for proposed projects involving facility/product re-design. iii. Storyboards for Six-Sigma projects are formatted as (DMAIC) along with charts/documents illustrating the activity of each area.

Tools used to Translate Customer Requirements

Tools used to Translate Customer Requirements 1) Critical-to-Quality (CTQs) characteristics---- are those product features that are deemed important from an examination of customer needs. a. Must translate customer feedback into project goals and objectives, including critical to quality (CTQ) attributes and requirements statements ******Key Takeaway: CTQs are important characteristics that are developed from an examination of customer needs. Unlike other measures of customer wants, customer needs that are directly related to product features are an important component of this Six Sigma methodology. 2) Use voice of customer tools such as quality function Deployments (QFD) to translate customer requirements into performance measures. a. QFD= tool used to determine which processes, products, and/or services will be impacted based on customer feedback. b. VOC will help determines which requirements that customers deems critical 3) Kano Analysis

Tools used to find potential causes of failures:

Tools used to find potential causes of failures: a. Tools (brainstorming, NGT, multivoting, & fishbone) used to list all possible causes for every failure mode b. CEDAC (cause-and-effect diagram with addition of cards)= alternative version of Fishbone diagram where people are encouraged to write their ideas or causes on the wall. Success depends on organizational culture and communication.

What is the % of Tolerance variation (SV/Toler) associated with a difference between measurements when one person repeatedly measures the same item? See graph in the answers 6.71 10.67 8.29 31.66

Total Gage R&R: SD-0.35559 ; [(6 X SD) =2.13354] [SV= 31.93] [SV/ %Tolerance= 10.67] Repeatability -- SD=0.27648; 6SD=1.65888 ;SV =24.83 SV/ %Tolerance=8.29 Reproducibility SD=0.22362; 6SD=1.34169; SV=20.08 SV/ % Tolerance= 6.71 Part-To-Part SD=1.05525; SD/ 6= 6.33151; SV= 94.76 SV / % Tolerance= 31.66 Total Variation SD= 1.11355;SD/6= 6.68132; SV=100.00 SV/ % Tolerance=33.41 Repeatability measures the variability of the instrument when one person repeatedly measures the same item, and is the correct source of variation to use for the SV/Toler column 10.67 is associated with Total Gage R&R, not Repeatability. 6.71 is associated with Reproducibility variation, not Repeatability. 6.71 is associated with Reproducibility variation, not Repeatability.

Traditional Business process

Traditional Business process 1) Vertical Silos---built around functional boundaries 2) Production starts with the process inputs and flow across many vertical functions and horizontal business levels to produce process outputs (products, goods, and services) 3) Differences in function, organizational structure, vocabulary & location are sources of confusiton and defects. 4) Business process can fail if the functional relationships not clearly understood.

Decision Tree Diagram

Tree Diagram 1) Breaks a general topic into number of activities that contribute to it via a series of steps. 2) Suggestion of specific topics that contribute to the general topic. 3) A decision tree is a visual decision making tool that can prove useful across a number of business settings. The main principle behind a decision tree is the presentation and comparison of a number of competing alternatives. It also provides potential outcomes and their probabilities.(Does not provide risk analysis)

Two types of statistical studies:

Two types of statistical studies: a. Descriptive studies i. Purpose is to present data in a way to facilitate understanding. b. Inferential studies. i. Analyze data from a sample to infer properties of the population from which the sample was drawn

Two- Tail Test

Two- Tail Test Hypothesis test that determines whether a population shift has occurred in either direction---> allowable alpha error is then divided into two equal parts.

Two-Way ANOVA

Two-Way ANOVA Two Way ANOVA is used to analyze the effect of two random factors on a Critical-To-Characteristic (see concept Factors and Levels). A factor is said to be random when levels are randomly chosen from a population of possible levels and we wish to draw conclusions about the entire population of levels, not just those used in the study. For example, Two Way ANOVA type of analysis is generally used in Gage R&R studies.

Types of Variation in Statistical Study:

Types of Variation in Statistical Study: a. Calibration -Drift in average measurements of a absolute value. b. Stability—drift in absolute value over time. c. Repeatability - ( "within appraiser" variation) ----variation in measuring equipment when measured by one appraiser in the same setting at the same time. Equipment measurement variation expressed as standard deviation

Typical DFSS Roadmap Steps

Typical DFSS Design Process Steps followed by Design team: 1. Need---> Analysis of problem--> Statement of problem--> Conceptual Design (combines engineering, science, practical knowledge, production methods, practices to generate solutions)---> Embodiment of schemes----> Detailing Step (consolidates the fine points of producing product) ----> Working Drawing

"Typical application of run Charts

Typical application of individual-X charts: 1) Normal distribution---> run tests are applied as they were written in the Western Electric Statistical Quality Control Handbook. 2) Non-normal distribution ---> X- chart has been used & the median substituted for the average. 3) Must establish zones that produce the same probabilities as the normal curve for the given standard deviation. 4) When applying run tests to the upper and lower halves of a control chart, run tests 1, 2, 5, and 6 should be applied separately. 5) Run tests (3, 4, 7, and 8) are applied to the entire chart. The option to use run tests is typical of statistical process control programs."

Usage of velocity data

Usage of velocity data 1) tells degree to which a process responds to customer demands. 2) If less work in progress---> shorter lead times & greater velocity. (Longer lead times---> slower velocity). 3) Slow velocity --> quick business response to new customers order. 4) The formula for process velocity (number of value-added steps divided by process lead time) suggests approaches for increasing velocity: a) Reduce amount of work in progress b) Increase number of completed processes per hour c) Analysis of process efficiency will indicate which of these strategies is appropriate.

Suppose I have a process in Statistical Control. When I compare the natural process limits to the specification limits I find that there are 10% of the parts above the USL, but none below the LSL. What is the first step I should take to improve the % defective? Retarget the process. Nothing. We're screening the defectives out and reworking them later Talk to engineering about changing the specifications. Reduce the process variability.

Usually changing the process center is an easy adjustment. Try this first.

Lean Concepts & Tools-- Value Stream

Value Stream a. Activities that organization performs (design, produce, deliver products or services) b. Three Main Components: 1. Flow of materials from receipt of supplier material to deliver of finished goods/ services 2. Transformation of raw materials-→ finished goods 3. Flow of information required to support the flow of materials and transformation of goods/services (eg. Shipping notices, work orders, purchase orders) c. Value Stream Map= illustrates the movt of materials, information, inventory, work-in-progress, operators, ect. d. Value Stream Analysis-→ uncovering of hidden waste through the use of lean tools.

Value Stream & Muda

Value Stream & Muda: 1. Womack described application of value stream beyond the boundaries of a typical plan, involving suppliers, organizations, and customers. 2. Must eliminate Muda from value stream 3. Risk of price concession requests cause firms in the value stream to be reluctant to disclose information--> Solution= voluntary alliance of value stream member to eliminate Muda hidden the value stream.

Win-Win Agreements

Win-Win Agreements If an unsatisfactory agreement is reached, then other side will not feel compelled to keep the agreement. Better to use legal force to win a judgement but will not reach a decision in the original required time. Steps for making Win-Win Agreements " 1) Start by confirming each other's goals 2) Verify areas of agreement 3) Consider alternative solutions in areas of disagreement 4) Resolve differences.

Methods in how work instructions can be documented as:

Work instructions can be documented as: 1. Written instructions 2. Checklists 3. Flowcharts 4. Photographs 5. Drawn pictures 6. Videos 7. Electronic screen shots 8. Electronic software-driven process steps

Written Procedures

Written Procedures 1) ISO 9001 ---States that internal procedures shall control nonconforming product so that it is prevented from inadvertent use of installation. ------> Quality Department Responsibility but functions carried out by multiple departments 2) Procedure should be developed by those having responsibility for the process on interest.

Assume two events A={1,2} and B={3,4,5} in rolling a fair dice with six possible outcomes S={1,2,3,4,5,6}. Select the best option. Events A and B are mutually exclusive and dependent Events A and B are independent. Events A and B are exclusive but not dependent. Events A and B are dependent but not exclusive.

p(A ∩ B) = 0 So they are mutually exclusive p(A ∩ B) ≠ P(A).P(B) therefore they are dependent.

46. During which stage of DMAIC is the 5S method used most often? a. Define b. Measure c. Analyze d. Improve

"46. D: Improve. The 5S method is used most often during the improve stage of DMAIC. 5S is a Japanese lean tool for reducing cycle time. In Japanese, the five aspects of 5S are organization, purity, cleanliness, discipline, and tidiness. In English, these words are often translated as sort, straighten, shine, standardize, and sustain. The most common targets of 5S programs are processes that tend to lose materials or require unnecessary movement.

A Six Sigma Manager is handling product redesign project. Which feedback forum is best suited to address obstacles encountered by the team? Status report Customer audit Management Review Storyboards

A management review provides management with the status of the project, the opportunity to review the project charter and project team mission, and a chance to discuss management responsibilities likely to have an impact on the progress of the team, etc. Status reports tell what the project is based on, and where it is supposed to be relative to the plan.

Average and Range Method and ANOVA R & R Method

Average and Range Method and ANOVA R & R Method

Describe the importance of identifying key decision points during Measure stage

Describe the importance of identifying key decision points during Measure stage 1) After creating a process map, must identify the important decision made during the process. 2) Team members are alert to areas of inefficiency, requiring excess decision-making. 3) Team should try to reduce the number of decision made every time a process is performed. 4) Since process has not been measured, then all measurements must reflect the process as typically performed----> to ensure proper targeting of improvement efforts. 5) Team leaders communicate info to stakeholders.

DOE--

DOE---best used for optimizing a process versus as problem-solving tools. a. SDCA ---standardize, do, check, act— i. Most commonly used once a process has improved to update control plans and process sheets to lock in the improvements & standardize the changes throughout the organization. ii. Outcome of applying sound recommendations based on the correct interpretations of valid effects obtained from proper design of experiments.

Deming & Business Process Management

Deming & Business Process Management Deming defines the supplier-process-customer Model: 1)Process outputs, controls, & outputs are inter-dependent 2) Statistical methods can improve process control and guide improvements 3) Process feedback can be used to re-design products and processes and improve overall business results ***BPM represents a major advance in the quality improvement thinking by managing the entire process---as opposed to traditional management process that operate as silos.

Deming's chain Reaction

Deming's chain Reaction Improve Quality --> Decrease costs (less rework, fewer delays)---> Productivity improves---> Capture the market with better quality and price---> Stay in business--> provide jobs

Deming's philosophy on people and economic benefits

Deming's philosophy on people and economic benefits: 1. Deming's philosphy focused on individuals as members of the system, which benefited the individuals of the firm 2. Economic benefits to the company : a. Reduction of economic burden b. Expansion of Markets c. Survival of organizations that serve customers

Setting up Control Charts:

Setting up Control Charts: i. Characteristic to be tested is based on what is defective or what can be controlled by worker ii. Attribute data (percent defective) and Variable Data (numerical measurements) are both selected to diagnose the cause) 1. Chart for attributes requires discrete measurements (pass/fail, counts)-→ data will usable only if the data has high defective rate and comes from a reasonable subgroup size. 2. Variable charts are on continuous scale (length, weight)

Shewart & Statistical Control Charts

Shewart & Statistical Control Charts 1. Used to monitor processes and to determine when process changes occur. 2. Process changes are only made when points on the control chart are outside the specification limits.

5 States of Team Development Forming

Forming i. "Honeymoon" period of team members getting to know each other ii. Group is immature, trying to please each other, and too much agreement on initial topics. iii. Not much work accomplished

Frequency Distribution

Frequency Distribution a. Used to illustrate the location & spread of numerical data. b. Peaks represent concentration of data i. Unimodal (one peak) ii. Bimodal (two peaks) iii. Multimodal (mixture of populations) iv. No peak/ flat curve (rectangular distribution) c. Width of curve represents spread of data i. Thinner distribution= less variation

Metrics Selection & 9 dimensions of Quality Measurement

Metrics Selection & 9 dimensions of Quality Measurement 1) Performance ---> Product's primary features 2) Features ----> Secondary features added to the product 3) Conformance ---> product that meets fit, form, & function 4) Reliability------> Dynamic quality of product over time 5) Durability ---> useful life 6) Service -------> Ease of repair 7) Response------> Human Interface 8) Aesthetics ----> Product appearance 9) Reputation----> Based on past performance

Metrics Selection--- Measurements important in the Marketplace

Metrics Selection---Measurement important in the Marketplace 1) Price / Demand Increases 2) Product Range/Brand name/ Design/ Color Range 3) Conformance Quality 4) Reliability of delivery / Delivery Speed 5) Technical Support/ After-sales Support

7 Metrics for measuring Profit

Metrics for measuring Profit 1. Stockholder value 2. Capital Investments 3. community comparision 4. ROI 5. Sales Dollars 6. Profit margin on sales 7.. Personnel Costs

Metrics of Six Sigma?

Metrics of Six Sigma—Six sigma quality performance means 3.4 defects per million opportunities (accounting for 1.5 sigma shift in the mean)

Poisson Distribution

Poisson Distribution Basic assumptions: Discrete distribution Length of the observation period (or area) is fixed in advance Events occurs at a constant average rate Occurrences are independent Rare event Uses include: Number of events in an interval of time (or area) when the events are occurring at a constant rate Number of items in a batch of random size Design reliability tests where the failure rate is considered to be constant as a function of usage

QFD Structure

QFD Structure 1) Left side= customer needs 2) Ceiling = design features and technical requirements 3) Right side=Customer priorities (comparisons) 4) Foundation = target values, benchmarking 5) Roof= matrix describing the relationship between design features.

Quality Cost Bases

Quality Cost Bases **Two or Three Cost base comparisons is normal. Quality costs are summarize as a report on monthly basis. 1) Labor Bases ------> Standard labor (planned) , Total direct labor worked 2) Manufacturing Cost Bases ----> Shop Cost Output (direct labor, materials, indirect costs) + Manufacturing Cost of Output (packing/shipping, provision for complaints, total shop cost output. production engineering expenses) 3) Sales bases--------->Net sales billed, Contributed Value (aka. Net Sales minus Direct Materil) 4) Unit bases------> quality costs, dollars per unit of production.

A major airline is exploring the potential for expanding its operations into a Midwest city. For this purpose, it commissioned an internal team, and a team of external consultants to collect feedback from customers on their interest in the new route. Which of the following tools can ensure that both teams will collect consistent data? Control charts Stratification Design of Experiments Check sheets

Stratification is a technique that separates data gathered from a variety of sources so that patterns can be seen. Check sheets are a structured, prepared form for collecting and analyzing data.

Creation of moving-range charts

"Creation of moving-range charts 1) Plotted statistic is the moving range, or the difference between the immediate observation and the immediately preceding observation of the sub-groups in an individual-X chart. 2) The moving range is calculated (see formula) . The centerline is calculated (see formula) , where m is the total number of subgroups and MRj is the moving range for subgroup j. 3) The upper control limit is calculated and the lower control limit is calculated as (see formula) , in which is the average of the moving ranges, d3 is a function of n, and σx is the process sigma. 4) If a process has a normal distribution, then the moving-range chart will not display any special causes not also displayed on the individual-X chart. "

"Reducing adjustment time

"Reducing adjustment time 1) Good process controls best way to reduce adj. time. a) Less adjustment if a process can be repeated in exactly the same way but may require process to be redesigned. 2) Statistical process control charts and designed experiments identify those areas of the process that necessitate adjustment. 3) Make sure the targeted process components are on the critical path to avoid making pointless modifications

Schedule, stakeholders, and team composition of Project Charter

"Schedule, stakeholders, and team composition 1) Project charter----> includes schedule made at the beginning of project with specific dates on which established objectives and goals are to be met. 2) Should include a complete identification of the stakeholders in a project ( persons who influence a project or are affected by a project) 3) list of the team members (may overlap with stakeholder list)

12. Which method of creating a prioritization matrix is appropriate when time is limited? a. Partial analytical method b. Consensus-criteria method c. Full analytical method d. Summary method

12. B: Consensus-criteria method. When time is limited, the consensus-criteria method should be used to create a prioritization matrix. In this method, a group of people is each allotted a hundred points, which they then allocate across a series of criteria according to perceived importance. Prioritization matrices are used to identify those projects that will create the most value improvement over the long term. Also, organizations use participation matrices to identify the projects that will contribute the most to the achievement of the organizational goals. Besides the consensus-criteria method, the other method for creating a prioritization matrix is called the full analytical method. In this method, all of the various options are listed, and the members of the team assign a numerical value to each. "

"29. If there are 32 observations in an experiment, it is typical to run autocorrelations from lag 1 to: a. Lag 4 b. Lag 8 c. Lag 16 d. Lag 32

29. B: Lag 8. If there are 32 observations in an experiment, it is typical to run autocorrelations from lag 1 to lag 8. The basic calculation for the number of autocorrelations in an experiment is lag 1 to lag x/4, in which x is the number of observations. Since there are 32 observations in this experiment, autocorrelations should run from lag 1 to lag 8. The lag is the difference between correlated observations. In lag 1, for instance, observation[...]"

3 Elements of Standard Work Operation

3 Elements of Standard Work Operation 1) Cycle Time ---> time allow to make a product; based on takt time---Improvements are evaluated by comparing actual time with takt time 2) Work Sequence----> order of operations that the worker must use to produce a part---> Standard time provided for each element 3) Standard inventory---> minimum allowable in-process material in work area, including the amount of material on the machine that is needed to maintain smooth flow.

"43. Which goodness-of-fit test focuses on the relationship between number of data points and distributional fit? a. Nonparametric test b. Kolmogorov-Smirnov test c. Chi-square test d. Anderson-Darling test

43. B: Kolmogorov-Smirnov test. The Kolmogorov-Smirnov test focuses on the relationship between number of data points and distributional fit. A nonparametric test is used instead of a hypothesis test for comparing the means from samples with different conditions and for assessing the effects of changes on process averages. A chi-square test is the simplest form of goodness-of-fit test. An Anderson-Darling test is excellent for obtaining information from the extreme ends of a distribution."

5 reasons that require standardized work conditions

5 reasons that require standardized work conditions: 1) Takt time 2) Ergonomics 3) Parts Flow 4) Routines 5) Maintenance procedures ***Standardized work is the documentation of each action required to complete a task . ***Standardize work in the form of work instructions should be available to the operator

5S--Sustain

5S--Sustain 1) Commit to the 4 previous stesp and continually improve on them 2) Develop Self-discipline 3) Peform evaluations of each of the 4 steps **** Management commitment will determine the control and self- discipline areas for an organizaiton. ****5S program can be set up and operations within 5-6 months but the maintenance effort is continuous.

8 Advantages of Computer/ Automated Project Management Methods

8 Advantages of Computer/ Automated Project Managment Methods: 1) Can model "what ifs" scenarios 2) Can show impact of alternate options 3) Diversity of formats/ detail of information presentation 4) Automatic scheduling and data collection 5) Real-time availability of variances in plan 6) Project status reports easily generated 7) Multi-location data input and information sharing 8) Easier summarization of project

When trying to decide what's in or out of cope for a project, the most likely tool the Green Belt would use is: Raci Chart SIPOC Fishbone Diagram Box Plot

ANSWER= A SIPOC chart (suppliers - inputs - process - outputs - customers) allows the GB to create a high level view of the process of interest and identify boundaries activities, stakeholders and other elements. A Raci Chart, while typically used in the define phase, is used to analyze stakeholders and their requirements for communication (not project scope) A box plot is more likely to be used in later phases, such as Measure and Analyze, to analyze data.

Balanced Scorecard

Balanced Scorecard a. Measurement system + Management System (translates vision & strategy into action) b. Provides feedback on both internal business processes and external outcomes for continuous improvement. c. Selection of project metrics to ensure customer & business needs are met. d. Metrics (4 perspectives)= financial, customer, internal processes, and employee learning/growth. e. Uses lagging & leading measures across the 4 perspectives i. Lagging measures= measured at end of the event. ii. Leading measures= measures that help reach objectives (measured upstream of the event)

Bottleneck and location of Quality Control Check

Bottleneck and location of Quality Control Check 1. After bottleneck is identified , make sure that the bottleneck is always filled with materials from previous steps to make sure that the it is always running. I 2. It is critical to arrange things so that when a part gets to a bottleneck, it is likely to make it the rest of the way. 3. Avoid placing quality control check after the bottleneck that causes parts out of the bottleneck to be discarded. a) Do the QC before the bottleneck so that things going into it are as good as possible. b) Avoid using bottleneck to make things that are not immediately needed.

Box-and-whisker plot Uses?

Box-and-whisker plot Used to: i. To visually compare two or more populations and see the shift in median and variation. ii. To mine information from a database.

Brainstorming

Brainstorming a. Method of generating large number of creative ideas in short time period. b. Variation in Brainstorming= NGT, Idea mapping, Mind mapping. Best used when broad range of options and ideas desired. f. Advantage= encourage team participation.

Classification of experimental designs: Blocked Factorials

Classification of experimental designs: Blocked Factorials ---reduces the number of runs and uses blocking to run the experiment in subsets.

Control Charts Information Use

Control Charts Information Use 1) Used to determine the natural range of process and to compare it to the specificaiton limits 2) If the Control limits are wider than specification limits----> Adjust the specification limits or need to improve the process to narrow its natural control limits. 3) Ideally, the control limits should be within the specification limits for the process to be "in control" 4) If the control limits are outside the specification and your process is in control and producing consistent results, you may be producing defect-free products (within the spec limits). On the other hand, you may be in control (within the control limits) but producing defects nonetheless if you are falling outside the spec limits

Control charts

Control charts a. Graph to study how process changes over time b. Current data vs. historical control limits compared to conclude about whether process variation is consistent ( in control) or is unpredictable ( special cause variation) c. Classified by the type of data: i. Variable Data (continuous scale)---eg. Weight, distance, temperature---has the possibility of measuring with greater precision; measured in decimals and fractions ii. Attribute data (can have decimals or fractions)---used to determine if something is present or absent d. Variable Charts—Moving average, Cumulative Sum cart, Expontentailly moving average chart, R-chart (range, X-bar chart

Which hypothesis test can be used to determine if the variance calculated from machine #1 data on Shift A is the same as on machine #1 during shift B? Chi-Square Test z-Test F-Test t-Test

Correct! An F-test compares the variance from one population (Shift A) to the variance from another population (Shift B).

In hypothesis testing, the Null hypothesis is: Accepted when the test statistic falls within the rejection region An assumption that is claimed to be true. Influencing the level of significance for the test. Used to calculate the test statistic

Correct! Correct definition of null hypothesis--assumption that is claimed to be true

When any statistic is used to estimate the parameter, what property of the statistic is called the standard error: Variance Parameter Mean Standard Deviation

Correct! Standard Deviation of the statistic is called the standard error.

A chemical process is run at 3 different temperatures. Five trials at each temperature. The average product yields are 89, 91 and 93%. Assuming you have the complete data set, which of the following would be the best analysis to determine whether there is a statistically significant difference between product yields? Process capability Analysis of variance Run charts Linear regression models

Correct! ANOVA used to determine whether the differences among population means are statistically different. Run charts may show difference, but not quantitatively. Process capability is used to measure process repeatability e.g. to meet specifications. Linear regression models show relationship between 2 variables.

The difference between the observed result and the predicted value for that result based on the empirically determined assumed model is called which of the following? Residual error Precision error Remainder error Relative error

Correct! Residual error or experimental error is the variation in the experimental response under identical test conditions Precision error is a measure of how well repeated measurements under unchanged conditions show the same results, not differences compared to an assumed model. Relative error gives an indication of how good a measurement is relative to the size of the item being measured.

Identify two of the benefits of single-minute exchange of dies (SMED). Decreased waiting time and increased quality. Decreased changeover time and increased safety. Decreased costs for purchasing dies and increased production. Decreased inventory and increased capacity.

Correct! SMED decreases the time to change dies, increases capacity and throughput, and improves on-time deliveries to customers. SMED increases production, but it does not reduce the costs to purchase dies Waiting time may be decreased due to SMED; however, it is not implemented for quality. Although changeover times are decreased, safety is not the driving force in implementing SMED.

When manufacturing a product, which of the following options is an element of continuous flow? Takt time Kaizen Mass production Muda

Correct! Takt time begins the movement in continuous flow and sets the pace Muda is waste; this may be reduced as an effect of continuous flow, but it is not a part of continuous flow. Kaizen is a method of gradual improvements.

If all of the sample sizes are equal, which one of the following charts is the most appropriate to substitute for a u chart? c chart p chart I chart np chart

Correct! The c chart is appropriate when the statistic of interest is the number of occurrences of an attribute for a given area of opportunity, where the area of opportunity remains constant for all subgroups. he np chart is appropriate only when the statistic in question is the number of nonconforming items out of the total number of items inspected and the subgroup size is constant. An I chart is designed for individual measurements from a process in time (or space) order. An I chart is designed for individual measurements from a process in time (or space) order.

Creation of Gantt Charts

Creation of Gantt Charts: 1) 1st = make a list of all the tasks that have to be completed in the project. 2) Decide whether some tasks may be performed simultaneously or whether some tasks require the completion of previous steps before they may be initiated. ( jobs that can be done at the same time are placed on parallel paths, and other jobs are placed in sequence.) 3) Within each parallel path, there may be serial tasks. However, if one of the parallel paths can be completed more quickly than another, then some slack time can be built in to that path.

Crosby's 4 absolutes of Quality Management

Crosby's 4 absolutes of Quality Management: 1. Quality means conformance to requirements---customer based requirements and do-it-right-the first time 2. Quality comes from prevention ---- opportununities are present to correct problems 3. Quality performance standards is zero defects---- Without zero defect, you are making it okay to send out nonconforming parts. If non-conformance is present, then you must prevent and eliminate it. 4. Quality measurement is the price of non-conformance---- Measurement of quality is needed to get management's attention, prioritize problems, correct problems and measure progress

Cross-functional Teams

Cross-functional Teams Structure= 8-12 members from different areas, departments, or disciplines 1) Thoughtful selection of members crucial to building an effective team made up of subject matter experts----> who have knowledgeable about policies, practices, and operations of department or functional area. 2) Team involvement promotes sharing of problems and reduces "finger pointing" 3) Representation from various departments promotes acceptance and implementation of change throughout the organization. 4) Solutions designed with active participation of affected departments tend to superior and accepted more readily by those who must implement them.

Current State Mapping

Current State Mapping 1) Current state map of process is developed to facilitate process analysis. 2) Provides information on lead and value-added time 3) Typical process data: a) Time ----> work time, uptime, cycle time, changeover time b) Scrap rate, WIP, pack size.

Customer Data Analysis

Customer Data Analysis 1) Green Belt analyzes customer data to compare customer attitudes over time or between groups---> insight into market niches & changes 2) Data on customer feedback examined using : a. Statistical tests b. Line graphs c. Control Charts d. Matrix Diagrams e. Pareto Diagrams

Customer Expectations

Customer Expectations Along with listening to customers, must uncover the subjective reasons that customer made a purchase---> Maslow's Heirarchy 1) Basic Needs---> Bare essential product/ service attributes (eg. basic software in laptop) 2) Expected Needs--->Attribute that are provided as part of the package ( Eg. technician giving operating instructions, tutorial on product startup, or rental car policies explained) 3) Desires--attributes that are attractive but not a necessity ---> (Eg. Car rental employee telling you how to save money or giving you good directions) 4) Unanticipated needs---> surprise attributes that go beyond the customer expectations. (eg. Technician calls and tells that he'll personally home-deliver the product or rental agency drops the car by your place) ***As an individual's needs (basic needs) are fullfilled, then they advance up the heirarchy and looks for newer attributes.

External measure of operational Effectievness? Internal measure of operations efficiency?

Customer Satisfaction ---external measure of operational effectivenss) Sigma levels, WIP inventory, or Rolled Throughput Yeild--Internal measure of operations efficiency

DFSS Roadmap & GE's Product Design Development Process

DFSS Roadmap & GE's Product Design Development Process 1) Understanding CTQ characteristics for external & internal customers 2) FMEA 3) Use Design of Experiments to find key variables 4) Benchmarking facilities using survey and competitive analysis

Which of the following statistical expressions represents a method to summarize information about a data set? Descriptive statistics Regression analysis Scatter diagram Confidence interval

Descriptive statistics is a method to summarize data in a way that will facilitate understanding. Regression analysis is a statistical technique for estimating the parameters of an equation relating a particular variable to a set of variables. Scatter diagrams are used to chart relationships between two variables. A confidence interval provides a set of limits within which a population parameter lies.

Design FMEA

Design FMEA 1. Focused on reducing the risk failure modes caused by design deficiencies by examining the functional requirements and design alternatives. a) 76% of engineering changes are due to poor design and 24% due to improvement. 2. Conducted before the release of production drawings for tooling and to manufacturing (Manufacturing issues are part of process DFMEA, NOT DFMEA) a) Manufacturing process capabilities--- tolerance, tooling, finishes, & specifications 3. Design flaws are not corrected via process controls 4. Design team should be multidisciplinary--including design, test, materials, service, reliability, and manufacturing expertise.

Design for Scalability

Design for Scalability --- Deployment of product/systems within growth market with anticipation of expansion/ rapid adoption. (eg. Auction Website that goes blank during peak usage)

Determine Critical Customer Requirements

Determine Critical Customer Requirements 1) To prosper, business must do well in at lease of of the 4 critical customer requirements---Cost, Quality, Features, or Availability (CFQA) 2) The level of importance for each of the 4 requirements is constantly changing due to competition & environment ---> Must continuously determine the critical customer requirements. 3) Cause-and effect Matrix and Quality functional deployment ----Tools used to understand the entire system of customer expectations/ needs/ priorities along with the critical relationships and interactions of all these factors

Drawback of Projects Under Consideration

Drawback of Projects Under Consideration 1) May conflict with customer needs, industry standards, product specification, personnel safely or laws/ regulatory requirements

External Customer Identification

External Customer Identification: 1) Can separate business customers as --Nonprofit vs. Profit 2) Can review customer groups by ---market competition, market growth, market risk, or high profit margin

External Customers

External Customers 1) CFQA= cost, quality, features, and availability factors--used to weigh the value of goods/services 3) 3 Types of External Customers a. End Users b. Intermediate customers c. Those impacted but do not use/purchase the product

FMEA Severity Vs Occurrence ?

FMEA----Severity Vs Occurrence Severity rating -----Rating of the severity of the effect of failure can cause (scale 1-10; high number, more severe impact) Occurrence rating ---Likelikhood that cause can occur (scale 1-10; high number= high occurrence)

When calculating a correlation coefficient, which of the following steps is performed first? Calculate the mean for all x-values and the mean for all y-values Sum the values and divide the sum by Sx x Sy Calculate x- x-bar, and y- y-bar for each pair (x.y) and then multiply those values together Calculate the standard deviation for all x-values and the standard deviation for all y-values

For the first step, one would calculate the mean for all x-values and the mean for all y-values Calculating the standard deviation for all x-values and the standard deviation for all y-values is the correct 2nd step. Calculating the standard deviation for all x-values and the standard deviation for all y-values is the correct 2nd step Sum the values and divide the sum by Sx x Sy is the correct 5th step

Gage Reproducibility & Repeatability Testing

Gage Reproducibility & Repeatability Testing c. Gage repeatability and reproducibility (R&R) is a method for finding out the variations within a measurement system. Basically, there are 3 sources for variation: variation of the part, variation of the measurement device, and variation of operator. Variation caused by operator and interaction between operator and part is called reproducibility and variation caused by measurement device is called repeatability. d. Check for only the precision of a measurement system. e. Variation due to Reproducibility - operator problems (training, skill, knowledge) or the machine problem (faulty design) f. Variation due Repeatability= equipment problems

Geometric Mean

Geometric Mean is a special type of average where we multiply the numbers together and then take a square root (for two numbers), cube root (for three numbers) etc. The geometric mean is NOT the arithmetic mean and it is NOT a simple average. It is the nth root of the product of n numbers. That means you multiply a bunch of numbers together, and then take the nth root, where n is the number of values you just multiplied. Did that make sense? Here's a quick example: What is the geometric mean of 2, 8 and 4? Solution: Multiply those numbers together. Then take the third root (cube root) because there are 3 numbers. cube Root ( 2 *8* 4) = cube root (64) = 4 Geometric Mean

Goal of Rational Subgrouping?

Goal of Rational Subgrouping? The goal should be to minimize the chance of special causes in variation in the subgroup and maximize the chance for special causes between subgroups. Subgrouping over time is the most common approach; subgrouping can be done by other suspected sources of variation (e.g., location, customer, supplier, etc.) v. For example, an equipment leasing business was trying to improve equipment turnaround time. They selected five samples per day from each of three processing centers. Each processing center was formed into a subgroup. vi. When using subgrouping, form subgroups with items produced under similar conditions. To ensure items in a subgroup were produced under similar conditions, select items produced close together in time.

Graphical, statistical, and qualitative tools to analyze customer feedback:

Graphical, statistical, and qualitative tools to analyze customer feedback: a. Affinity Diagrams b. FEMA c. Pareto Diagrams d. Interrelationship diagrams e. Matrix diagrams f. Priority matrices ****Tools used to identify group issues and determines which issues have the greatest impact for the resource expended.

Histograms

Histograms 1) Frequency column graphs that display a static picture of process behaviours with a minimum of 50-100 data points 2) "Frequency" =Data points fall withing a given bar or interval 3) Stable Process---> Predictable process; Uni-modal or Bell-shaped curve. 4) Unstable Process---> Exotic shape---> Exponential, Lognormal, Gamma, Beta, Weibull, Poisson, Binomial, hypergeometric, Geometric

How Are KPIVs Identified?

How Are KPIVs Identified? The variables which are important enough to control are selected Structured experiments are conducted to model process behaviour and understand the cause-and-effect relationships that are present (design of experiments) Processes have to satisfy customer requirements (e.g. price, quality, durability). Therefore, changes to input can have a negative impact on customer satisfaction These experiments find the balance between both input and output, and those that are key are ones which will not affect output

How are Stakeholders impacted with project's stated goals are not met?

How are Stakeholders impacted with project's stated goals are not met? 1) Stockholders ---company's net worth reduced 2) Suppliers---delayed payments and never get full payment 3) Management --Frozen/ diminished wages and reduced staff 4) Customers may impose fines and seek legal action or seek other places to be.

How was the West introduced to term " lean production"?

How was the West introduced to term " lean production"? 1. Womack introduced the term " lean production" to West in his publication of "The Machine that Changed the World" 2. This book descirbes basis of lean manufacturing as practiced in best companies.

What % efficiency can process reach after using Lean strategies?

However, it is more typical for processes to approach 25% efficiency after the implementation of lean strategies. The reasonable value for efficiency depends in large part on the type of activity. a) Creative activities usually have efficiency values of approximately 5%, although lean management strategies have been known to increase this figure up to almost 25%. b) Transactional activities, on the other hand, sometimes can reach efficiency values of up to 50%."

Implementation Planning

Implementation Planning 1) Can use Gantt chart to illustrate the future state plan. 2) Speed of implementation planning is affected to: ======>Available resources/ funding

Kano Model

Kano Model Analsysis 1) Used to Analyze Customer Requirements 2) Based on 3 categories: a) Dissatisfiers or "Must Be's "------> customer expects that basic requirements are part of the basic package (eg. steering wheel in car) b) Satisfiers or "More is Better" ----> when these requirements are met, the more it is met then the better (eg. Tourist on a cruise expect personal attention) c) Delighters----> services/features that go beyond expectations. ***Improvement projects are selected among the satisfier and delighter categoreis.

Lean

Lean is a comprehensive system for decreasing waste and increasing cycle times and quality. Many of the ideas of lean date back to Henry Ford, but the credit for assembling and updating the system goes to Taiichi Ohno, an employee of Toyota in the 1970s.

Measures Stage of DMADV

Measures Stage of DMADV i. Technical & competitive product management analysis ii. Specification of the design most valued by customer & industry iii. Imposition of expectations by regulators, partners, & stakeholders.

Metrics Selection--5 Secondary Metrics

Metrics Selection--Secondary Metrics 1) DPU 2) DPMO 3) Average age of receivables 4) Lines of error free software code 5) Reduction in Scrap

NGT

Nominal Group Technique (NGT) a. Type of brainstorming technique with limited team vocal interaction. b. Best used to encourage equal participation in discussion, esp. when there is dominance by some group members c. Subject identified and problem defined to keep focused on idea. Facilitator explains the rules. 10-15 minutes to write down ideas and ideas are posted in a space for all to read. Round-Robin format of reading the ideas aloud but no judgment can be passed. Idea submissions are kept anonymous if the topic is controversial. d. Consolidation=expansion and clarification o d ideas to eliminate redundancy.

Non-normal data distribution vs. Normal distribution

Non-normal data distribution i. Box-Cox technique & Johnsons Transformations used to change data from non-normal to normal distribution. c. Normally Distributed Data--→ use normal distribution table to estimate process capability + estimate the mean and sigma using the control chart data

Kano Model & Normal Needs

Normal Needs = These are the qualities, attributes, and characteristics that keep a supplier in the market. a. These next higher level expectations are known as the wants or the satisfiers because they are the ones that customers will specify as though from a list. They can either satisfy or dissatisfy the customer depending on their presence or absence. b. The wants include voice of the customer requirements and other spoken expectations (see table below).

Normal Probability Plots

Normal Probability Plots 1) Most of the points are near the center line, average 2) Some points are at the minimum and maximum points 3) When all the special causes of variation are eliminated, the process will produce a product that when sampled will produce a bell-shaped curve.

Objective of the House of Quality

Objective of the House of Quality: 1) House of Quality is a kind of conceptual map that provides the means for inter-functional planning and communications. 2) Primacy benefit of the House of Quality ---> quality-in-house which get people thinking in the right direction as well as thinking together.

Outer Array

Outer Array In Taguchi style fractional factorial experiment, these are the factor that cannot be controlled in a process.

P-test

P-test 1) When testing a claim about a population proportion and we have a fixed number of independent trials with constant probabilities, and each trial has two outcome possiblities (bi-nomial experiment) 2) When np < 5 or n(1-p) <5, the binomial distribution is used to test hypotheses relating to proportion.

Pattern and Trend Analysis

Pattern and Trend Analysis 1) Control charts or Trends can used to display changes in data patterns. 2) Data can be either summary (statis) or time sequence (dynamic) 3) Trend charts show patterns that indicate if a process is running normally or whether desirable or undesirable changes are occurring.

Points Near or Outside Limits

Points Near or Outside Limits A. X-Bar Chart Causes B R- Chart Causes C. Corrective Action

Precision/ Tolerance

Precision/ Tolerance 1) Ration between the estimated measurement error (precision) and the tolerance of the characteristic 2) Best to have a small P/T ratio to be small to reduce the effect of measurement error<-------or the net effect of all sources of measurement variability that cause an observed value to deviate from the master value. 3) 6sigma is the standard deviation due to measurement variability and the assumptions are: a) Measurement errors are independent b) Measurement errors are normally distributed c) Measurement error is independent of the magnitude of measurement.

Precision/ Total Variation

Precision/ Total Variation (P/TV) 1) Like P/T, it evaluates the acceptability of a measurement system 2) At P/T or P/TV increases, the ability to discriminate a change in process reduces. 3) When the measurement process is inadequate to detect part-to-part variation----> must use a smaller measurement variation

Process Analysis & Documentation

Process Analysis & Documentation 1) Set of inter-related resources and activities which transform inputs into outputs with the objective of adding value 2) Tools used- flow charts, process maps, written procedures, and work instructions.

Process Owner

Process Owner a. Sponsor or champion who is responsible and accountable for execution and results of process. Training in 6 Sigma concepts/ tools/ strategies/operations methods (emphasis on management tools)

A newspaper reported that the revenue trends for the past five years were up 5%, down 1%, up 7%, up 3%, and 0%. Which of the following measurement scales would be most appropriate to analyze this trend? Categorical Ratio Nominal Ordinal

Ratio data have the property of an absolute zero and the proportions of two numbers can be compared. Thus, the comparing the proportion change of revenue requires ratio data. Categorical is the same as Nominal data which are for numeric or non-numeric identifiers used to label items Ordinal data are numeric or non-numeric but the items are not always uniform and the numbers are not meaningful.

Team Recognition

Recognition a. Teams should be recognized by senior management via bulletin boards, thank-you notes, bonus, stock option, ect. b. Organization culture can lengthen/shorten team stages of formation.

Recording Check Sheets

Recording Check Sheets 1) Used to collect measured or counted data 2) Data is collected by making tick marks -- ( tally sheet) 3) Caution--leave enough room for individual measurements to be written. 4) Measured data = physically measured information ---eg. pH, airpressure, or amount of downtime in hours.

Results of Cause-and-Effect Matrix

Results of Cause-and-Effect Matrix: 1) Listing & Evaluating KPOV's in a capability summary 2) Listing & Evaluating KPIV'S in a control plan summary 3) Listing and investigating KPIVs in a FMEA ***Can provide overview of the most important KPOV to KPIV relationships and help determine critical customer requirements.

Risk analysis studies the impact on:

Risk analysis studies the impact on: a. Meeting goals/objectives b. Planned schedule c. Resources d. Safety e. Produceability f. Serviceability g. Reliabiity h. Meeting customer expectations & requirements.

Role of project chart in the project documentation process?

Role of project chart in the project documentation process? a. Serves as top-level quick overview of project b. Small Projects-→ critical areas in project charter

Role of the Facilitator in Brainstorming:

Role of the Facilitator in Brainstorming: i. Enforce the ground rules during the brainstorming session and encourage ideas (keep ideas flowing) ii. Selected ideas are categorized : personnel-machine-material-methods-measurement-methods

Rolled Throughput Yield

Rolled Throughput Yield 1) Cumulative calculation of yield or defect through multiple process steps 2) RTY helps team to focus on the major problem source. 3) Significant difference in individual yields can suggest improvement opportunities.----> Eg. If the Yeild of Welding= 90% ---> Fab (yeild= 86%) ---> Assembly (Yield= 65%) ***The large drop in yield for the Assembly step means that must do an improvement project for Assembly step.

"Scatter diagrams

Scatter diagrams 1) Simple plot on two axes, useful for investigating the correlation between two variables. 2) Analyze stage of DMAIC--. The x-axis of a measures the independent variable (is the variable manipulated in the experiment) and the y-axis measures the dependent variable (the variable not manipulated in the experiment). 3) The general method of an experiment involves making small adjustments to the independent variable and observing the effects on the dependent variable. a) The independent variable should be manipulated throughout the entire region of interest, so data points achieve sufficient spread on the diagram. b) Also, the data used on a scatter diagram should be generated especially for that purpose. ******Experimenters should be cautious when one variable appears to change along with another, since correlation "

Statistical Process Control

Statistical Process Control 1) Use of control charting methods to conduct statistical analysis. 2) Assumes that all process are subject to variation 3) Key Benefit------> monitors stable process and detects changes that occur due to factors other than random variation and locates the source of assignable cause variation so ti can be eliminated.

Statistical control

Statistical control-→ process that exists as having only common cause variation only after all the special cause variation is removed. It also means that the process is between the upper and lower control limits. i. Overadjustment-→ adjustment of a process in response to common cause variation leads to greater variation. ii. Underadjustment -→ Failure to respond to special cause variation results in addition process variation.

Statistical process control errors:

Statistical process control errors: a. Any process can be monitored using basic SPC technieques. b. Control charts== separate assignable causes from random variation. c. Type I error= special cause that has not effect on the process. d. Type II= failure to address a defective process created by special cause variation. ****Use historical data with caution.

Steps for Process Capability Studies Process Stability

Steps for Process Capability Studies Process Stability a. Process is stable if the control charts shows no special causes after 20-30 subgroups plotted (more points plotted = greater confidence in conclusion) b. Control chart monitoring (process stability) = not affected by non-normal distribution of data c. Process capability-→ normality is required for continuous data

Team Life Characteristics--Develop Phase

Team Life Characteristics--Develop Phase (Norming) 1. Group takes on task-related work---> tasks, presentations, relationships 2. Groups must work to involve non-participating members 3. Leadership= Low task/ high relationship style

car maker is working on the design of a new sport utility vehicle. Which of the following tools can help the design team in gaining more understanding of their customers' needs? A) Kaizen B) Quality Function Deployment C) Check sheets D) Kano model

The Kano model is useful in gaining a thorough understanding of a customer's needs. Quality Function Deployment (QFD) integrates customer requirements (voice of the customer, or VOC) into the design and delivery of a service offering, but these requirements need to be collected first using other methods

Tools for Business performance measures:

Tools for Business performance measures: 1) Balanced Scorecard 2) Performance to established goals 3) Process Performance.

Types of FMEA

Types of FMEA 1. System FMEA 2. Design FMEA 3. Process FMEA 4. Service FMEA

When to use DOE?

When to Use DOE Use DOE when more than one input factor is suspected of influencing an output. For example, it may be desirable to understand the effect of temperature and pressure on the strength of a glue bond. DOE can also be used to confirm suspected input/output relationships and to develop a predictive equation suitable for performing what-if analysis.

Written Survey

Written Survey: 1) Written surveys can be sent to randomly selected customers or to potential customers , but getting responses from all those selected rarely occurs 2) Carefully worded survey can shed significant light on customer's reactions.

X-bar & R-charts:

X-bar & R-charts: a. X-chant detects any process shift b. R-chart only common cause variation. c. High probability of variation can occur between successive samples while the sample size is small-→ Best to use the same process to reduce "Within-in Sample" variation. d. Between- sample variation= process shifts.

A1C is a measure of sugar in the blood. The Mayo Clinic says a healthy person should have an A1C reading between 4.5% and 6.0%. Someone who already has diabetes will typically have an A1C level of 8%. A common treatment goal for people with diagnosed diabetes is an A1C of less than 7%. A pharmaceutical company is working on a new diabetes drug to reduce A1C and is ready to do atrial comparing the drug to a placebo. What significant difference in A1C should the company look for? 1.0% 0.6% 1.5% 0.6%

because 1.5% is enough to make a practical difference for the patient with diabetes and will require fewest samples. A 1.0% difference would help many patients however the sample size would be more than twice as big as when testing for a difference of 1.5%, driving up the cost of the experiment.

Gage R&R study Calculations

l. Calculations: (See formula on page191) You must calculate the following: i. The average for each trial for each appraiser ii. The average and range for each part for each appraiser iii. The overall average and average range for each appraiser iv. The overall average and the average range for the part You then determine the average range for the three appraisers. Then determine the difference between the maximum appraiser average and the minimum appraiser average. A has the maximum average (3.157). C has the minimum average (2.695). Thus, the difference is 3.157 - 2.695 = 0.462. Next determine the range of the part averages (Rp). The largest part average is for Part 3 (4.099). The smallest part average is for Part 5 (1.936). So, Rp = 4.099 - 1.936 = 2.163

"Setup time

"Setup time 1) To reduce cycle time ---> Target setup time & preparation time for activities. 2) Setup time = interval b/w the completion of the last item & beginning of the next item. 3) Setup time consists of four components: a) Preparation= adjusting & reconfiguring tasks for equipment before starting next item. b) Replacement= Tasks for adjust & reconfigure equipment before processing next item (eg. replacing printer paper) c) Location= positioning or moving NVA tasks that must be completed between iterations of a process. d) Adjustment =monitoring or fine-tuning tasks that must be performed between iterations to ensure correct performance of a process.

4 Benefit of Improvement Teams

4 Benefits of Improvement Teams: 1) Fuller understanding of the process needing improvement 2)Larger issues can be solved better than working alone 3) Mutual support/ cooperation from team member when working on common project. 4) Immediate access to the technical skills & knowledge of all team members (black belts, green belts, ect)

Taguchi

16) Taguchi-→ any departure from the target value represents a loss to society + emphasis on robustness + fractional factorial designed experiments. a. Coined the term "Design of Experiments" (DOE) b. The Taguchi method for improving product quality is optimally used during the design stage of the manufacturing life cycle. c. Taguchi curve can be interpreted as a phenomenon where any deviation from a quality measure target incurs costs that increase at an increasing rate as the actual measure moves away from the target value. From the quality cost perspective, the closer a company can get to its quality targets, the more it reduces quality related costs. Theoretically, if a company can consistently attain its quality targets, quality-related costs will be minimized and will consist only of those costs incurred to sustain the target quality level.

3 Awards given to Deming

3 Awards given to Deming: 1. Shewart Medal, ASQ, 1955 2. Second Order Medal of the Sacred Treasure, First American to get this award (1960) 3. Honorary ASQ member

Are Residuals Well-behaved?

Are Residuals Well-behaved? 1) Residuals=-----> elements of variation which are unexplained by the model----------> estimates of the experimental errors obtained by subtracting the observed response from the predicted response.-(predicted response calculated from the chosen model, all the unknown parameters estimated from experimental data) 2) Residuals are assumed to be normally distributed with mean of zero and some constant variance. 3) Assumptions for ANOVA and classical regression analysis: a) Regression model will err in predicting a response in a random fashion. b) Model should predict values higher and lower than actual with equal probability. c) Level of error should be independent of when the observation occurred in the study, OR the size of the observation being predicted, or even the factor settings involved in making the prediction.

An insurance company is attempting to address minor issues with its claims payment process. If it chooses to use the PDSA cycle popularized by Deming, what would be the correct order of steps to take? A) Implement the desired changes with a pilot program. B) Measure the performance of the modified process. C) Identify what changes to the claims process should be made. D) Integrate successful results into the existing claims process.

Answer= 3, 1, 2, 4 Identify, Implement, Measure and Integrate are the correct order of steps in the PDSA improvement cycle.

A point outside the control limits is a signal of the likely occurrence of: A trend A chance cause Special-cause variation Common-cause variation

Answer= Special-cause variation A trend is a continual movement in one direction, for example, six points in a row constantly increasing.

Champion

Champion a. Upper-level managers with training in 6 Sigma concepts/ tools/ strategies/operations methods (emphasis on management tools) that function as the liaison to senior management b. Vital in the implementation of quality management methodologies. Champions also serve as mentors to Black Belt certified professionals.

FMEA

FMEA: 1) Deatiled analysis of the sytem down to the component level as either a) failure mode b) Effect of Failure c) Risk Priority Number. 2) Used to manage risk due to failures and it effects; tool that provides the probability of a failure modes occurring and the effect the failure will have on rest of the system 3) Allows the criticality function of the failure effect on the entire system be ranked

Graec0- Latin Designs

Graec0- Latin Designs 1) Useful to eliminate more than 2 sources of variability. 2) It is an extension of the Latin Square design, but one extra blocking variable is added for a total of 3 blocking variables; it is an example of completely orthogonal square 3) Application --------> Useful when experimental units are used repeatedly ----Graeco-Latin squares are used in the design of experiments, tournament scheduling, and constructing magic squares 4) Disadvantage----> a) Size of the experimental design is restricted by degrees of freedom for the error term. Thus the order of the design must be one greater than the number of blocking factors---a severe restriction if there are only a few levels of the factor of interest that are to be considered. b) Unable to detect differences in factor levels effects because of the small number of observations from which each factor level iaverage is computed.

Importance of standardizing the new methods during control stage

Importance of standardizing the new methods during control stage 1) Prevention of backsliding (falling back on old habits) can occur if : a) Written policy documents the new standardization of method and the advances made. b) All process procedures and duties are written on process maps, flowcharts, and set of work instructions. 3) Employees should be trained in the new methods and new staff should be give comprehensive orientation

Improvement Teams

Improvement Teams Structure= 8-10 members from single department Application---> Process improvement team can consist of multi-department membership and focus on process flow and product issue a) Solve quality/productivity problem b) Problem may be management selected, but the solution is team directed.

Perfection is accomplished via 4 activities

Perfection is accomplished via 4 activities: 1. New product Development 2. Product teams working with customer feedback on specifying value, enhance flow, and achieve pull 3. Using technology to eliminate muda 4. Joint Collaboration between Value Stream Partners (suppliers, subcontractors, distributors, customers, employees to reduce Muda)

Perfection can be achieved in the business process by which of the following? A) Conducting a one-time brainstorming session B) Adding more resources in the process C) Iteratively and continuously improving the process D) Having a kaizen blitz session

Perfection is not a one-time effort. It goes through many iterations as it is continuously improved. Kaizen blitz is a one-time problem-solving process. Adding more resources will not make the process perfect

Statistical Process Delay.

Statistical Process Delay. a. Misapplication of statistical techniques to pretend that information is being used to run a process, when in reality it is only for show instead of actual application to improve a process

To use Kano Model Analysis, follow these steps:

To use Kano Model Analysis, follow these steps: 1) Brainstorm all of the possible features and attributes of your product or service, and everything you can do to please your customers. 2) Classify these as "Threshold", "Performance", "Excitement" and "Not Relevant". 3) Make sure your product or service has all appropriate Threshold Attributes. If necessary, cut out Performance Attributes so that you can get these - you're going nowhere fast if these aren't present. 4) Where possible, cut out attributes that are "Not Relevant". 5) Look at the Excitement Attributes, and think how you can build some of these into your product or service. Again if necessary, cut some Performance Attributes, so that you can "afford" your Excitement Attribute. 6) Select appropriate Performance Attributes so that you can deliver a product or service at a price the customer is prepared to pay, while still maintaining a good profit margin

Work Breakdown Structure Time Constrained Projects vs. Resource-Constrained Projects

Work Breakdown Structure 1) Plan that expands the project (Statement of Work) into a detailed listing of activities needed to complete the project 2) Project Team leader assigns tasks/ responsibilities and creates the work breakdown structure 3) Predececsor Event---> an activity that must be completed before another begins 4) If time constraints are fixed, then resource constraints must be flexible to accommodate variations in project.---> Contract may impose penalties for each day that project is late and therefore time delays require offstting increase in resources and costs to keep the original deadline 5) Resource-Constrained (equipment / manpower) Projects-----> If shared resources, then must schedule and coordinate resources---> Objective is to meet the project deadline without exceeding resource limits

What is the difference between work procedures and work instructions?

Work Procedures describe: i. What is done during the process ii. Why it si done (business reason) iii. Where it is done (location/ process map) iv. When it is done (trigger) Work Instructions explain 2 things: i. Who does what (personnel with specific skill set) ii. How it is done (step by step)

"X-bar charts

X-bar charts 1) Control charts for variables data taken from the averages from subgroups (of observations) to get a view of the process location over time. 2) Measure stage,---> used to baseline processes by identifying the inevitable amount of common-cause variation. They also are used in this stage to evaluate the repeatability and reproducibility of a measurement system. 3) Analyze stage---->these charts are used to distinguish special and common causes of variation. 4) Improve stage---->, X-bar charts are used to confirm that implemented changes have improved the process metric. Finally, in the control stage, X-bar charts are used to ensure that processes remain statistically stable. "

"14. Which type of diagram is used to eliminate unnecessary movement during a process? a. Spaghetti diagram b. Scatter diagram c. Ishikawa diagram d. Matrix diagram

a 14. A: Spaghetti diagram. A spaghetti diagram indicates the physical travel of employees, resources, and equipment during a process. These diagrams are used to identify unnecessary movements and to streamline processes as much as possible. A scatter diagram displays the correlation between two variables, with the independent variable on the x-axis and the dependent variable on the y-axis. An Ishikawa diagram, also known as a fishbone or cause-and-effect diagram, is used to outline the causes of a particular event, as well as the possible results of particular actions. Finally, a matrix diagram depicts the relationships between a group of different items in several groups. A matrix diagram looks a great deal like a table of data looks a great deal like a table of data, with the strength between relationships indicated by the values in each cell.

The F distribution has two parameters:

a. The F distribution has two parameters: degrees of freedom numerator (dfn) and degrees of freedom denominator (dfd). b. The dfn is the number of degrees of freedom that the estimate of variance used in the numerator is based on. The dfd is the number of degrees of freedom that the estimate used in the denominator is based on. The dfd is often called the degrees of freedom error or dfe. In the simplest case of a one-factor between-subjects ANOVA, (dfn = a-1)

Benchmarking

e. Benchmarking (1988--1996)--- 1. Definition = An improvement process in which a company measures its performance against that of best-in-class companies, determines how those companies achieved their performance levels, and uses the information to improve its own performance 2. Process whereby project Key Performance Indicators (KPIs) are set against internal and external goals. a) Key Takeaway: An important aspect of initial project definition is to set KPIs that track performance across a number of areas such finance, operations, and human resources. Project teams can set KPI figures that compare against previous company projects (internal goals) or those of external competitors. 3. Company measures its performance against the best-in-class 4. Determines how companies achieve their performance levels 5. Uses the information to improve its own performance.

Application of Run Charts

Application of Run Charts 1) On m and individual X-charts where the normal distribution is used, the median is substituted for the average. 2) Must establish zones that produce the same probabilities as the normal curve for the given standard deviation 3) When applying run test to the upper and lower control chart-------> Separately apply run tests 1,2, 5, &6 4) Run tests (3,4,7,8)-----> Apply to the entire chart.

Hypothesis Tests for Means Chi-square

Hypothesis Tests for Means----Chi-square a. Conditions: i. All expected frequencies are at least 1 ii. At most, 20% of the expected frequencies are less than

Improve Stage: Implementation and verification of changes

Improve Stage: Implementation and verification of changes 1) Realization of the solutions crafted by the team that requires constant communication & documentation to detect/ resolve problems. 2) Get sponsor's explicit authority before making changes so other employees recognize the importance of buy-in. a) Must give detailed instructions to participating employees with close supervision during the initial implementation phase. b) Best if the 1st results verify the solution and generate sufficient data for statistical analysis. (After process confirmed as stable & acceptable, it becomes the new method of operation.)"

Sensitivity

Sensitivity 1) A gage should be sensitive enough to detect difference in measurement as slight as 1/10 of the total tolerance specification or process spread, whichever is smaller. 2) Inadequate discrimination will affect both accuaracy and precision of reported Values

Steps for documenting Value Stream Mapping

Steps for documenting Value Stream Mapping: 1. Product development --->Id customer requirements, delivery methods, and typical quantities 2. Process Design---> Note machine times, cycle times, operators, changover times, WIP, available time, scrap rate, machine reliability. 3. Record Current Status .4 Create Future State Map

Two -level Experimental Design

Two -level Experimental Design: 1) Ideal for screening designs and give most of the information required to do a multi-level response surface experiments. 2)Include some center points (for quantitative factors)

Selection of Variable

Selection of Variable 1. May be the "leading indicator" of choice of special cause variation. 2. Contractual requirement specify the variable to be monitored. 3. If the root cause of the special variation is unknow, an input variable can be monitored. (best to pick the input variable that is most difficult to hold and has been identified by capability analysis) 4. Computerized control charts aid in the monitoring of multiple variables.

7 Metrics for measuring Marketplace

7 Metrics for measuring Marketplace respones 1. Market survey of customers 2. Analysis of returns 3. New product Development 4. Customer retention 5. Cutomer losses 6. Facilities ratings 7. Coursey ratings

Quality Function Deployment Application

Applied/ Uses of Quality Function Deployment: a. Adoption of customer value within the product. b. Supports customer-driven quality and total customer experience by providing objective and traceable matrix linking customer wants/expectations to technical details--→reinforces the robustness of design

Charter Negotiation and 7 examples

Charter Negotiation Upon presenting the charter to upper management, team may propose a different approach than what was envisioned by management. ---Examples 1) Objectives --eg. final customer may want a huge redesign that was not anticipated. 2) Scope-->Project boundaries may have to be expanded or broken up into small sections or require additional project teams 3) Resources---Management may be called to decide on how to prioritize resources outside of the team's control 4) Project Transition---Additional time or supervision may be needed to transition the project to routine staff. 5) Project Closure---> May have re-consider project closure date due to shifts in customer preference or unexpected succuess or diverse events.

Challenges linked to interactions between process suppliers & customers:

Challenges linked to interactions between process suppliers & customers: a. Process ownership ( two or more areas may think that they own the process and have final decision to make changes) b. Sharing of information (eg. Proprietary issues, hiding poor performance) c. Commonality of measures (eg. Finance measures in dollars, production uses defect /productivity) d. Process knowledge or expertise (manufacturing may not understand supply chain management)

Chi-Square & Probability Distribution:

Chi-Square: 1) The chi-square distribution has the following properties: a. The mean of the distribution is equal to the number of degrees of freedom: μ = v. b. The variance is equal to two times the number of degrees of freedom: σ2 = 2 * v c. When the degrees of freedom are greater than or equal to 2, the maximum value for Y occurs when Χ2 = v - 2. d. As the degrees of freedom increase, the chi-square curve approaches a normal distribution. e. Chi-square is non-negative & non-symmetric f. Many different chi-squares distributions, one for each degree of freedom

Classification of experimental designs:

Classification of experimental designs: a. Completely randomized b. Factorials --- c. Blocked Factorials --- d. Fractional Factorials ---- e. Randomized Blocks--- f. Mixture Desings g. Response surface h. Latin square/ Youden square

Common interpretations of Cp and Cpk:

Common interpretations of Cp and Cpk: 1. Higher values best for Cp and Cpk 2. Cp value does not change as the process is being centered to target unless something in the process changes. 3. Cp and Cpk values will be equal if the recess if perfectly centered. 4. Cpk is always equal to or smaller than Cp. 5. Negative CPk→ process average is outside the engineering specifications. 6. If the process with one-sided specification-→ either Cp upper limit or Cp lower limit is calculated. 7. High Cpk from small sample may not be of much uses as the confidence intercal for Cpk will be very wide.

Design Improvement to mistake proof products & processes.

Design Improvement to mistake proof products & processes. 1) Amplification of human senses 2) Redundancy in design (Back-up systems) 3) Simplification by using fewer components 4) Consideration of functional and physical environmental factor 5) Providing fail-safe, cut-off mechanisms 6) Enhancing product producibilty and maintainability 7) Selecting components and circuits that are proven

Examples of common cause variation:

Examples of common cause variation: a. Sequencing of the process b. Manufacturing equipment design c. Natural variation in incoming material supply d. Measuring equipment design

FMEA Followup

FMEA Followup: Green-belt should followup on FMEA to verify that implementation actions completed Followup-Steps 1.) Review FMEA for function/purpose./ objectives 2 )Note high risk, high severity, high RPN values 3.) Identifying the critical, significant, and major characteristics of the process 4.) Seeing the control plan is in place 5) Conducting capability studies, if needed 6) Working on processes below Cpk of 1.33 7) Working on improving other process with Cpk of 1.33

FORD used FMEA

FORD used FMEA for design and manufacturing to safeguard against safety and regulatory-related failure modes (resulting in reduction of risks threatening product quality) i. Chrysler, Ford, GM created a document with process and design FMEA-→ QS9000 (ISO/TS 16494) standard expectations for automotive suppliers. ii. Updated with inputs from SAEJ1739 work group. iii. Automotive Industry Action Group (AIAG) described FMEA Activities as intended to: 1. Recognize/evaluate the potential failure or product/process 2. Document entire process 3. ID actions that can eliminate/reduce failures.

Which of the following team situations requires facilitator intervention? Recording critical data from meetings Staffing the team Watch the use of allocated time Discussion develops into multiple conversations

Facilitator intervention is the function of a facilitator who must intervene if a discussion develops into multiple conversations Watching the use of allocated time is the function of a timekeeper. Staffing the team is the team leader function Recording critical data from meetings is the function of a scribe or note taker.

IDOV

IDOV =( Six Sigma method) = identification, design, optimization, and validation. In the identification phase, the business determines the needs of customers. 1) Also, the business will create an effective business model and establish a Six Sigma team. 2) Design phase= the team uses critical-to-quality metrics to determine the best methods for processes. 3) Optimizing phase= the Six Sigma adjusts the factors that contribute to processes until the best outputs are achieved. 4) Validation phase, the Six Sigma team makes any necessary changes to the new process.

IDOV verses DMAIC:

IDOV verses DMAIC: a. IDOV—Used to create completely new product or business processes to meet customer needs b. DMAIC—used to improve an existing product or service via the use of new processes.

Ideal Six Sigma Design

Ideal Six Sigma Design→ 1, exceeds customer requirements at the lowest cost. 2. Number of sigmas that will fit between a process Mean and the desired specification limit will decrease over time. Key Takeaway: Due to a number of factors including machine wear and tear, the number of sigmas that will fit between a process mean and the desired specification limit will decrease. The challenge for managers is to chart this reduction and provide solutions in order to maintain high quality in production standards.

Comparison of the natural process range to the tolerance range tells you: If the process is centered on target. That the process needs no adjustment. asq/learn_check.gif If the process is capable of meeting the specification If the process actually meets the specification.

If the natural process range is narrower than the tolerance range, then the process is capable of meeting specification, albeit it might need re-targeting. Comparison of two ranges doesn't tell you how well the process is doing, just how well it could do if centered appropriately.

Interval Estimate

Interval Estimate In statistics, interval estimation is the use of sample data to calculate an interval of possible (or probable) values of an unknown population parameter, in contrast to point estimation, which is a single number. It is the interval within which the population mean is predicted to fall

Master Black Belt Duties

Master Black Belt Duties: i. Input to rewards committee ii. Engage in 15-20 projects yearly iii. Coaching/mentoring to green & black belts iv. Selection of projects with champions & process owners v. Budget matter expert vi. Remove project stagnation and project barriers vii. Approval of completed projects.

Measurement Scales----Ratio

Measurement Scales----Ratio ) Central location = Geometric or Harmonic Mean (both are not the same thing) 2) Dispersion of Percent Variation 3) Significance Test= Any Test 4) Data consists of names or categories without any ordering scheme

Measurement of Project Activity

Measurement of Project Activity 1) Upper management will require scheduling briefing session that can range in depth from project milestones to comprehensive reports.

Methods of reducing errors:

Methods of reducing errors: a. Data collection plan b. Calibration schedule for data collection equipment c. Repeatability & reproducibility (R& R) studies on measurement system d. Statistical testing to remove outliners e. Clear & complete instruction & training f. Redundant error correction system for digital data transmission g. Auxiliary information recording (units, time of collection, condition, measurement equipment, data recorder)

Multi-Collinearity

Multi-Collinearity 1) Occurs when 2 or more input factors are expected to independently affect the value of an output factor, but are found to be highly correlated. 2) Example-------> experiment is being conducted to determine the market value of the a house. Input factors are square feet of lining space and number of bedrooms.

Non-Bottle Neck Stage & Inventory of Intermediate parts

Non-Bottle Neck Stage & Inventory of Intermediate parts 1. Flawed old concept ---->High inventories of intermediate parts previous considered beneficial if each part made as efficiently as possible, and that would make the final manufactured good as efficient as possible. ---> Each station set up for long runs to produce an excess intermediate parts. 2. This harms the plant in several ways: a) Money is invested in these parts that are not immediately needed, b) Excess parts take up space, create costs of moving them around, are susceptible to theft loss, but most importantly, can cause steps to be unavailable when needed to make critical bottleneck parts. 3. Excess inventory of intermediate parts should be viewed as a cost, not an asset. 4) A non-bottleneck machine should not work faster than needed for immediate production, because that will just pile up inventory. If you have excess production capacity, you must waste it rather than use it.

Objective of "Define" Stage

Objective of "Define" Stage 1) Fcuses on identifying and articulating a project's important characteristics, including identification of the goals, objectives, and scope of the project. 2) Selection of the team members and sponsors, as well as the proposed schedule for the project. 3) Project deliverable (desired result) is defined.The Six Sigma team should define the desired result of the project, otherwise known as the deliverable. 4) Stakeholders, the input/output structure, and the functions involved are identified 5) Selecting and assembling a team able to accomplish all of the proposed goals. (must be able to work together and have a solid understanding of the task , should all desire to successfully complete project.)

Owners & Stakeholders

Owners & Stakeholders 1) Process owners a. Responsible for the execution and implementation of specific process. b. May be subject matter experts c. Are formally recognized via documentation (job description or organizational chart) i. An organizational structure defines the relationship amongst project participants within an organizational unit while setting guidelines for duties, responsibilities, channels for authority and internal and external communication. In simple terms, an organizational structure aims to provide answers to the following questions, among others: who is the leader of a particular organizational unit? To whom do members of the unit report? And how does the unit function? ii. Project team, matrix, and functional structures are all examples of organizational structures.

PDCA

PDCA 1. Every activity is part of a process 2. Flow diagrams divide wok into stages and stages together form a process. 3. Each stage has a customer 4. Improvement cyvle will send a superior product/service to ultimate customer

Define Performance Benchmarking

Performance Benchmarking 1) Enables managers to assess their competitive positions through product and service comparisons 2) Focuses on elements of price, technical quality, ancillary product or service features, speed, reliability, and other performance characteristics.

Design of Experimentation Planned Grouping

Planned Grouping a. Practice done to promote uniformity within blocks and minimize the effect of unwanted variables. This will make experiment more effective in determining assignable causes.

Quality Cost Improvement Sequence---9 steps

Quality Cost Improvement Sequence 1) Define Company's quality goals ----> Position among competitors + desired long-term reputation 2) Translate Quality goals in Quality Requirements ---> Specific tests, Control types, Outgoing Quality levels 3) Estimate the capability of process/ machines/ systmes 4) Create a program aligned with company goals 5) Determine resources requirements for project 6) Set up quality cost categories of prevention, appraisal, and failure 7) Get estimates of the 3 quality cost categories 8) Analyze the quality cost data for major improvement candidates 9) Pareto Principle to isolate specific areas for investigation.

Recorder Role

Recorder Role 1) Usually full-fledged team member selected by team member 2) Coordinates preparation of letters/reports/ documents 3) Duty may be rotated among team members

Relationship between DPMO & Sigma

Relationship between DPMO & Sigma: 1. Motorola noted that operations tended to shifted 1.5 sigma over time 2. Process, with normal distributions and normal variation of the mean, need to have specification limits of +/- 6 sigma in order to produce less than 3.4 DPMO. 3. Companies with less than 3 sigma capability and with +/- 1.5 sigma shifts won't be around long enough to take on a six sigma improvement effort.

Risk Analysis tools can be combined with other tools (eg. Brainstorming) to ensure:

Risk Analysis tools can be combined with other tools (eg. Brainstorming) to ensure: a. Effective coverage of risk identification b. Analysis of potential impact if the risk is realized c. Potential Mitigation plan.

Risk Management Phases & Definitions ---Risk Monitoring

Risk Management Phases & Definitions ---Risk Monitoring Definition-----> Continuous Evaluation & Tracking system to ensure that implemented actions to mitigate risk are working as planned. Inputs: "DD PRS" ---Danger List, Decision Analysis, Performance tracking, Reaction Models & Simulations.

SIPOC can help everyone "See" the business from overall process perspective in 4 ways

SIPOC can help everyone "See" the business from overall process perspective in 4 ways: 1) Displaying cross-functional activities in simple diagrams 2) Providing a framework applicable to process of all sizes. 3) Helping maintain the big picture business perspective 4) Providing methods for adding additional detail as needed ** Must consider the levels of business process.

Sampling Plan

Sampling Plan (Define Phase) 1) When looking at large quantities, either distractions or other problems can prevent correctly identifying the items that are being studies. 2) To prevent this, then a sampling plan can be used to take samples of the population in question to get idea of what the population contains. Various factors are considered to ensure the process, product, and customer needs are met. Checklist can be used to ensure that sampling plan is followed.

Simplex Design

Simplex Design Spatial design used to determine the most desirable variable combination in a mixture.

Which of the following does a Process Capability Study provide? A tool to set engineering specifications A comparison of a process' performance to output requirements A tool to help reduce process variability A determination of short-term process performance, not long term performance.

Since a process capability study should only be used on a process that is stable, it can estimate long-term performance. When compared to outputs, process capability helps to determine overall performance. This comparison is quantified by the process capability index (Cp) among others. Engineering specifications should be set based upon engineering requirements. Process capability should only be considered when engineering specification are too tight for the process to achieve

List the 4 Tools in Six Sigma

Six Sigma ToolSet-→ 1) Statistical process control, 2) control charts, 3) FMEA, & 4)process mapping.

Steps for getting good results from DOE:

Steps for getting good results from DOE: a. Watch for process drifts and shifts b. Perserve all raw data---not just averages c. Avoid unplanned changes d. Get buy-in from all parties.

Steps in Project documentation

Steps involved in project documentation: a. Goals & objectives b. Project spons and stakeholders c. Project plans and schedules d. Key project milestones e. Project budget f. Project boundaries g. Roles and responsibilities of project team members h. List of deliverables i. Metrics used to assess the project's performance

Steps involved to get good results from DOE

Steps involved to get good results from DOE 1) Set Objectives 2) Select process variables 3) Select Experimental Design 4) Execute the design 5) Check the data are consistent wit the experimental assumptions 6) Analyze and interpret the results 7) Use/ present the results ( may lead to more runs or DOEs)

Which of the following is an example of a process that can benefit from applying value stream mapping to identify waste in space? A production unit requires the storage of metal parts awaiting rework on the shop floor. A metal plating process producing excessive scrap parts as a result of a poor maintenance program. A raw materials release process requires the approvals of the purchasing, production, and warehouse managers. A production process that requires moving parts from the warehouse to a staging area, then to the production floor for assembly.

Storage is a type of waste associated with space. Value stream mapping can be used to identify such waste. Scrap is a type of waste in materials. Requiring too many permissions is an example of complexity waste. A production that requires moving parts from the warehouse to a staging area, then to the production floor for assembly is an example of transportation waste (movement) that may not add value.

Team Dynamics --- Floundering & Countermeasures

Team Dynamics & Floundering a. Teams functioning in a unsteady, faltering manner b. Causes: i. Lack of team direction (high-profile team leaders don't participate in meetings) ii. Major organizational changes where the future uncertain. iii. Overwhelmed team leaders (change in leadership, downsizing, new mergers, off-shore transfers) c. Countermeasures: i. Provide more direction in early stages of team formation. ii. Team leaders visible presence in meetings iii. Reinforce management support when teams challenges purpose of team iv. Team leaders address the concerns of team members and keep the team focused by not letting other events to distract.

Team Dynamics: Group Think

Team Dynamics & Group Think a. Symptoms= No public disagreements, Doubt expressed in private b. Cause= Fear of losing group cohesiveness if disagreement c. Countermeasures= Bring outside members to participate + Management by fact + rotation of roles

Taguchi--View of Product Quality

Taguchi--View of Product Quality 1. Evaluation of Quality -----> a) Costs that fails to meet targets can be evaluated with loss function and signal-to-noise ratio. b) Quality loss increases parabolically as the product strays from single target value (traditional view is the product is or isn't in control limits) 2. Improvement of Quality & Cost Factors a) Statistical methods---QFD, signal-to-noise characteristics, and DOE---can be used for system design, parameter design, and tolerance design of the product. 3. Monitoring & maintaining quality a) Reduce production line variability ---> Stress Consistency on the floor---> Measure the quality characteristics from the floor and use the feedback

Which of the following statements is true in describing the house of quality structure? A) The left side of the house has the customer needs B) The right side of the house has design features and technical requirements C) The foundation contains customer priorities D) The roof of the house contains the VOC

The right side of the house contains customer priorities, the foundation contains target values and the roof contains a matrix that describes relationships. The roof contains a matrix that describes relationships. The foundation contains target values.

The various contributors to the measurement system variation can now be calculated. There are five that need to be calculated:

The various contributors to the measurement system variation can now be calculated. There are five that need to be calculated: • Equipment variation (EV) • Appraiser variation (AV) • Repeatability and reproducibility (GRR) • Part variation (PV) • Total variation (TV)

Tools for top-level process analysis

Tools for top-level process analysis 1) Flowchart = The most common tool for top-level process ----> Represents process sequences graphically, with a minimum of verbal description. 2) ) SIPOC analysis identifies (suppliers, inputs, process, outputs, customers 3) Process map (more detail than flowchart) indicates who is responsible for each activity in the process. ****A comprehensive Six Sigma project might include more than one of these tools for top-level process definition. The primary aim of these representations is to inform stakeholders and other people peripheral to the project. A top-level process definition will not contain enough information to explain each process in depth."

Two reasons that one should move from discrete to continuous measurement:

Two reasons that one should move from discrete to continuous measurement: a. Control charts based on continuous data are more sensitive to process changes than those based on discrete data. b. When designed experiments are used in process improvement, changes in continuous dta may be observed even though the discrete measurement hasn't changed.

Two reasons why it is difficult to implement an improvement program

Two reasons why it is difficult to implement an improvement program: 1. Bad times when low resources, strategic plan in survival mode, and resources costrained. 2. Good times because high profitability and resources focused on current cash flow lower the importance of improvement programs

Two-tail test

Two-tail test a. Test used to test whether a population shift has occurred in either direction.

Upward Flow of Communications

Upward Flow of Communications 1) Information relayed from the bottom/grassroots to higher levels 2) If management acts on the information from below, then lower levels gets motivated or empowered. 3) To encourage upward communication, managers should keep an open-door policy, surveys, questionaires, suggestion systems, shift meetings, breakfast meetings, ect.

17. In which method of sampling is a population divided into groups and a sample taken from each group? a. Systematic sampling b. Stratified sampling c. Judgment sampling d. Cluster sampling

a "17. B: Stratified sampling. In stratified sampling, the population is divided into groups, and a sample is taken from each group. In systematic sampling, on the other hand, there is a particular order to the selection of samples. In judgment sampling, an expert or group of experts selects the samples. In cluster sampling, experts create a representative group from which a random sample is drawn.

"Central limit theorem

""Central limit theorem 1) In some cases, there will not be enough (or good enough) data to establish a distribution. 1) Apply central limit theorem is applied when the true distribution of a population is unknown. 2) The central limit theorem, in short, asserts that the probability distribution of the sample means will approach a normal distribution as the number of samples increases, provided that they are simple random samples of uniform size. 3) The central limit theorem is applied when the number of samples is relatively small or when the true distribution is unknown. a) After about thirty samples, the data should approximate a normal distribution. b) The central limit theorem is extremely useful in control charting and in the calculation of process capability.

23. Which of the following is a disadvantage of using engineering process control devices to prevent deviation? a. The devices must be monitored by human operators. b. The use of these devices precludes the use of statistical process controls. c. These devices require constant maintenance. d. These devices cannot handle multiple inputs.

"23. B: The use of these devices precludes the use of statistical process controls. One disadvantage of using engineering process controls to prevent deviation is that the use of these devices precludes the use of statistical process controls. An engineering process control is a mechanism that automatically adjusts inputs when it detects variations in the process. A thermostat is a basic example of an engineering process control. It is not necessary for these devices to be monitored by human operators, and in most cases engineering process controls do not require constant maintenance. The constant adjustments made by these devices, however, mean that any data related to their activities is not independent, and therefore cannot be analyzed with statistical process control charts. However, the engineering process controls used by heavy industry are capable of handling a number of different inputs and outputs simultaneously. "

31. In kaizen, the idea that one step in a process should be completed only when the subsequent steps are ready is referred to as: a. Flow b. Poka-yoke c. Pull d. Perfection

"31. C: Pull. In kaizen, the idea that one step in a process should be completed only when the subsequent steps are ready is referred to as pull. This is opposite to the typical arrangement in manufacturing processes, in which materials are pushed through the process chain as they are completed. Kaizen recommends instead that materials be drawn along by vacuums created in the production chain. A process chain in which this occurs is said to have pull. Flow, meanwhile, is the continuous completion of a process. Organizations that adopt the kaizen philosophy attempt to make flow constant in every department and stage of processes. Poka-yoke is a Japanese system for error-proofing, based on the premise that avoiding errors in the first run is worth a slightly higher cost. Perfection is the kaizen ideal of continuous improvement. Perfection is a goal that can never be attained but should be strived towards regardless.

"33. If all of the data points on an Np chart fall between the upper and lower control limits, the process is: a. Representative b. Stable "c. Efficient d. Erratic

"33. B: Stable. If all of the data points on an Np chart fall between the upper and lower control limits, the process is stable. So long as all of the variation is within these limits, it can be assumed to be the result of common causes. Assuming the chart is reliable, data points that fall outside the upper and lower control limits are the result of special-cause variation. At the least, the presence of data points outside the upper and lower control limits identifies areas where employees will need to conduct further research. Np charts are control charts for analyzing attributes data. These charts are used when the sample size is regular and the targeted condition may only occur once per sample. "

34. According to Little's law, the number of items included in a process divided by the number of process completions per hour is the: a. Process lead time b. Value-added time c. Velocity d. Process cycle efficiency

"34. A: Process lead time. According to Little's law, the number of items included in a process divided by the number of process completions per hour is the process lead time. Process cycle efficiency is calculated by dividing value-added time by process lead time. When every activity in the process adds value, the process may attain the maximum process cycle efficiency of 100%. Of course, very few processes actually reach this level of efficiency. It is much more common for a process to have a process cycle efficiency below 50%.

37. Which of the following factors is not included in the calculation of risk priority number? a. Detection level b. Severity c. Expense d. Likelihood"

"37. C: Expense. Expense is not one of the factors included in the calculation of risk priority number. Risk priority number is calculated by multiplying severity, likelihood, and detection level. The severity of the risk is the significance of its occurrence. Various industries have created standardized tables for indicating the severity of common risks. The likelihood of a risk is simply the chances of it happening. Finally, the detection level is based on the number of modes for identifying the error or failure, as well as the chances that any one of these modes will be successful in detection. A common formula for calculating risk priority number is to place all of these categories on a scale from 1 to 10, then multiply them together. In this scenario, the maximum risk priority number would be 1,000."

"38. Which of the following run tests identifies shifts in the process mean? a. Run test 4 b. Run test 6 c. Run test 7 d. Run test 8

"38. B: Run test 6. Run test 6 identifies shifts in the process mean. The other run tests provide information about sampling errors. Run tests 1, 2, 3, and 5 also identify shifts in the process mean. Run tests are typically used in statistical process control programs to identify errors in data collection. Unfortunately, run tests are only able to identify the presence of errors and are not very good at pinpointing their location.

4. What are the three most important characteristics of process metrics? a. Rationality, reliability, and repeatability b. Reliability, reproducibility, and repeatability c. Reliability, responsibility, and rationality" "d. Repeatability, responsibility, and reproducibility"

"4. B: Reliability, reproducibility, and repeatability. The three most important characteristics of process metrics are reliability, reproducibility, and repeatability. Reliability is the extent to which the results of an experiment can be trusted to represent accurately the process being measured. Reproducibility is the extent to which a metric can be applied in different situations and obtain a reliable result. Repeatability is the extent to which a metric can be applied to the same situation multiple times and achieve the same result. "

40. Which distribution should be used when the targeted characteristic may appear more than once per unit? a. Binomial b. Exponential c. Lognormal d. Poisson

"40. D: Poisson. A Poisson distribution should be used when the targeted characteristic may appear more than once per unit. In order for a Poisson distribution to be effective, the data should consist of positive whole numbers and the experimental trials should be independent. A binomial distribution is appropriate for situations in which the units in the population will only have one of two possible characteristics (for example, off or on). An exponential distribution is appropriate for measurement data, especially frequency. A lognormal distribution is appropriate for continuous data with a fixed lower boundary but no upper boundary. In most cases, the lower boundary of a lognormal distribution is zero.

44. In hypothesis testing, why is it better to set a p value than to select a significance level? a. It ensures that a true hypothesis will not be rejected. b. It is then easier to make adjustments later in the experiment. c. It enables the collection of more samples. d. It makes it possible to reject the null hypothesis.

"44. B: It is then easier to make adjustments later in the experiment. In hypothesis testing, it's better to set a p value than to select a significance level because it is then easier to make adjustments later in the experiment. In general, a p value allows for more freedom in the later parts of the experiment. There is always a possibility of rejecting a true hypothesis, in what is known as a Type 1 error. The number of samples collected is not dependent on whether a p value is set or a significance level is selected, and either method maintains the possibility that the null hypothesis will be rejected.

"Which pioneer of quality control wrote Quality Is Free? a. W. Edward Deming b. Joseph M. Juran c. Armand V. Feigenbaum d. Philip B. Crosby"

"5. D: Philip B. Crosby. Philip B. Crosby wrote Quality is Free, a book that revolutionized quality management by placing an explicit emphasis on getting processes right the first time. Crosby insisted that businesses are better served by investing more money in quality control on the first run, and thereby avoiding the costs of defective products. W. Edwards Deming is famous for enumerating the seven deadly diseases of the workplace and fourteen points of "emphasis for management. Joseph M. Juran stressed the importance of customer satisfaction as a goal of quality control. Armand V. Feigenbaum is known for emphasizing four key actions in the implementation of quality management: establishing standards; creating metrics for conformance to these standards; resolving issues that impede conformance; and planning for continuous improvement." REVIEW THE WORK OR BOOKS OF THE OTHER CHOICES

"ANOVA

"ANOVA 1) Table depicting sum-of-squares variance that can be credited to a particular source, to an error, and the total sum of squares from the data. 2) F statistics = significance of the source relative to the error. 3) Measure stage---ANOVA identify the origins of measurement errors, esp. for processes that damage/ diminish the resources involved, such that a repeatability and reproducibility (R & R) is not possible. 4) Analyze stage---,ANOVA provides information about the statistical significance of a regression models (+ ) identify differences between data subsets with the intention of identifying the causes of variation within a process. 5) ANOVA used to find possible process drivers.

Analytical statistics

"Analytical statistics 1) Provide information about processes in action and distinguish common- and special-cause variation. a) Common-cause variation occurs consistently and always influences data in a similar fashion. b) Special-cause variation, on the other hand, is unpredictable in its occurrence and effects. c) Statistical process control charts create an operational definition of special-cause variation by noting the location and level of variation within a subgroup at each time in the process. a) If enough subgroups can be collected, the statistical process control chart will be able to predict the location and extent of special-cause variation. b) The key feature of analytical statistics is the consideration of time, which makes it possible to pinpoint the moments in a process that need improvement. " "

"Basic objectives of the measure stage

"Basic objectives of the measure stage During the measure stage of DMAIC, the Six Sigma team will focus on gathering the information necessary to complete the project. First, the team will attempt to define each relevant process in great detail. At the same time, it will be necessary to develop a group of metrics appropriate to the processes. No metric is complete until accompanied by a measurement analysis system that identifies and quantifies any common errors in the metric. In other words, the team will need to determine the ways in which the measurement system likely is inaccurate. The final general objective of the measure phase is to estimate process baselines. This objective enables the team to identify a reasonable starting point for the project. " "Creation and interpretation On a process map, a symbol represents every step in a process. Although it is possible to use the entire set of ANSI symbols for flowcharts, it is more common simply to depict decisions with diamonds and other tasks with rectangles. All decisions should be framed as binary; that is, only two possible answers exist. In more sophisticated process maps, shading or shaping will indicate delays or measurement intervals. As with flowcharts, one of the clearest signs of inefficiency on a process map is a glut of decision points. The goal of process maps is to spot redundant, unnecessary decisions and tasks and to discover ways to simplify processes"

"Binomial and Poisson distributions

"Binomial and Poisson distributions 1) Binomial distributions are useful when the units in a population exist in only two states.( e.g. if the only possible characteristics of a population are "off" and "on," a binomial distribution can be used to estimate the total number of "offs" and "ons" in a population.) 2) A binomial distribution only applies when trials are independent and the number of samples in the population is fixed. 3) Distributional parameter ( average proportion)= . dividing the (number of sample items that meet the condition) / (total number of items in the sample) . A Poisson distribution, on the other hand, can guess the number of times a particular condition will occur for a given process or population or the number of events in each unit. a) The distinguishing feature of the Poisson distribution is its appropriateness for situations in which the targeted condition may occur more than once in each unit. b) The Poisson distribution accurately estimates the number of events in each sample unit. As with the binomial distribution, the trials be should be independent, and the data should be composed of positive whole numbers. In a Poisson distribution, the distributional parameter is the[...]"

"Box-whisker charts

"Box-whisker charts 1) Box-whisker charts =graphs used for the comparison of summary data from numerous data sets. ( data sets may be organized by any general principle or characteristic) 2) Analyze stage,--> box-whisker charts may be used to compare the placement and characteristics of services, products, or processes. 3) Improve stage, box-whisker charts may be used to compare process performance before and after improvement efforts. **Note that box-whisker charts do not contain statistical control limits and therefore cannot be used to baseline or set statistical controls for a process. In other words, box-whisker charts are not control charts.

Deming's 14 points on management: Constant quality improvement and on-the-job training:

"Constant quality improvement and on-the-job training: a) Deming's fifth point for management is that production and service systems should be improved continuously so costs can be decreased continuously. b) Constant quality improvement Via unceasing attention to the desires of customers. ----> Customer desires change over time, so data collection never should cease. c) Deming: & continuous quality improvement Vs. solving problems? ---problem solving=ineffective way to achieve constant, long-term improvement d) Deming's sixth point for management= all employees given on-the-job training in all areas that pertain to their work, and managers always should understand the tasks of their subordinates. d) Effective training requires steady and productive communication between all layers of the organization. e)Also, training programs should be based on concrete and specific performance standards.

Control charts to identify sources of variation

"Control charts to identify sources of variation 1) Analyze stage --->look closely at the statistical control charts created during the measure stageto distinguish special and common causes of variation. a) Special causes of variation do not occur during every performance of the process, but when they do, they cause significant delay. b) Control chart, special causes of variation= points that lie far outside the normal range. The location of these out-of-control points will suggest --location at which point during the process the variation occurred. b) Common causes variation will influence each iteration of the process and therefore not obvious on control chart---> Difficult to detect b/c. Several reasons for common cause variation may exist.

Creation and interpretation of Flow charts

"Creation and interpretation of Flow charts 1) Flowcharts simplify communication by representing each task in a sequence with a symbol. a) Most organizations endorse the use of the symbols created by the American National Standards Institute.----> diamonds= decisions and processes =rectangles. b) Use of different colors suggest departmental divisions and time distinctions. C) Look for is a preponderance of decision points, as this occurrence may lead to unnecessary delays or adverse variations. *****The goal of process improvement is to reduce complications and unnecessary lags. Therefore, the interpretation of a flowchart always should be performed with an eye toward simplifying as much as possible.

Creating Box-whisker charts

"Creation"Box-whisker charts 1) A box-whisker chart features horizontal and vertical axes. a) Whiskers= bisected box (above each mark on the horizontal axis), that extends from the top and bottom; indicate the upper and lower limits according to special formulae. b) The upper and lower edges of the box indicate the dimensions of the first and third quartiles of data. c) Formula for lower limit: Lower Limit = Q1 - 1.5 × (Q3 - Q1). d) The upper limit is calculated with the formula Upper Limit = Q3 + 1.5 × (Q3 - Q1). 2) Typically, the box and whiskers should be based on quartiles; however, with normal distributions it is also possible to use metrics like mean and standard deviations. a) Eg-the box could define 2 standard deviations, and the whiskers could indicate 4 standard deviations. b) Some box-whisker charts will include dots beyond the upper and lower limits, indicating the greatest and least data points, respectively. c) Before creating and interpreting a box-whisker chart, establishing appropriate statistical controls is essential. "

Critical-to-schedule metrics

"Critical-to-schedule metrics 1) Cycle time/ Delivery time/ downtime/ order-processing time (MC critical-to-schedule metric) = the duration required for the completion of a defined process. 2) The improvement of critical-to-schedule issues begins with making the distinction between value-added & non-value added steps 3) Six Sigma professionals often use process efficiency and velocity numbers to assess value added relative to cycle time. 4) In most Six Sigma projects, cycle time is considered secondary to metrics related to quality or cost. *****If project focuses exclusively on reducing cycle time , quality compromised but useful to baseline quality and cost metrics and then work to reduce cycle time. so that quality will not be affected by any time-saving adjustments.

Desirability function & Response surface analysis

"Desirability function 1) Response surface analysis can attain optimal performance relative to multiple responses using overlaid contour plots or desirability functions. 2) Desirability function = analytical tool for finding the shared optimal point of multiple functions. a) Must define each optimal value as a minimum, maximum, or specified target. b) d = degree that each response obtains its optimum condition , and the composite of these responses is designated as D, which is calculated as the geometric mean of each response desirability function: D = (d1 × d2 × d3 × ↑ × dm)1/m. c) This composite value is maximized in each case so d values near the maximum of 1 suggest that all responses are within the desirable range at the same time. "

"EVOP strategy

"EVOP strategy 1) Analyze stage------>, Assess the significance of various process factors. 2) Improve stage-----> Used to set new process factor specifications. **Disadvantage= many repetitions required to isolate the effects of changes in factor levels & time consuming ( longer than other designed experiments), Advantage= Productive operations may continue even during implementation of EVOP strategy, unlike designed experiments. ***Also, evolutionary operations strategies assess the process as currently performed, which means that any significant factors overlooked in a designed experiment automatically will become part of the system."

Demi's 14 points of Management: Eliminating dependence on inspection and ceasing to award business based on price alone:

"Eliminating dependence on inspection and ceasing to award business based on price alone: a) Inspection should be eliminated as much as possible.---> Inspection programs ineffective because they spot mistakes only after they occur. ( prevention is better) b) Inspection programs costly --->Deming: "better to spend this money on improving production processes. c) The fourth of Deming's points for management is that business should not be awarded solely because of price---> better create a measure of total cost and use this measure to make their buying decisions and buy top quality goods & part for savings in long-run d) Best to have single supplier for each part/ product, / service, ---> enable us monitor quality and establish beneficial relationships. (relationships with suppliers create greater opportunities for customization and optimization.

Encouraging self-improvement and universal buy-in: Deming's

"Encouraging self-improvement and universal buy-in: 1) Deming's thirteenth point for management is the creation of self-improvement programs. ----> organizations thrive when employees feel that their personal goals align with the goals of the organization. b) In-house education opportunities. Employees always should believe that effort and self-improvement can lead to promotion within the organization. 2) The last of Deming's fourteen points for management is universal buy-in. ---> all members of should subscribe to the new vision of quality improvement. a) Managers are responsible for informing and encouraging their subordinates by telling that adherence will advance the employees as well as the organization's interests. b) Moreover, Deming felt that managers always should be alert for opportunities to increase employee participation in quality improvement."

Enumerative statistics--Process baseline estimation

"Enumerative statistics--Process baseline estimation 1)In process baseline estimation, enumerative statistics are useful for evaluating random samples from populations----esp to determine if two samples were drawn from a known population or even from the same population. 2) Can determine if samples are free of bias and represent the population, . 3) Can provide a confidence level, an assumed distribution, and a set of confidence intervals as well. 4) CI= Upper & lower value for a given statistic data (CI= 95% in most cases) Drawback= enumerative statistical methods get data from a static, unchanging population. In Six Sigma, often it is necessary to obtain information about dynamic processes. For this, analytical statistics are required."

The poka-yoke philosophy =Error-proofing strategies

"Error-proofing strategies/ poka-yoke 1) System for preventing defects in production processes. ----> emphasizes error prevention so processes should be analyzed rigorously and improved so mistakes occur less often. 2) Type s of error-proofing a) Auto-correction systems= reflexive adjustments to changing external conditions. b) Automatic shutdown, which stops a machine or process if conditions for an error exist. c) Warning signal (requires human operator) = alert that an error is about to occur. *****Error"proofing strategies can be expensive to implement and may slow processes at first, but in the long run these strategies will save money and time that would have been wasted on mistakes."

Deming's 14 management tips Establishing leadership and eliminating fear:

"Establishing leadership and eliminating fear: a) The seventh of Deming's fourteen points for management is to establish effective leaders----> . The purpose of management should be to set a proper example rather than to monitor the behavior and performance of subordinates. b) If training is effective and standards are clear, employees should not need constant supervision. Managers always should do their jobs with an eye toward improving quality. c) In addition, managers should have the authority to resolve issues they observe in "always should do their jobs with an eye toward improving quality. In addition, managers should have the authority to resolve issues they observe in the workplace. d) The eighth of Deming's points for management is that fear should be eliminated as a source of motivation. --->Deming observed that employees were terrorized with the constant threat of demotion or firing; ineffective way to inspire loyalty & quality e) Employees should feel comfortable enough in their jobs to consider possible innovations or to raise questions with their superiors. Likewise, managers should not be afraid of receiving comments or complaints from their subordinates.

"Evaluation of process failure modes

"Evaluation of process failure modes The initial goal during the improve stage of DMAIC is to optimize the process flow. 1) Failure Modes ----New operating conditions or process flows examined for possible sources of deviation after completing optimization 2)Brainstorming for solutions to conditions or flows identified as having deviations---> problems and potential solutions often are depicted on a process decision program chart, which looks a bit like a tree diagram. 3) FMEA is used to supplement process decision program chart 4) FMEA concludes with the generation of a risk priority number (RPN), indicating the influence of a particular failure on overall performance. The team then must develop mitigation or elimination strategies for the most significant potential failures. "

Focus groups

"Focus groups 1) Answers questions about the services or products of a business or provides insight into customer perceptions 2) Typical focus group format = the attendees asked to give opinions on product sample and best when when more specific and targeted information is obtained. In most cases, the members of a focus group will be compensated, but the leaders should make it clear that participants are to be honest. ***Members of a focus group may be customers or non-customers. ----> a random group of citizens or members of a target demographic.--- or people from demographics in which they do particularly well or particularly poorly (used to learn the secrets of their success or failure)

"Gauge repeatability and reproducibility analysis

"Gauge repeatability and reproducibility analysis 1) Assigns a numerical value to the errors caused by a particular measurement system. 2) MC in measure and control stages of DMAIC. a) Measure stage--->, this analysis 1) used to assess how much measurement error contributes to common- and special-cause variation when estimating process baseline. 2)) Also used in Measure stage to confirm gauge discrimination. 3) Control stage, ----> used to evaluate the accuracy of specific measurement systems.---> success depends on the collection of representative samples. ******* A measurement system cannot be evaluated correctly except by its performance with respect to typical samples. "

"Histograms

"Histograms 1) Presents data pictorially---. looks like a standard bar graph, except that each bar represents the total number of observations that lie within a range of ranked values. 2) Measure & analyze stages of DMAIC---. histograms are useful for identifying multiple distributions for applying a distribution to capability analysis. a) First created by placing all data in order from least to greatest. b) The number of bars in a histogram should equal the square root of the total set of data values. c) The width of each bar is the quotient of the range of data and the number of bars. d) Calculate the range of the data= (Max - Min) e) Find the number of data observations in each bar. --------->The horizontal axis = indicates the data values for each bar, and the vertical axis will indicate the number of observations. "

"Hypothesis testing

"Hypothesis testing 1) Establishes a degree of confidence and then compares a sample statistic against a historical value or another sample statistic. 2) Allows one to make statistical inferences about the characteristics of a population. 3) Useful for distinguishing the factors that contribute to variation in data. 4) Used during designed experiments and regression analysis. 5) Improve & Analyze stage,=compares the averages of improved processes with baseline estimates. (assume population constant and homogenous + random sampling).

"Individual-X and moving-range charts

"Individual-X and moving-range charts 1) control charts ( variables data) --- depicts changes in variation between consecutive subgroups over an interval. 2) An individual-X chart assesses the process location over an interval using a subgroup with a single observation. 3) Individual-X and moving-range charts are used during the measure, analyze, improve, and control stages of DMAIC. a) Measure stage---> these charts used to baseline processes by creating an objective measure of inherent process variations. b) Analyze stage---->, these charts are used to distinguish special and common causes for variation. c) Improve stage--->, these charts are used to confirm the results of improvements. Finally, in the control stage, these charts are used to confirm the stability of improved processes."

"Interpretation of U-chart

"Interpretation 1) Upper and lower control limits= indicate the boundaries of expected process behavior. 2) Common cause variation= points that lie within the control limits 3) Special cause variation= points outside the statistical control (eliminate)--->need to brainstorm possible sources of special-cause variation and then design experiments to test these hypotheses. 4) If no special-cause variation exists, then the process is stable enough to be predictable. *** In the ""meantime, it is possible to predict the behavior of the process once the special causes of variation removed. ---->To make such a prediction, simply ignore all out-of-control points when calculating the average and the upper and lower control limits."

Interpreting Histograms

"Interpretation Histograms 1) Histograms effectively pinpoint the process location and variation. 2) Histograms also indicate when data is symmetrical or bounded. a) A symmetrical distribution= data spread evenly about the histogram's center in an arrangement similar to a bell curve. b) If data unevenly distributed--->histogram is more skewed. i) data from multiple sources --> multiple peaks. 3) A histogram only indicates the performance of the process as it is being measured.---> Get broader output by extending sample period . 4) Histograms ignore sequence of the points --> must combine with measures of statistical process control.---->use a control chart to identify statistical control before fitting a distribution.

"Interpretation of EVAP

"Interpretation of EVAP 1) 1st step---> Examine if range chart is out of statistical control---> Can't trust EWMA's control limits a) Find the sources of special cause variation (if any points found outside statistical control) b) Compare EWMA to its control limits after reviewing rang chart c) Broader variation in the process make run tests and specifications are irrelevant in the interpretation of EWMA charts. b) For stable process, consider EWMA's chart capability relative to requirements may be useful.

Interpretation of Gantt charts

"Interpretation of Gantt charts 1) Gantt charts help businesses create the fastest-possible schedule for a process. 2) Key benefit =indicates areas within the process that can tolerate lag. ---- For every task, though, there will be some time by which it must be initiated and some time by which it must be finished. 3) To obtain a precise measure for the duration of a particular task.,use PERT analysis to gauge the probability of a task being completed within a certain amount of time."

"Interpretation of Np charts

"Interpretation of Np charts 1) An Np chart depicts a stable process when all of the data points lie between the upper and lower control limits. a) Any variation within these limits is due to common causes, but any variation that results in data points outside the control limits must be attributed to special causes. b) Data points that lie outside the control limits must be explained. 2) Typically, an Np chart is succeeded by experimentation aimed at diagnosing particular causes. a) Until these special causes of variation have been determined, the capability of the process can be predicted by eliminating the data points that lie outside the control. b) This elimination will remove the extreme data points from the formulae for average and control limits, which will provide a more reasonable estimate of future performance."

"Interpretation of Pareto Chart

"Interpretation of Pareto Chart 1) Isolate the categories that contribute the most to count or cost. 2) Cumulative line aspect of the chart= degree to which resolving problems with the most pressing issues would improve performance. a) Steep cumulative line is better because values concentrated in the left-most categories. b) Flat cumulative bar not helpful, because this arrangement indicates no problems as more important than the rest.---> aka the categories have been subdivided too much.--Solution= regroup into a few large categories for better result. 3) The traditional method for using a Pareto chart involves identifying the left-most categories that approximate 80% of the count or cost( most important areas).

"Interpretation of Run Tests

"Interpretation of Run Test: 1) Run tests either identify shifts in the process mean (1, 2, 3, 5, and 6) or provide information about sampling errors (4, 7, and 8). 2) Run test 4 --->suggests that samples have been taken from a multi-stream process, while run test 7 suggests that sample stratification exists. 3) Run test 8 suggests that samples have been taken from a mixture. 4) Run charts incorrectly may identify errors in the statistical control chart or problems in data collection. 5) Run tests alert the user to errors but not location of those errors. 6) For many of these tests, a good number of data points must pass through before the run test is activated. Therefore, when a run test is violated the user should go back and analyze the data to discover its location.

Interpreting the X-bar chart,

"Interpretation the X-bar chart 1) Examine the range chart before interpreting the X-bar chart. a) X-bar control limit useless if the range chart is out of control b) Range chart with few distinct values---> chart needs better resolution. c) Eliminate special cause variation---> by removing points outside of statistical control on the range chart d) After any points outside statistical control have been removed,---> Interpret the X-bar chart relative to control limits & run-test rules. e) To eliminate any points on the X-bar chart outside statistical control-----> identify the special causes of variation through brainstorming and data analysis, and the operating conditions are adjusted accordingly."

"Lean metric velocity

"Lean metric velocity 1)Rate at which value is added during a process phase. 2) Analyze stage ---can place cycle time improvement opportunities in order of importance. Formula for Velocity= (No. of Value added steps) / (process lead time). Process lead time (Little's law) : (No. items in the process ) / ( number of times the process is completed per hour) . .

"Matrix diagrams

"Matrix diagrams 1) Table in which the value in each cell indicates the strength of the relationship within multiple categories. 2) Matrix diagrams often are used during the define and improve stages of DMAIC. a) Define stage---> these diagrams are used to pick projects that will contribute to the achievement of organizational objectives and goals. + b) used in the define and improve stages to examine the similarity between customer desires and process metrics. The first step in the creation of a matrix diagram is to identify the items for comparison. Six Sigma teams often transfer the categories from some other chart, such as an affinity diagram. The next step is to use consensus decision-making rules to evaluate the relationships between each pair of values. In this subjective process, the team should perform carefully and according to an organized protocol.

"Merits of process simplification

"Merits of process simplification MC strategies ---> Simplify needlessly complex processes. 1) If same process is used to create a wide variety of products and services, the process probably is too complicated. 2) Best to limit the variation in deliverables so processes can be standardized more effectively. 3) Longer cycle times when customers have more choices, 4) Simplify processes by restricting sales to a single type of customer. 5) By focusing on a particular customer demographic, a business can preclude frequent requests for variation. ****Advantage of reducing process complexity---> need less inventory (If fewer modifications or customizations are performed during the creation of a product, fewer spare parts will be required as well)

"NGT

"NGT 1) Nominal group technique (NGT) is a system for ranking non-objective data and is used primarily to create consensus or agreement in groups. most commonly during the define and analyze stages of DMAIC. 2) Define stage--->used to simplify project loads by combining redundant projects and eliminating unnecessary projects. 3) Analyze stage-->, helps teams agree on which solutions should be pursued. 4) NGT Process: a) The first step in nominal group technique is to distribute paper to each team member. In the event of more than 35 options, each team member should receive eight pieces of paper; if there are 20 to 35 options, each member should receive six pieces of paper; and if there are less than 20 options, each member should receive four pieces of paper. Following discussion, each team member will select one option for each piece of paper he or she has received and indicate the option's rank on the paper. Then the papers are collected, and the weighted rankings are tabulated. The group then can focus on the most important and preferred options.

"Nonparametric tests on equality of means

"Nonparametric tests on equality of means a) Substitute for traditional hypothesis tests for the equality of two means. b) best used if assumptions associated with common statistical distributions cannot be met. c) Analyze stage: ---nonparametric tests are used to compare the means from samples with different conditions. d) Improve stage: ----Assesses whether process averages improved over baseline estimates after changes implementated Advantage= Statistical normality not needed. Disadvantage: larger sample size needed.

"Normal distributions

"Normal distributions 1) Used for continuous data that have neither an upper nor a lower boundary. 2) For a normal distribution, the average of a sample, X-bar, is calculated by adding all the measurements and dividing by the total number of measurements, N. 3) The standard deviation of N is calculated with the following formula:(see formula) 4) Need to calculate a z value so this particular standard deviation may be considered a standardized normal distribution with a standard deviation of 1 and a mean of zero. 5) The z value is calculated with (see formula) . Once this z value is obtained, it is possible to estimate the likelihood of being greater or less than a particular value for x. "

"Np charts

"Np charts 1) Control chart ( attributes data) measures the number of times a condition exists in each sample, when the condition may occur only once and the sample size is consistent. 2) Measure stage--->, attributes used to data to estimate the process baseline. ( usually Np chart combined with variable control chart for this purpose) 3) Improve stage---->, Np charts are used to determine the number of errors in process samples. but are useless most process have small number of errors

"Obtaining customer information

"Obtaining customer information 1) Areas businesses tend to ignore a) Fail to gather information about non-customers.-- > Attract non-customers by focusing on characteristics of people who choose not to buy their products b) Customers who switch to a competitor without notice or complaint ---> possibly due to disliking some aspect of their original purchase or special offer from competitor c) Product features that cause customers to defect. d) Reasons customers become dissatisfied with a product after its warranty period ends. ----> Information about customer desires and product failures lost when customers don't contact manufacturer

"PERT analysis

"PERT analysis 1) Used to estimate the time required by various processes. 2) PERT is an essential part of scheduling and cycle time analysis in which hard data on the duration of certain process steps may be unobtainable. 3) Uses probability techniques to determine a best guess about durations. 4) Used during the define, analyze, and improve stages of DMAIC. a) Define stage,----PERT analysis identifies the process steps with the most influence on the overall duration of the process. b) Analyze stage,--- PERT analysis determines the critical path of a process cycle. c) Improve stage, ----PERT analysis confirms improvements in the cycle time."

Process capability index

"Process capability index 1) A process capability index quantifies the ability of a process to meet the expectations of customers and other stakeholders. a) This index usually is converted into a standard deviation or estimate of defects per million opportunities. 2) Measure stage--->, process capability indices can create a baseline estimate for a controlled process. 3) Improve stage--->, a process capability index can confirm process improvements and assure that process is in statistical control. 4) Control stage--->, these indices are used to monitor processes, generally to confirm that they remain in a state of statistical control. ****For all of these uses, the process must be in statistical control so measures of process capability are valid. *****If the process has both upper and lower specifications, the statistics Cp and Cpk can be used. If only one of the specifications is known, Cpk must be used. "

Process of Hypothesis Testing

"Process of Hypothesis Testing: 1st step = State Ho: ( represents the value that the test aspires to prove). 2nd step= define (H1). ( covers all of the area excluded from the null hypothesis) 3rd Step = either set p-value (p-value more flexible for adjusting experiment) or selec significance level (α). *Type I error= significance level= chance of incorrectly rejecting a Ho: 4th Step = Collect samples, calculate the stats, and draw conclusion/

"Process of Response Surface Analysis & its 3 phases

"Process of Response surface analysis & its 3 phases Phase 0 - ( prerequisite phase)---Screening designs create a critical set of significant factors (+) a first-order regression model created. Phase 1 = steepest ascent methodology used to define the operating region at present and identify the direction of maximum response. ---Phase 0's first-order regression model helpful b/c its great scale ensures data points are affected by first-order effects. Phase 2 =application of ridge analysis and 2nd-order model to locate the optimal conditions at stationary points in a small region. a) In response surface analysis, a stationary point is defined as anywhere that the slope of the second-order response surface model is zero for each of the factors. b) Stationary points may be a maximum value, a minimum value, or a "mini-max" value, which is the highest or lowest point in a saddle curve. If the stationary point is significantly outside[...]"

"Project scope of project charter

"Project scope 1) Better for projects to have a limited scope to prevent team objectives from being disgruntled and resources being wasted when a project stretches out indefinitely. 2) In general, Six Sigma projects should take no more than four months. --->Longer projects = risk of losing key members. 3) If the team members have a vision that cannot be realized within four months,---> consider dividing grand project into multiple smaller projects. 4) If solving a particular problem depends on solving a collection of smaller problems---> conduct a series of smaller projects rather than grouping the entire problem in one project.

Promoting employee pride and emphasizing quality over production:

"Promoting employee pride and emphasizing quality over production: 1) The twelfth of Deming's fourteen points for management is divided into two parts. a) 1st part= employees will do a better job when they are given the chance to feel responsible for their work.+ businesses should look for opportunities to link employees with their work ( giving employees control over over multiple tasks in production or service provision. Deming felt that fear and anxiety in the workplace were obstacles to employee pride and therefore to quality. Employees always should be taught to emphasize quality over numerical production. b) 2nd part= the promotion of quality over production. ----> via abolition of merit systems, which reward success within the business rather than improvement of the business. ----->rewarding employees for the volume rather than the quality of their work set up bad incentives. In other words, this improper emphasis would encourage employees to make decisions that harm quality."

Quality function deployment

"Quality function deployment, ---- 1) system for ensuring that customer requirements are aligned with product and process requirements. 2) used in the analyze and improve stages of DMAIC. and to plot strategies for fulfilling customer requirements. Basic function of QFD= establish customer requirements, known as whats ( what needs to be achieved) , and design requirements, known as hows (how to achieve it) ***A number of tables are used to assess the importance and significance of various design requirements in the promotion of customer requirements. ***If a customer requirement, or what, has very few or no design requirements, or hows, then improving the process may be possible. ******When there are hows without whats, the process may contain some non-value-added activities that can be eliminated.

Deming's fourteen points for management Reducing interdepartmental barriers and reducing pressure on the workforce:

"Reducing interdepartmental barriers and reducing pressure on the workforce: a) Deming promoted interdepartmental communication as a method for reducing waste and improving quality in all areas.--->, employees trained in all other departments + managers ' familiarity with the tasks and responsibilities of the other departments. c) Deming observed that many businesses segmented their departments and evaluated performance as such, so departments in a failing business might appear to be thriving. + Deming favored linking all departments together. d) The tenth of Deming's fourteen points for management = don't pressure workers with short-term production goals---> better to embrace a holistic, long-term program for quality improvement. e) Constant exhortations and work targets, in Deming's view, contribute to anxiety and lower production among workers. Goals never should be set for individual employees, but instead for larger groups. "

Regression analysis

"Regression analysis 1) System for identifying when independent variables are influenced by one or more dependent variables. a) Measure stage: --- used to evaluate the degree to which a measurement system is linear. b) Analyze stage----used to explore the connections between metrics and process factors. c) Improve stage--- useful for confirming these connections after improvements have been implemented. ****A scatter diagram often is interpreted with a simple linear regression analysis. If more than one factor influences the value of the independent variable, then multiple regression is necessary.

Required calculations for X-bar

"Required calculations for X-bar 1) Must determine the appropriate subgroup size and sampling frequency before creating an X-bar chart 2) Subgroup average=The plotted statistic. 3) Centerline = grand average. 4) Calculate upper & lower control limits ****In these equations, is the grand average and is process sigma. Process sigma is calculated with either the subgroup range or the subgroup sigma statistic."

"Required calculations for range and sigma charts

"Required calculations for range and sigma charts The plotted statistic for a range chart is calculated as follows: Range= Max- Min 1) . Centerline= average range. 2) Calculate The upper & lower control limits (look up the formula) 3) In these equations, is the average range, is a function of n, and is process sigma. 4) In the calculation of process sigma, the average range is set as /d2. 5) In a sigma chart, the plotted statistic is the subgroup standard deviation, calculated , where are the observations in subgroup j, is the subgroup average for subgroup j, and n is the subgroup size. The centerline of the sigma chart is the average sigma. The upper control limits are calculated , and the lower control limits are calculated , where is the average sigma and is a function of n."

Responses to conflict: Accommodation

"Responses to conflict: Accommodation 1) Accommodation is the suppression of one's own interests in order to serve the interests of another person may be served. 2) Accommodation can be a valuable tool in conflict resolution, as long as it does not engender resentment. 3) One should accommodate if he or she a) recognizes personal error. b) If the issue is clearly much more important to the other person. c) When an issue is trivial to one person in a disagreement, d) when the group is fragile and more conflict could be very damaging. **** Drawback = Of course, excessive accommodation can inspire some members to dominate the team, and in some cases accommodation will lead the team to accept the incorrect solution."

Responses to conflict:

"Responses to conflict: Collaboration 1) Collaboration is the response to conflict wherein team members work together to find a solution and is effective when: a) both parties have good points. b) When members of a team have different areas of expertise, and so it is a good idea to combine their perspectives. c) When the conflicting parties have some personal disharmony; collaborating may help them work through these problems. . d) when cooperation will increase buy-in from other team members and stakeholders. ***Time-consuming b/c makes room for too many viewpointsOne problem with collaboration is that it can take a long time. For this reason, it should not be used to resolve unimportant conflicts."

Responses to conflict:

"Responses to conflict: Compromise 1) Compromise is a useful response to conflict: a) when maintaining team harmony is more important than coming up with the best possible solution. b)) Not the right approach to ending crucial conflicts. c) when two important members of the group are diametrically opposed on a key issue.( If neither of these parties will give ground, a compromise may be the only way for the group to move forward) d) When no time exists for a lengthy debate or collaboration. Drawback of Compromises---can leave the participants with lingering regrets, may disillusion team members, may produce mish-mash solutions that fail in the long run due to a lack of clear, unified approach,

Ridge analysis techniques in phase 2

"Ridge analysis techniques in phase 2 1) Conduct a first-order model near-maximum at the end of phase 1 then estimate "lack of fit" using several redundant runs a) Insignificant " lack of fit"---> new path of steepest ascent may be found by adjusting the intervals, starting point, or direction. b) Significant "lack of fit"---> Curvature present so the point likely is close to a maximum, minimum, min-max. 2) Ridge analysis = At this juncture ( defining lack of fit as significant or not) is time to begin phase 2 of the response surface analysis: ridge analysis. a) In this process, a second-order regression model is generated with a central composite design near the optimum. 3) Create response surface and contour plots for each two factors. Then, identify the stationary point in the response surface and contour plots. 4) Predict the response at the optimum using the second-order regression model. Finally, verify the model by gathering new data in the region around the optimum.

"Sampling methods

"Sampling methods 1) Simple random sampling occurs when every unit in the population has the same chance of being selected. 2) Stratified Sampling= When the population is divided into groups and a sample is taken from each of the groups. 3) Systematic sampling program=, some method exists regarding the selection of samples.( eg. every third unit might be selected) . 4) Cluster sampling =a representative group is selected out of the population, and then a random sample is drawn from that group. This method depends on the ability of the sampler to select a truly representative group. 5) Judgment sampling programs rely on expert opinions in their selection of a sample group. This mode of sampling is appropriate when the samples must have particular characteristics not common to every member of the population. "

"Simulations

"Simulations 1) (Improve stage)--cheap way to predict the likelihoods and effects of variable levels or confirms the solutions derived from analysis and can indicate the possible interactions between variables. 2) Uses of Simulations a) To evaluate changing situations because the dynamics of the change can be built into the simulation. b) To create Frequency Distribution---> Simulations use process model and a rough probability distribution for every variable to create frequency distribution. c) Simulations must be confirmed against hard data before or after they are run. (not fool-proof method of research but cost-effective)

Statistical Distributions

"Statistical Distributions 1) Statistical distributions allow Six Sigma teams to make performance assumptions with a minimum of supporting data. 2) Discrete data --> use the binomial and Poisson distributions. 3) Continuous data (derived from measurement; indicates not just the number of events, but also the extent of each event. )= apply the normal, exponential, Johnson, and Pearson distributions are applicable. 4) Discrete data is obtained from processes that can be counted. ( eg. collections of error data usually will consist of the number of times the error occurred) . 5) Distributions may be useful in the measure, analyze, improve, and control stages of DMAIC. 6) The primary utility of these tools involves determining the characteristics, most notably"the sigma level, of processes and resources. ---Six Sigma professionals also may use these tools to create random data for use during process modeling.

"Statistical process control charts

"Statistical process control charts 1) Past performance used to predict future variations & also can confirm the stability of a process. a) Measure stage--->, they convert common-cause variation into objective data that can be used to baseline processes. Also, statistical control charts can be used during the measure stage to evaluate the repeatability and reproducibility of a measurement system. b) Analyze stage----> used to distinguish common and special causes of variation. c) Improve stage---->can be used to confirm the success of process improvements. d) Control stage of DMAIC, statistical control charts can be used to confirm the stability of adjusted processes and to confirm that the adjustments have improved the process.

"Team leader

"Team leader 1) Establishes & maintaines protocol, the set of rules by which the team will operate. 2) Continuously realigning the team's efforts with the original goals and objectives by reviewing first principles to stay on track. 3) Resolving conflict 4) Planning and administrating the team's meetings. 4) Keeps stakeholders and project sponsors informed of group's progress. 5) Must continuously to improve the team's performance, first by helping members brainstorm and then by refining solutions through the use of data analysis."

10. Which parameter of a statistical distribution relates to the sharpness of its peak? a. Central tendency b. Kurtosis c. Skewness d. Standard deviation"

10. B: Kurtosis. Kurtosis is the parameter of a statistical distribution related to the sharpness of the peak. In a normal distribution, where the points resemble the standard bell curve, the kurtosis value is one. If the peak is sharper, the kurtosis value will be higher than one; if the peak is less severe; the kurtosis value will be less than one. Central tendency is the general trend of the data: in an asymmetrical distribution, the median is roughly equivalent to the central tendency, while in an asymmetrical distribution, the mean is a better marker. Skewness is basically the difference between the mean and the mode of a data set. The mode of the data set is the value that appears most often. Finally, the standard deviation of the data set is the average amount of variation from the mean.

24. Which type of chart is appropriate when sample size is variable and each sample may contain more than one instance of the targeted condition? a. P chart b. Autocorrelation chart c. U chart d. X-bar chart

24. C: U chart. A U chart is appropriate when sample size is variable and each sample may contain more than one instance of the targeted condition. These are control charts most appropriate for handling attributes data. A P chart, on the other hand, is better for measuring the percentage of samples with a particular characteristic when sample size is variable and the characteristic will either be present or absent. An autocorrelation chart indicates the relationships between various factors in the process. An X-bar chart, finally, is a control chart for variables data, in which the subgroup averages are assessed to determine the process location variation over time.

25. From whose perspective is value defined in the lean methodology? a. Customer b. Chief executive c. Entry-level employee d. Competitor

25. A: Customer. In the lean methodology, value is always defined from the perspective of the customer. This was a radical shift in perspective when it was first introduced. Most businesses assessed value from the perspective of executives or in-house experts. In lean methodology, value is defined as the qualities or characteristics for which a customer is willing to compensate the business.

26. On an X-bar chart, what variable is always represented on the x-axis? a. Variations b. Errors c. Length d. Time

26. D: Time. On an X-bar chart, time is always represented on the x-axis. X-bar charts are control charts for variables data. The chart should resemble a chronological model of the process: as the bars move away from the y-axis, they represent the advancement of time. In order for an X-bar chart to be possible, any variation must be assigned a time value. Outlying values on the X-bar chart indicate the presence of special-cause variation.

27. Which of the following diagrams indicates the critical path of a process? a. Gantt chart b. Work breakdown structure c. Value stream analysis d. Matrix diagram

27. A: Gantt chart. A Gantt chart indicates the critical path of a process. The critical path is the sequence of steps that have a direct bearing on the overall length of the process. Some steps can be delayed without elongating the overall duration of the process: these steps are not considered to be on the critical path. A work breakdown structure depicts the organization of a process. To create a work breakdown structure, one isolates the various components of a problem and then considers the various contingencies associated with each component. A value stream analysis determines the elements of a process that add value to the finished product. These elements are targeted for special attention. Finally, a matrix diagram depicts the relative strengths of the relationships between the items in different groups. A matrix diagram might indicate causal relationships between various factors in a process or might simply indicate which of the factors are related.

28. Which of the following is a disadvantage of higher-order multiple regression models? a. These models do a poor job of defining the area around a stationary point. b. Comprehensive and detailed experiments must be performed on the main effects. c. These models rarely have clear peaks and valleys. d. Small regions are difficult to perceive. "

28. B: Comprehensive and detailed experiments must be performed on the main effects. One disadvantage of higher-order multiple regression models is that comprehensive and detailed experiments must be performed on the main effects. Otherwise, it will not be wise to assume that the results of the higher-order multiple regression models are useful or accurate. However, higher-order multiple regression models have a number of advantages. For one thing, they are excellent at clearly defining the area around a stationary point. They typically have well-defined peaks and valleys, which facilitates analysis. Also, they are very effective at mapping small regions in the process, so they are able to achieve a high level of precision and detail.

Design of Experimentation Treatment

3. Treatment a. Single level assigned to a single factor or experimental unit during the experiment b. Specific combination of factor levels whose effect is to be compared with other treatments c. Treatment combination= series of levels for all factors in the experiment

32. Which type of Pareto chart would be the least useful? a. One in which the bars represent costs b. One in which the cumulative percentage line is steep c. One in which all the bars are roughly the same height d. One in which the bars on the left are significantly taller than the bars on the right"

32. C: One in which all the bars are roughly the same height. The least useful type of Pareto chart would be one in which all the bars are roughly the same height. A Pareto chart is used to identify the most important and urgent problems in a process. It is based on the Pareto principle, which is basically that a process can be improved dramatically through attention to the few most important problems. It is essential that the bars on a Pareto chart represent fungible values, like cost or count. A Pareto chart will not be useful if it is based on percentages or rates. The most useful Pareto charts have several large bars on the left, indicating problems that are significantly more important than others. Similarly, a steeply ascending line on a Pareto chart indicates that a few of the identified factors are very important, and therefore that the chart will be useful. If all of the bars on a Pareto chart are roughly the same height, no one factor is more important than another, meaning it will be impossible to generate an unusual amount of benefit by solving a single problem."

47. In a histogram, the number of bars is equal to: a. The square root of the total number of data values b. The square root of the range of data c. The range of data divided by the total number of data values d. The number of data observations"

47. A: The square root of the total number of data values. In a histogram, the number of bars is equal to the square root of the total number of data values. Histograms look like bar graphs, but the bars on a histogram represent the number of observations that fall within a particular range. Histograms are often used to locate multiple distributions or apply a distribution to capability analysis. The width of each bar in a histogram is calculated by dividing the range of data by the number of bars. The range of data is determined by subtracting the minimum data value from the maximum data value. On a histogram, the x-axis represents the data values of each bar, and the y-axis indicates the number of observations."

49. In a contour plot, what is indicated by a series of evenly spaced parallel lines? a. First-order main effects b. Second-order main effects c. Interactions between two responses d. Interactions between three responses

49. A: First-order main effects. In a contour plot, a series of evenly spaced parallel lines indicates first-order main effects. Interactions between responses are indicated by curving contour lines. Very few contour plots consist of evenly spaced parallel lines. The most general use of contour plots is during the improve stage of DMAIC, when they are used in response surface analysis to predict minimum and maximum response values for specific data ranges."

5 Stages of the life cycle of customer

5 Stages of the life cycle of customer: 1) Acquisition--Converting prospect to customer (high costs) 2) Retentionof existing customers (1/4 of the cost of getting new customers) 3) Attrition (Fading customer enthusiasm with growth of dissatisfaction) 4) Defection= losing customer 5) Re-acquisition--getting back old customer at even higher cost.

5 activities of the new product development

5 activities of the new product development: ***Multi-functional team activities involving all departments needed for effectiveness and speed to market (obsolete is the single function teams passing the product) 1) Concept Study= uncovers the unknowns about market/technology/ manufacturing process 2) Feasibility investigations---need to determine the limitation of the concept and find out if unknowns are resolvable and if new research improves project 3) New product Development---Includes specifications, customer needs, target markets, multi-functional teams, and key stage gates 4) Maintenance--post-delivery activities of product development 5) Continuous learning

5 types of matrix diagrams

5 types of matrix diagrams: 1) L-type--L-shaped matrix relates two groups of items to each other (or one group to itself). 2) T-type--T-shaped matrix relates three groups of items: groups B and C are each related to A. Groups B and C are not related to each other. 3) X-type ---X-shaped matrix relates four groups of items. Each group is related to two others in a circular fashion. 4) Y-Type----Y-shaped matrix relates three groups of items. Each group is related to the other two in a circular fashion. 5) C-type ( 3D)----C-shaped matrix relates three groups of items all together simultaneously, in 3-D.

5S (Housekeeping), Workplace Organization

5S (Housekeeping), Workplace Organization 1) Indicative of commitment of senior management to workplace organization, lean manufacturing, and elimination of waste 2) Mandates that resources be provided in required location and support work activities. 3) 5S a) Sort-----separate all that is unneeded and eliminate b) Straighten---put things in their place c) Scrub or shine d) Standardize---make cleaning and checking routine e) Sustain--improve on the first 4 steps

5why method---

5why method----- question-asking technique that helps reach the root cause of a business or manufacturing problem used for exploring the root cause of a business or manufacturing problem. It is based on the understanding that by the fifth 'why', major aspects of the root cause of the problem will already be revealed, such as what may be causing low productivity in a waste management facility.

6 Sources of Variability

6 Sources of Variability 1) Lot-to-Lot Variability ---long-term variation due to difference between process average and the variation. ----> Control charting aims to reduce this type of variation. 2) Stream-to-Stream Variation ---->Distribution of products flowing from different streams (machines, tanks, ect) produce variability greater than individual streams 3) Time-to-Time variability 4) With-in Piece Variability---> Differences found Physical inspection measurements at many different points on a given unit ----> If there is significant positional variation, then must change the material or machinery. 5) Piece-to-Piece Variability ---variation of a single production unit due to inherent error of measurement by both human and equipment components. 6) Inherent Process Variation---> instant reproducibility of the machine and represents the ultimate capability of operating under virtual laboratory conditions.

8 Advantages of DOE

8 Advantages of DOE: 1) Economical and less interruptive to normal opeactions b/c allows simultaneaous evaluation of multiple factors 2) Noise factors ---uncontrollable factors that influence the output ----requires the use of other controllable input factors to make output insensitive to noise factors. 3) Detailed statistical knowledge not always needed to benefit from standard planned experimentation . 4) Fewer experiments needed to prioritize the important factors. 5) Since designs are balance, there is confidence in the conclusions drawn.----> factors are usually be set at the optimum levels for verification. 6) Results will aid in identifying any factors that were overlooked. 7) Precise statistical analysis can be run using standard computer programs 8) Cost Savings---> quality and reliability improved with minimal trial costs.

8 Common Errors in performing Gage R&R:

8 Common Errors in performing Gage R&R: a. In process control situations, not selecting samples covering tolerance spread i. Best to either pick samples outside the specification limits or random samples from the process to study. b. Non-random samples measured-→ Results in Bias in the repetitive measurement trials. c. Use of untrained appraisers or process-unrelated employees in the experiment due to a lack of appraisers.-→ Results in inflated reproducibility errors. d. Samples altered during the study. e. Experimenters absent during the R&R study i. Assigning appraisers or remotely studying the process. ii. Important for experimenter to be present during the human interaction portion of the process (loading, setting, aligning, unloading, and ect) iii. Risk of invalidating results and having to start over f. Publishing the results with appraiser's names-→ risk of unhealthy comparisons between appraisers or appraisers feel uncomfortable and refuse to work for future studies; Best to represent appraisers as ABC g. Assuming that the Gage R&R results are valid forever GR&R results must be periodically validated as gage calibration is done with regular frequency. ii. Changes in measurement methods, appraisers, setting, or software/ firmware of equipment may occur. h. Assuming that Gage R&R performed on a specific piece of equipment is the same for all other equipment of that kind.

8 Disadvantages of Computer/ Automated Project Managment Methods

8 Disadvantages of Computer/ Automated Project Management Methods 1) Learning curve for software user 2) Costly 3) Data entry & updates time-consuming 4) Computer's data inaccuracies lead to incorrect decisions 5) Manager's focusing on computer may lose sight of the project 6) Software options for specific projects not available 7) Limitation of environments for computer use 8) Unusual events or task relationships may not fit the software model

8 reasons why six sigma works?

8 reasons why six sigma works? 1. Bottom line results 2. Senior management involved 3. Disciplined approac 4. Short project completion time (3 to 6 months) 5. Clearly defined measures of sucess 6. Infrastructure of trained individuals (green & black belts) 7. Customer and process focused. 8. Sound statistical approach

Image displayed next to each question Critical equipment breakdown is identified as a risk in the project at the start of the project and risk probability is 4, risk severity is 2, risk cost is $12. Which of the following risk priority number is correct? 8 3 6 96

8= The risk priority number = equal to the Risk probability multiplied by Risk severity. Incorrect calculation (Risk probability multiplied by Risk severity multiplied by Risk cost)= 96 Incorrect calculation (Risk cost / (Risk probability + Risk severity)= 6 Incorrect calculation (Risk probability + Risk severity)/2= 3

9 Disadvantages of Project Management

9 Disadvantages of Project Management 1) May not be portable depending on format 2) Only one site has project status information 3) Can't display large/ complex projects 4) Possible to overlook potential problem activities 5) Costs may not be tracked 6) Erasing status board--> permanent data loss. 7) To get interim status report, must be done manually 8) Difficulty doing project analysis 9) No flexibility if reporting format change or if new monitoring control identified.

An automobile manufacturer wants to collect field intelligence to address and improve customer satisfaction, customer retention, and customer loyalty. It has 3,000 dealers nationwide. Which of the following will provide the most information at optimum cost? Develop a statistical sample size and collect the field intelligence from dealers that are well-distributed geographically. Request field intelligence information from all the dealers spread throughout the country. Create detailed survey forms and send it to customers who have purchased their cars in the last year. Use the data from publications like consumer reports and develop strategy to attract and retain customers.

ANSWER=Developing a statistical sample size and collecting the field intelligence from dealers that are well-distributed geographically will provide prompt feedback at a much lower cost and will facilitate making the right decision. Requesting field intelligence from 3,000 dealers will be cost prohibitive and the quality of the data might be poor. Sending a detailed survey to recent consumers will not represent the customers who have purchased cars more than a year ago.

In a PERT chart, activities which may be done in parallel are indicated by which of the following? Colored line between activity boxes Identical predecessor step Square activity boxes Dashed arrow between activity boxes

ANSWER=Parallel activities will have the same predecessor step Colored line between activity boxes doesn't indicate parallel activities. It indicates Critical Path Square activity boxes don't indicate parallel activities. They indicate events. Events are nodes that separate tasks. Dashed arrow between activity boxes doesn't indicate parallel activities. It indicates a dummy activity or extra event. A dummy is an arrow drawn with dashed lines used to separate tasks that would otherwise start and stop with the same events or to show logical sequence. Dummies are not real tasks.

According to Shingo & Sharma, standard charts should include 5 key elements

According to Shingo & Sharma, standard charts should include 5 key elements: 1) Capacity Chart by parts ---> chart on order of processes, process names, numbers, basic times, tooling needs 2) Standard task combination sheets---> Order of operations for ajob 3) Task manuals----> detailed instructions on tool changes, setup changes or parts assembly. 4) Standard operating sheets----> details from task instruction manual with information on equipment layouts, cycle times, order of operations, standard on hand stock, net work times, safety checks, and quality checks 5) Task instruction manuals---> training manual for training workers and contains equipment layouts, quality check methods, operational procedures and standard stock required

What must be done if a failure mode is found during the first run of the FMEA?

Action Plan a. Improvement of current controls or reduction in the frequency of occurrence of cause. b. Product or process redesign to reduce the severity. c. Assignment of completion dates for tasks. 11) Take Action a. Key step where FMEAs fall apart due to lack of management support, conflicting priorities, lack of resources, and lack of team leaderships b. Best to conduct prototype to pilot test process. 12) Recalculate RPN a. Use objective evidence (customer feedback, reliability tests, warranty return rate, yield tracking) to assess the score 13) Review & update new risks a. Evaluate customer feedback, warranty analysis, internal nonconformance reports, ongoing reliability test responds to find new risks and update FMEA

5 Advantages of PERT

Advantages of PERT 1) Planning required to identify the task information for the network and the critical path analysis can identify inter-relationships b/w tasks and problem areas. 2) Probability of achieving the project deadline can be determines, and by developing alternative plans, the likelihood of meeting the completion data is improved. 3) Changes in project can be evaluated to determine their effects. 4) large amount of project data can be organized and presented in a diagram for use in decision-making 5) PERT can be on unique, non-repetitive projects.

Advantages of the SIPOC Diagram:

Advantages of the SIPOC Diagram: a. Displays a cross-functional set of activities in a simple diagram b. Uses a framework applicable to process of all sizes—even the entire organization. c. Helps maintain a "big picture" perspective, to which additional detail can be added.

Affinity Diagrams

Affinity Diagrams i. Affinity diagrams are business tools used to organize ideas, especially in the brainstorming and development stages of a project. The goal of these diagrams is to organize ideas based on similarity and simplify them for management and analysis. ii. Tool used to analyze customer qualitative data & feedback. iii. Organizes ideas when issues are too large & complex, overwhelming facts, or when group consensus needed.

Affinity Diagrams / KJ Method

Affinity Diagrams/ KJ Method 1) Techniques that individual/ team can use for problem-solving, especially for new or complex problems 2) Similar to mind-mapping in that it generates ideas that link with other ideas. 3) Can be used int he planning states of a problem to organize ideas and information.

Affinity Diagram

Affinity diagrams 1) Organize a collection of issues, problems, or ideas so they may be utilized effectively. 2) The diagram is so called because it identifies the affinities, or similarities, between seemingly disparate ideas or problems. 3) Helps organizations to discover unforeseen relationships between variables. 4) Used at the beginning of a process to help team to gain a primary understanding of the problem. 5) Helps build a consensus when members of a team have divergent interests and knowledge bases. 6) However, affinity diagrams may be used in every phase of the DMAIC process. One common result of an affinity diagram is the identification and elimination of redundancies in thinking."

One member of a team frequently displays highly disruptive behavior, despite attempts by the leaders and facilitator to get this person to follow the team's ground rules. The team's performance has suffered as a result. As a team leader, what action would you recommend? A) Do not invite the person to future meetings B) Ask for guidance from the team sponsor C) Remove the person from the team D) Do not acknowledge the disruptive behavior and continue to lead the meetings

Although a last resort, removing the person from the team is probably the best action for the team leader to take, with the full support of the sponsor.

Which of the following statements describes a reason why companies find Six Sigma valuable? A. Six Sigma provides a customized improvement methodology that can be applied anywhere throughout the business. B. Six Sigma fundamentally changes how a company improves. C. Six Sigma makes no overall changes to the quality of the company's products and services. D. Six Sigma is a long-term approach to improvement that does not fundamentally change the way a company measures success.

Answer= D= long-term approach to improvement that does not fundamentally change the way a company measures success. Six Sigma focuses on achieving financial targets or reducing costs within 12 months, which is not necessarily a change in how a company improves the quality of its products or services. Additionally, simply changing an approach with no regard for results is not part of the Six Sigma value proposition. Six Sigma is a standard (not customized) improvement methodology that can be applied to many businesses and throughout an individual business. Six Sigma projects almost always target quality improvement.

Armand Feigenbaum Current employment

Armand Feigenbaum--Current employment 1. Founder of General Systems ---provides quality management & strategic planning worldwide 2. States that USA should strive to be strong on the marketplace domestically via proper design, production, selling & servicing to reach marketplace supermacy.

Customer satisfaction is primarily driven by which of the following? A) Delivering quality and product features that exceed those provided by competitors B) Capturing customer feedback through multiple methods (e.g., surveys, complaints, etc.) C) Identifying both the stated and unstated needs of customer segments D) The ability of a product or service to meet the needs of a user or purchaser

As with identifying needs, capturing customer feedback is important. However, unless the feedback is acted upon in a meaningful way, it will not promote customer satisfaction. Quality and product features are important, but there are other considerations such as providing unnecessary features, value for price, etc. The ability of a product or service to meet the needs of a user or purchaser is the correct answer.

Basic purposes of communication

Basic purposes of communication 1) Influence the work of peers or employees 2) Inform employees of necessary job performance information 3) to control the organization's progress toward the objectives 4) To inspire employees through displays of values, attitudes, and modeling

A process in which a company measures its performance against that of other companies is referred to as: A) Reengineering B) Shewhart Method C) Ishikawa Diagramming D) Benchmarking

Benchmarking is the process of measuring products, services, and processes against those of organizations known to be leaders in one or more aspects of their operations. Reengineering is defined as a breakthrough approach involving the restructuring of an entire organization and its processes.

Benefits of Pull System

Benefits of Pull System: 1. Cycle times decrease in areas of: a) Sales Delivery b)Concept to launch c) Raw material to customer 2. Reduced Finished Inventories 3. Reduced Work-in Progress (WIP) 4. Pricing is stabilized 5. Customer stabilizes their ordering

FMEA and Process Capability

Best to perform comprehensive process FMEA to identify the characteristics that merits a full process capability study + id the characteristics that need statistical process control + Control plan for detailed SPC planning. 5) Specification and tolerance are obtained from engineering drawings and customer contracts. a. Examples of specifications and tolerance (restaurant wait times, next day delivery) b. Unlike manufacturing, service providers required study of the market and customer expectations through surveys and informal interviews and identify the specifications and tolerances

Black Belt --Duties in a Project

Black Belt --Duties in a Project i. NPV, ROI, and payback calculation on projects ii. 4-6 project per year iii. Supervision of projects iv. Project selection with input from process owners, Master Black Belt, and champions v. Address stagnation/ barriers to project success. vi. Presentation of project progress to management.

Black Belts

Black Belts 1. Name from Karate; Black belts are coaches or trainers 2. Six sigma Black belts--demonstrated skill in implementation & principles/ practices. via significant positive financial impact & customer benefits on multiple projects. a) Team Leaders who measure/analyze/ improve key process for customer satisfaction, growth, or productivity b) Internal consultants for multiple teams c) Instructors for statistics classes and problem solving

Blocking

Blocking When structuring fractional factorial experimental test trials, blocking is used to account for variables that the experimenter wishes to avoid. A block may be a dummy factor which doesn't interact with the real factors. A nuisance factor is used as a blocking factor if every level of the primary factor occurs the same number of times with each level of the nuisance factor. The analysis of the experiment will focus on the effect of varying levels of the primary factor within each block of the experiment. Blocking used for nuisance factors that can be controlled---->When we can control nuisance factors, an important technique known as blocking can be used to reduce or eliminate the contribution to experimental error contributed by nuisance factors. The basic concept is to create homogeneous blocks in which the nuisance factors are held constant and the factor of interest is allowed to vary. Within blocks, it is possible to assess the effect of different levels of the factor of interest without having to worry about variations due to changes of the block factors, which are accounted for in the analysis.

Box- and -Whisker Plots (Box Plots)

Box- and -Whisker Plots (Box Plots) ---John W. Tukey 1) Most simple and useful way to summarize data-----> 5 number summary of the data with a central line representing the median 2) Upper and lower quartiles of the data define the end of the box ---------> min and max data points are the end of the line "whiskers" 3) Notch Widths are calculated so that if 2 median notches do not overlap------->means are different with 5% significance. 4) Variable width notches are proportional to the log of the sample size. 5) Outliners (Asterricks)------Points that are more than 1.5 times the inter-quartile distance from each quartile.

Box-Behnken designs

Box-Behnken designs Box-Behnken designs are experimental designs for response surface methodology, devised by George E. P. Box and Donald Behnken in 1960, to achieve the following goals: Each factor, or independent variable, is placed at one of three equally spaced values, usually coded as -1, 0, +1. (At least three levels are needed for the following goal.) The design should be sufficient to fit a quadratic model, that is, one containing squared terms and products of two factors. The ratio of the number of experimental points to the number of coefficients in the quadratic model should be reasonable (in fact, their designs kept it in the range of 1.5 to 2.6). The estimation variance should more or less depend only on the distance from the centre (this is achieved exactly for the designs with 4 and 7 factors), and should not vary too much inside the smallest (hyper)cube containing the experimental points. (See "rotatability" in "Comparisons of response surface designs".) The design with 7 factors was found first while looking for a design having the desired property concerning estimation variance, and then similar designs were found for other numbers of factors. Each design can be thought of as a combination of a two-level (full or fractional) factorial design with an incomplete block design. In each block, a certain number of factors are put through all combinations for the factorial design, while the other factors are kept at the central values. For instance, the Box-Behnken design for 3 factors involves three blocks, in each of which 2 factors are varied through the 4 possible combinations of high and low. It is necessary to include centre points as well (in which all factors are at their central values).

Business Level Metrics

Business Level Metrics 1. Requires Balance Scorecard 2. Typically financial (external) and operational (internal) summaries of management and shareholders that are reported monthly/quarterly/ annually 3. Traditional end-of-period cutoff reports and comparing year-to-date totals to the same period last year are not enough for 6 sigma projects

Business Process Mapping

Business Process Mapping produces a business process illustration. 1. Key Takeaway: Business Process Mapping (BPM) is an analytical tool designed to map out activities that are involved in business processes. The goal of BPM is to determine the requirements of each process in terms of people, products and information. A business process illustration is obtained from the mapping which defines what each process does.

After obtaining the mean for all x-values and the mean for all y-values when calculating correlation coefficient, what is the next step? Sum all the values and divide the sum by Sx x Sy Calculate the standard deviation for all x-values and the standard deviation for all y-values Calculate x- x-bar, and y- y-bar for each pair (x.y) and then multiply those values together Sum all the values and divide the sum by Sy x Sx

Calculating the standard deviation for all x-values and the standard deviation for all y-values is the correct 2nd step

A Six Sigma Green Belt is evaluating the correlation between two variables. She has heard that it is critical to be aware that correlation does not always imply causation. The best way to determine if there is a causation is to: Conduct an observational study or a well-designed experiment Plot the variables on a scatter plot Calculate the correlation coefficient, and if over 1, it is causation Make the independent variable the X-Variable

Causation cannot be detected from a correlation coefficient.The best way to be certain that causation is part of correlation is to conduct an observational study or well-designed experiment. The best way to be certain that causation is part of correlation is to conduct an observational study or well-designed experiment.

Check sheet vs. check list:

Check sheet vs. check list: People sometimes confuse a check sheet with a check list. The list we use for groceries and the report you get from the auto repair shop with things checked off after service (oil, filter, tire pressure, tread, etc.) are examples of a check list. The following table highlights some key differences between a check list and a check sheet. 1) Check Sheet a) A tally sheet to collect data on frequency of occurrence b) Custom designed by user c) One of seven quality tools d) E.g.: Check sheet to document reasons for interruptions in OR ***When to use the check sheet:----> when the data can be observed and collected repeatedly by either the same person or the same location. It is also an effective tool when collecting data on frequency and identifying patterns of events, problems, defects, and defect location, and for identifying defect causes. Types of check sheets: Commonly used check sheets are tabular check sheets or tally sheets, location check sheets and graphical or distribution check sheets. 2) Check List a) A tool used to ensure all important steps or actions have been taken b) Often a standard form is used c) Not one of the seven quality tools d) E.g.: All items in case cart are present before surgery in OR

Comment cards

Comment cards and formal surveys : 1) Information about how customers view the business' products and/or services. 2) The most basic metric obtained in these ways is , how customer satisfaction is affected by various characteristics. 3) Comment cards are left in places where customers will be, and customers voluntarily complete them. ***For this reason, very satisfied and very dissatisfied customers are overrepresented, because they are more likely to be motivated enough to fill out a card.

Conflict Resolution Matrix

Conflict Resolution Matrix 1) Avoiding a) Lose-Lose ---unassertive/ uncooperative = individual withdraws from the situation b) Best for less important issues or when the potential damage of the conflict outweighs the benefits of the goal 2) Accommodating a) Win-Lose ---unassertive/ cooperative = individual yields to the wishes of others b) Best used when one party is wrong or when the issue is more important to the others than it is to you. 3) Competing a) Lose- Win ---assertive/ uncooperative = the individual tries to sin, even at the expense of others 4) Collaboration---- a) Win-Win --- assertive/ cooperative= individual wants things done their way, but is willing to explore solutions which satisfy the other person's needs as well b) Used when two opponents have equal power and the goal are not worth the effort of disruption of mutually exclusive solution.

Contrasting Theory of Constraints and Lean Manufacturing

Contrasting Theory of Constraints and Lean Manufacturing Both methodologies have a strong customer focus and are capable of transforming companies to be faster, stronger, and more agile. Theory of Constraints Lean Manufacturing 1) Objective a) TOC =Increase throughput. b) LEAN =Eliminate Waste. 2) Focus A) TOC--Singular focus on the constraint (until it is no longer the constraint; B) LEAN = Broad focus on the elimination of waste from the manufacturing process. 3) Result A) TOC = Increased manufacturing capacity. B) LEAN =Reduced manufacturing cost. 4) Inventory Maintain--- TOC = sufficient inventory to maximize throughput at the constrainT; LEAN -Eliminate virtually all inventory. 5) Line Balancing TOC= Create imbalance to maximize throughput at the constraint. LEAN=Create balance to eliminate waste (excess capacity). 6) Pacing TOC = Constraint sets the pace (Drum-Buffer-Rope). LEAN = Customer sets the pace (Takt Time). ****From the perspective of the Theory of Constraints, it is more practical and less expensive to maintain a degree of excess capacity for non-constraints (i.e. an intentionally unbalanced line) than to try to eliminate all sources of variation (which is necessary to efficiently operate a balanced line). Eliminating variation is still desirable in TOC; it is simply given less attention than improving throughput.

What is a control plan?

Control Plan : 1) Document describing the critical to quality characteristics, the critical X's and Y's, of the part or process. 2) Through this system of monitoring and control, customer requirements will be met and the product or process variation will be reduced. 3) Control plan should not be replacement for detailed operator instructions in the form or work instructions or standard operating procedures. 4) Each part of process must have a control plan. ----> a group of common parts using a common process can be covered by a single control plan.

"Standardizing new methods & Control Stage

Control Stage & Standardizing new methods 1) Standardizing the new methods.= creating written policy out of the advances or continuous improvement gained with new protocols and processes 1) Prevent backsliding (employees fall back into old and unproductive habits) after the main thrust of the Six Sigma project is complete by using process control to monitor process variation. a) Options----> i) define the methods of control with a control plan. ii) Write out all process procedures and duties on process maps, flowcharts, and sets of work instructions. iii) Process variation always should be monitored with process control. iv) Train all employees should in the new methods, and new employees should receive a comprehensive orientation."

On a control chart, 15 consecutive points in a row falling within one sigma from the center line indicates which of the following? You should investigate the cause; The process is out of control because this is statistically unlikely. You should investigate why the process variation has increased. You should investigate why the mean has shifted 1.5 sigma. You should do nothing, the process is in control and the parts are in spec.

Correct! A run of 15 points in a row within one sigma of the e center line is statistically unlikely, time to investigate. Something has changed in the process and it may have improved the process or it may be someone is making up the data, either way, we should investigate because statistically speaking the process is not in control. The process variation has decreased.

In addition to visually showing how much time is spent at each step, what additional data may be used on a cycle time chart? RPN Costs and value/non-value added steps Names of each operator Only time; no other data can be used

Correct! Costs and value/non-value added steps are two examples of what can be useful for a team to visualize while analyzing a cycle time chart. Costs and value/non-value added steps can be useful and would not be restricted from a cycle time chart.

You are a green belt planning a kaizen event to reduce cycle time for trauma patients in a busy emergency room. You think several different departments may be involved. What approach might be most useful to help identify all the stakeholders needed to attend the event? Announce the kaizen event and include a sign-up sheet. Sketch out a high-level process map. Ask an emergency room supervisor. Create a SIPOC.

Correct! Create a SIPOC in addition to providing a high-level scoping of the process for the event to help identify all stakeholders that touch the process. High- level process map ---While it is part of a SIPOC, it does not identify stakeholders

Data has been collected every day for the last 3 months from all three shifts. How should the data be organized if it is to be analyzed using an Xbar and s control chart to compare the shifts? Create an Analysis of variance table to calculate the shift to shift contribution to variation Create a matrix of the Process Capability by shift of key quality characteristics Organize the data by type of defect and compare frequency of defects from shift to shift Organize the data using rational subgroups from each shift

Correct! Rational Sub Grouping will allow you to compare both sample to sample process shifts and establish within sample variation. Since all product made on a shift is assumed to be similar we can compare the Xbar S charts for each shift for common and special causes of variation. Analysis of variance is not a control chart but a statistical technique with a similar objective to an Xbar and s control chart. Process capability is concerned with process performance. Although it may be useful it was not what the question asked which was to comparing the shifts with an Xbar and s control chart. Pareto analysis uses a Pareto chart (a "bar chart" arranged in decreasing order of frequency or amount) which is not an Xbar and s control chart.

What is the fewest number of samples required to achieve a confidence level of 99% that when fed a new diet, the average weight gain of a sheep over a 90 day period had increased by 5 pounds over the current average of 20 pounds? The weight gain per sheep is normally distributed with a standard deviation of 4.3 pounds. 31 101 71 123

Correct! Sample = n = (Z2 σ2)/(E)2) Where E = the minimum value to be

The most effective method for ensuring similar products are segregated during manufacturing in the same production area is to: Install painted lines between work stations running similar products Use a unique color coded label to identify each similar product Schedule and run production of only one similar item at a time Train personnel to recognize the differences between products

Correct! Scheduling and running production of only one similar item at a time prevents the possibility of mixing items if only one is being manufactured at a time and presents the cleanest solution to the problem. The possibility of mixing product is eliminated by this method. Scheduling and running production of only one similar item at a time is a more effective method since only one similar item is being produced at a time

Suppose you have completed an Ishikawa diagram. Which is the best way to search for true root causes out of all the possible causes identified? Use a multi-vote process to select highly likely root causes and then use force-field analysis on them. Gather data to confirm that the selected Xs affect the Ys of concern. If none of the investigated Xs turn out to significantly affect the Ys, then return to the fishbone and start over. Only look at the deepest level on the fishbone. Gather data to confirm/deny each identified possible cause on your fishbone. Investigate the Xs on the major bones. The root cause will be at that level.

Correct! Start with investigating the most likely causes, prioritizing them in order of likelihood. You need to confirm with data the root cause nature of suspected X. If none of them are highly effective in moving Y then you should go back and review your fishbone and start the multi-vote process again, removing those Xs already disproven. The true root cause is almost always deeper than the major bones, which are the 5Ms and E. The remaining choices are too burdensome to conduct.

A control chart is being kept on the total number of scratches on five inspected car hoods. The most appropriate control chart is: p chart np chart c chart u chart

Correct! The c chart is appropriate when the statistic of interest is the number of occurrences of an attribute for a given area of opportunity, where the area of opportunity remains constant for all subgroups, which is the case here on five hoods. The u chart could be used here for the ratio of the number of occurrences of a scratch to the area of opportunity, one hood. But since the subgroup size is constant (at size 5), a c chart is more appropriate.

A spaghetti or work-flow diagram is used to identify which one of the following types of wastes? Defects Motion Waiting Overproduction

Correct! The movement of paper, a file, a person, a piece of information, or materials can be measured by a spaghetti diagram. Overproduction measured with an analysis of work-in-process (WIP). Waiting = cycle time study Yield rate= defects

The chief operating officer wants to know if the 17th quarter's customer satisfaction survey result, 88, "is a statistically significant improvement of the mean at the 95 % confidence level and why?" The following facts apply: The data are collected by a proper survey method and are normally distributed and two-tailed. The mean satisfaction result for the last 16 quarters is 85.8. The standard deviation of the population is 3.06. The standard error calculated by the data analysis software is 0.79 (n=15) (two-tailed t-statistic is 2.13). No, the mean is within the 95% confidence interval. Yes. The upper limit of the 95% confidence interval for the mean is less than 88 when corrected for the use of a population instead of a sample. Unable to form a conclusion. To determine a statistically valid answer, at least 30 quarters of data are required. No. The lower limit of the 95% confidence interval for the mean is greater than 88.

Correct! The standard error of the population data will be smaller than the standard error treating the data as a sample

Suppose your team has just completed a relationship matrix on a current process problem. The team is having difficulty identifying where to start searching for the root cause. Which of the following is the best way to guide team members? Explore the cells where the relationship between potential root cause (X) and outcome (Y) is strong. If there are any rows that have more strong relationships in them than others guide the team to consider those root causes (Xs) first. You need the team to break up and work as individuals or small groups on each strong relationship identified. Let them figure it out. Just make sure that everyone gets a chance to talk. Look for low hanging fruit, i.e., easy fixes and Just Do It, whether it has a high impact on the Ys.

Correct! These root causes seem to affect more than one problem so there is a lot of potential to reduce waste if you can identify how to mitigate or prevent these issues. Choice B= Remember that if your team is cross-functional, as it should be, then the combination of all the experience and knowledge will lead to better solutions. Also, this approach is resource-intensive. It is better to prioritize the potential causes and work on them one or two at a time. C= A good facilitator has a plan when the team gets stuck. It is your job to help them move forward. D= If it doesn't have a high impact you really shouldn't do it. It's a waste of resources.

How is a typical kaizen event structured? Observe the current condition, identify and validate solutions, address the root causes of problems found, and establish standard work. Observe the current condition, address root causes of problems found, identify and validate solutions, and establish standard work. Address root causes of problems found, observe the current condition, identify and validate solutions, and establish standard work. Address root causes of problems found, establish standard work, identify and validate solutions, and observe the current condition.

Correct! These steps follow plan-do-check-act.

A work team wants to set up an X bar R chart for the finished length of tubes used in a camera tripod. They have data from 25 samples of 4 pieces each, What statistics do they need to calculate before they can set their control limits for their X bar chart? The average of the sample averages for finished length and average range across the samples. The average of the range for finished length. The average of the sample averages for finished length and average range across the samples, and the square root of the sample size. The average of the sample averages for finished length and the standard deviation across the samples.

Correct! You need to know the grand average finished length and the average range.

A fishing lure manufacturer precisely controls the weight of each lure by adding lead to the lure before sealing it. The manufacturer has decided to use SPC charts to understand and control its manufacturing processes. Which of the following charts would give them the best opportunity to prevent making lures of the wrong weight? An X bar R chart constructed using a sample of 5 lures pulled off the manufacturing line every hour and measured for weight. An X Bar S chart based on weights of a box of 12 lures per shift packaged and ready to ship. The number of customer returns per 100 pieces shipped. A P chart of non-conforming lures rejected each shift for weight out of tolerance.

Correct! You want to choose characteristics that provide data for diagnosing problems and test at the earliest point in the production process where you can get information on assignable causes to prevent non conformances. The frequency assures sample to sample and within sample variations can be measured. C& D = This is an attribute, which occurs long after the product has been shipped so does not provide useful information for controlling the process. Close

Which of the following is a group of experimental runs conducted under relatively homogeneous conditions? Response variable Interaction Experimental error Block

Correct! A block is a factor in an experimental program that has influence as a source of variability under homogeneous conditions. A response variable is the variable being investigated in the design of experiment. Interaction is a condition where the effect of one factor depends on the level of another factor. Experimental error is variation caused by extraneous variables. It is often called experimental noise.

n DOE, which of the following best describes a "blocking factor?" Factor thought to have an effect but not easily controlled Factor selected for greatest response Factor excluded in design Factor identified for least response

Correct! A blocking factor limits the number of experiments to a block, e.g., time in a day or shift, size of a batch, machine capacity. Blocking factor is included in the design but it has no impact on response level.

A control plan is best defined as: The natural boundaries of a process within specified confidence levels, expressed as the upper control limit and the lower control limit. A written description of the method for controlling part and process quality by addressing key characteristics and engineering requirements. A solution meant to reduce or eliminate an identified problem. A document specifically details the level of compensation that each stakeholder should receive after the process has been improved.

Correct! A control plan provides a documented description of the direct relationship between any highlighted characteristics and their controlling process setting or parameter. While upper and lower control limits may be featured in a control plan, this statement best describes control limits.

Which of the following best describes a controlled or uncontrolled variable whose influence on a response can be studied? Level Treatment Factor Unit

Correct! A factor is a variable that can have influence on a response. A treatment is a single level assigned to a single factor during an experimental run. Resolution is a term that describes the degree to which factors are aliased (or confounded) with interactions. Level refers to the values or settings selected for the factor being examined in the experiment

When is it appropriate to use a block factorial design? When the number of runs is too large to be carried out under homogeneous conditions. When a more illustrative design to display factors under the experimenter's control influences the response. When there are many factors and levels and it is impractical to run all combinations. When there is a need to study relative variability instead of mean effect of sources of variation.

Correct! A situation in which the number of runs is too large to be carried out under homogeneous conditions is appropriate for a block factorial design. When a more illustrative design to display factors under the experimenter's control influences the response, a response surface application is appropriate. When a more illustrative design to display factors under the experimenter's control influences the response, a response surface application is appropriate. A nested design application is appropriate when there is a need to study relative variability instead of mean effect of sources of variation.

An X bar control chart used in a machining operation is trending toward the upper control limit for the past 10 weekly samples. Which of the following changes is the most likely cause of this trend? The plant start time was changed from 8:00 AM to 7:00 AM 10 weeks ago. The sample size for X bar was changed from 5 pieces to 8 pieces 12 weeks ago. The semiannual preventive maintenance action due 15 weeks ago was skipped. A new measurement tool that is more repeatable was introduced to measure the samples 3 weeks ago

Correct! A trend would be expected when something on the machine is wearing out, which could be caused by lack of maintenance. Changing the sample size would not cause a trend. A more repeatable measuring device would reduce common cause variation in the measurements and reduce the range. If it did produce a change in the average, it would be a step function, not a trend. Since it happened only 3 weeks ago, it does not account for the 7 points in the trend before it was introduced.

Which chart would you use to track the number of defects found per identical printed circuit board inspected? np chart u chart c chart X Bar R chart

Correct! C-charts! These charts are concerned with the counts of whether an attribute is present (attribute data) and the sample size is constant. np Charts are used to study the number of defective items. U charts are used to monitor the proportion of defects for example defects per 1000 opportunities. The X bar R chart is a chart used for variables data (also called measurement data).

A company is testing a new material it developed and has designed an experiment to determine how temperature and pressure impact the strength of the material. Temperature and pressure have two levels (high and low), and the experiment is set up to be repeated once. Which comparison will provide the company with an estimate of the experimental error? Difference measured between first and second runs of the same combinations Difference measured between high pressure and low pressure results Difference measured between high pressure and high temperature results Difference measured across all combinations

Correct! Comparing first and second results for the same combinations will provide an estimate of error. Comparing high and low pressure results will estimate the effect of the pressure factor only. High pressure compared to high temperature will provide a partial estimate of the effect of each factor. Comparing the difference across all combinations will estimate the total variation.

The Billing department of a firm has been keeping an np chart on defective invoices. They sample 50 invoices a week and over the past year their average number of defective invoices has been 4.1. In order to save time, they are going to reduce the sample size to 40. What are the new control limits for the np chart with sample size of 40? 0 and 7.936 2.182 and 6.018 0 and 9.854 0 and 9.920

Correct! Control limits for the np charts = 0 and 9.854 Using the formula from page 323 of the CSSGB Handbook, with np bar = 4.1, n = 40 Upper control Limit= np-bar + { 3 * [square root of product of np-bar* ( 1- (np bar/n)]} 4.1 + { 3* [square root of [4.1 * (1 - (4.1/ 40))]} 4.1 + 3* [square root of [4.1 * (1 - (0.1025))]} 4.1 + 3* [square root of [4.1 * (0.8975)]} 4.1 + 3* [square root of [3.67975)]} 4.1 + 3* [1.9182]= 4.1 + 5.75= 9.85 Lower Control Limit np-bar - { 3 * [square root of product of np* ( 1- (np bar/n)]} = 4 - (5.75) = - 1.75

Sources of unscheduled downtime include which of the following? Machine breakdown and part shortages. Quality failures and part shortages. Setup time and maintenance. Machine breakdown and maintenance.

Correct! Machine breakdown and part shortages cause unscheduled down time. Part shortages cause unscheduled downtime, but quality failures are not a source of unscheduled downtime. Setup time is a source of unscheduled downtime, but maintenance is scheduled.

Since experimental error is almost invariably present, what is required to increase the precision of estimates of the effects? Replication Redundancy Random sampling Regression

Correct! Replications are test trials that are made under identical conditions. Redundancy is product- or process-focused. Regression is associated with an analysis technique.

A long coil of coated wire is being inspected. The number of defects or breakdowns in successive lengths of 1,000 feet each is being recorded. It is desirable to take advantage of the equal (constant) size subgroups of 1,000 feet. Which control chart is most appropriate to use? u np p c

Correct! The c chart is most appropriate when the statistic of interest is the number of occurrences of an attribute for a given area of opportunity, where the area of opportunity remains constant for all subgroups. In this case it is the number of nonconformities (breakdowns) in a 1,000 foot section. The u chart is only the most appropriate when the statistic of interest is the ratio of the number of occurrences of an attribute to the area of opportunity which varies from subgroup to subgroup. Here the area of opportunity (subgroup size) is constant (1,000 feet). The np chart is appropriate only when the statistic in question is the number of nonconforming items out of the total number of items inspected and the subgroup size is constant. The p chart is appropriate only when the statistic in question is the proportion of nonconforming items out of the total number of items inspected.

Each day, 100 items are inspected and the number of rejected items is recorded. You are asked to make a control chart, where the statistic in question is the number of nonconforming items out of the total number of items produced, that takes advantage of the equal size subgroups. Which control chart is most appropriate to use? c u np p

Correct! The np chart is appropriate when the statistic in question is the number of nonconforming items out of the total number of items inspected and the subgroup size is constant (here the subgroup size is 100). The c chart is appropriate only when the statistic of interest is the number of occurrences of an attribute divided by the area of opportunity, where the area of opportunity remains constant for all subgroups. The u chart is appropriate only when the statistic of interest is the ratio of the number of occurrences of an attribute to the area of opportunity. The p chart is appropriate when the statistic in question is the proportion of nonconforming items out of the total number of items inspected.

A Six Sigma Green Belt is creating a control plan for a manufacturing process. What should the Six Sigma Green Belt consider? What is the delivery rate for the material? How many parts will be produced per hour? Where is the material stored? Are packing issues for the part addressed?

Correct! The packaging may also need to be inspected. The number of parts to inspect is in the control plan, not the production rate.

An X Bar R control chart over time shows that the process average is centered in between the spec limits, and the process average range is greater than the distance between the upper and lower spec limits. Which statement below is the most correct? The process does not predictably meet customer requirements; the process variation needs to be reduced. The spec limits are too tight. The process is not in control. The process is in control

Correct! The process may be in control, but the customer requirements are not being met because some points fall outside the upper or lower spec limit. [ Range > (USL-LSL)] Reducing the variation in the process will reduce the range. If the variation is due to special causes, then removing them will bring the process into control and reduce the variation. The process mean is already centered; hence it is proper to focus on variation. The spec limits should reflect the needs of the customer, they may be too tight but we cannot tell that from the information.

Which of the following best describes the structure of a response surface design of experiment? Each treatment occurs one in every row. Several factors are investigated. Factor settings are viewed at defined points. Factor settings are constrained.

Correct! The response surface design of experiment has the following structure= factor setting are viewed at defined points.A map can be constructed as a result of the defined points Mixed designs employ the structure where factor settings are constrained. Fractional factorial= Several factors are investigated. A row-column design also referred to as a Youden square= each treatment occurs one in every row.

A control chart is used to control a paper towel line. The end product is produced in rolls of varying length 12 inches wide. Nonconformities include tears, improper printing, improper perforations, etc. The control statistic is defects per 100 sheets with one roll constituting a sample. Which control chart is most appropriate to use? c p np u

Correct! The u chart is appropriate when the statistic of interest is the ratio of the number of occurrences of an attribute to the area of opportunity. In this case, it is the number of nonconformities divided by the length of the roll in hundreds of sheets. Here, the area of opportunity (subgroup size) varies because one roll constitutes a sample and the rolls are of varying length. The c chart is appropriate only when the statistic of interest is the number of occurrences of an attribute in a given area of opportunity, where the area of opportunity remains constant for all subgroups. In this case the number of nonconformities is divided by the length of the roll in hundreds of sheets. Thus, the subgroup size varies because one roll constitutes a sample and the rolls are of varying length. The p chart is appropriate only when the statistic in question is the proportion of nonconforming items out of the total number of items inspected. The np chart is appropriate only when the statistic in question is the number of nonconforming items out of the total number of items inspected and the subgroup size is constant.

The number of blemishes in the paint finish on the hood of a car is being monitored. Each day a sample of between 20 and 25 hoods is inspected. To monitor the number of blemishes per hood in the daily sample, which control chart is most appropriate to use? p c np u

Correct! The u chart is appropriate when the statistic of interest is the ratio of the number of occurrences to the area of opportunity. In this case it is the number of blemishes divided by the number of hoods inspected. Here the area of opportunity (subgroup size) varies because a sample varies between 20 and 25 hoods. The p chart is appropriate only when the statistic in question is the proportion of nonconforming items out of the total number of items inspected. The c chart is appropriate only when the statistic of interest is the number of occurrences of an attribute in a given area of opportunity, where the area of opportunity remains constant for all subgroups. This case of interest is the number of blemishes divided by the number of hoods inspected. Here the area of opportunity (subgroup size) varies because a sample varies between 20 and 25 hoods. The np chart is appropriate only when the statistic in question is the number of nonconforming items out of the total number of items inspected and the subgroup size is constant.

A manager is struggling to meet customer expectations in processing orders timely. The manager has asked the team how to reduce lead time with kaizen. What would be the least effective method to reduce cycle time? Focus on working each step faster; the problem is unimportant. Reduce interruptions and delays between steps. Change the sequence of steps. Look for ways to perform steps in parallel.

Correct! Trying to work each step faster is not the best approach as it may not be sustainable and doesn't reduce any barriers or constraints that may be affecting the process. Reducing interruptions or delays can improve the continuous flow of work which will help to reduce lead time. Wait times and transport times can sometimes be reduced by changing the sequence of steps, which can help decrease lead time. Many processes are worked in a serial approach that results in the cycle time for the entire process being the sum of each step; finding ways to work in parallel can significantly shorten lead time.

How can visual process controls improve and control a workplace? By quickly identifying abnormal conditions. By being a part of the control phase of a Six Sigma project. By keeping the work area clean. By statistically monitoring a process.

Correct! Visual process controls can be used to quickly identify abnormal condition SPC is used to statistically monitor a process. 5S is used to keep a work area clan. 5S may be implemented as part of a visual factory, but it is not a requirement for a visual work area. The implementation of a visual workplace can be performed during the control phase of a Six Sigma project; however, simply implementing a visual workplace is not how a process is improved or controlled.

A quality team has finished a brainstorming session and would like to document the results. The best tool for this is which of the following? Pareto chart Check sheet Cause and effect diagram Dot chart

Correct!---A cause and effect diagram documents the final list of causes from a brainstorming session. The Pareto chart separates the "vital" few from the "trivial many." It is created based on data and not brainstorming results. Dot charts are used to display the values of categorical data. Check sheets are used for data collection.

Which technique plots data to see how well it follows a straight line? Fitting mixtures Central Limit Theorem Normal Probability plot Test of model validity

Correct= Normal Probability Plot=By constructing a histogram and comparing it to a normal curve a normal probability plot to see if the sample data came from a normal distribution. Test of model validity is a test of the validity of an assumed discrete model, not a technique for plotting data. Fitting mixtures is a term used to describe one or more peaks within a distribution of data. Central limit theorem is a statistical principle not a technique used to plot data.

Tools used to Translate Customer Requirements-- CTQ

Critical-to-Quality (CTQs) characteristics---- are those product features that are deemed important from an examination of customer needs. a. Must translate customer feedback into project goals and objectives, including critical to quality (CTQ) attributes and requirements statements ******Key Takeaway: CTQs are important characteristics that are developed from an examination of customer needs. Unlike other measures of customer wants, customer needs that are directly related to product features are an important component of this Six Sigma methodology.

Crosby

Crosby a. Described quality costs as the price of making things right the first time plus the price of making things right if they were not. b. Developed the process cost model (PCM) that describes the quality costs as the combination of the price of conformance and the price of nonconformance . Crosby (1984) identifies the price of conformance (POC) represents the costs of making things right, and the price of non-conformance (PONC) represents the cost of doing things wrong c. Quality awareness + "zero defects day" + all employees involved in quality improvement + Individual improvement goals + measure both current & potential quality problems

Crosby's opionon of Quality?

Crosby stated that corporate management must make the cost of quality a part of the company's financial system.

Curvature

Curvature 1) Refers to non-straight-line behavior between one or more factors and the response. 2) Curvature is usually expressed in mathematical terms involving the square or cube of the factor. Y= B0 + B1 X1 + B11 (X1 * X1) + E B11 (X1 * X1) describes the curvature

Customer Priorities

Customer Priorities 1) Data gathering tools ----customer interviews, surveys, focus groups, phone surveys, mail surveys, audits, sales reports---aid companies to identifying customer priorities. 2) Questions that should be asked: a) What attributes are valued? b) Rating of each desirable attribute c) How do we compare with competitors? d) What other features & services would be of value? ****Management can use this data to enable company to provide timely response and also direct the improvement team towards addressing weaknesses.

Customer Retention

Customer Retention 1) Most companies spend money on getting new customers, rather than customer retention. 2) High customer satisfaction scores---unreliable indictor--does not translated into strong customer loyalty or high customer retention. 3) Cheaper to retain existing customers than to find new ones. (Existing customers are 5X more valuable)

Customer needs related to the use of the product

Customer needs related to the use of the product 1) Conveinance --> technology can bring new product and services but some society sections limit extent of technology in their live (Amish) 2)Safety Needs---> Products/ Services for customer needs ---eg. sun screens protection due to thin ozone layer 3) Product simplification features---> Product/ services that facilitate the conversion to a new product 4) Communications---> Need to be informed and to be given access to rightful information--> eg. open door government meetings. 5) Service for product failures--> warranties, returns, exchanges, replacement policy 6) Customer Service---> Company agent should be empowered to satisfy the customer at the point of complaints.

Which of the following is an example of an activity that adds to the Cost of Quality? A) Addition of safety relief valve to pressure vessels being produced. B) Refining of crude oil to produce high quality unleaded gasoline. C) Regular online updates to anti-virus software. D) An inline pH test of milk before final formulation.

D= CORRECT!! quality costs are the total of the cost incurred by (a) investing in the prevention of nonconformance to requirements; (b) appraising a product or service for conformance to requirements; and (c) failure to meet requirements". Product testing is part of cost of quality. Refining of crude oil to produce high quality unleaded gasoline is part of the oil refining process and does not specify any product testing or other cost of quality activities. Safety relief valves are part of the product and not an added cost of quality. Regular online updates to anti-virus software is an added value to the product and a customer expectation, does not represent part of cost of quality.

Which of the following is most important when developing a series of measurements for an organization? A) Identify measurements common to other organizations for benchmarking purposes. B)Establish process capability Cpk for core processes first. C) Measure all processes across the organization to surface potential issues. D) Align measurement priorities with goals and available resources.

D=Correct! Measurements should support organizational goals and be implemented within the constraints of available resources. While an organization with sufficient resources might be able or desire to attain this level of measurement, it would still need to make sense within the context of goals and resource availability. Additionally, measurement should only be undertaken when there is a reason to measure. Close Core processes should be measured, but Cpk is only useful when the process is known to be in statistical control with a normal distribution.

Which of the following is an example of sub-optimization of the whole? A) Focusing on satisfying customer requirements will necessarily optimize the whole organization. B) The best way to optimize the whole organization is to optimize its parts either one at a time or using some design of experiments approach. C) The only way to optimize the whole is for all to sacrifice to work together to produce the best output (products/services) for their customers. D) Organizational processes are interdependent; the requirements specific to a sub-process may not align with organizational objectives.

D=The interconnectedness or interdependence of each process may require an individual process to sub-optimize to fulfill broader organizational performance goals (e.g., one sub-process may overproduce for downstream processes, increasing levels of waste within the organization). Because processes within an organization are interdependent, the whole organization cannot be optimized by optimizing its parts. For all to sacrifice to work together to produce the best output (products/services) for their customers is not the way to optimize the whole; rather, it restates the question as a statement.

DFSS Roadmap & IDOV

DFSS Roadmap & IDOV 1) Design---> a) Transfer functions used to develop product's layout & geometry; Statistical Quality Approach used in this step b) Tools in this stage i. DOE ii. Computational Fluid Dynamics iii. Statistical Intfernce= ANOVA or Hypothesis testing iv. Finite Element analsysis (FEA) 2) Optimize---> a) Focuses on minimizing the new design's sensitivity to CTQ design features 3) Optimize---> PQS used as input to show the required level of quality has been achieved. Data from quality control and field studies confirm that the product satisfies the required specifications.

DFSS Roadmap & Sequence of development phases for new projects

DFSS Roadmap & Sequence of development phases for new projects: 1. Training & Launch a) Team learn DMAIC method and projects selected represent "Lowest hanging fruit) yeild gains from obvious improvement opportunities with easy solutions 2. Implementation a) Teams work on core business objecting using statistical tools to enhance process efficiency and predictability in improvement project 3) Design for Six Sigma a) Teams attach most complex projects with greatest potential gains. Solutions require more than improvement and become a matter of new process and product designs. ****DMAIC is easier & faster than design projects...DMAIC & DFSS projects are run concurrently.

DFSS Roadmap & Steps to determine an organization's readiness to deploy DFSS

DFSS Roadmap & Steps to determine an organization's readiness to deploy DFSS 1. Monitor Sigma Levels ----> a) Rising Sigma levels = Effective Program b) Sigma increases slowing down= existing process maximally improved so consider redesigning process 2. Written Schedule of Prioritized ideas a) Plan & prioritize projects by complexity b) If simpler improvement run-out--> Train Staff for DFSS applicaiton 3. Remain aware of marketplace changes that could obsolete product/process a) If customer requirements, market demand, or technology changes---> Use DFSS to develop new products/ processes 4. Gauge organizations capability for success with DFSS a) Evaluate if project deadlines are meeting b) Check if resources are sufficient and management team can more involved in the design process.

DOE & SDCA

DOE---best used for optimizing a process versus as problem-solving tools. a. SDCA ---standardize, do, check, act— i. Most commonly used once a process has improved to update control plans and process sheets to lock in the improvements & standardize the changes throughout the organization. ii. Outcome of applying sound recommendations based on the correct interpretations of valid effects obtained from proper design of experiments.

A total of 4000 closures were manufactured on machine #7. 200 closures had one defect each. A total of 3800 closures were packed. What are the defects per unit (DPU) and throughput yield? A) 1.05 DPU and 95% throughput yield B) 1 DPU and 99% throughput yield C) 0.05 DPU and 95% throughput yield D) 0.0526 DPU and 94.7% throughput yield

DPU is determined by 200/4000 = 0.05 and throughput yield is determined by 1.00 - 0.05 = .95 x 100 = 95% Defects per unit (DPU) is determined by total number of defects divided by the total number of products produced in some time period. Throughput yield is determined by 1- % of closure defects * 100.

Data Coding

Data Coding Efficiency of data entry and analysis is frequently improved by data coding---Sometimes it is more efficient to code data by adding, subtracting, multiplying or dividing by a factor. 1) Types of Data Coding a) Substitution - ex. Replace 1/8ths of an inch with + / 1 deviations from center in integers. b) Truncation- Ex. data set of 0.5541, 0.5542, 0.5547 - you might just remove the 0.554 portions. 2) Effects of Coding Data a) Will affect the mean to the extent that the mean must be uncoded for reporting purposes. b) Coding and uncoding of the mean will be exactly opposite. (Ex. Add X, subtract X or multiply by X, divide by X.) c) The effect the coding has on standard deviation depends on how the data is coded.

Data collection Methods:

Data collection Methods: a. Surveys -----Low response rate (10-15%) b. Face-to-Face interviews & Focus Groups------High data integrity and can clarify with respondents c. Mystery shopping d. Customer feedback-------Can be reactive after product failure; best to gather up-front information before designing product/service e. Automatic data capture i. Can eliminate errors involved with manual data capture. f. Manual data capture (prone to errors)

Quality Function Deployment: Sources of Data

Data sources for Quality Function Deployment: a. Data of customer needs/wants obtained by customer experience, focus groups, surveys, and joint development b. Prioritize wants by quantification and weight c. Research and customer experience or independent neutral agents-→ customer perceptions relative to competition. d. Critical process characteristics-→ how things are done, design requirements, operational factors, and human factors. 8) Relationships are tracked and the key links are determined to find out the customer wants are correlated with specific design approach. -→ Ranking of relationships to find OFI, innovation, or Technical importance.

Project Charter & Defining the business need

Defining the business need 1) Identifies business processes that will be improved by the successful completion of the project and with the use of objectionable data. 2) It may be difficult for the author to estimate the value of the project to specific business processes, particularly if this estimate is based on minimal and preliminary data in the problem statement. 3)Charter should refer to specific metrics ( number of defects, schedule, or profit) **** Many Six Sigma theorists recommend that all projects have a clear and measurable benefit to the customer. Comprehensive project charters will address business needs related to the customer, stakeholders, and other employees."

Definition of Value--multicultural view

Definition of Value--Multicultural view 1. German idea---Engineers in control of business; product features & processes & enhancements are most important; German's explain customer disinterest in complex enhancements that customer is not smart enough to undersand new features. 2. Japanese view--Value is defined by where the value was created and stressed using Japanese suppliers. Customers are only concerned with getting their needs met --> therefore many weakened Japanese companies 3. Once values is define, the target cost of product can be determined 4. Target cost is more than market cost; Target cost is mixture of current selling prices by competitors and the cost of eliminating waste via lean methods; Target price should below the current selling prices. 5. Market price= Profit + selling prices+ manufacturing cost.

Deming & 7 deadly diseases that management must cure

Deming & 7 deadly diseases that management must cure: 1. Excess medical costs 2. Excess for to warranty, fueled by lawyers working on contingency fees 3. Use of visible figures for management, with little or no consideration of figures that are unknown or unknowable 4. Mobility of management: job-hopping 5. Personal evaluation appraisal, by whatever name, for people in management the effects are devestating 6. Emphasis on short-term profits 7. Lack of constancy of purpose to plan a marketable product and service to keep the company in business and provide jobs.

Describe Process Definition

Describe Process Definition 1) First step in Measure Stage= Create a comprehensive process level map of processes currently performed.----aka. group describes all the activities that they plan to improve 2) Expert employee participation needed to create detailed process map because employees have a different conception of process sequence than their superiors. 3) Employees, often , would have streamlined and modified process with notification of management---> harmful to criticize or judge the customization made by employees------> best to focus on creating an accurate process map.

Describe control plans and their application in control stage of DMAIC

Describe control plans and their application in control stage of DMAIC 1) Control plan----general summary of the detection and/or prevention strategies used to control processes or materials a) Used to record the strategy used to control the key process variables. b) Derived from data compiles from DOE results and FMEA (FMEA indicates key sources of failure) c) Lists the characteristics that need to be monitored and controlled------provides information bout the specifications, measurement technique, sample size, sample frequency, analytical tool and reaction protocol.

Design Using QFD

Design Using QFD 1) Design Team ----By describing the product in the language of the engineer, along the top of the quality, design team list those engineering characteristics that is most like to impact the customer attributes. ---> " Roof Matrix" helps engineers specify various engineering features that have to be improved.---> Customer satisfaction must be considered when examining engineering characteristics 2) Interfunctional Team---> fills the body of the house or the "relationship matrix" that indicates how much the engineering attributes affect each of the customer attributes 3) Weighted characteristics are compared to the actual component costs to prioritize improvement components. 4) Increasing one engineering characteristic may have a negative impact on another engineering characteristics which may also indicate that a different solution is needed

Design of Experimentation Difference between replication & repetition:

Difference between replication & repetition: i. Replication= process of running the experimental trials in random manner. Resetting of each trial condition. ii. Repetition= running experimental trials under the same setup of parameters

"Exponential and lognormal distributions

Exponential distributions 1) used for continuous data, or data obtained by measurement. 2) MC application for this distribution is in the measurement of event rate, or the frequency with which a particular event occurs. 3) If the event rate essentially is constant, the exponential distribution is appropriate. 4) The distributional parameter is λ (lambda), calculated 1/μ, where μ is the interval between events. 5) Lognormal distributions are appropriate for continuous data that has a fixed lower boundary, usually zero, and no upper boundary. This distribution is used often for reliability data. 6) As with other distributions, a goodness-of-fit test can determine whether a lognormal distribution credibly summarizes the data.

Factors to be considered for initial project selection

Factors to be considered for initial project selection: 1) Broad appeal to team members and management 2) Fairly simple, but no trivial 3) Opportunity for quick benefits 4) Project scope under group's control 5) Selected using time and resource constraints 6) Major benefits are project resolution and learning teamwork

An FMEA is being constructed for the manufacture of a syringe cartridge. The team has developed risk ranking scale criteria for calculating the RPN. The team has assigned five values for ranking likelihood of occurrence (O), 10 values for ranking the risk associated with severity (S), and five values for ranking the risk associated with detection (D). Using this method will most likely do which of the following? A) Give severity a disproportionate representation in RPN. B) Ensure all values for O, S, and D are equally represented in RPN. C) Give occurrence and severity an equal representation in RPN. D) Ensure RPN reflects the priority for addressing failure modes.

Failure modes with high severity rank values will result in larger RPN numbers than failure modes with high occurrence or detection values. Since O, S, and D have different levels of ranking, RPN may be biased and not provide correct priority.

Full Factorial

Full Factorial **Experimental designs which contain all combinations of all levels of all factors. No possible treatment combinations are omitted. As their name implies, full factorial experiments look completely at all factors included in the experimentation. In full factorials, we study all of the possible treatment combinations that are associated with the factors and their levels. They look at the effects that the main factors and all the interactions between factors have on the measured responses. If we use more than two levels for each factor, we can also study whether the effect on the response is linear or if there is curvature in the experimental region for each factor and for the interactions. Full factorial experiments can require many experimental runs if many factors at many levels are investigated.

Full Verses Fractional Factorial

Full Verses Fractional Factorial 1) Full Factorial -------> Experimental design which contains all levels of all factors. No possible treatments are omitted. 2) Fractional Factorial is a balanced experimental design which contains fewer than all combinations of all levels of all factors. -----Will require an equal number of plus and minus signs in each column

Full-Factorial Experiments

Full-Factorial Experiments a. Looks for every possible combination in order to compete a full study of interactions. (contains all levels of all factors; no treatment is omitted) b. Two-way ANOVA used to evaluate the statistical significance of the results. --→ compares the between treatment variation with the within- the -treatment group c. The larger the effect, the more likely the results are significant.

Green Belt's FEMA Role

Green Belt's FEMA Role 1. Common for Greenbelts to conduct and supervise FMEA 2. FMEA is a team effort so team of 10-15 used a) All team members must participate b) Multi-disciplinary expertise and input is beneficial Input from all engineering fields is desirable c ) Representatives from all areas (not just technical disciplines) are generally included as team members 3. After project completion, Greenbelt relinquishes control of the process documents, then the project is transferred and assigned to Process Owner to ensure that the processes continue to be supervised and monitored after project completion----> Process owner updates the process documents & FMEA documents + leads FMEA team. a) The DFMEA will be refined and evolve with the product--->Numerous revisions are required to obtain the full benefit of the DFMEA b) The DFMEA should include all systems, sub-systems, and components in the product design ***Without transfer of ownership, the monitor and control of the evolving DFMEA document would be lost.

How is Nominal Group Technique conducted?

How is Nominal Group Technique conducted? 1) 5-9 people generate ideas and then write it down 2) Facilitator than records the ideas and then facilitator leads the discussion 3) Voting on the best solution (rank ordersing, priority rating, ect) a) Two Voting methods----> Repeat voting till best idea is found or 2nd method= use of cards & Pareto breakdown of favored ideas.

C. Hypothesis Tests for Means P-test

Hypothesis Tests for Means-----P-test a. Used when testing a claim about a population proportion and required a fixed number of independent trials having constant probabilities., with each trial having two possible outcomes (binomial distribution) b. When np < 5 or N(1-p), binomial distribution used to test the hypothesis relating to proportion.

Hypothesis Tests for Means Contingency Tables

Hypothesis Tests for Means----Contingency Tables a. Two dimensional classification tables with rows ad columns with original frequencies and count data that can be analyzed to determine whether the two variables are independent or have significant association. b. Chi-square will test if there is dependency between the two classifications. ****Contingency coifficient- show the strength of correlation while the chi-square test shows significant dependency.

Hypothesis Tests for Means Paired T-test

Hypothesis Tests for Means----Paired T-test a. Measures whether means from a within-subjects test group vary over 2 test conditions. b. This t‐test compares one set of measurements with a second set from the same sample. It is often used to compare "before" and "after" scores in experiments to determine whether significant change has occurred.

C. Hypothesis Tests for Means Student T- test

Hypothesis Tests for Means----Student T- test a. Used to make inferences about a population mean when the population variance is unknown and the sample size is small. b. Only applies to samples drawn from a normally distribted population. c. Can be applied to any sample size but best applied to samples greater than 30.

Lean & Muda

In Japan, the philosophy is called muda. The general strategy of lean is to identify activities that add value and activities that do not add value. As much as possible, lean businesses try to eliminate activities that do not add value. These judgments always are made from the perspective of the consumer. Lean methodology is very similar in this respect to Six Sigma."

Performing is the stage of the team evolution in which the team members do which of the following? A) Begin to understand the need to operate like a team rather than as a group of individuals B) Work together to reach their common goals C) Express their own opinions and ideas, often in disagreement with others D) Struggle to understand the goal and its meaning for them individually

In the performing stage team members work together to reach their common goal When team members express their own opinions and ideas, often in disagreement with others is the definition of the forming stage. Struggle to understand the goal and its meaning for them individually is the definition of the storming stage. The process of beginning to understand the need to operate like a team rather than as group of individuals is the definition of the norming stage. When team members express their own opinions and ideas, often in disagreement with others is the definition of the forming stage.

Juran--Triology

Juran--Triology Managing for quality requires the same attention that other functions obtains. 1. Quality planning 2. Quality control 3. Quality improvement Juran saw these items as keys to success. Top management can follow this sequence just as they would use one for financial budgeting, cost control, and profit improvement **Quality planning used to create the process that will enable one to meet the desired goals. ****Quality Control used to monitor and adjust the process. Chronic losses are normal in controlled state, while sporadic spike will initiate investigation. **Quality improvement reduces chronic losses and move the process to better and improved state and that's the "last award"

"Kaizen philosophy vs. US. business model

Kaizen is a Japanese philosophy that governs all aspects of business; more effective TQM systems 1) Continuous, small improvements. 2) Improvements based on the insights and experiences of lower-level employees, in contrast to the Western business model, in which changes typically result from executive orders. 3) The initial concern of the kaizen business is attracting and maintaining good employees essential for good processes 4) Kaizen requires with adequate training, defined operating practices, and buy-in from all employees. 5) Constant communication exists between employees at all levels.

Kanban

Kanban a. System is best controlled when material and information flow into/out of processes in smooth & rational manner. b. Premature arrival of inputs→ confusion, inventory loss, and excess costs. c. Properly administered Kanban system will improve system control by assuring timely movement of products & information. d. Implemented using visual indicator called Kanban cards, which indicate the quantity to be replenished once the minimum level is reached. e. Kanban card inside empty bin--→ signals production to pull material from the previous step. f. Lean processes must be in place before initiating Kanban.

Lean Concepts & Tools-- Define Value

Lean Concepts & Tools--Value a. Definition= customer's perception of te usefulness and necessity of a given product/ service b. After customer's perception of value is defined, the target cost of the product/service can be determined. c. Target cost= mixture of current selling prices of competitors + examination of waste by Lean methods.

Level Loading

Level loading 1) Lean Tool; purpose is to regulate and moderate the flow of orders in a particular process or to eliminate wait time at beginning of processes a) Improve stage = can be useful for diminishing the need for inventory checks during a process. b) Before level loading can be implemented, protocols must be standardized and employees must be trained in multiple areas. 2) Process of Level Loading: a) Calculate Takt time ( target process time). Take Time= (demand) / (amount of time available). Takt time should be posted at the work station, and resources should be aligned with it. When variations in demand exist, needs for increased resources also will exist. *****The intention of level loading strategies is to eliminate wait time at the beginning of processes. Each completed unit should begin the next phase of the production process immediately. There should be a protocol in place for adjusting resources to meet the natural fluctuations in demand.

"Linearity analysis

Linearity analysis 1) Detects chance that error bias in particular system of measurement will be present throughout the equipment's entire operating range. ( can measurement system be trusted for large & small values?). 2)Measure stage ---> linearity analysis is useful for assessing the accuracy of a measurement system within the range of values likely to be observed during the process. 3) Automotive Industry Action Group created standard procedure for analyzing linearity e . a) 1st step = select multiple parts to examine throughout the operating range. b) Use precise equipment to find reference value c) Have different employees measure the parts throughout the range with the intended measurement system. i) Find the average of these measurements to to calculate bias: Bias= (average measurement) - (reference value) . ****This data then should be depicted on a scatter diagram. The adequacy of the measurement system is assessed using R2, the coefficient of determination. A value of greater than 70% is acceptable in most cases."

Long-term process capability---→

Long-term process capability---→ same measurements my include a variety of samples (different streams of machines, multiple operators/ spindles/ cavities/ measuring equipment) i. Process capability calculation (Pp, Ppk) performed using sample standard deviation of values. ii. Parameters not following normal distribution (flatness, wait time) are normalized with tranformation techniques before analyzing the data. iii. Variation and shift in the mean occurs in long-term process iv. The same capability indices that you calculate for short-term variation can also be calculated for long-term, or total, variation. To differentiate them from their short-term counterparts, these long-term capability indices are called PP and PPK. (The P stands for "performance.") v. The only difference in their formulas is that you use σLT in place of σST. These long-term capability indices are important because no process or characteristic operates in just the short term. Every process extends out over time to create long-term performance.

Measurement correlation

Measurement correlation a. Used when measurements are taken simultaneously with multiple measurement devices of the same type for parts coming from multiple streams of manufacturing. 6) Percent agreement: a. Attribute Gage R&R Study -→ Compares Appraisers b. Go/no go gage.= This gage simply tells you if the part passes or it fails. There are only two possible outcomes. i. This gage will simply tell if the part is within specifications. It does not tell you how "close" the result is to the nominal; only that it is within specifications. To determine the effectiveness of Gage, then must conduct a attribute gage R&R study to assess the variation. Variations can be either due to human judgment or due to machine variation (automatic measurement gauging where parts are screened as good/bad by the machine) ii. Results can be expressed as accept/reject or rating 1-5

Measuring Process Capability

Measuring Process Capability i. Process variation < specification limits-→ must reduce the non-conformances + reduce variations ii. Crux of Six Sigma Methodology= reduction of variations. 1. Containment Action= In some cases, may have to off-center the process distribution in a direction that required rework and salvage rather than scrapping of parts. Used to buy time while trying to find a way to reducing the variation. 2. Can also revisit the specification limits of the customer and engineering standpoint. (usually due to tight, unrealistic specifications created by the designers who have reviewed the limitation in technology or the process capability)

NP Chart--Attribute Data Control Chart

NP Chart i. Control charts dealing with the number of rejected items expressed as an integer. 1. If sample size is constant and defectives are being used, then np can be used rather than p chart. ii. When each data point is based on the same sample size, a special version of the p chart can be used. The np chart follows the same principle as the p chart, but actually plots the number of instances in a category over time rather than the proportion in the category. iii. The np control chart is used to determine if the number of defective items in a group of items is consistent over time. The subgroup size (the number of item in the group) must be the same for each sample.

NVA activities in the analyze stage of DMAIC

NVA activities in the analyze stage of DMAIC Identification 1) Tools such ( matrix diagrams & quality function deployment effort) identify non-value-added activities. ( value based on customer's perspective) 2) Types of NVA activities a) Improper / unsuccessful designs. b) Unnecessary or redundant steps in process c) Any work that does not translate into a product or service for the customer ( eg. inventory reorganization provides nothing to the customer) d) Any unnecessary transport of people or resources

Normal Distributions

Normal Distributions 1) Continuous distribution used for Variable data (length, mass, time, ect) 2) Bell Shape Curve=Averages of measurements of individual data follow normal distribution even if the individual data are from a different distribution. 3) Standard Deviation (Z-scores)= represents the area under the bell curve 4) Standard normal distribution has a mean=0 and a SD=1 5) According to the central limit theorem, the sampling distribution of a statistic (like a sample mean) will follow a normal distribution, as long as the sample size is sufficiently large. Therefore, when we know the standard deviation of the population, we can compute a z-score, and use the normal distribution to evaluate probabilities with the sample mean.

Organizational Performance and related strategic goals and objectives may be determined by 5 measures

Organizational Performance and related strategic goals and objectives may be determined by 5 measures: 1) Profit 2) Cycle time 3) Resources 4) Marketplace response 5) Short-term or Long-term emphasis

Origin of Quality Cost Measurements

Origin of Quality Cost Measurements---Reasons: 1) Products become more complex 2) Customers' expectations of product become more significant 3) Customers demanded service after the sale and expected failure remedy 4) Both supplier and customer costs expanded due to labor and mainence. 5) Management alternatices needed to be moneary terms.

Orthogonal

Orthogonal 1) Design is orthogonal if the main and interaction effects in the given design can be estimated without confounding the other main effects or interactions. 2) Full factorial is said to be balanced, or orthogonal, because there are an equal number of data points under each level of each factor.

4 PERT Requirements

PERT Requirements: 1) All individual project tasks must be included in the network 2) Events & activities must be sequenced in the network to allow determination of the critical path 3) 3 estimates must be made for each activity in the network----optimistic, most likely, and pessimistic elapsed times. 4) Critical path & slack times for the proejct are calculated. ------> Critical path= sequence of task that needs the greatest expected time S= T(L) - T(E) ** Slack time= [Latest time event can occur / finished w/o project extension] - [Earliest date an event can occur]

Parametric & Nonparametric tests

Parametric & Nonparametric tests a. Parametric tests= descriptive measure calculated using population data. i. Assumes that data are normally distributed. b. Nonparametric tests i. Distribution-free testing-→makes no assumption regarding the population distribution. ii. Applied to ranked data in which data are not specific in any continuous data or attribute data.

Pareto Diagrams

Pareto Diagrams i. In a Pareto chart, the data is organized in a descending order which allows the emphasis of the significant 'few' from the trivial 'many'. In other words, if the data shows many observations that are very small but you have a number of important, large ones, then this type of chart will emphasize the latter. ii. Arranges data in descending order of frequency of occurrence and creates a cumulative percent line. iii. Data presented on the basis of frequency of occurrence iv. Data are assigned weights based on criticality and multiplied by occurrence and pareto diagram is creased based on a weighted score. v. Avoid making data too specific to best leverage Pareto's strength of identifying systemic issues that require root cause resolution.

Poke-Yoke/ Mistake Proofing

Poke-Yoke/ Mistake Proofing 1) Shingo 's Concept ---that human error does not necessarily create defects 2) Success of Poke/ Yoke depends on some intervention device ------vaiaadaptive approaches, mechanical screening devices, signaling mechnaism or control mechanisms------------that detects the mistake before it is translated into non-conforming product. 3) Characteristics of Poke/ Yoke Device: a) Permits 100% inspection b) Avoids sampling for minitoring and control c) Are inexpensive. 4) 4 ways that errors can occur: a ) Skipping an operation b) Failing to properly align a bolt c) Using wrong parts/ materials d) Positioning parts in wrong direction.

Poke/ Yoke---Types of Countermeasures for Human Errors

Poke/ Yoke---Types of Countermeasures for Human Errors 1) Forgetfullness (Cause= Poor concentration ) ----> Solution: Checklists/ Visual Aids 2) Misunderstanding (Cause= unfamiliar siutaitons) ----> Solution: Checklists/ Visual Aids / Training/ Work Standardization 3) Indentification (Cause= Similar Appearance ) ----> Solution: Training / Visual Aids 4) Beginners Errors (Cause= No experience ) ----> Solution: Checklists/ Visual Aids / Training/ Work Standardization 5) Willful error (Cause= Ignoring Rules ) ----> Solution: Training/ Work Instructions 6) Slowness (Cause= Judgement Delays ) ----> Solution: Work standardization/ Visual Aids / Work Instructions 7) Surprise Error (Cause= Erratic Equipment ) ----> Solution: Work standardization/ Work Instructions 8 ) Intentional Errors (Cause= Crimes/ Sabotage ) ----> Solution: Education/ Discipline

Process Performance indices such as Pp and Ppk use more data in the calculation of the index than Process Capability indices like Cp and Cpk. requires the process to stabilize over time can be used for comparison and prioritization of improvement efforts. provide an estimate of future process operation

Pp uses root mean square standard deviation in its calculation, therefore it looks at overall variation rather than process average within and between subgroups which Cp does through the use of estimated standard deviation. Because they do not require the process to be in statistical control, that is stable over time, they only tell you how well the process did in the past. Both process performance indices and process capability indices can be calculated from the same data, but use different calculations of sigma.

Practical Significance vs. Statistical Significance

Practical Significance vs. Statistical Significance 1) Levels of 5% or 1% are used to determine whether a hypothesis has statistical significance. 2) If p-value less than 5%---> statistically significant 3) Possible to have a hypothetical claim that is statistically significant but has no practical value.---generally when the sample size is inadequate.

A lab supervisor determined that performing 5 quality control tests on Product A instead of 6 lowers the confidence level that the batch is acceptable by a very small percentage. The elimination of the 6th QC test saves 20 minutes per lot release. What advice would you give to the lab supervisor? Statistical significance is more important than economic factors. Leave the sixth test in place. The economic factors outweigh the statistical significance. Eliminate the sixth test. More data is required. Leave the sixth test in place. 6 QC tests are the minimum required for lot release.

Practical significance outweighs the small change in confidence level. Statistical significance must be balanced with economic and engineering considerations.

"Problem statement of Project Charter

Problem statement 1) Defines an adverse situation in the business, or the target of the proposed project. 2) Provide measurable evidence of the problem's existence. (problem description in terms of poor outputs. 3) should include preliminary data as possible, with the understanding that it will be updated as the project moves along and more data becomes available. ****The measure stage of the DMAIC model is an opportunity for the team to refine its charter through direct acquisition of data. ****The presence of objective data is important because it justifies the decision to direct resources toward the project.

Which of the following would typically be described on a written procedure or a work instruction? Procedure: What ,Why; Work instruction: Who, How, When, Where Procedure: What ,Why, Who, How; Work instruction: When, Where Procedure: What, When, Where, Who; Work instruction: How , Why Procedure: What, Why, When, Where; Work instruction: Who, How

Procedures are written to describe: What is done during the process Why it is done Where it is done When it is done Work instructions explain two other important aspects: Who does what How it is done Who does what" is the main concept of a work instruction. Usually "when and where "are considered on a procedure.

Process A = C/T: 1 minute Process B =C/T : 4 minutes Which process generates excess inventory?

Process A is pushing more inventory to process B. Process B has more cycle time to complete. The output generated by process A is excess inventory. Process B has more cycle time to process the inventory than process A. It is not generating excess inventory.

Process Behavior Charts:

Process Behavior Charts: a. Pictorial of process variation while the work is being done. b. Ensures that process is stable and continues to operate within the process boundaries established for that process. c. Variable data is continuous and comes from scales or measures d. Attribute data is discrete and comes from indicators. e. Ensures that the measurement from the process are recorded, calculated, and plotted correctly.

Define Process Benchmarking

Process Benchmarking 1) Focuses on discrete work processes and operating systems, such as the customer complaint process, billing process, or the strategic planning process. 2) Used to identify the most effective operating practices from many companies that perform similar work functions.

Process Capability Index, Cpk

Process Capability Index, Cpk Description: Cpk is a short term process index that numerically describes the "within subgroup" or "potential" capability (Ppk is long term indicator) of a process assuming it was analyzed and stays in control. It is an option (along with Z-score and PPM) when describing process baseline measurement in the MEASURE phase or in the CONTROL phase when obtaining the final performance analysis. As with all the process capability indices, the process should be in control before assessing capability. Use time-series and SPC charts to determine process control. If the process is out of control (i.e. still rising with upward trend), then assessing the current process is not likely to reflect the long term performance. Cpk Calculation The Cp is the best a process can perform if that process is centered on the midpoint and the Cp = Cpk. The addition of "k" quantifies the amount of which a distribution is centered. A perfectly centered process where the mean is the same as the midpoint will have a "k" value of 0. The minimum value of "k" is 0 and the maximum is 1.0. Both Cpk and Ppk relate the standard deviation and centering of the process about the midpoint to the allowable tolerance specifications. An estimate for Cpk = Cp(1-k). and since the maximum value for k is 1.0, then the value for Cpk is always equal to or less than Cp. Cpk will never exceed the Cp. Similar to Ppk, the Cpk capability index is only a function of the standard deviation and mean of the data, not a nominal (target) value that may be historical or provided by the customer. The Cpm capability calculation accounts for a nominal value. Cpk also requires input from the customer for the lower specification limit (LSL) and upper specification limit (USL). There are two calculations from the formula providing two values for Cpk. Select the MINIMUM value as the Cpk and to serve as the baseline value. This minimum value must be equal to or greater than the minimum acceptability level. Unlike Cp (and Pp), the Cpk (and Ppk) index can be calculated using unilateral or bilateral tolerances. If only one specification is provided (unilateral) the use the value that involves that specification limit. There will not be a minimum (or maximum), just calculate using the formula that has a specification and use it for the Cpk value. The standard Cpk level is 2.0 for a process in six sigma quality control . The acceptability levels for Cpk depends on your customer.

Process Capability Indices

Process Capability Indices a. Used to quantify process capability in a single number. b. Focus on variable (continuous) data c. C(pk) ≥ 1-→ process is capable (natural process limits lie inside the specification limits); Six Sigma requires specification limit C (pk) = 2 or +/- 6Φ d. 6 Φ process= where σ ≤ [1/12 * specification) + process average ≤ 1.5 σ e. Percent violating the specification limits is based on Z-values corresponding to 4.5 σ f. C(p)--→ shows how good C(pk) could be if process were centerd (Cp--doesn't take into account whether the process is centered in the specification. ) g. C( r) --→ shows percent of specification used up by process variation; (Lower Values of Cr better) ----------------------→ [Cr= inverse of C(p)];

Process Capability Indices

Process Capability Indices a. Used to quantify process capability in a single number. b. Focus on variable (continuous) data c. C(pk) ≥ 1-→ process is capable (natural process limits lie inside the specification limits); Six Sigma requires specification limit C (pk) = 2 or +/- 6Φ d. 6 Φ process= where σ ≤ [1/12 * specification) + process average ≤ 1.5 σ e. Percent violating the specification limits is based on Z-values corresponding to 4.5 σ f. C(p)--→ shows how good C(pk) could be if process were centerd (Cp--doesn't take into account whether the process is centered in the specification. ) g. C( r) --→ shows percent of specification used up by process variation; (Lower Values of Cr better) ----------------------→ [Cr= inverse of C(p)];

Process Capability Study Objectives

Process Capability Study Objectives 1) Objective-----> to establish a state of control over the manufacturing process and then maintaining the state of control through time. 2) When natural process limits are compared with the specification range, the following options for course of action : a) Do Nothing------>if process limits are within the specification limits b) Change Specification Limits------> If they are too tight and need customer's approval for modification . c) Center the Process-----> if the process spread is the same as specification spread, then centering will bring the bulk of product within specifications. d) Reduce Variability -------> Partition the variation (within piece or batch-by-batch) and tackle largest offender first by using Experimental design to find the source. e) Accept Losses------> Accept the high loss rate and focus on efficient handling of scrap and rework

Process Performance Measures: Process Capability indicies

Process Capability indicies C(p)= ratio of the tolerance to six sigma or the upper specification limit minus the lower specification limit dived by six sigma (USL- LSL)/ 6SIGMA 1. C(p) >1 means that process has the potential of having > 99.73% of outcomes within specifications 2. Upper Specification Limit (USL)= largest specification limit that can occur in a process ans still meet the customer specifications. C(pk)= lesser of the USL minus the mean divided by 3 sigma minus the LSL divided by 3 sigma. The greater the C(pk), the better. 1. Processes are 3 SD away from the mean when the Cpk =1 *****Capability ratio (Cr) is the ration of 1 divided by C(p). The lower the value for Cr, the better, with 1 being historical maximum.

Process Identification

Process Identification 1) Two primary tools used to identify the process boundaries are: a. Basic process model (input→ process→ output) b. Supplier-input-process (SIPOC)—output model 2) Projects flounder due to lack of clear boundaries for the project 3) Processes can affect multiple departments and organizations

Process Owners

Process Owners 1) Coordinates improvement activities & regularly monitors progress. 2) Works with black belts to improve processes 3)Trained in core statistical tools but need to gain understanding of techniques to improve individual processes. 3) May be six sigma sponsors or champions

Process cycle efficiency metric

Process cycle efficiency metric The measurement of process cycle efficiency is used to determine the most useful and positive ways to improve cycle time. The process cycle efficiency metric is used most often during the analyze stage of DMAIC.

Process input, Outputs, and Feedback

Process input, Outputs, and Feedback 1) Must measure a process before it is improved 2) Process is measured by identifying process input variables, process outputs, and documenting their relationship via cause-and-effect diagram, relational matrices, flow charts and other tools 3) Measurement of Process Inputs (raw materials, human resources, or results from upstream process) can be used to optimize and control an upstream process. 4) Process input requirements should be stated so key measures of input quality can be controlled.---> Once the process capabilities are known, output measures can be used to determine if the process has remained in control. 5) Feedback from downstream process measurements can be used to improve an upstream process. 6) Planned experimentation ----isolating the effects of different, independent process variables----and Designing for Six Sigma ---eliminating potential sources of error-----are applied to complex inter-relationships found in the organizational feedback system

Process of Autocorrelation Function

Process of Autocorrelation Function: 1st Step= testing for autocorrelations between each of the isolated observations. a) Lag 1 autocorrelation= Each step will be considered in relation to the steps immediately before and after it. n. b) Autocorrelations for distances----> lag 3 autocorrelation = examines relations b/w the 1st & 4th observations, 2nd & 5th observations, and so on. c) Formula for Estimation of lag m autocorrelation function --->, where n is the total number of observations and is the X-bar average of these observations. ( know the formula for estimation of autocorrelation) ***The general recommendation suggests testing for autocorrelations from lag 1 to lag n/4, where n is the total number of observations

Process of Benchmarking

Process of Benchmarking Benchmarking is a sequential process to assess efficiency and productivity against known targets. The first step in benchmarking is to select KPIs that are relevant to the process and to proceed until data is gathered and compared against targets. The goal of benchmarking is to improve business and manufacturing strategies. Select KPIs > select benchmarking targets > collect data > analyze data against benchmarks > improve strategies

Process of estimating CI on proportion

Process of estimating CI on proportion 1) To estimate the confidence interval on proportion---> start calculating the average error rate for n sample units. 2) The confidence interval is then calculated with the formula (see formula) 3) As with estimate of the confidence interval on proportion, this operation assumes that the samples are taken from a population with a normal distribution. 4) Also, the confidence interval is two-sided, running from one side of the mean to the other. The z value can be obtained from a table in appendix

Processing/ Muda

Processing/ Muda Addition steps in manufacturing process such as: 1. Having to remove burrs from manufacturing process 2. Reshaping a piece due to poor dies 3. Adding extra handing process due to lack of space 4. Performing inspection step since all inspection is NVA 5. Keeping extra copies of information.

Define Project Documentation

Project Documentation 1) Proposal usually in response to an improvement objective, project plan and budget. 2) Approval of proposal is management's indication of support for the project objectives and commitment to provide funding and resources. 3) During project implementation, status reports are the communication vehicle to management/ customer on the progress of the project

Project Evaluation & Review Technique and Critical Path Method

Project Evaluation & Review Technique and Critical Path Method 1) Software packages that identify the critical path for projects with multiple path 2) Delay in critical path-→ entire project delayed 3) "Crashing the project"=Putting more resources in the activities along the critical path --→ project completion in shorter time because reduced time for one of the activities along the critical path

Project Planning tools

Project Planning tools ---The number of tools and depth of data is determined by the size and scope of the project. 1) Project Charters 2) Project management tools 3) Milestone charts 4) Gantt charts 5) Critical path method (CPM) 6) Program evaluation and review technique (PERT) 7) Project schedules 8) Goal & objective statements or bullets.

Project Team member

Project Team member a. Selected by process owner and trained in 6 Sigma methodologies, quality, basic statistical tools, and process improvement techniques. b. Duties: i. Charter and project scope definition ii. Project meeting input + brainstorming iii. Data collection iv. DMAIC process v. Provide input to green & black belts

Project Teams/Task forces/ Ad Hoc Teams

Project Teams/Task forces/ Ad Hoc Teams 1. Team members selected based on experience and directed by management to look into specific areas of the project---> can be broad or specific member selection and may consist of all or part management. 2. Team disband on project completion 3. Team membership can be all management, all work area, or composite of the two---> assignment boundaries are tightly drawn for the team.

Project Charter

Project charter =overview of project a) Changes as the project moves along, so team members and sponsors will remain on the same page. b) Has clear deliverables, so its success or failure can be measured. c) Directs employee efforts to the right areas because it will identify the key variables that affect performance at all levels. d) Ensure that a proposed project does not interfere with other work performed in the company, because it will make public the intentions of the team. e) Ensures that Six Sigma efforts are directed to the most important aspects of performance, and that they deal with processes integral to the business in the future. " ****** Requires Management approval to give the project legitimacy and informed guidance.

Which of the following statements best explains how Six Sigma projects are related to organizational goals? A) Organizational goals determine the Six Sigma projects that should be executed for all processes of the organization. B) Project selection criteria should be aligned with the mission of the organization and its related quality, service, and cost goals. C) Project goals are determined by the process being measured and will necessarily optimize the organization for achieving its goals. D) Six Sigma projects identify process output measures that are used to establish broader goals for the organization

Project selection criteria aligned with the mission of the organization and its related quality, service, and cost goals is the correct answer. Organizational goals should be used to identify valuable Six Sigma projects to be launched for key processes that support the goals. Organizational goals should be used to identify valuable Six Sigma projects to be launched for key processes that support the goals. Setting goals based simply on the process in question can sub optimize the organization. Six Sigma projects being identified as process output measures that are used to establish broader goals for the organization is a backward response. Without consideration of organizational goals, a Six Sigma project may pick goals that are inappropriate or that have adverse impact on the organization.

Pull System

Pull System a. Just-in-time concept b. Links accurate information with the process to reduce waiting and overproduction. c. Traditionally, inventory overstocked and finished goods pushed to the next process. d. In pull system, process produces only when there is a pull from the subsequent process, which is signaled either by empty bin or kanban card.

A Green Belt is preparing a SIPOC for the purchasing process in a construction company. She interviews the Purchasing Manager, who is concerned that she does not receive accurate and complete purchasing information. This prevents her from processing purchase orders in a timely manner. Which of the following is correct regarding the purchasing information? The purchasing information is an input to the purchasing process. The purchasing information is outside the scope of SIPOC for this process. The purchasing information is owned by the Purchasing Manager. The purchasing information is an output of the purchasing process.

Purchasing information is a key input to the purchasing process and will appear on the SIPOC chart.

A process improvement team is reviewing the purchasing process at a manufacturing site. The project team includes a Green Belt, the Production Manager, the Warehouse Manager, the Purchasing Manager, and three Line Operators. Which of the following is considered a process owner in this case? A) The Production Manager B) he General Manager of the site C) The Warehouse Manager D) The Purchasing Manager

Purchasing manager is the process owner of the purchasing process. The general manager can be the sponsor but not the process owner in this case. The production manager & WAREHOUSE MANAGER = stakeholder and not the owner.

Purpose of FEMA

Purpose of FEMA--→ to understand the opportunities for failure and the impact of risks in a product or process design, prioritize the risks, and take actions to eliminate/reduce the impact of risks (front-end tool). f. Reviewing FMEA on continual basis ensures sustainable success. g. Use of the scale criteria and calculate the risk priority number (RPN)

Lean Concepts & Tools-- "Push - pull Strategy"

Push- Pull Strategy: 1) A push-pull system in business describes the movement of a product or information between two subjects. On markets the consumers usually "pull" the goods or information they demand for their needs, while the offerers or suppliers "push" them toward the consumers a) Move towards value streams in which production decisions are based on the pull of customer demand. 8) Perfect pull-based flow-→ customer order will trigger production of a replacement item in finished goods inventory before it is needed by the customer and be delivered in timely manner.

5 Quality Developments over the Years:

Quality Developments over the Years: a. Quality Control Circles -(Japan origin)---self-improvement study groups consisting of employees & their supervisors b. Statistical Process Controls -the use of statistical techniques to control processes . c. International Standards Organizations---set of international standards on quality management and quality assurance used to help companies to document the quality system elements that increase efficiency. 1. ISO 9000--- set of international standards on quality management and quality assurance developed to help companies to effectively document the quality system elements to be implemented to maintain an efficient quality system. d. RE-engineering—restructuring of the entire organization and its processes e. Benchmarking

Quality Function Deployment

Quality Function Deployment 1) Tool --"Voice of Customer" or "House of Quality"----ensures that customer wants and needs are heard & translated into technical characteristics by a company through design function or via crossfuntional team of sales, marketing, design engineering, manufacture engineering, and operations. 2) Flexible, customizable, used by all staff, and works for both service & manufacturing industry. 3) First applied by Kobe shipyards (Yoki Ako ) but introduced to US by Don Clausing 4) Matrix/graphical method of expressing relationship between customer wants and desired product/ service attributes & design features. 5) Relationship matrix= primary matrix b/w customer wants and design. 6) Can be used for product features in a model or to reduce cycle time as a concurrent engineering tool

An electronics company is working on the design of a new smart phone. The company collected feedback data from its customers on their requirements and expectations on the new phone. Which of the following tools can help the company in integrating customer feedback into the design features of the phone? Control charts Design of Experiments Quality Function Deployment Stratification

Quality Function Deployment (QFD) integrates customer requirements (voice of the customer, or VOC) into the design and delivery of a service offering. Stratification is a technique that separates data gathered from a variety of sources so that patterns can be seen. DoE are for planning, conducting, analyzing and interpreting controlled tests to evaluate the factors that control the value of a parameter or group of parameters, not for data collection

Rational Subgrouping

Rational Subgrouping 1) Subgroups are selected in a away that makes each subgroup as homogeneous as possible and that gives the maximum opportunity for variation from subgroup to another. 2) Selection depends on knowledge of the component of the total process variation 3) In production control charting, must maintain the order of production. -----> "Out of Control" charted process may be mixed to create new X-bar -R chart which demonstrates remarkable control ------Why? By mixing, chance causes are substituted fro original assignable causes as a bases for the difference in the subgroups. ***The basic idea behind rational subgrouping and forming subgroups is that you want to minimize the opportunity for variation to occur within a subgroup. This means that you want to form the subgroup under conditions that are essentially the same. As explain in the last newsletter, the average range is used to set the control limits on the X chart. So, the within subgroup variation from the range chart is used to determine how much variation there can be between the subgroups on the X chart. So, the X-R control chart is really answering the following question: Are there any significant differences in the subgroup averages when we take into account the within subgroup variation?

Rational Subgrouping Definition

Rational Subgrouping: 1. Definition: Method used to select samples for control chart. Rational Subgroup is a sample set that is sufficient to determine the common cause scenarios. It is applied to enhance the randomness and reduce " piece-to-piece" Variation.

5 Reasons for intentional overproduction:

Reasons for intentional overproduction: i. Unreliable machinery ii. Long setup time iii. Just-in-case philosophy iv. Unbalanced workload v. Amoritization of capital cost

Reasons that process improvement methods use teams to represent stakeholders & process owners:

Reasons that process improvement methods use teams to represent stakeholders & process owners: a. Stakeholders have best knowledge about the process., have best ideas for process improvements, are most aware of unintended consequences of process changes, and b. Stakeholder buy-in required to implement real process improvement.

Recognition & Rewards

Recognition & Rewards 1) Enforces repetition of positive behavior. 2) Individual rewards/recognition best received when they are personal to the individual receiving them 3) Unique awards are move valued than repeated recognition. 4) Sincere"thank you" is the best reward 5) Team rewards should be the same for everyone 6) Intangible rewards are given as result of activities

Repeatability: Equipment Variation (EV)

Repeatability: Equipment Variation (EV) This is the "within appraiser" variation. It measures the variation one appraiser has when measuring the same part (and the same characteristic) using the same gage more than one time. The calculation is given below. where K1 is a constant that depends on the number of trials. For 2 trials, K1 is 0.8862. For 3 trials, K1 is 0.5908. For this example:

Residuals analysis

Residuals analysis Residuals = differences b/w observed value and a regression model's predicted value for that response. Residuals analysis of a regression model = shows unusual patterns (aka error in the model). a) Statistical software programs such as Excel will calculate a standardized residual , such that the variance is set to 1. This makes outliers more obvious. b) Normality Test (MC technique of residuals analysis) = test for randomness of error by creating a distribution of the residuals. i) Random error follow a normal distribution with a mean of zero. ii) Residuals on a scatter diagram = assess the independence of a variable. "

"Response surface analysis

Response surface analysis 1) Techniques for calculating the best response value or setting for processes/ products. (eg. best possible result of an industrial or service process) or the service response. 2) Improve Stage : Maps the response surface so the effects of varying certain factors can be predicted (+) . to find the operating conditions that produce the desired specifications. 3) Response surface analysis cannot be begun until a 1st-order regression model has been created from the critical set of significant factors---> Screening designs used to develop the critical set of significant factors.

Response surface analysis Interpretation

Response surface analysis Interpretation a) Response surface analysis generates charts that can be subjected to analysis of variance. a) Example= F statistic used to compare the sum of squares variation caused by pure error with the sum of squares variation caused by curvature. ----> 1) If the curvature is significant--->point is close to a local maximum, minimum, or mini-max (that is, a stationary point). 2) If the stationary point falls outside the experimental region---->, get new data to enlarge the region 3) If data point falls in a region that would be impossible to reproduce---> Apply constrained optimization to the steepest ascent methodology. 4) If the stationary point is determined to be a mini-max---> Apply constrained optimization techniques to the steepest ascent methodology. 5) Analyze the full results after adding axial points on design at the new centerpoint (aka at places where they will achieve rotatable orthogonal design) ----> Collect new data around the optimal point to verify the new counterpoint

Risk Management--7 Continuous Process steps

Risk Management-- 7 Continuous Process steps" 1) Identify : Search/ Locate risk before they become a problem 2) Analyze ---Transform risk data into decision-making information 3) Plan---Translate risk information into decision actions 4) Track-- monitor risk indicators and action throughout the project 5) Mitigate--Reduce the impact 6) Communicate--Provide visibility & feedback data, internal and external to project on current or emerging risk Communication = Key element to a successful Risk Aversion Program.

Risk assessment assignment matrix

Risk assessment assignment matrix a. Ranks each risk on sale 1-3 for severity & probability. b. Assignment number = (severity rating) (probability rating)

Risky Shift

Risky Shift Definition ---a group collectively agrees on a course of action that is more extreme than they would have made if asked individually. Two Solutions: 1) Discuss Risky-Shift openly in the training 2) Have team member ask "If it were my personal mony, would we still risk it on proposed solution"

Role of Stakeholders in a process:

Role of Stakeholders in a process: a. Maintenance staff b. Process design staff c. Internal & External Suppliers d. Internal & External customers e. Process operators & managers from all shifts.

Run Charts Interpretation

Run Charts ***A Run Chart is a basic graph that displays data values in a time sequence (the order in which the data were generated). 1) Identify shifts int eh process mean or provide information about sampling errors. 2) May identify errors in the statistical control chart 3) Generally, they find error in the data collection stage before control chart generated. 4) Poor at locating the source of error----> user has to go back and analyze data to find the location.

A plot of data arranged in time sequence is a: Histogram Scatter Diagram Box-and-Whisker Plot Run Chart

Run charts are arranged in time sequence to reveal patterns in the data over time.

An improvement team is looking into ways to improve the approval timeline of personal loans at a major bank. Loan applications have to be reviewed and processed by several internal departments. These departments in turn will request feedback and credit reports from various external parties before approving the application. Which of the following tools will help the team in formulating a high level understanding of the process and its key players? A)Matrix Diagram B) SIPOC Diagram C) Relations Diagram D) Prioritization Matrix

SIPOCs help improvement teams in defining the boundaries of the project and defining the suppliers-inputs- process- outputs and customers of a process. Matrix Diagram shows the relationship between 2, 3 or 4 groups of information Relations Diagram helps analyze the natural links between different aspects of a complex situation Prioritization Matrix is a decision making tool

Sampling Methodis with Respect to Statistical Process Control

Sampling with Respect to Statistical Process Control 1) Random Sampling a. Measurements made within a batches are not a subgroup b. Used for batch processing-→ (1) pick random samples (2)average the measurements of the samples (3) plot the samples as one data point of a subgroup. 2) Systematic Sampling a. Used when performing individual moving range SPC monitoring by smapling every nth part. b. Used when pats are coming out of the conveyor c. Used in transactional process situtions-→ assessment of service quality by sampling every nth customer. 3) Subgroup Approach to Sampling a. Used for plotting X-bar & R-charts or X-bar & s charts. b. "Within group" variations should contain only chance causes-→ reason shy consecutive parts are sampled in the X-bar chart. c. Subgroup intervals should be planned to capture special causes.

Select & Scale Process Variables

Select & Scale Process Variables Process variables include both inputs & outputs ---ie. factors and responses. Team should: 1) Include all important factors ( based on engineering/ operator judgements) 2) Be bold in choosing low and high factor levels 3) Avoid factor settings for impractical or impossible combinations 4) Include all relevant reponses 5) Avoid using responses that combine two or more process measurements. For range of settings for input factors---> avoid extreme values.

Selecting Team Members

Selecting Team Members When selecting a team, upper management identifies those parts of the organization that are associated most closely with the problem. There are four places to look: 1) Where the problem is observed or the pain is felt 2) Where sources or causes of the problem might be found 3) Among those with special knowledge, information, or skill 4) In areas that can be helpful in developing the remedy ***In forming cross functional people, must look for people with expertise.

Shewart--Quote

Shewart--Quote Both pure and applied science have gradually pushed further and futher the requirements for accuracy and precision. However, applied science, particularly in the mass production of interchangeable parts, is even more exacting than pure science in certain matters of accuracy and precision.

Short-term Vs. Long-term Capability Traits of Short-term capability index (CP)

Short-term capability index (CP) Determining the width between the two rigid specification limits is easy; it is simply the distance between the upper specification limit (USL) and the lower specification limit (LSL). But with variation that trails out at the tails, how do you determine the width of the process? To get over this hurdle, Six Sigma practitioners have defined the effective limits of any process as being three standard deviations away from the average level. At this setting, these limits surround 99.7 percent, or virtually all, of the variation in the process. Figure 13-9 shows these limits graphically. image0.jpg So to compare the width of the specification to the short-term width of the process, you use the following formula: Cp= (USL - LSL) / (6 * Standard deviation (st)

Which of the following is true about short-term process capability? It is generally calculated under controlled conditions with a small number of subgroups (20-30). It is represented by Pp and Ppk. It is usually about the same as long-term process capability Cp and Cpk are usually smaller than Pp and Ppk

Short-term process capability is usually just a starting point to understand the process. Further exploration is needed to understand sources of variation and drift over time/operating conditions. Actually Cp and Cpk are usually larger than Pp and Ppk due to variations in the process over time and under less controlled conditions. Actually Cp and Cpk are usually larger than Pp and Ppk due to variations in the process over time and under less controlled conditions.

Short-term vs. Long-term Capability

Short-term vs. Long-term Capability 1) Process Average and Spread----> dependent on number of units measured or duration that process is measured. 2) Small Variation ----> if process with one operator on one shift with one equipment 3) Increased variation as more factor are included----time, multiple operators, lot materials, environmental changes, ect. Smaller the data amount, lesser the variation. 4) Control limits for short-term process evaluation are closer together than control limits for long-term process and produce false, out-of-control patterns. 5) Shorter Runs ---> use 3-10 peces without adjustment and modified X-bar and R chart-----> Calculated value is compared with critical value and then Control limits are established using inflated D4 and A2 values 6) Attribute Data vs. Variable Data---->variable data gives more information about the process----aka a resonable estimate of the process mean and variation using 25-30 groups of 5 samples each ****Better to use Variable Data for estimating Process Capability. 7) Attribute Data----> requires 25 groups of 50 samples each

A one-sample T study was performed on a supplier's lot: T study μ = 10 vs ≠ 10 and Alpha Value=0.05 The mean of 10 samples tested is 9.85. The 95% confidence interval of the population mean is 9.75, 9.95. P-Value = 0.046. A discussion with the process expert determined that the population mean can vary from 9.7 to 10.1 without impacting the product performance. Choose the best course of action: Fail to reject the null hypothesis since it is not statistically significant and reject the lot irrespective of practical significance. Reject the null hypothesis since it is statistically significant and reject the lot irrespective of practical significance. Reject the null hypothesis since it is statistically significant and accept the lot since it is not practically significant. Fail to reject the null hypothesis since it is not statistically significant and accept the lot since it is not practically significant.

Since the P value is lower than the Alpha value, the null hypothesis of μ = 10 vs ≠ 10 is rejected. However, due to the practical significance that there is no impact to the product, the supplier's lot is accepted

Term: One complete product proceed through various operations without interruptions, backflows, or scrap. (opposite of batch flow)

Single Piece Flow One complete product proceed through various operations without interruptions, backflows, or scrap. (opposite of batch flow)

Define Six Sigma,,,,,What is the Goal of Six Sigma?

Six Sigma (1995- present)—structured and disciplined process designed to deliver perfect products and services a. Goal of Six Sigma= Improves the bottom-line by eliminating the defects in the processes. b. Associated with process capabilities of Cpk>1.5 i. Six-Sigma Philosophy= All processes can be defined (DMAIC). If you control the inputs, then you can control the outputs ***Sigma= means Standard Deviation

4 Six Sigma Metrics & Mike Harry

Six Sigma Metrics & Mike Harry Mikel Harry and Richard Schroeder, two pioneers of Six Sigma: ----> "An organization's profitability is determined by what it chooses to measure and how it measures it." 1) They point out that most companies need to change what they measure. Many companies use customer satisfaction to measure success. They believe that the metrics of customer satisfaction can lead to the identification of Six Sigma projects, and that the results of those projects should increase customer satisfaction. 2) Harry introduced new group of metrics: a) Measure customer opionons b) Determine Customer CTQ factors c) Measure product outcomes (throughput yield, rolled throughput yield, normalized yield) d) Correlate process outcomes to CTQ (measure processes with metrics that correlate to the company's fundamental outcomes)

Process Performance Measures: Six Sigma, Tolerance limits, & Yield of Sigma process...

Six Sigma Process= 1. Example= Tolerance limits are 5.000 +/- 0.012 that is, 4.988 to 5.012. Data collected from the process from the second shift indicates that the process mean is 5.000 and its standard deviation is 0.004. + or - sigma fits inside the tolerance shift. 2. To calculate yield of Sigma process, must determine the area under the normal distribution curve that is + or - 3 standard deviations and gives a yield of 0.99973 3. If the process variation is reduced and sigma equals 0.002 and there is + or - 6 sigmas between the tolerance limits, the process is 6sigma process.

Which of the following best describes the Six Sigma philosophy? A) Daily activity sustaining continuous improvement in which people do rapid experiments to learn how to eliminate waste in business processes. B) Regularly scheduled rapid improvement events in which the right people come together to find ways to eliminate waste in business processes and implement their solutions very quickly. C) Teams working on short- to medium-term projects, using a standardized methodology to solve problems that directly impact the bottom line, exposing the "hidden factories," and using statistical methods to drive their decisions. D) A structured management approach for the entire organization, centered on quality, based on the participation of everyone, and aimed at long-term success through customer satisfaction.

Six Sigma emphasizes solving problems using statistical methods to drive the solutions using the DMAIC process. Correct = D) A structured management approach for the entire organization, centered on quality, based on the participation of everyone, and aimed at long-term success through customer satisfaction. Kaizen philosophy= Daily activity sustaining continuous improvement in which people do rapid experiments to learn how to eliminate waste in business process LEAN= Regularly scheduled rapid improvement events in which the right people come together to find ways to eliminate waste in business processes and implement their solutions very quickly is the philosophy behind lean.

Six Sigma vs. LEAN-???

Six Sigma vs. LEAN-: a. Six Sigma→ reduces process variation to enhance process control 1. Motorola developed Six Sigma in 1981 to improve the quality / effectiveness of business & manufacturing processes. 2. Quality management tools & statistical tools included---eg. creating special designations for individuals within an organization ("Green", "Belt") that reflect expertise in these tools and methods. 2. Unlike other quality improvement methodologies, Six Sigma is designed to yield measurable & quantifiable financial returns + creates strong leadership & mentorship---in both current and future business processes.. a) Designates ownership of certain skills to individuals within the organization, developing distinct expertise. LEAN-→ Remove wasts (Defect in process) + Standardization of work processes and value stream mapping. 1. Value stream mapping analyzes the flow of information /materials to develop a product/ service. 2. Value stream mapping =lean manufacturing concept= found in many industries i.e., healthcar & software production. The goal of value chain mapping is to quantify the input (information and materials) necessary to bring about a product or a service.

Special Roles---Gatekeeper & Boundary Spanners

Special Roles 1) Gatekeepers= individuals who are at the crossroads of communications channels and are center of information because of their jobs. 2) Boundary Spanners= individuals who have positions that link them with others outside of the work units and exist to exchange information between groups

Which of the following is true about using an Xbar and R chart to control and monitor a process? Specification limits apply to individual values while control limits apply to subgroup averages When the subgroup average falls between the control limits, the process isn't producing out-of-specification products. Subgroup averages fall between the control limits resulting in the process producing 3500 ppm out of specification When some subgroup averages fall outside the control limits then the process is producing at least some product out-of-specification.

Specification limits almost always apply to individual values. Control limits are used to compare subgroup averages over time to detect when a process changes. If the natural process limits are wider than the specification limits then even a process in statistical control will produce product out-of-specification. The subgroup averages being in statistical control doesn't mean the process is operating at a 6-sigma level. It could be worse or better. Remember control limits are for subgroup averages and specification limits are for individuals.

Stakeholder

Stakeholder a. Vested interest in the process and/or product's outputs. b. Customers, suppliers, employees, investors, and communities. c. Economic or contractual influences can change stakeholder interest and involvement.

"Stakeholders and team members

Stakeholders and team members 1) Stakeholders are all of the people and groups affected by the problem being addressed and potentially affected by the project as it progresses. 2) In Six Sigma, the team should use a top-level process map to identify all of the affected groups (customers, suppliers, other employees, and specific departments within the organization) . 3) Most projects are headed up by a single black belt, with one or more green belts representing each relevant department. 4) Some projects may require the participation of experts or skilled employees. 5) In rare cases, a Six Sigma project team will include a facilitator to ensure that all members of the team work together effectively. "

Statement of objective and identification of resources

Statement of objective and identification of resources 1) A complete project charter should include a statement of objective in which the author provides a definition for the intended result of the project. a) Six Sigma experts recommend expressing this result in financial terms. b) Other measures (defects per million or return on investment) have more variation and unforeseen influences. 2) A complete project charter also will identify the resources ( equipment, materials, computer time, databases, and other employees) required for the project but excludes the team's efforts/ time ***Resources required by a project are not only those resources necessary to improve the process, but also those necessary to collect and analyze data along the way.

Statistical Process Control & Preventing deviation

Statistical process control & Preventing deviation 1) Detection after the mistake = more expensive, time-consuming, and difficult. 2) Must control the inputs that lead to process variation. 3) Statistical process controls monitor the stability of input and output variables, although this process is more effective at monitoring inputs. a) If the process capability index Cpk is 1.5 or more, then a control chart may be used to assess the Six Sigma-level outputs are good enough. b) Statistical process control is one of the three common methods for controlling processes: the other two are engineering process control and operational procedures.

Steepest ascent methodology in phase 1 applied first-order regression model.

Steepest ascent methodology in phase 1 applied first-order regression model. 1) Data points gather at the steepest path beginning with the design center, or the spot where (x1, x2) = (0, 0). 2) Design center is the first test condition. 3) Steepest ascent is determined by moving β1 coded units in the x1 direction for every β2 coded units in the x2 direction, where β1 and β2 are the coefficients of the x1 and x2 terms, respectively. a) If the small changes, then possible to get a good picture of the response will be possible. b) Given a specific change in uncoded units for x1-->, can find values for x2, β1, and β2. c) Can determine the path of steepest ascent relative to the physical limitations of the system, such as if either x1 or x2 cannot proceed beyond a certain point. c) Determine the local maximum conditions by looking for the point at which the response begins to diminish---> Next arun another experiment near this point to obtain a first-order[...]"

Steering Committee Roles--

Steering Committee Roles 1) Setting goals----> top mgmt identifies opportunties, strategic goals, and improvement needs 2) Identifying projects---> Selection of projects critical to meeting quality and other goals 3) Selecting Teams 4) Supporting Project Teams 5) Monitoring progress.

Stem and Leaf Plots

Stem and Leaf Plots-- John Tukey 1) Manual method for plotting both categorical and variable data sets. 2) Data are grouped by class intervals as "Stem" ------smaller data increments = " Leaves" 3) Allows for data to be read directly from the diagram whereas histogram may lose individual values as frequencies within the class interval.

Stem-and-leaf plot

Stem-and-leaf plot a. Used to quickly identify repetitive data within class interval b. May get measurement errors fi the data values are not evenly distributed in the cells. c. Like tally column but the last digit of the data is recorded i. Leaf unit= tells which unit is used (pg. 163 + Khan Academy video on reading stem-plot)

What kind of plot takes the data points of 12, 13, 18, 18, 22, 22, 27, 29, 31, and 35 to show: 1: 2388 2: 2279 3: 15 Box-and-whisker Stem-and-leaf Weibull Histogram

Stem-and-leaf plot is text-based analysis with the last significant digit of each value becoming the stem and other digits the leaf. The box-and-whisker is a graphical plot showing the population.

Steps for Process Capability Studies Identification of rational sample subgroups for the plotting a control chart

Steps for Process Capability Studies Identification of rational sample subgroups for the plotting a control chart a. Subgroups size of 2-10 (usually less than 5 is best) b. Stability of process is observed on the control chart by plotting consecutive samples taken at equal intervals from a process VS. average/ range c. Must make sure that "within subgroup variation" is less than "between subgroup" variation. d. Low-volume processes (Subgroups size= 1): i. Individual charts are plotted to monitor the stability where the subgroup size= 1 ii. When subgroups size= 1-→ less sensitive in detecting shifts in processes e. High volume processes (Subgroup size > 10)-→ use Average Standard deviation charts (best for detecting shifts in processes but costly) f. Average range chart--→ more economical choice for detecting shifts in processes

Steps for conducting Gage R&R study

Steps for conducting Gage R&R study g. Planning: Check equipment calibration & availability of appraisers & supervisors. Inform apprasiers of measurement criteria and inspection method h. Sample Selection: Hand-pick samples covering the spread (avoid random sampling). Appraisers should not be involved in sample selection process. i. Data tracking: Create table for experimenting and comparing samples between trials and between appraisers j. Trials -Each appraiser within the group must conduct measurements of all samples for every trial to ensure completeness of the study (data imbalances if the study incomplete) k. Data Collection Sheet- data can be collected either in the randomized data collection sheet or by using the tabular calculation sheet. l. Calculations: (See formula on page191)

Recent sales reports for an online store show steep decline in sales figures. The store sells three main categories of products: smart phones, computers and printers. The store customers are concentrated in four regions: North, South, East and West. The store performs online surveys for its customers, the data is lumped together and no reasons for decline in sales can be seen. Which of the following tools can help this store in finding patterns in the collected feedback data? A) Kano Analysis B) Quality Function Deployment C) Stratification D) Kaizen

Stratification is a technique used in combination with other data analysis tools. When data from a variety of sources or categories have been lumped together, the meaning of the data can be impossible to see. The Kano model is useful in gaining a thorough understanding of a customer's needs. QFD is for determining customer preferences and not analyzing data. Kaizen is a process improvement tool not data analysis methodology.

Stratified sampling

Stratified sampling i. No homogeneity in the lot (Mixture of parts from different machines, different streams, different raw material lots, different process settings) ii. Like random samples, stratified random samples are used in population sampling situations when reviewing historical or batch data. iii. Stratified random sampling is used when the population has different groups (strata) and the analyst needs to ensure that those groups are fairly represented in the sample. iv. In stratified random sampling, independent samples are drawn from each group. The size of each sample is proportional to the relative size of the group. 1. For example, the manager of a lending business wanted to estimate the average cycle time for a loan application process. She knows there are three types (strata) of loans (large, medium and small). Therefore, she wanted the sample to have the same proportion of large, medium and small loans as the population. She first separated the loan population data into three groups and then pulled a random sample from each group

List the 8 ways the Steering Committee Supports the Six Sigma team

Supporting Project Teams 1)Training a) Makes sure that improvement teams are well equipped to carry out the project b) Training in team tools and techniques + training black belts & green belts 2) Logistics/ meeting sites 3) Communicating project results throughout organization 4 ) Securing Resources 6) Monitoring Progress a) Feedback on group effectiveness b)Summarizing Points made by group 7) Coaching leader or participants 8) Managing Relationships & conflict a) Assessing change processes, cultural barriers, b) Helping with interpersonal difficulties c) Clarifying points of view on issues d) Asking feeling on sensitive issues

Systems Thinking

Systems Thinking 1) Definition= Recognition/identification and consideration of all the various individual elements that interrelate with a common purpose toward a whole function of a unit. The use of tools and methods available to understand what is being done at a specific operation and how that activity affects tasks and products further downstream and how the products/tasks affect the process being reviewed. 2) Process behavior charts (aka control charts) must go out of control to show an improvement in the system. 3) Thinking that we can have both continuous improvement and control charts that are always in control is not systems thinking.

Team Dynamics: Reluctant participants

Team Dynamics & Reluctant participants a. Symptoms=No feedback, uncommitted, disinterested. b. Causes: i. Fear of job loss for voicing opinions ii. Moving out of organization or current job function iii. Intimidated by team leaders iv. Team members don't have stake in team's outcome. 5) Unquestioned acceptance of opinions & facts a. Symptoms= Members present information without backing up data or anlysis b. Cause= organizational culture or lack of management by facts c. Countermeasures= Team leaders request for data analysis and conclusions that are statistically valid. Assumptions behind analysis are questioned.

Team Leadership & Norming

Team Leadership & Norming Norming-→ Supportive leadership style with less directive behavior a. Emphasizes ground rules, scope, roles, and responsibilities b. Team members allowed decision-making responsibilities c. Progressing towards Performing Stage. d. Norming is an important stage in team development and evolution. After disagreement and differing approaches, team members realize that they must agree on basic rules, values, professional behavior and methodology in order to bring their project to a successful completion. This is a participative stage whereby team members realize that collaboration as a team is just as important as the individual contributions of members.

Team Life Characteristics---Optimize Phase

Team Life Characteristics---Optimize Phase 1. High task/ high relationship team style---->Members prioritize and perform tasks 2. Team decisions are done in a caring way.---> Conflict is accepted but cooperation is preferred 3. Team leadership= delegator; low task/ high relationship

Team Problems & Solution----Attributions

Team Problems & Solution----Attributions 1) Members make casual inferences ---> Challenge assumptions 2) Members don't seek real explanations ---> Challenge assumptions 3) Members make psychological judgements ---> Ask for data to support conclusions

Team Problems & Solution----Feuding

Team Problems & Solution----Feuding 1) Win-lose adversaries---> Confront adversaries alone 2) Team takes sides---> Enforce team operating guidelines 3) Group pursues tangents---> Re-direct the discussion

Team Problems & Solution----Jump to Solutions

Team Problems & Solution----Jump to Solutions 1) members rush to accomplish something ---> Re-enforce need for data analysis 2) Members avoid data collection ---> Ask for alternate solutions 3) Members want immediate decisions ---> slow process down

Team Problems & Solution---Negative Nellies

Team Problems & Solution---Negative Nellies 1) Members say " We tried that already" ---> Reinforce positive 2) Members defend their turf ---> Ask for other points of view 3) Members are negative of suggestions ---> Separate idea generation from criticism.

Team Problems & Solution-Putdowns

Team Problems & Solution-Putdowns 1) A member's comments ignored ---> Encourage active listening 2) Members are not listening---> Encourage equal participation 3) Meaning of a suggestion is missed---> Talk to parties privately 4) Sarcasm noted---> Promote uniform idea consideration.

Techniques for assuring data accuracy and integrity 9 Common causes of errors: ???

Techniques for assuring data accuracy and integrity 1) Common causes of errors: a. Units of measure not defined b. Poor legibility c. Loss of precision (rounding errors) d. Distortion of data (emotional bias) e. Inconsistency f. Poor training g. Ambiguous terminology h. Clerical / typo errors i. Guesswork/ personal bias (inadequate use of validation techniques) j. Multiple points of data entry---inconstistency and errors

Techniques for getting to know customers better

Techniques for getting to know customers better 1) Don't use your instincts as research data 2) See the world from customer's side 3)The higher you are in the organization, the more out of touch you are 4) Get customers to talk 5) Do research to retain customers 6)Conduct research on customer expectations 7) Develop customer profile 8) Share results of customer research studies 9) Don't go overboard on the details and measurements 10) Coordinate and use research efforts (research may not help all situations)

When to use DMADV methodology, instead of the DMAIC ? Vice versa?

The DMADV methodology, instead of the DMAIC methodology, should be used when: a. Managers launching a new project are more likely to use DMADV project methodology. b. A product or process is not in existence at your company and one needs to be developed c. The existing product or process exists and has been optimized (using either DMAIC or not) and still does not meet the level of customer specification or Six Sigma level methodology, should be used when a product or process is in existence at your company but is not meeting customer specification or is not performing adequately 2) The DMADV methodology, instead of the DMAIC methodology, should be used when: a. Managers launching a new project are more likely to use DMADV project methodology. b. A product or process is not in existence at your company and one needs to be developed c. The existing product or process exists and has been optimized (using either DMAIC or not) and still does not meet the level of customer specification or Six Sigma level

A Green Belt is comparing two machines against one another to determine if the variance introduced by one machine is different than the variance introduced by the other. Which of the following tests would best be used in this application? ANOVA Chi-square test Correlation Analysis F test

The F test makes a statistical comparison between the variances of two data sets. The null hypothesis states that if two normal populations have the same variance or not . Correlation analysis often measured as a correlation coefficient, indicates the strength and direction of a linear relationship between two random variables. One-way ANOVA is used to test for differences among two or more independent groups. Typically, however, the One-way ANOVA is used to test for differences among at least three groups, since the two-group case can be covered by a T-test. Pearson's chi-square (χ2) test It tests a null hypothesis that the frequency distribution of certain events observed in a sample is consistent with a particular theoretical distribution. The events considered must be mutually exclusive and have total probability 1. A common case for this is where the events each cover an outcome of a categorical variable.

The T distribution has the following properties:

The T distribution has the following properties: a. The mean of the distribution is equal to 0 . b. The variance is equal to v / ( v - 2 ), where v is the degrees of freedom (see last section) and v > 2. c. The variance is always greater than 1, although it is close to 1 when there are many degrees of freedom. With infinite degrees of freedom, the t distribution is the same as the standard normal distribution. d. T-distribution is bell-shaped but with smaller sample sizes show increased variability (flatter) e. As sample size increases, the distribution approached normal distribution f. Mean = 0 g. Population SD unknown h. Variance >1 but approaches one from above as the sample size increases.

A company that makes axles wants to reduce the variation in the diameter of its axles. The long term standard deviation in diameter has been 1.98 mm. The data is normal. A new process has been developed which produced a standard deviation of 1.41mm on 12 samples What is the confidence that the company should place on the conclusion that the new process will produce a lower variation? 10% 90% 94% 80%

The Test Statistic X2=((12-1)*1.412)/(1.982)=5.578 From the Chi Squared Table with dof =11, the 90 % value for 90% confidence is 5.58. The test statistic is just less than the Chi squared vale for 90%, so 90% is the best answer 94% would be true if the dof = 12 but dof = n-1 = 11

You are conducting an experiment to compare the means of two different treatments of a product. You have computed a Z test statistic of 1.645 and reject the null hypothesis that the two means are equal. What is the probability that you have made a type 1 error? 90% 95% 10% 5 %

The Z statistic of 1.645 corresponds to a probability of a type one error in each tail of a two tailed test of α/2 = 5 % Therefore α = 10 %.

What does the DIAMOND symbol above on a flowchart represent? A yes or no decision point A pre-defined process A starting or ending point in a process. A connection of 2 process steps

The diamond symbol indicates a yes or no decision has been made based on a question asked. A starting or ending point in the process is symbolized by an oval. A predefined process is symbolized rectangle with a vertical line at each end of shape. A connection of 2 process steps is shown with arrows or other connectors.

What should be considered when designing a process capability study? Data collection should be planned to isolate any potential sources of variation so they can be evaluated first. If the output from the process is statistically stable, then a process capability study isn't necessary. Process capability studies are only useful if the process is producing a low level of non-conforming parts. You can set the process up to the most frequently used conditions for the study. Any other conditions will produce similar results.

The first step should always be to thoroughly review the process to understand all potential sources of variation. You may need to take preliminary data to understand if your capability study sampling plan needs to be able to separate out all the sources. Not at all true. A process can be in statistical control but still produce out of specification product. A process capability study to identify and reduce potential sources of variation may be useful. Actually the opposite is true. If the process is producing few non-conforming parts, then it might be better to spend your limited resources on a more troublesome process. It is well established that processes produce different results under different conditions such as speed, temperature, humidity, incoming materials, etc.

A green belt is reviewing the raw materials ordering process at a ready-to-eat food processor. She discovers that large quantities of ingredients are ordered and stored at different locations to be used at various steps. A visit to the processing floor shows congested and clattered space. The green belt creates a value stream map for the process and discusses it with the production manager, who says that buying large quantities is necessary to ensure availability of ingredients. Which of the following should be the focus of the green belt next? A) Ensure proper storage conditions of the ingredients. B) Identify and reduce storage space waste on the processing floor. C) Review the purchase agreements with the suppliers for better prices based on ordered quantities. D) Monitor and track expiry dates of ingredients.

The key identified issue is the waste in storage space occupied by all the raw materials and how this can be managed. Ensuring proper storage conditions of the ingredients is a quality assurance function and no information in the question refers to issues with storage conditions. Reviewing the purchase agreements with the suppliers for better process based on ordered quantities is irrelevant as the problem is the quantity of orders, not their prices. Monitoring and tracking expiry dates of ingredients is a quality assurance function.

After writing down their ideas and turning them in to the facilitator, a customer experience improvement team begins to collectively rank the results. This is an example of which of the following? Brain writing Nominal group technique Round robin brainstorming Popcorn brainstorming

The nominal group technique is an idea generating process. Team members write their ideas privately, and then each idea is written on a flip chart. Team members rank all ideas with point values which are totaled at the end. The idea with the highest number of points is selected for implementation. Brain writing is a nonverbal form of brainstorming. Team members write down their ideas on paper, and then exchange the papers with other team members. These ideas are used to motivate the team members to write down more ideas. The popcorn technique is a freewheeling method of idea generation. It is used when the group is comfortable expressing ideas in front of others. It encourages creative thinking. The round robin technique obtains ideas from team members by going around the group and having each person verbally offer one idea at a time and repeated until the group runs out of ideas.

Theory of Constraints & Modified Operations

Theory of Constraints & Modified Operations: 1) If the operations are modified so that each station requires the same amount time, the sequence of operations is "balanced"---> Any delay in Station 2 will effect the subsequent station. 2) Best to have plant unbalanced with greater capacity at all operations except at the designed constraint. ---keep available capacity at the end of the production sequence. 3) Economically, it is better to have operating unit that allows improvement to other stations---It is acceptable to have a machine wait on an operator if the overall customer objective is achieved. if total productivity maintenance (TPM) and human motivation are 100% levels and process variability is low

Theory of Constraints terms ---Dependent events & Statistical Fluctuations

Theory of Constraints terms ---Dependent events & Statistical Fluctuations: 1. Dependent events and statistical fluctuations in manufacturing plants are difficult to manage so the organization achieves its goal. 3. Drum-buffer-rope scheduling (DBR) is a plant scheduling approach, incorporating the inevitable dependent events and statistical fluctuations, used to maximize/ manage the productivity of a manufacturing facility. a) Differs from other manufacturing techniques---> Focuses on finding the relationships among resources to resolve conflicts and create a smooth flow of product b) Applicable to all types of process- es whether they are repetitive, process, or job shop. c)Drum-buffer-rope = protects/ minimizes impact of disturbances on smooth production flow

Theory of Constraints terms--Techniques used in TOC

Theory of Constraints terms--Techniques used in TOC: 1) Effect-cause-Effect---> Brainstorming used to gain intuitive sense of problems & causes. Make assumptions about each effects & speculate the causes. If the cause is a constraint, then eliminate. 2) Evaporating Clouds---> By re-examining the basic foundation of the problem, a simple solution may be found to complex problem. After changes made, problem evaporates. 3) Prerequisite trees---Something must occur before something else can occur. TOC is a transition tool from old way of doing things to the new way.

Three ways that calculations for process capability are different from process capability calculations

Three ways that calculations for process capability are different from process capability calculations *** Both are exactly the same in calculation process except for : 1) Historical data from control charts should be excluded but okay to include other forms of variation 2) If multiple machines are producing the same part, then the capability of each machine should be determined independently 3) Machine capability should come from consecutive part measurements from the same machine at or near the same time (perhaps 20 to 40 parts).

Tools used understand the organizations and functional areas involved with the process.

Tools used understand the organizations and functional areas involved with the process. a. SIPOC can identify the organization & functional areas as process suppliers and customers b. Flow chart c. Process map

Two Level Full Factorial Designs

Two Level Full Factorial Designs These are factorial designs where the number of levels for each factor is restricted to two. Restricting the levels to two and running a full factorial experiment reduces the number of treatments (compared to a general full factorial experiment) and allows for the investigation of all the factors and all their interactions. If all factors are quantitative, then the data from such experiments can be used for predictive purposes, provided a linear model is appropriate for modeling the response (since only two levels are used, curvature cannot be modeled). Two Level Fractional Factorial Designs This is a special category of two level designs where not all factor level combinations are considered and the experimenter can choose which combinations are to be excluded. Based on the excluded combinations, certain interactions cannot be determined.

Types of Communication flow-→ top-down, bottom-up, horizontal

Types of Communication flow-→ top-down, bottom-up, horizontal f. Top-Down Communication i. Communication of instructions, policies, or project performance feedback by senior management or executive sponsor. ii. Written letter or meeting best to present dilution of the message iii. Sensitive information should be communicated via chain of command to prevent middle-level management from feeling loss of importance.

Types of Constraints

Types of Constraints 1) Physical Constraints----Typically equipment or tangible items, (eg. material shortages, lack of people, or lack of space). 2) Policy Constraints--- Required or recommended ways of working. May be informal (e.g. described to new employees as "how things are done here"). Examples include company procedures (e.g. how lot sizes are calculated, bonus plans, overtime policy), union contracts (e.g. a contract that prohibits cross-training), or government regulations (e.g. mandated breaks). 3)Paradigm Constraints----Deeply engrained beliefs or habits. For example, the belief that "we must always keep our equipment running to lower the manufacturing cost per piece". A close relative of the policy constraint. 4) Market Constraints----Occurs when production capacity exceeds sales (the external marketplace is constraining throughput). If there is an effective ongoing application of the Theory of Constraints, eventually the constraint is likely to move to the marketplace.

Types of Matrix Diagrams

Types of Matrix Diagrams Types of matrix diagrams 1) Directional Matrix system----, arrows suggest the nature of the relationship between each pair of items. 2) Sophisticated directional systems ---have weighted arrows that indicate the direction of causation as well as the importance" 3) Numerical matrix diagram,----- numbers perform the same functions as weighted arrows. For example, on a ten-point scale a 5 might indicate a moderate relationship in one direction, while a 1/5 indicates a moderate relationship in the opposite direction." 4) Plus-minus system----- a plus sign indicates a relationship and a minus sign indicates the absence of a relationship. 5) Symbol system------, finally, triangles, circles, and squares indicate characteristics of each relationship. *****Matrix diagrams typically have final rows and columns that tally up the total importance of each item or process. The team then can identify the items and processes that have the most significance."

Types of Statistical Control Charts:

Types of Statistical Control Charts: a. P-chart is used to illustrate proportions b. C-chart is used when counting the number of nonconformities in an area of opportunity. ******Shows the total number of nonconforming items per unit such as the number of errors per page of code.. A C-chart assumes that the size of each area of opportunity from which the nonconforming items are drawn remains constant. c. Xbar charts are used measuring numeric data such as means

Value Stream (Value Chain)

Value Stream (Value Chain) 1. Kaizen will create islands of improvements; Real improvement is hard to identify 2. For product or business, there are 3 required value streams: a) Problem Solving Stream--->solve concept, design, develop prototype, plan review and determine mechanism of product launch b) Information Management Stream---> order taking from customer, raw material sequencing from suppliers, in-house scheduling, and delivery to the customer. c) Physical Transformation Stream--->Product realization in ISO 9001 ; conversion of raw material to customer finished goods

Value Stream Map

Value Stream Map 1. Identifies all activities in the product 2. Includes suppliers, production activities, and final customer 3. Activities are viewed in terms of 3 criteria. a) Adds values as perceived by the customer b) Adds no value but is required by the process c) Adds no value and can be eliminated.

Control Charts for Variable Charts

Variable Charts X-bar--R Chart (When data is avaiable) Run Charts (limited single point data) MX(bar) - MR Chart (limited data---moving average/ moving range) X- MR chart (limited data) X-S chart (when sigma is readily available) Median Charts Short Run Charts

Variable Charts:

Variable Charts: a. Data is reported from a particular characteristic of the process output in small subgroups of 2-5 sequentially taken methodically. b. Data to be plotted results from measurement on a variable or continuous scale. c. Generally used when each pair of values have infinite number of possible values. d. Smaller subgroups can be used because data is always continuous and variable. (use 25 subgroups

Given the batch size (lot size), inspection level (LTPD), and AQL value, which of the following sampling plans would be most appropriate to determine the sample size and acceptance criteria? ANSI Z1.4 Sequential plan Variables single for defective Stratified plan

Variable plans (ANSI Z1.4) are a plan based on measurement of individual items, not whether they are conforming or nonconforming. A sequential plan is a plan that treats each item as a sample of one. With this technique, the sample is built up one item at a time. After inspecting each item, ask yourself: "Can we be sure enough to accept or reject this batch on the information so far collected?" Stratified sampling is used if the population of parts to be sampled is naturally divided into groups. Variable SingleThis type of sampling is applicable only for measurable characteristics that are normally distributed. (1) Take a representative sample of n units from the lot to be inspected. (2) Measure each unit and calculate the average of the samples, denoted X. (3) For a lower spec limit (LSL), accept if X ≥ LSL + k S. For an upper spec limit (USL), accept if X ≤ USL - k S. For two spec limits, accept if LSL + k S ≤ X≤ USL - k S. Otherwise, reject the lot.

Design of Experimentation Variables, Independent Variables, & Independent Variables.

Variables: a. Dependent Variables i. Independent variable cause of the apparent change in the dependent variables ii. Dependent variable values are compared in research. iii. Dependent variable can't change without independent variable b. Independent Variables i. Most commonly an input in the experiment and does not have interactions with other variables

Verifying Stability and Normality

Verifying Stability and Normality 1) Common Cause Variation----> process output forms a distribution that is repeatable and stable and predictable over time 2) Special Cause Variation -----> process output not stable and process distribution changes over time and MAY be unstable if the process average/ variation is out of control 3) Chi-square test----used to test the validity of the normality assumption and data is partitioned into data ranges

Visual Factory (Lean Concept)

Visual Factory a. Visual identification of the status of materials/information throughout the value stream b. Examples-→ (showing units produced, status of materials in/out of raw material warehouse, use of lights to indicate machine status) + Status of work orders (logbook references, internal meetings, and updating line supervisors)

Visual indicator of nonrandom patterns 3 uses?

Visual indicator of nonrandom patterns Used to: i. ID patterns in process data ii. Detection of non-random variations. iii. Detection of trends, osciallation, clusters, & mixtures. d. Typically used when subgroups= 1 i. Subgroups > 1-→ Means or Medians must be calculated and connected (like control charts) e. Provides real-time feedback using variable data. f. P< 0.05-→ Statistically significant non-random pattern g. Number of runs above or below the median= present of cluster, mixture, trend, or oscillation.

How to create Voice of Customer

Voice of Customer To create a VOC: 1) Id. customers and their needs 2) Analyze reactive data ( complaints, service calls, warranty claims) and then consider proactive approaches (interviews, focus groups, surveys) 3) Convert collected data into customer needs 4) Sort out the most important attributes that the customer wants, that is, information critical to quality characteristics. 5) Get specifications from teh CTQ characteristics.

Voice of Customer

Voice of Customer a. Definition= process for capturing customer-related information. i. Proactive and innovative to capture stated, unstated, and anticipated customer requirements, needs, and desires. ii. Goal= customer loyolaty iii. Involves web-based data, compliant logs, survey data, focus group findings, and other data the affect customer's purchasing & relationship decisions. b. Key organizational drivers= customer + market knowledge.

Voice of External Customer

Voice of External Customer--> means constant contract with customer. 1) Complaints may be the only way that company listens to customer 2) Ways that companies can listen to customer" a) Immediate customer surveys and Customer follow-up Surveyss (6, 12. 24 months) b) Personal customer contract c) Electronoic mail d) employee interviews e) Test Marketing f) Use of 800 phone numbers or suggestion boxes g) Ombudsmen --advocate for customers h) Inspectors--ie. mystery shoppers, auditors i) Community Surveys j) Quality garauntees

Demi's 14 points of Management: "Maintaining consistent purposes and adopting a new philosophy"

W. Edwards Deming outlined fourteen points for management. ----basic guidelines for the promotion of quality. 1) Managers should maintain consistent purposes to maintain high product or service standards, to be competitive in the marketplace, and to keep people employed. a) Deming was reacting to what he perceived as pervasive short-term thinking in the business world. b) He advocated the establishment of research and development programs enable constant innovation and progress within the organization. 2) Deming's second point is that businesses (meaning the entire organization from senior to new employees) must embrace whole-heartedly the new quality philosophy, including lobbying governments for laws that foster quality. ------- a) This point was in part fueled by nationalist sentiment during a period in which American businesses felt threatened by foreigners, particularly the Japanese

"Weibull and Johnson distributions

Weibull distributions are 1) Best for continuous data with a set lower boundary, usually zero, and no upper boundary. 2) Like the lognormal distribution, the Weibull distribution often applies to reliability data, in which the interval between failures is recorded. 3) A Johnson distribution, on the other hand, is appropriate for continuous data on which neither a normal nor an exponential distribution may be used. a) Johnson distributions often are useful for data obtained after quality improvement campaigns, because the adjustments to process create non-results. b) Assumes that the data represents the process during the period of collection and that one distribution accurately can represent the data. ***These assumptions are key because they assert that enough data has been collected to make visible any common causes of variation. ****Johnson distributions are more provisional than others, although any statistical distribution should be taken with a grain of salt.

What Are KPIVs? Difference between KPIV and KPOV?

What Are KPIVs? Key Performance Input Variables Input that significantly impacts the variation found in KPOV (Key Performance Output Variables) When the input variable (e.g. fabric quality) is held constant and controlled, the process should produce consistent output results (e.g. satisfied customers) Key Process Output Variable (KPOV) ---The process outputs and/or metrics that are most closely linked to customer CTQs. CTQ trees, fishbone diagrams and affinity diagrams are typically used to link CTQs to KPOVs in the late Define and early Measure phases....a.k.a the big 'Ys'. Key Process Input Variable (KPIV) ---The process variables that have the most influence on KPOV performance. The purpose of the Analyze phase is to determine and validate process KPIVs....a.k.a the big 'Xs'.

What are run Test rules?

What are run Test rules? ---->Western Electric created run-test rules as additions to the standard control chart to be enable identification of patterns in the plotted points on control charts **** Total 8 run tests-----> 1st four test used for P,U,Np, and C charts 1) Run-Test #1--->Shift in the process mean ------> shows One subgroup with more than 3 St. deviations from mean 2) Run Test #2---> 9 consecutive subgroups on one side of the average 3) Run test #3------> 6 subgroups in a row either increase or decrease 4) Run Test #4-------> 14 consecutive subgroups alternate being greater or smaller than the preceding and succeeding subgroups 5) Run Test #5 -------> 2 or 3 consecutive subgroups are greater than 2 st. deviation from the mean 6) Run Test # 6-------> 4 out of 5 subgroups in a row are greater than 1 st. deviation from the mean 7) Run Test #7------> 15 consecutive subgroups are within 1 std. deviation 8) Run Test #8 -------> 8 points ina row are more than 1 std. deviation from the center.

What is an R chart?

What is an R chart? An R chart plots the process range over time for variables data in subgroups. This control chart is widely used to examine the stability of processes in many industries. For example, you can use R charts to examine process variation for subgroups of part lengths, call times, or hospital patients' blood pressure over time. Examine the process variation using an R chart before interpreting the process average with an Xbar chart. The process variation must be in control to correctly interpret the Xbar chart because the control limits of the Xbar chart are calculated considering both process spread and center. If the R chart is out of control, then the control limits on the Xbar chart may be inaccurate and may falsely indicate an out-of-control condition or fail to detect one. You can use the R chart when your subgroup size is 8 or less. Use the S chart when your subgroup size is 9 or more.

What is an Xbar-R chart?

What is an Xbar-R chart? An Xbar-R chart plots the process mean (Xbar chart) and process range (R chart) over time for variables data in subgroups. This combination control chart is widely used to examine the stability of processes in many industries. For example, you can use Xbar-R charts to monitor the process mean and variation for subgroups of part lengths, call times, or hospital patients' blood pressure over time. The Xbar chart and the R chart are displayed together because you should interpret both charts to determine whether your process is stable. Examine the R chart first because the process variation must be in control to correctly interpret the Xbar chart. The control limits of the Xbar chart are calculated considering both process spread and center. If the R chart is out of control, then the control limits on the Xbar chart may be inaccurate and may falsely indicate an out-of-control condition or fail to detect one. You can use the Xbar-R chart when your subgroup size is 8 or less. Use the Xbar-S chart when your subgroup size is 9 or more.

What is an Xbar-S chart?

What is an Xbar-S chart? An Xbar-S chart plots the process mean (Xbar chart) and process standard deviation (S chart) over time for variables data in subgroups. This combination control chart is widely used to examine the stability of processes in many industries. For example, you can use Xbar-S charts to examine the process mean and variation for subgroups of part lengths, call times, or hospital patients' blood pressure over time. The Xbar chart and the S chart are displayed together because you should interpret both charts to determine whether your process is stable. Examine the S chart first because the process variation must be in control to correctly interpret the Xbar chart. The control limits of the Xbar chart are calculated considering both process spread and center. If the S chart is out of control, then the control limits on the Xbar chart may be inaccurate and may falsely indicate an out-of-control condition or fail to detect one. Use the Xbar-S chart when your subgroup size is 9 or more. You can use the Xbar-R chart when your subgroup size is 8 or less.

What is the difference between types of attribute charts ? That is C- chart , P -Chart , Np-Chart and U-chart ?

What is the difference between types of attribute charts ? That is C- chart , and U-chart ? Attribute consists of two cause 1. Defect - No of defect in a given unit. (Units still can be used) 1. Defective - No of Units which are defective in a given bunch or bulk of unit. (Unit cannot be used as it is completely defective) Charts Chart for Defective are 1. P Chart - It calculate the proportion of defectives in each subgroup * Chart - It calcuate the number of defectives in each subgroup Chart for Defects are 1. C Chart - It calcuate charts the number of defects in each subgroup. Use C Chart when the subgroup size is constant. 2. U Chart - It calcuate charts the number of defects per unit sampled in each subgroup. Use U Chart when the subgroup size varies.

Cost/Benefit Analysis - & Traditional Quality Curve

e. Traditional quality curves (Cost vs. Quality graph)-→ Higher quality products result in higher costs. i. The traditional economic model of quality of conformance graph, proposed by J. M. Juran in 1962, demonstrates how the cost of producing your product drops when a quality assurance program begins, only to swing back up as the cost of the quality assurance program increases. A dotted vertical line at the point where the cost of quality assurance intersects with the cost of noncompliance represents the optimum level of quality. This line continues straight up passing through the total cost curve, where it marks the optimal cost of producing your product. At this intersection, you have invested just enough in quality assurance, but not so much to increase the total cost.

X-charts & S-charts

e. X-charts & S-charts → Sigma available i. An Xbar-S chart plots the process mean (Xbar chart) and process standard deviation (S chart) over time for variables data in subgroups. This combination control chart is widely used to examine the stability of processes in many industries. ii. The Xbar chart and the S chart are displayed together because you should interpret both charts to determine whether your process is stable. Examine the S chart first because the process variation must be in control to correctly interpret the Xbar chart. The control limits of the Xbar chart are calculated considering both process spread and center. If the S chart is out of control, then the control limits on the Xbar chart may be inaccurate and may falsely indicate an out-of-control condition or fail to detect one. iii. Used to monitor variables data when samples are collected at regular intervals from a business or industrial process.


Ensembles d'études connexes

Midterm #2: Characteristics of High-Performing Teams

View Set

ATI Comfort, Rest, and Sleep test

View Set

mcpA customer has an active Enterprise Agreement (EA) for Professional Desktop, Windows Server, Microsoft SQL Server, and Microsoft SharePoint Server. Which benefit requires activation?

View Set

CHAPTER 9: Negligence and Strict Liability

View Set

Final Exam - Intro to Environmental Science

View Set

Business Ethics Final-Dr. Sanders

View Set